Follow the reluctant adventures in the life of a Welsh astrophysicist sent around the world for some reason, wherein I photograph potatoes and destroy galaxies in the name of science. And don't forget about my website, www.rhysy.net



Q & A

I do quite a lot of attempting to answer astronomy questions on the internet, so I've decided to collect as many of these as I can. This is by no means a complete list. These are mostly short, often simplified answers to complicated questions - no fancy graphics, lengthy discussions or excessive lists of links here. I've edited the questions and answers here for clarity.

Sometimes there were follow-up questions. In these cases I've edited them so that each question has a standalone answer, you don't need to read the question immediately above to get a more complete understanding. I've also added simplified versions of the questions (which are often quite detailed themselves) in bold, so readers can quickly decide if they want to read a question or not.

Also I've removed multiple punctuation marks so that the questioners look less like a bunch of crazy people. :)

Unfortunately, while the majority of the time the discussions end to everyone's satisfaction, sometimes people believe their ignorance trumps science. This can involve long, unproductive discussions, from which I've here tried to salvage what I can.

If you find an error or would like more detail / links on any particular topic, let me know in the comments !


CONTENTS

The Solar System (and others)
Can something in space rotate the Earth ?
Could you pee around the Moon ?
When NASA moves an asteroid to a lunar orbit, will they need to give it a push to start it orbiting ?
Could the asteroid belt have formed by the destruction of a planet 10,000 years ago ?
Are the numerous moons of the outer planets evidence for an exploding planet in the asteroid belt ?
Was there an asteroid-induced tsunami in the Indian Ocean about 5,000 years ago ?
Will a "tidal wave" from space affect our Solar System ?
Could a large earthquake shift the axis of the Earth ?
Can we calculate gravity on other planets ?
What size asteroid would cause a mass extinction ?
Is there anything that could stop the Earth from going around the Sun ?
Why do we use the Roman names for the planets and not the Greek ?
Are planets born in nurseries like stars are ?
Could the Sun split in two and spit out a mini-Sun ?
Could we stop an asteroid with three weeks warning ?
Would a 400m-wide asteroid just break up harmlessly in the atmosphere ?
Why use radar for searching for asteroids ?
If stars were larger in the past, were planets bigger too ?
Does the Sun make a corkscrew motion as it moves around the centre of the Galaxy ?
Do all the meteorites that hit the Earth make it heavier and slow down its rotation, and maybe also push it out of orbit ?
If you dropped a match into Neptune, wouldn't it catch fire because of all the methane ?
If the Moon eventually decides to leave (because it's slowly getting further away), will we be in trouble ?
OK fine, but if the Moon did leave, could we make a new one ?
If we replaced the Sun with another star during a collision with another galaxy, would we all die ?
What's the speed of the Earth through space if the whole Solar System is moving ?
Is Planety McPlanetface real ?
Why don't you like Planety McPlanetface ?
What will happen if the earth suddenly vanishes from its orbit ?
Why do meteors always land in craters ?
How can we sometimes see the inner and outer planets at the same time ?
Which planets are the most and least spherical ?
Are the most massive planets the most spherical ?
What's the truth about Nibiru ?
Could Venus just be cooling down rather than experiencing a super-greenhouse effect ?
If Callisto and Hyperion were orbiting the Sun, how would they be defined ? Planets ?
How can we detect planets around other stars given that Solar System images have such lousy resolution ?
Is there a chance the IAU will adopt a new definition of "planet" that would include Pluto ?
Where do you think is more likely to harbour some form of organic life in the Solar System, other than Earth ?
Can we see the Apollo landing sites in enough detail to see if the flags have turned white ?
Is asteroid 2017 EA a cause for horror ? What about panic ?
Is our Solar System expanding as the Universe expands ?
Could you jump off Saturn's moon Pan ?
What's the proper term for other solar systems ?
Is the Moon a giant asteroid ?
What is the Sun ?
How many planets are in the Solar System ?
Could we use Lagrangian points to move around the Solar System ?
Can we work out how fast the meteor that killed the dinosaurs was going ?
Why is the Earth still rotating ?
Could an asteroid make the Sun implode ?
Is gravity a constant and if so why did the Apollo astronauts bounce higher on the Moon ?
Why is the Moon leaving Earth ?
Does the Moon have a real name ?
What do you think of Planety McPlanetFace ?
What makes the Northern Lights appear green ?
How many habitable planets are known ?
Could the asteroid belt be the remains of an old planet ?
Bah, you're just not trying hard enough to figure out why the asteroid belt actually was a planet.
How can tidal locking depend on how bright a star is ?
How does the composition of a planet affect its gravity ?
Is a planet's gravity affected by its spinning liquid core ?
Is a planet's gravity affected by the spin of its core ?
Is there a nice simple formula to work out the characteristics of a planet ?
Is this asteroid really from beyond the stars ?
This is a nice picture of Mars isn't it ?
Do scientists know where the Sun was born ?
Do blue moons really look blue ?
Was Kepler being silly when he tried to model the planets of the Solar System as having circular orbits ?
Would an asteroid impact at the Mariana Trench be worse than if it hit somewhere else ?
Are all things big enough to pull themselves into spheroids just different types of planets ?
Is this asteroid impact video accurate ?
How hot does a pole get moving at Mach 1 through the atmosphere ?
Are there really only four planets in the Solar System ?



Stars
Are the stars all dead ?
Can there be dark matter stars ?
How can stars form when gravity is so weak ?
How does dust form ?
Will stars keep recycling gas forever ?
Can a supernova destroy a planet, and if so, at what range ?
Can debris from supernovae destroy planets in other star systems ?
How do we know the distances to the stars ?
Do stars in stellar nurseries have very different compositions ?
What do supernovae have to do with planets ?
Can neutron stars and proton stars share the same space ?
When the giant star UY Scuti explodes, will we feel the effects ?
Can we measure the Sun expanding ?
How many stars are there in our galaxy ? How many new ones are born each year ?
Do stars explode when they're young or old ?
Can "rogue stars" knock stars out of their own star systems ?
Can stars merge ?
Do rogue stars pose a threat to our Galaxy ?
Were there bigger stars in the past ?
How come I can see Polaris all year round ?
Will we ever see a naked-eye supernova ?
Are we looking for gravitational waves as advance warnings of supernovae ?
Would a supernovae at the distance of Alpha Centauri be too close for comfort ?
Would we feel the blast from a supernova at the same time we saw it ?
Could gamma ray bursters explain the Fermi paradox ?
What's the deal with the star with the alien megastructures ?
If we dropped a small wormhole into a star, would it cause a supernova ?
What if we dropped a large wormhole into a star, or into a neutron star ? Would that cause a supernova ?
If a star turned into a black hole would it suck in all its planets ?
If there was a teaspoonful of white dwarf material on Earth we'd all get sucked in, right ?
If there is no oxygen in space, then, how the sun and other stars are burning ?
Are the stars we see in the night sky just in a small part of the Milky Way ?
If the distances between the stars are so large, why do they look so close together in photographs of other galaxies ?
If a star dies , does it still have the same mass ?
If a star becomes a black hole does its mass increase ?
What's it called when you live in a binary star system and one star occults the other? Is it still an eclipse ?
What's the closest two stars can get without colliding ?
What's going on with Tabby's Star ?
Shouldn't Dyson spheres resist absorbing and re-radiating heat ?
Could the dimming of Tabby's Star be caused by evaporating comets or sunspots ?
Could solar winds prevent dark matter from entering star systems ?
Why does a star's gravity get stronger when it dies ?
Could we cool down a white dwarf by blowing cold gas over it ?
Could we cool down a white dwarf by increasing its internal convection ?
Okay, why can't we cool down a white dwarf by increasing its internal convection ?
Okay fine, but what if we shot a black hole through the dwarf to mix its hot and cold layers ?
Right then, what if we put a black hole in orbit of the white dwarf to cancel out its gravity ?
Okaaay... what if we put LOTS of black holes in orbit to cancel out a white dwarf's gravity ?
Sheesh this is tough. What if we fill our white dwarf with teeny-tiny wormholes ?
Okay. I'll give up on making an artificial black dwarf FOR NOW. But could we build a giant ball of iron and drop hydrogen on it to initiate fusion ?
Well, what I'm really looking for is and object that's really hard to detect at a distance of about 1 LY unless something impacts it. Black dwarfs seem to be out but have you got any other substitutes ?
How many stars are in the sky ?
What are the stars ?
If the Universe is expanding then how come the constellations don't change ?
How are these two stars moving on a W-shaped trajectory ?
Could massive stars in the early Universe have created supermassive black holes ?
This article about a star that keeps exploding kills dark matter, doesn't it ?
You're lying about the size of Betelgeuse.
What would we see in a spaceship travelling a very long way from a star ?


Galaxies
Are rogue stars parts of mostly dark matter galaxies ?
In the Universe is expanding, how can galaxies evolve through mergers ?
Can radio signals travel faster than light ?
When exactly will we collide with Andromeda, given that both galaxies have finite size ?
Will the gravity of Andromeda cause our collision to occur more quickly ?
Can a supernova damage our Galaxy ?
What happens to a galaxy when its central black hole evaporates ?
Could galaxies be rotating too quickly due to electical forces rather than dark matter ?
Will we really collide with the Andromeda galaxy in the future ?
How can there be hot gas between galaxies if space is so cold ?
Why study dwarf galaxies as opposed to their larger companions ?
How come we can see other galaxies given that there are no so many stars in our own galaxy ?
What will happen when the Milky Way collides with Andromeda ?
Will the collision of the Milky Way and Andromeda cause dangerous gravitational waves ?
How big is the error when weighing a galaxy ?
Would the estimated mass of a galaxy change using General Relativity instead of Newtonian gravity ?
How is the mass measured for a group or cluster of galaxies ?
What are you planning to research for the rest of the year ?
Should we get the sausages ready for when the the Milky Way collides with Andromeda ?
If there was a quasar in our own galaxy, could we see it with the naked eye ?
Will gravitational waves be significant for studying galaxy evolution ?
If the Sun moves out of the plane of the Galaxy, wouldn't it be pulled towards the centre more strongly and so spiral inwards ?
How do we know about the Sun's motion through the Galactic plane ?
Why does every galaxy have a black hole at the centre ?
Are rogue stars and planets created by galaxy collisions ?
Is there an ordered sequence to galaxy evolution ?
Could these newly-discovered ultra diffuse galaxies be ordinary galaxies at a different stage of their evolution ?
Could Unruh radiation explain the direction of galactic rotation ?
How many gas clouds are there around M33 ?
Is Keenan's giant gassy Ring really all that giant ?
What do you think of this press release about dust ?
What do you think of this silly meme about a void in space ?
Why don't galaxy-sized turbulent gas clouds look like smoke ?
What do you think of the idea of spiral arms being density waves ?
If Andromeda were to go nova instead of colliding with the Milky Way, what would happen ?
What do you think about this article about Dragonfly 44 being mostly made of dark matter ?
Could Dragonfly 44 be full of Dyson spheres ?
Are you very skeptical about Dragonfly 44 ?
What do you think about Dragonfly 44 ?
What would happen if dark matter started absorbing light ?
Does dark matter cause ram pressure stripping ?
Will star formation in galaxies be rejuvenated when all the stars go out and more gas can fall in from their surroundings ?
Could some dark matter halos just be made of normal matter that isn't glowing or reflecting anything ?
What's your take on this press release about a new law of nature ?
Do irregular galaxies have black holes in their centres ?
Are there any new advances in astronomy that you're excited about ?
Is this black hole heading for Earth and if not why are the media talking about it ?
Can black holes travel through space ?
Are these ionised gas clouds evidence of galaxy destruction ?
When galaxies interact are some interactions more important than others ?
Do galaxies and galaxy clusters move ?
Where are you looking for Ultra Diffuse Galaxies ?
Is the standard model of dark matter now looking more likely thanks to this paper ?
Can we measure the distances to gas clouds in our galaxy ?
Is there a plane of satellites around the Centaurus A galaxy ?
What are you planning to use the IRAM 30 m telescope for ?
How do they know the Fast Radio Burst was five billion light years away ?
How did this little gas cloud get so many metals ?
Why didn't this simulated gas cloud collapse ?
Does this simulation include a central supermassive black hole ?
What's neat about this galaxy without any dark matter ?
A galaxy can't have more than one black hole !
Could you make a 3D version of this galaxy fly-through video ?
Is this a picture of the Andromeda galaxy ?
Can turbulent gas clouds really do anything other than explode, since space is a vacuum ?
What's the data set used in this pretty movie ?
Can you shroud a galaxy ?
So what's the difference between "enormous dwarf" and "small regular sized"?


Cosmology
What is dark matter ?
How can we prove dark matter exists ?
With the right telescope, could we see the beginning of the Universe ?
Is the Solar System expanding along with the rest of the Universe ?
Is "dark matter" the wrong name ? Should it be "modified gravity" instead ?
Why didn't the young Universe collapse ?
Was gravity always present ?
Is cosmology a load of mumbo-jumbo ?
Is the expansion of the Universe a certainty, or might this be overturned by future discoveries ?
Could it be that space is moving rather than the galaxies ?
Is space infinite ?
What's the latest progress in detecting dark matter ?
How can we measure the age of the Universe since gravity distorts time ?
How can we measure the age of the Universe since time and space are expanding ?
Can the Universe expand faster than the speed of light ?
Can we see objects when they fall into a black hole ?
If space is flat, can we keep travelling and travel into another dimension ?
When measuring the Cosmic Microwave Background, does the telescope need to point away from the Galaxy ?
Is it really possible to tear a hole in the space-time continuum, cap'n ?
Could dark matter just be normal matter that exists in a parallel universe ?
How fast are we moving away from where the Big Bang happened ?
Can we see back to the very beginning of the Universe ?
Does the inertia of the galaxies affect the expansion of the Universe ?
Does ordinary matter become repulsive at high relative velocities ?
Is there any technology we could use to see right back to the Big Bang ?
As the Universe expands will its chemistry change significantly ?
How does the size of the Universe change over time ?
Is there evidence for the expansion of the Universe besides redshifts ?
Is there any other scientific explanation for redshifts and the CMB that could bring back a static, steady state model to explain our cosmological order ?
Why are distances in space so enormous ?
What's your take on the gravitational wave detection ?
I'd like to know more detail about deciphering the age, direction and source of the gravitational waves.
Since energy and mass are equivalent, are dark matter and dark energy really just the same thing ?
Since matter can never be created or destroyed, are you the same age as the Universe ?
What do you think of the idea that Universe had no initial singularity ?
Does space have limits ?
Why do black holes bend space ?
If space was curved enough could I see the back of my own head ?
How do we know space is curved ?
Is space globally curved ?
Is the Universe shaped like a doughnut ?
Yeah but really, could Steady State be the answer after all ?
How does the expansion of space cause resdshift ? Where does the energy of each photon go ?
Are we sure the Tired Light Theory was wrong, then ?
Who can explain to me why Einstein was trying to find the Gravitational constant of the universe ?
Isn't G related to the total amount of gravity in the Universe ?
Is gravity the weakest force ?
But if we DID have an FTL drive, could we explore the unobservable Universe ?
How do you know that dark matter is diffuse ?
How small was the deuterium fraction created during Big Bang Nucelosynthesis ?
Could the Big Bang be a white hole ?
How can we see light from the Big Bang, shouldn't it have overshot us ?
Does the total amount of energy in the Universe really add up to zero ?
How can the Universe be expanding if it's infinite ?
Could frame dragging explain dark matter ?
Do models of dark matter predict it should be self-interacting ?
If dark matter is self-interacting then shouldn't it behave like a normal gas ?
What's your take on Verlinde's new theory of gravity ?
Does the Bullet Cluster rule out self-interacting dark matter ?
Is gravity caused by protons and electrons spinning through space ?
Is gravity caused by protons and electrons spinning through space causing magnetic fields ?
What's beyond the Universe expanding ?
Is that new theory that does away with dark matter just bunk or is it a real thing ?
Is dark matter any better than the rival theories ?
Can modifying gravity explain away dark matter ?
How can you get something out of nothing, i.e. from the Big Bang ?
How come gravity is stronger on smaller objects ?
Could dark matter really just be electromagnetic forces ?
Yes, but could electromagnetic forces at least be important than people think ?
Don't quasars prove that distant galaxies aren't that old ?
I've disproved the expansion of the Universe, haven't I ?
Why doesn't the Cosmic Microwave Background look homogeneous ?
Why doesn't surface brightness vary with distance ?
But, surely surface brightness does vary with distance ?
Could time dilation due to redshift be making our estimates of star formation rate all wrong ?
Does the Radial Acceleration Relation provide a way to test which model of gravity is correct ?
What do you think of this paper on missing matter ?


Space Exploration
Why are space probes so slow ?
Are there any ideas for faster spaceships ?
Can we use the expansion of the Universe to travel faster ?
Could a big volcanic eruption launch a small moon into space ?
Could a volcanic eruption launch small particles into space ?
Should we send children into space to see if it affects them differently ?
Shouldn't the Voyager spacecraft be accelerating due to the nearest star system ?
When will faster-than-light travel become a reality ?
Are astronauts time travellers ?
Could we find new, dangerous elements on distant planets ?
Do space probes still use radioactive materials as a power source ?
How fast would you have to go to get to Mars in 15 minutes ?
When will Voyager I reach the heliopause, and will we lose communications with it when that happens ?
What if we moved to a planet with more helium ?
How can rockets work in space when there's no air to push against ?
How tall must a building be for someone on top to be weightless ?
Have NASA supressed better technology than rockets ?
What determines escape velocity ? Is it just speed and angle, and is there a better way to get high enough to see the curve of the Earth ?
What if aliens have a different name for planet Earth ?
If I started firing off relativistic projectiles into the Universe and random, should I worry about hitting something ?
Would a relativistic projectile be stopped, fragmented and/or destroyed by interstellar/intergalactic drag ?
Would nuking a dangerous asteroid at the last minute just make things worse ?
Why does the Cygnus spacecraft need to do a de-orbit burn ?
Is there any research into wormholes and negative energy going on ?
Can you spin a ball to make artificial gravity ?
Why do Space X love boats ?
Should astronauts really be running marathons on the International Space Station ?
Why is Martin Rees calling for an end to human space missions ?
Could we build a space battle cruiser if we really, really needed to ?
Is the Orion drive as stupid as it looks ?
Could the ISS rescue astronauts if they fell off ?
How can we improve communication with distant space probes ?
How fast could an Orion drive get us to Mars ?
What's your take on this article about generational starships being unworkable ?
Isn't it just too much effort to ever bother building a generational starship ?
Will the expansion of the Universe limit how much of it we can explore ?
How can we build a gravity rocket ?
Why don't we send some bacteria to Mars to see how they get along ?
Could Star Wars have saved a lot of needless trouble if there'd been an astronomer aboard the Millennium Falcon ?
Nuclear bombs as propulsion ? Isn't that mad ?
Have we imaged the event horizon of a black hole yet ?
Does FAST have a fundamental design flaw ?
Can I shoot down satellites using a radio wavelength laser from a submarine ?
Can your country build some space telescopes please ? It seems like America builds all of them !
If you were an evil dictator, what horrors of space exploration would you unleash ?
Is the solution to the Fermi Paradox actually very simple ?
How can radar get such amazing resolution while being so teeny-tiny ?
Have Virgin Galactic ceased manned flights ?
Could a rocket engine be recovered with a big parachute ?
Nuclear weapons against asteroids are a bit too risky unless we have no alternative.
Is the Orion drive ineffective because the bombs have spherical explosions ?
Is NASA's sterilisation procedure just the same as natural daylight and therefore pointless ?
Could a giraffe take over the Arctic ?
Could we build a solid ring around the Earth for aliens to detect ?
Is anyone trying to build an Alcubierre drive ?
Your Orion variant of 2001's Discovery is all wrong, isn't it ?
Are telescope eyepieces overrated ?
Should we nuke the ISS from the ground to take out the xenomorphs ?
What do you think of this idea to search for alien civilisations by looking for rings of orbiting satellites ?
Could a multi-generational ship be made smaller by taking along frozen genetic material instead of living humans ?
Wouldn't it be terribly difficult to dock with a rotating space station ?
Why did the Chinese send plants on the Moon only for them to die within days ?


Miscellaneous
What's the chance that aliens exist ?
Can we travel through time by eating the right plants ?
How can we detect gravity ?
Is it worth worrying about stupid memes ?
What is time ?
What's the difference between gravitational waves and tidal forces ?
Does an infinite Universe mean that impossible things will happen somewhere ?
What will the first aliens we meet look like ?
How do photons last for billions of years ?
Space is jolly cold. Don't photons run out of energy travelling through it ?
Would two charged black holes repel each other ?
Is there any known way of reversing time ?
If the speed of light isn't constant, why do we measure distance in light years ?
Can you ever escape the graviational field of an object ?
Do gravitational waves keep going for ever or do they eventually stop ?
What's the latest in the search for gravitational waves ?
If two gravitational waves collided, would they cancel each other out or create a new wave ?
Have there been any signals from aliens yet ?
Are you sure it's not aliens ?
Why are we assuming aliens are using technologies we understand to communicate, instead of something more advanced ?
Do gravitational waves cause winds in space ?
If gravitational waves are so weak, does that mean they couldn't have destroyed the Klingon moon Praxis ?
Why do different planets move at different speeds ?
Was Giodano Bruno a scientist ?
Are the physical constants really constants ?
What's the opposite of a black hole ?
What would happen if a black hole collided with a white hole ?
What causes inertia ?
If a black hole and a white hole have the same gravity, surely that makes them the same dang thing ?
Where does matter falling into a black hole go ?
How can we prove there's life or other civilizations on other planets ?
What is the escape velocity of Jar Jar Abrahams and is there a way tp reduce it to zeo ?
How do you know black holes exist ?
How long will it take for all the heavy elements to decay ?
Is it possible stop microscopic black holes from shrinking ? I am totally not plotting to destroy the world.
Can something become a black hole by going really fast ?
Does this nebula look like a brain ? This hour-long documentary is worth watching !
Why didn't you watch this 25 minute video on why gravity doesn't exist and the Earth is flat ?
Could there be black holes made of anti-matter ?
Apart form looking for aliens, what will China's new 500m telescope do ?
What do you think of this UFO report ?
What do you think of this 80 minute documentrary about UFOs ?
Do black holes rip you apart because they're made of antimatter ?
If we can see back in time, can we also see forwards in time ?
Would it take infinite time to cross the galaxy at the speed of light since time stops ?
When black holes merge, do they form a wormhole ?
Do black holes convert energy into mass ?
Can you do science without the scientific method ?
Should the Arecibo radar be subject to ethics review boards to prevent us accidentally signalling our presence to aliens ?
Will the new FAST radio telescope be better at signalling/detecting aliens ?
How about these space facts, eh ?
If nothing can escape from a black hole, why do they have jets ?
Does centripetal gravity affect different masses differently ?
Have we reached the end of physics ?
If aliens visited Earth, what would be the hardest universal human behaviour to explain ?
Are there any known lifeforms which can survive in space ?
Are black holes gateways to other dimensions ?
Could there be a white hole in the centre of our Galaxy ?
Why do you want physics to be broken ?
How do we know black holes are spinning ?
Do all astronomical imaging systems have the same contrast range ?
How could we definitely detect dark matter with an infinite budget and/or resources ?
Can mobile phones take images as good as giant telescopes ?
Is there sound in space ?
Time doesn't exist.
What do you think about UFOs ?
Why doesn't FAST have a radar transmitter to study asteroids ?
Could you survive falling in to a supermassive black hole ?
Can you hear the sounds of space using a radio telescope ?
Can black holes fling supermassive black holes around the Universe ?
Could we move a black hole using a powerful electric charge ?
Could the reported speed of this black hole be in error ?
When will we get these much-promised pictures of the black hole in Sag A* ?
Surely radio bursts in space aren't the least bit surprising ?
Is there an easy way to tell the difference between stars and planets just using the unaided eye ?
What's the difference between dark matter and anti-matter and are there any practical applications ?
Why are black holes such messy eaters ?
Should we all turn out lights off to enjoy the stars and galaxies ?
Could we all be aliens ?
Should I eat more doughnuts ?
Why don't we turn into buckets of goo ?
What do you think of this video about NASA being speechless about the discovery of a super-powerful object billions of light years away ?
Does the Arecibo telescope need to be kept clean ?
What does Physicist Valhalla look like ?
What do you know about this graph ?
Is this article about a wormhole any good ?
What do you think about this article about everything being conscious ?
Nuclear winter must be just a theory, not a fact, right?
What astronomical events are you looking forward to in 2018 ?
Will we get our first picture of a black hole's event horizon in 2018 ?
What do you think of the idea that theories have to be falsifiable to be scientific ?
Does the IRAM 30 m telescope complain when it gets too cold ?
Does that telescope have its own ski lift ?
I'm an Aries. Any advice for my love life ?
Have you tried giving the IRAM 30 m telescope some nice hot chocolate when it's cold ?
Are you a rah-rah scientist ?
Is that my timeline of the Universe behind Stephen Hawking ?
What do you call the points on an orbit around a black hole which are furthest from and closest to a black hole ?
Is mainstream science being a douchebag by pretending the Big Bang is real ?
What shape are black holes ? And what is the reasoning for that shape ?
Did you predict the battle of Callisto from the Expanse ?
How much stuff has been found through microlensing ?
Is the Fermi Paradox really much of a paradox ?
Do we need a better measure of brightness to describe the naked-eye visibility of astronomical objects ?



THE QUESTIONS

THE SOLAR SYSTEM

Can something in space rotate the Earth ?
Q : If earthquakes are strong enough to alter the rotation of the planet, is it not reasonable to infer that a spacequake could alter not only the rotation of the Earth,  but it could also move the Earth, if only a few feet. What I'm getting at is earthquakes can cause great big faults in the ground. Well, is it not the same with spacequakes ? Could they not disrupt the gravity well our planet sit within by moving it thereby changing the degree of axis our planet rotates on?

Currently, the axis is at around 33 degrees. Yet, what is to say that the axis could be changed to say 35 or even 40 degrees, with a large enough spacequake. Say, something as big as a star going nova. It is not in the realm of possibilities?

A : I confess I'd never heard the term "spacequake" before, so I had to do some research. A spacequake is a variation in the Earth's magnetic field. And they can be terrifying.
http://blogs.discovermagazine.com/badastronomy/2009/06/18/ok-so-maybe-we-can-be-a-little-frightened/#.VQiJLI7F98E

However, I don't see any way a magnetic event could affect the gravity well of the Earth in any way. Gravity is caused by mass, and mass alone. To alter the axial tilt of the Earth, you'd need to hit it with something massive or at least have a close flyby.The tilt of Uranus is interesting in that regard.
http://www.universetoday.com/18955/tilt-of-uranus/


Could you pee around the Moon ?
Q : Because of gravity on the moon, does this mean your urine can even go further!? Maybe a circle around the moon ?
[This was obviously a joke thread, but I decided to take it seriously anyway, because it's funny]

A : Let's do this properly. Typical speed of urine, according to the internet, is around 15 ml per second. For water that's a mass of about 0.015 kg, or a volume of 0.000015 cubic metres. Assuming that the hole is a circle 2 mm across, and the urine is ejected in a continuous cylindrical stream, that's a cylinder of length 1.2 metres (so a speed of 1.2 m/s). That seems a little high to me, but sod it I can't be bothered to check because this is a stupid exercise anyway.

Assuming that's correct, the orbital velocity is given by v = sqrt(GM / r) where G is the gravitational constant, M is the mass of the Moon and r is the radius of the Moon. Which turns out to be 1700 m/s. So no, you cannot pee around the Moon.

... HOWEVER you could on a sufficiently small asteroid. Asteroid densities vary considerably, but can be close to that of ordinary water, so let's go for that. We need an orbital velocity v to be equal to 1.2 m/s. We know the density of the asteroid is 1000 kg/m^3. Assume the asteroid is a sphere, then its density will be M / (4/3 * pi*r^3) where M is the mass of the asteroid and r its radius. Re-arranging the equation for the orbital velocity, it can be shown that v = sqrt(G*density*4/3*pi*r^2) which we can re-arrange to find r...

The answer is that you probably could pee all away around an asteroid about 2 km across.


When NASA moves an asteroid to a lunar orbit, will they need to give it a push to put it in orbit ?
Q : Hearing the discussion about placing an asteroid in orbit around the moon got me thinking. Would the asteroid have to be pushed to get the orbit started? I imagine if you simply placed it near the moon, it wouldn't just begin to rotate given that an object at rest remains at rest unless acted upon by an outside force. However, one simple push should get it going after which it would continue on its orbit. Am I correct in this thought process? 

A : Essentially yes, with a but. If you did somehow simply place it near the Moon, it would just fall to the Moon. But in reality everything is already in motion. What NASA's proposed mission will involve is moving an asteroid from its current orbit around the Sun to an orbit around the Moon - so they will give it a push to move it from one orbit to another, rather than making it be in orbit at all (if that makes sense).


Could the asteroid belt be a very recent feature of the Solar System, formed by the destruction of a planet due to a collision with supernova debris ?
Q : Could a supernova explosion have ejected fragments which caused utter chaos in the Solar System about 10,000 years ago, including destroying a planet between Mars and Jupiter which then formed the asteroid belt ?
[This question was part of a larger discussion including archeoastronomy and mass extinctions. I decided to concentrate on the purely astronomical aspects of the questions - i.e. the bits I have some hope of answering !]

A : The energy needed to not just blast apart a planet but also send its fragments hurtling off into deep space (out of the gravity well of the Sun) is significantly greater than just smashing the planet. At 2.7 AU (the distance to the middle of the asteroid belt) the escape velocity of the Sun is about 26 km/s. For an Earth-sized planet, this means the energy required is about 10 times greater than "merely" smashing the planet to bits. (as a ballpark, this corresponds to raising the temperature of an Earth-sized mass of granite by about half a million degrees, but you only need to raise it by a few thousand K to vaporise it - so it's not clear to me whether such an event could result in large numbers of mountain-sized fragments)

I don't see any way of accelerating a planet in the Solar System to collide with another at that kind of velocity, or to get a smaller object up to a much higher speed to carry the same energy. A planet ejected from another star system, possibly by a supernova, might be able to do it - but this is such a fantastically unlikely event that it will never, ever happen (see http://astrorhysy.blogspot.cz/p/q-a.html#supernovaddebrisamageplanet)

But let's assume it did. In that case the composition of the asteroids should resemble that of a planet, i.e. distinct differences between asteroids that were originally part of the crust, mantle and core. Asteroids within those categories ought to have very similar compositions to one another. Each category should be found uniformly throughout the belt since there's no way a collisional origin could cause asteroids with different compositions to be distributed differently from one another. This is not the case. Asteroid composition varies significantly (e.g. meteorite studies : http://www.permanent.com/meteorite-compositions.html) and does vary within the belt, as a function of distance from the Sun : http://www.esa.int/Our_Activities/Space_Science/Asteroids_Structure_and_composition_of_asteroids

I'd also guess (and it is just a guess) that the distribution of asteroids produced by a recent collision should look completely different to the distribution of asteroids formed in situ billions of years ago. Unless the planetary geologists are all massively incompetent, someone would have noticed this. :P

Ceres and other large asteroids are believed to have differentiated interiors and high volatile contents. Volatiles would not have survived the collision, nor would there be time for the asteroids to form differentiated interiors (or probably even form at all).

Observations of the rotation rates of asteroids are consistent with the Yarkovsky / YORP effects acting over billions of years (e.g. http://en.wikipedia.org/wiki/Yarkovsky%E2%80%93O%27Keefe%E2%80%93Radzievskii%E2%80%93Paddack_effect).

Radiometric dating of meteorites has found that they were last molten billions of year ago.

In a word, no.


Would an exploding planet explain why there are so many moons around the outer planets ?
Q : Could the origin of the asteroid belt and the many, many moons of the outer planets be explained by an exploding planet ?

A : Well, the asteroid belt wasn't formed like this (see http://astrorhysy.blogspot.cz/p/q-a.html#recentasteroidbelt). But for the sake of it, let's investigate the issue from the perspective of the moons of the gas giants.

It's widely accepted that many of the small rocky moons of the gas giants are captured asteroids. There are very clear differences between their giant moons (especially the Galilean moons of Jupiter) and their smaller moons. The giants tend to absolutely dominate the mass, with the smaller moons having only a few percent or less of the total moon mass. The orbits of the moons tend to vary. The innermost moons (which includes the giants) tend to all orbit within a very narrow plane, while the outer moons are something more like a swarm of bees. Exactly what you'd expect if some formed in situ and others were captured later.

The moons which are all orbiting in a plane very likely formed at the same time as their parent planet did. The gas and dust which formed the planet would have collapsed into a disc because of its rotation, though the details of how planets and moons formed are not at all well-understood. Even discounting the more obviously captured-asteroid moons, the gas giants still have more moons than the terrestrial planets. But they're also a lot larger, so they must have formed from more material - so there would have been more material around to form more moons as well (even if we don't understand the process very well).

I am not at all sure that an exploding planet is going to produce large chunks of debris, since the energy required to blast it apart is far higher than that needed to vaporise it. But let's assume it does anyway.

If so, we can work out the mass of the postulated planet by considering the masses of the moons and their distance from the planet - assuming that a few fragments didn't have as much velocity so were able to be captured (more on that in a minute). That gives us a mass flux which we can multiply over the area of a sphere centred on the asteroid belt (2.7 AU from the Sun). I did a very crude, quick calculation of this. I have to admit, I was surprised by the result. Ignoring the giant moons (and the small moons in the same plane), the estimated mass of the planet is about 2% the mass of the Earth for the Jovian moons, 7% for the moons of Saturn, 2% for Uranus and 12% for Neptune.

Given the enormous errors in this process, the masses of the gas giant moons are not incompatible (not the same thing as "evidence that this happened") with their origin being a planet in the asteroid belt. Of course, if you pick and choose which moons to accept and reject for the calculation, the results will vary dramatically. And remember that this is assuming a spherical explosion - if you restrict it to something more compatible with the 1 AU thickness of the asteroid belt, those mass estimates will fall considerably.

If you want to explain the asteroid belt (as well as the outermost moons, which are moving much too slowly to have been ejected as solar escape velocity), there must have been a wide spread in the velocities of the debris. But that implies there should also be moons that are on highly elliptical orbits, but there aren't. Most of the orbits are rather circular. I also note that there's a rather sudden cut-off in the outer moons. If the debris had a continuous distribution of velocities, there should also be moons even further from the planet where they could be captured at lower velocities. Saturn displays a similar sort of cut-off in the distribution of its moons (at a very similar distance, 24 million km compared with 30 million for Jupiter), as does Uranus (21 million km) while Neptune has moons at up to 50 million km away.

This really requires a specialist in orbital dynamics to answer properly. My naive impression is that the rather circular obrits on a limited range of inclination angles aren't compatible with a spherical explosion origin.


Was there a giant tsunami in the Indian Ocean about 5,000 years ago, caused by an asteroid impact ?
Q : Could a giant impact explain Noah's Flood ?
[I chose to answer this since I know the questioner is not a Creationist and not set on proving faith-based beliefs. I would probably have ignored it if I thought it would lead to a pointless science vs religion debate.]

A : This idea comes from a press release a few years ago by a group of geologists claiming to have explained Noah's Flood. The "Holocene Impact Working Group" (http://tsun.sscc.ru/hiwg/hiwg.htm) is a group of geologists trying to prove that major asteroid impacts are more frequent than everyone else thinks they are. Readers should beware that this one is well outside my area of expertise !

Let's break this into two questions : 1) Was there a tsunami ?; 2) Was there an impact ? I read four main documents to answer this one : 1) Press release about a crater candidate and the tsunami (http://www.nytimes.com/2006/11/14/science/14WAVE.html?pagewanted=all&_r=0); 2) Book chapter by Gusiakov et al. 2010 about the same (http://www.academia.edu/1451625/Mega_Tsunami_of_the_World_Oceans_Chevron_Dune_Formation_Micro-Ejecta_and_Rapid_Climate_Change_as_the_Evidence_of_Recent_Oceanic_Bolide_Impacts); 3) Peer-reviewed article in the journal "Geology" by Bourgeois & Weiss 2009, describing a simulation of an impact at the proposed location (http://faculty.washington.edu/jbourgeo/BourgeoisWeiss2009final.pdf); 4) A response by some disgruntled geologists who are annoyed by the claims of an ancient tsunami / impact, published in GSA Today, Pinter & Ishman 2008 (http://www.geosociety.org/gsatoday/comment-reply/pdf/i1052-5173-18-6-e14.pdf). Here I'm giving a shortened version of my analysis, you can read my full discussion here : https://plus.google.com/u/0/+RhysTaylorRhysy/posts/akxd8cNuEer.

1) Was there a tsunami in the Indian Ocean about 5,000 years ago ?

Gusiakov 2010 discuss many different features produced by tsunamis : boulders, numerous different rock formations, and the chevron-shaped dunes. However, so far as I can tell they only cite the dunes as evidence for the Indian Ocean tsunami. They make several different claims why these chevrons indicate a tsunami, but they rarely consider any alternative explanations (e.g. the internal structure of chevron-shaped dune features, the lack of sediment in a river, the orientation of river sediments, the wide range of rock fragments in the chevrons, the presence of oceanic microfossils in the chevrons, the orientation of the chevrons with respect to the coastline).

The orientation of the chevrons is categorically dismissed by the simulation of Bourgeois & Weiss 2009, which Gusiakov 2010 appear to be completely ignoring. The presence of oceanic microfossils can be explained simply by the wind according to Pinter & Ishman 2008, who note other examples where this is the case. Annoyingly, though Gusiakov 2010 spend a long time describing all the signatures of tsunamis, they completely fail to mention any of them in this case. For example, it appears to be that there aren't any massive boulders transported high up the shoreline as there were with other tsunami events. What's really worrying is that they rarely even consider other explanations besides a tsunami.

So far as I can tell, they have no evidence of the age of the chevrons and therefore no evidence of when this hypothesised tsunami may have happened - besides ancient myths.

2) Did an asteroid impact the Indian Ocean about 5,000 years ago ?

Leaving aside the evidence for a tsunami, which I find very unconvincing, the evidence for an impact rests on a crater candidate. Even Gusiakov 2010 describe this as "subtle", and - if I may venture so far - it could also be called "wishful thinking". It's half an arc in some very complex terrain - you'd need some really good supporting evidence that an impact happened. By their own admission, the team don't have this. They have samples of rock from near the crater, but : "We have learned from our work on other oceanic impacts that the minimum weight of sample we need to confidently identify the presence (or absence) of impact ejecta is between 10 and 20 grams. The cores we have been working with are old and heavily sampled. Thus, in many cases we could only obtain a 2 to 5 gram sample."

It seems to me that this is taking the flimsiest of flimsy evidence and using it to infer an extraordinary event that is in flat contradiction to the rest of geology and astrophysics. Even without knowing geology, reading Gusiakov 2010 did not feel like reading a scientific paper. Most of the time they don't consider other explanations so they don't try to falsify their own conclusions. Quite honestly it didn't even feel like science. It's a house of cards, literally built on sand.


Will a giant tidal wave from space kill us all ??!?
Q : I either heard or read, recently, where there was a "tidal wave in space", that shook up a few galaxies. Wouldn't that affect our solar system in the next few hundred years ?

A : This was about the following press release : http://www.ras.org.uk/news-and-press/2619-giant-cosmic-tsunami-wakes-up-comatose-galaxies

Here the collision of two galaxy clusters has triggered a shockwave in their gas which may cause increased star formation. The term "tidal wave" is well-chosen to attract attention (that's what press releases are for, after all !) but causes confusion because it's such an ambiguous term with many different meanings : http://en.wikipedia.org/wiki/Tidal_wave
None of them really make much sense when applied to galaxy clusters. I think they're using it in the far more loose sense that's usually used in disaster movies - any large wave, even if it wasn't caused by tides. It's not a term normally used in astronomy.
it won't affect us in any way. It's 2 billion light years away and the effects are due to shocks within the gas in the galaxy cluster. Even in a galaxy in the cluster, individual star systems wouldn't really notice anything - not on human timescales, anyway. The effect of the shock would be extremely weak by human standards - but on the very low density gas, and over millions of years, it can trigger an increase in star formation.

There's pretty much nothing that could possibly happen in another galaxy that would have any effect on us. The distances are just too large.


Could a really large earthquake shift the axis of the planet ?
Q : What if over time, the axis of the world went from 33 degrees to say 0 degrees. Meaning that the world would shift onto its side and rotate that way. That would mean different climates all around. Water might shift, etc. Is there any kind of evidence that that could happen? OR did it happen already?

A : Earthquakes can and do shift the axis, but by a tiny amount. In order to shift it by an amount large enough to cause changes in the climate, you'd probably need enough energy to shatter the planet - at which point you wouldn't be worrying about climate variations.
http://www.bbc.co.uk/blogs/legacy/23degrees/2011/03/can_an_earthquake_shift_the_ea.html


Can we calculate what gravity is like on other planets ?
Q : Can we calculate gravity variables on other planet is that just speculation ?

A : The only variables are the gravitational constant, G, measured in laboratory experiments on Earth, and the mass of the planet. That can be measured from the motion of any objects orbiting it, or its affects on nearby objects.
http://www.scientificamerican.com/article/how-do-scientists-measure/

What size asteroid would cause a mass extinction ?
Q : What size asteroid or comet would cause an ELE (Extinction Level Event). Are we talking the state of Hawaii, the State of Rhode Island or the State of Texas ?

A : A large mountain-sized rock will do it. It did it for the dinosaurs at any rate.
http://en.wikipedia.org/wiki/Chicxulub_impactor
The energy of impact is more dependent on the speed than the mass, which for most asteroids is "bloody fast" (technical term). If I were you, I'd avoid following this twitter feed if you're of a nervous disposition.
https://twitter.com/asteroidmisses
If you're not of a nervous disposition and wondering what the energy of impact means in real terms, have a look here : http://www.projectrho.com/public_html/rocket/usefultables.php?#id--The_Boom_Table



Is there anything that could stop the Earth from going around the Sun ?
Q : Is there such an event that would cause a planet to stop revolving around it's star (ours being the Sun). If so, does that signal an end to a civilization or does it have anything to do with the magnetic shifts that go on over the course of several hundred thousand yrs ?

A : Not many. The most likely is that in a few billion years the Sun will expand to a red giant. If that doesn't completely incinerate the planet, it will shed a lot of mass from its outer layers and form a so-called planetary nebula. That might throw us out of orbit, if it doesn't incinerate us.

A collision or close encounter with a very massive object could do it. That definitely isn't going to happen anytime soon. Maybe over long time scale the solar system is unstable, but again we're talking billions of years.

A close encounter with a black hole from outside our solar system could do it, but there are so few black holes that the chances of that ever happening are close to nil.

It would definitely be very bad for civilization. Probably fatal. Nothing whatsoever to do with the magnetic shifts though, which are due to internal processes of the Earth.


Why do we use the Roman names for the planets instead of the earlier Greek ?
Q : For the naming of the planets in the solar system, why did they elect the names of the Roman gods ? Greek gods are more prevalent in modern culture than the Romans are. Actually, I believe Jupiter's moon Io is derived from Greek origin. The woman who transformed into a cow. It doesn't coincide with the Roman name Jupiter.

A : It wasn't an election, it was a military victory. It was the Romans who conquered Europe and spread their names for the planets, not the Greeks. So their names were established across Europe first. Moons were discovered much more recently.


Are planets born in planetary nurseries, like stars are born in stellar nurseries ?
Q : I'm not sure how to ask this, but I'll give it a shot, hoping I'm understood. When planets are being "born", what determines what goes inside the core ? I have heard of "nurseries", but wouldn't they all be similar in the core, because they are around the same environment ? Or do things interact differently in a nursery than say a singular planet being born ? I read that and it doesn't make much sense, to me... It's ok if no one else gets it.

A : I've heard of stellar nurseries but not planetary nurseries. Planet formation is not all that well understood, but generally they're thought to form around stars. The closer to the star, the hotter the material, so things like methane and ices aren't going to hang around long. Further out colder materials can survive without evaporation. So, there will be different compositions of different planets depending on where they form.

But then you have weird exoplanets like "hot Jupiters" which are gas giants orbiting very close to their star, which throws a spanner in the works.

I didn't read it all but wiki's article looks nice. http://en.wikipedia.org/wiki/Formation_and_evolution_of_the_Solar_System


Could the Sun ever split in two, like, spitting out a baby Sun ?
Q : If Jupiter can pop out "moons" like they are Chicklets, can the Sun also create a "mini Sun"??? OR what I'm getting at is can any star create another star or does that only happen in binary systems ? The reason I ask is I was hypothesizing that when the Sun is "nearing it's end", it might have enough energy to "have a baby"...Yeah I'm tired, and probably not making any sense... sorry.

A : "If Jupiter can pop out "moons" like they are Chicklets..."

It can't.

When the Sun dies, it will have used up most of its hydrogen fuel. At that point, splitting in two would be no help - it would be like chopping your car in two and hoping that would make things better... :P

... that said, it's thought that asteroids can gain enough spin that bits of them can fly off to create binary asteroids. Which would be quite a lot like spitting out moons like chicklets. But there's no known mechanism by which the Sun could spin up so fast that it would split in two. http://astronomynow.com/080710Newresearchputsasteroidsinaspin.html


Could we stop an asteroid in just three weeks ?
Q : The 400m-wide asteroid 2015TB145 nearly hit us ! If it's trajectory had been just slightly different, we'd have only had three weeks of warning. Would this be enough for us to do anything about it ?

A : Keep in mind that while yes, the trajectory would only have had to have been slightly different, that still doesn't mean a chance of a collision was ever very high. There are plenty of other slightly different trajectories if could have taken which would have sent it sailing safely past.

Suppose that the asteroid might have travelled anywhere within that 480,000 km radius of the Earth. Since the Earth itself has a radius of 6,371 km, the area of the Earth / area of the circle within which the asteroid might have passed = 0.00017. So, the chance that it would have hit is something like 0.017%, assuming it went anywhere at random within that 480,000 km.

That's a very simple approximation, mind you.

As to whether we could do anything about it, well really the only option at the moment is a nuke (probably always will be with that little warning - if you have more time, you could use a gentler, more controlled method). Could we go from zero to scrambling a nuclear-armed rocket able to hit a 400m target tens of thousands of kilometres away in three weeks ? I doubt it.


Would a 400m asteroid just break up in the atmosphere ?
Q : I mean, it's just not very big, right ?

A : What ? It's ENORMOUS ! I doubt very much that anything 400m across would break apart in the atmosphere. At 35 km/s it would travel through the entire atmosphere in less than 4 seconds - there's just not enough time for the heat on the surface to travel 200m into the interior. True, the atmosphere will slow it down - but not enough.

Even if it did break up, you really wouldn't want it to. All that energy has to go somewhere. Assuming a density equal to water (pretty decent assumption given known asteroid densities) and a spherical geometry, at 35 km/s that means it has an energy of nearly 5,000 megatons of TNT. Energy scales in direct proportion to mass, but in proportional to velocity squared - it's speed that matters, not mass. So even though it might not be a perfect sphere and the density may be different, this won't affect the calculation much. We're certainly talking about thousands of megatons of TNT.

5000 megatons = 200,000 Hiroshima bombs. It's about 20 times less than the dinosaur killer, but it would give everyone a really bad day. Hence, although asteroid impacts are rare, searching for dangerous asteroids is a worthwhile pastime.

For more scary numbers see +Winchell Chung's epic boom table : http://www.projectrho.com/public_html/rocket/usefultables.php#id--The_Boom_Table


Why would you use radar for hunting asteroids ?
Q : Well, why ?

A : Radar isn't used for finding asteroids, it's used for measuring their properties. Radar has a very narrow beam, so it can only survey a small part of the sky, so it's not a good way to look for new objects. Optical and infra-red telescopes have a much wider field of view, so they can look for moving objects over a very large area of the sky.
http://neowise.ipac.caltech.edu/

What radar is good for is measuring the properties of the asteroids (because it has very high resolution) and also their distance. With the distance the orbit can be constrained much more accurately, so the asteroid will be easier to track in the future.


If stars were bigger in the past, were planets also bigger ?
Q : You were wrote that it is quite possible for planets closer to the origin of the Big Bang, to be larger. Would that mean that there might be larger "Earth-like" planets with similar atmospheres closer to the beginning or am I not understanding something, yet again ?

A : For proper context see this : http://astrorhysy.blogspot.cz/p/q-a.html#idontunderstandwhyitslargerinthepast

It's thought (but not certain, because no-one has seen one yet) that stars in the early Universe were larger because there were less heavy elements around.

Heavy elements (meaning anything other than hydrogen and helium) are effective at cooling. Cooling is important for star formation. As gas clouds compress, they heat up. This heat pushes back against gravity, making it more difficult for them to collapse. If you throw some heavy elements into the mix, the gas cools more easily, so it has less heat holding it up against gravity, so it collapses more easily.

If you don't have heavy elements, the gas doesn't cool so much. This means you need a lot more gas for it to overcome the heat and collapse, so the stars are bigger. But, you also need those heavy elements to form planets. So you'll have bigger stars, but fewer (and probably smaller) planets. At least, that's definitely true for rocky, Earth-like planets. Not sure about gas giants though.


Does the Sun make a corkscrew motion as it moves around the centre of the Galaxy, like in this video ?
Q : That video being this one : https://www.youtube.com/watch?v=C4V-ooITrws

A : No it does not. As I wrote in this post :
http://astrorhysy.blogspot.cz/2013/12/and-yet-it-moves-but-not-like-that.html

Then there's his second video. This one is more objectively just plain wrong. He shows the Sun tracing out a corkscrew pattern as it orbits the galaxy, which makes no sense. The Sun simply goes around the center of the galaxy (and up and down a little bit) - nothing else. It's not orbiting anything else at the same time. For it to trace a helix is just nonsense. He seems to have an almost unique case of helix madness.

However you should read this follow-up post :
http://astrorhysy.blogspot.cz/2015/03/and-yet-it-moves-quite-lot-like-that.html


Do all the meteorites that hit the Earth make it heavier and slow down its rotation, and maybe also push it out of orbit ?
Q : I read that daily, our "Earth is bombarded with more than 100 tons of dust and sand-sized particles." (http://www.mnn.com/earth-matters/space/stories/meteor-asteroid-comet-whats-difference). If so, doesn't the mass on the Earth and consequently, its weight, increase leading to reduction in the speed of rotation of the Earth around itself and the Sun ?

A : The mass of the Earth is about 6x10^21 tonnes (6 followed by 21 zeros). So to increase the mass of the Earth by 1% would take about 6x10^19 days, or 1.6x10^17 years, which is much, much longer than the age of the Universe. Although the increase in mass does change the rotation rate and orbit, the increase is so small that it's negligible. Probably not even measurable.

To put it in slightly more comprehensible numbers, imagine that the Earth has been receiving the same mass in meteorites every day in its 4.5 billion year history. That comes to a total of 1.6x10^14 tonnes. If the ocean is around 5 km deep, then that mass is the same as an area of sea around 200 km on a side. So quite a bit, but still utterly tiny compared to the whole ocean, let alone the entire planet.

The mass of the Sun is about 300,000 times greater than the Earth. This is so much larger that the mass of the Earth basically doesn't matter. Even if its mass doubled, or increased a hundredfold, it would remain in exactly the same orbit.


If you dropped a match into Neptune, wouldn't it catch fire because of all the methane ?
Q : Neptune is basically a big cow fart in space, right ?

A : No, because you also need oxygen for combustion. It's the same reason Jupiter doesn't explode even though it has a lot of hydrogen and has lightning, which is much hotter than a match.


If the Moon eventually decides to leave (because it's slowly getting further away), will we be in trouble ?
Q : How would an "explosion" on the Moon affect life; as we know it; on Earth. I don't think the Moon is large enough to shove the Earth out of the "habitable zone", is it ? Yeah, the tides are going to be affected, but that's about it, right ?

A : Yes, removing the Moon probably isn't enough to change the orbit of the Earth around the Sun all that drastically. But tides are important, and I suspect removing them would have huge consequences. Life has evolved with tides as a constant background. Removing the tides would probably alter ocean currents, which would almost certainly be bad news for the entire ecosystem of the sea. Additionally, some animals rely on moonlight to hunt and navigate. My guess would be that without the Moon we'd be looking at an ecological disaster.

Actually, although the Moon is slowly getting further away from us, it's probably not going to escape at all. Just get a bit further away, until in about 50 billion years it reaches a maximum limit. The Sun will have died in about 5 billion years anyway, so it's a moot point.
https://www.physicsforums.com/threads/will-the-moon-escape-earths-orbit-in-billions-of-years.364921/


OK fine, but if the Moon did leave, could we make a new one ?
Q : Would it make sense to send up an "artificial Moon", to do the job that the real moon won't be doing ? Or is that something for future generations to talk about ?

A : Probably creating an artificial mirror to replace the light of the lost Moon (hey that sounds like a title for a novel...) would not be so difficult. In fact the Russians already tried something like this on a smaller scale, and it very nearly succeeded.
http://motherboard.vice.com/read/the-man-who-turned-night-into-day

But replacing the Moon's tides would require something of the same mass of the Moon. By the time we've got to the stage of being able to move that much mass around, there'll be no need to remove the Moon at all. We'd just go and mine asteroids or something instead. No point causing all that tidal chaos unnecessarily.

So leave the Moon alone. It's getting along just fine without us meddling.


If we replaced the Sun with another star during a collision with another galaxy, would we all die ?
Q : If "Sol" is replaced with a Sun from Andromeda, what is the likelihood that the Earth might change temperatures (either up or down), or would life just cease to exist, as an ELE ?

A : In the event that another star did get close enough to rip us out of the system / replace the Sun, we would all die. Any star that gets that close is going to cause massive disruption to the orbits of all the planets. Regardless of the temperature of the new star, as our orbit varies in distance we would experience extreme variations in temperature. We're talking the oceans boiling into space or freezing solid level of cataclysm. 


What's the speed of the Earth through space if the whole Solar System is moving ?
Q : If the solar system is moving at 600,000 MPH through the galaxy, how could the Earth be moving at only 66,000 MPH? Would it not be "left behind" ? Clearly, I'm not understanding the math behind the differences between the solar system movement through space and Earth's movement around the Sun. Why the discrepancy ?

A : t's about what things are moving relative to. If I walk down the street at 4 mph, I'm moving at 4 mph relative to the surface of the Earth. But the Earth itself is moving around the Sun at 66,000 mph around the Sun. I'm held to the surface by gravity, so I'm really moving at 66,004 mph around the Sun (well a bit less, depending on which direction I'm walking).

Similarly the Earth is going round the Sun at 66,000 mph, but the entire Solar System is orbiting the centre of the Galaxy at 600,000 mph. The Earth is also orbiting the centre of the Galaxy at roughly 600,000 mph, but it's only doing 66,000 mph relative to the Sun.

Or think of a CD. They can spin at anything up to 300 mph, but they don't go anywhere. Unless you've got one in a car, of course. Then it's moving forward at however fast the car is moving. But the speed at which the CD spins doesn't depend on how fast the car is going.

A simpler example might be one car overtaking another. If the first car is going at 60 mph and the second at 80 mph, then they go past each other at only 20 mph. But both of them are moving much faster than this. It all depends on if you're measuring speed relative to the road or to the other car.

It's the same with the Earth going around the Sun. It's going around the Sun at 66,000 mph, but simultaneously moving through the Galaxy at 600,000 mph. Now, if you wanted the true, total speed of the Earth, you'd have to combine them, so you could say the Earth is moving at 666,000 mph. But you wouldn't say that the Earth is going around the Sun at 666,000 mph, any more than you'd say the faster car is really only doing 20 mph. It all depends on your point of view.


Is Planety McPlanetface real ?
Q : I mean....something just ain't right here ! How is this big a$$ planet, just now being discovered ? They've found dwarf planets & all of these other structures, but missed this big a$$ planet?! Oh right...it only comes out every few millions of years right? Well how do they know that, if its newly discovered ?!

A : It hasn't been discovered, just predicted to exist based on the orbits of a few other objects. The whole thing is rather dubious, in my view.


Why don't you like Planety McPlanetface ?
Q : Would you care to elaborate why it is to be dismissed ? I get that it's far from the first time a "new planet is inferred!" but could you explain what makes this time's arguments flimsy ? I've heard about the Kuiper orbits all bunched up and a probe trajectory model that the planet would help, but with no idea what they are worth.

A : It just seems to me that they are making incredibly strong claims from extremely flimsy evidence : "“It's such a long history of people being basically wrong that standing up and saying we're right this time makes us almost look crazy," Dr Brown said. "Except I'm going to stand up and say we're actually right this time."
http://www.independent.co.uk/news/science/planet-9-what-we-know-about-this-new-mysterious-dark-world-a6826156.html

That sort of "we're definitely right" attitude may be fine if you're NGT but it's ridiculous if you're doing actual research. Coupled with Brown's self-labelling as a Pluto killer, the word, "grandstanding" leaps to mind like a kangaroo on a trampoline. The current definition of planet is bloomin' daft anyway. This smacks of glory hunting.
http://www.bbc.co.uk/news/science-environment-35365323 http://astrorhysy.blogspot.cz/2015/07/its-planet-deal-with-it.html

Then there's the evidence itself. Six objects. Six ! From which they infer a clustering probability of 0.007% by chance. Yeah.... really ? Seems to me that that's a crazy-small population of objects to draw any firm conclusions from. There could be any number of as-yet undiscovered objects out there which don't show the same clustering. The idea of a giant planet as the only explanation feels like building a house of cards.
http://www.scpr.org/news/2016/01/20/56964/caltech-researchers-answer-skeptics-questions-abou/


What will happen if the earth suddenly vanishes from its orbit ?
Q : Well, what ?

A : We'll all die. As for the rest of the Solar System, probably not all that much. The Sun is much the most massive component of the Solar System, so our sudden disappearance wouldn't have that dramatic an effect. The orbits of the other planets would shift, but (without checking the exact numbers) probably not that significantly. I'd cautiously speculate that the asteroid belt might do something a bit more funky though.


Why do meteors always land in craters ?
Q : Well, why ?
[It was a joke question, but I was asked for an answer so I gave one]

A : Of course, this is obvious. Every crater is home to a crater monster, sometimes known as an exogorth. A famous example of a such a creature can be seen in the 1980 documentary, Star Wars : The Empire Strikes Back (http://vignette3.wikia.nocookie.net/starwars/images/8/85/Hothslug.JPG/revision/latest?cb=20080201041919). However as passing starships are generally rare delicacies, most exogorths make do with a staple diet of rock flambé.


How can we see the inner and outer planets at the same time ?
Q : I have a question that if earth is after Mercury and Venus then how can we see them at night and moreover earth is between them and other planets so how can we see all at same time ?

A : You can only see Mercury and Venus just before sunrise or just after sunset. Sometimes it's possible to see the other planets at those times depending on which part of their orbit they're on. This online simulator probably explains things more clearly :
http://www.solarsystemscope.com/


Which planets are the most and least spherical ?
Q : What planet is closest to a perfect sphere and farthest?

A : Google tells me that Venus is the most spherical planet...
http://www.funtrivia.com/askft/Question88738.html
... while Saturn is the least.
http://www.geek.com/science/why-are-all-planets-spheres-1562346/
I guess that maybe the search terms aren't as obvious as they might seem.


Are the most massive planets the most spherical ?
Q : I would think that the planet with the most mass would be closest to a perfect sphere and the planet with the least mass would be the furthest from being a perfect sphere - due to gravity.

A : That would be true if all the planets had the same composition and were rotating at the same speed. But this is not the case. Earth is much smaller and denser than Saturn, but it's closer to being a perfect sphere - partly because it's mostly solid and rotates more slowly. Saturn is mostly gaseous, much less dense than Earth, and rotates very much more quickly (around 10 hours to complete one revolution at the equator). It's so much less dense than Earth that even though its mass is nearly 100 times greater, its surface gravity is about the same. But since it's also rotating much faster and made of much less rigid material, it's less spherical despite its greater mass.


What's the truth about Nibiru ?
Q : Planet-x, nibiru. What is the truth behind the rubish

A : The truth is it's utter nonsense. Fortunately I have a file prepared on this already*. The super-short version is that it's nothing but the ravings of crazy people making wildly contradictory claims that make absolutely no sense.
* This was for a group discussion in work back when some genuine (but probably wrong) scientific claims were being made for a distant planet in the Solar System. I decided to educate my colleagues as to why People On The Internet were so excited about this. [Please remember that I retain the absolute right not to answer questions and am under no obligation to discuss conspiracy theories, if I choose to do so that's up to me, but the purpose of these AAAAAAAA posts is primarily for education, not debate]

The long version :
Original idea from a mad lady in Wisconsin who said the aliens talk to her through an implant in her brain. Claimed :
""The Hale-Bopp comet does not exist. It is a fraud, perpetrated by those who would have the teeming masses quiescent until it is too late. Hale-Bopp is nothing more than a distant star, and will draw no closer."[9] She claimed that the Hale-Bopp story was manufactured to distract people from the imminent arrival of a large planetary object, "Planet X", which would soon pass by Earth and destroy civilization. After Hale-Bopp's perihelion revealed it as one of the brightest and longest-observed comets of the last century Lieder removed the first two sentences of her initial statement from her site, though they can still be found in Google's archives."
https://en.wikipedia.org/wiki/Nibiru_cataclysm

Lieder associated object with Nibiru, a planet apparently described in ancient Sumerian texts according to some dude named Stichin. Stichin denies the connection because obviously Nibiru won't return until AD 2900, though he did think the aliens might return in a spaceship before that.

Nibiru is also predicted in the Bible and will have astrological consequences. It's sending us a message that the stock market needs reform :
http://metro.co.uk/2016/01/25/people-reckon-the-new-planet-9-is-going-to-kill-us-all-heres-why-5642753/

Nibiru has been causing chaos for 510 TRILLION years !
"According to historians, Tiamat, a planet which had lain between Mars and Jupiter around 510,000,000 million years ago, was a victim of Planet X, as Tiamat collided with one of the moon's of Nibiru; it crashed, broke into half, as one half became the asteroid belt, and the other Phobos: Mars moon, while the other half is our home, planet earth. " [Yes, that's three halves]
http://www.krschannel.com/Nibiru-.html

"This strange planet has known to be twenty times bigger than Jupiter, with a burning moon which acts like Nibiru's personal sun. Since Nibiru goes much, much further away from our sun; this theory actually does make sense, and stands out."

"This is why the earthquake that are happening in Japan, Chile and other places, could be due to the fact that magnetic pull from Nibiru is increasing as it nears our plain. The pull from Nibiru will increase gravitational force of each planet in a rubber band effect. Most of this informaiton is not provided in Wikipedia."

Orbital period is 3,600 years. Or maybe it's 740,000. Could be it's four times bigger than the Earth, or maybe ten, or maybe 850 times bigger. Might be going to collide with the Earth, or just cause global warming by disrupting the magnetic field of the Sun. Aliens created humans by having sex with animals 300,000 years ago, or maybe it was only 25,000 years ago but who's counting. It's going to cause chaos later this year, or maybe next year, or maybe hundreds of years from now.
http://www.annunaki.org/nibiru/
http://www.nibiruupdate.com/


Could Venus just be cooling down instead of experiencing a super-greenhouse effect ?
Q : When the Venus Pioneer mission was sent to investigate the source of Venus' heat, it was initially determined that Venus is not in thermal equilibrium -- that in fact the planet is releasing 15% more heat than it is taking in, an apparent violation of Carl Sagan's Super Greenhouse Theory (arguably the primary focus of this mission was to generate evidence for the greenhouse effect). A glance at the original papers reveals that the data was corrected to reflect the greenhouse theory as an assumption. It seems that when the data did not cooperate, theorists simply assumed their hypothesis.

My question is: How do theorists rule out an alternative cause for this heat, such as a recent planetary catastrophe? Is it not possible that the planet is actually cooling down from some recent event?

See snapshots of the original papers here:
https://plus.google.com/+ChrisReeveOnlineScientificDiscourseIsBroken/posts/hdDa75Zk3sd

A : [Regular readers are well aware that I think this idea of re-writing the scientifc narrative is a load of old cobblers. On one hand we've got people worried that I'm being too dogmatic and others who think I'm not being dogmatic enough. Methinks that means I've got the balance just about right.]

There's no way I'm going to read 23 papers (and books !) just to answer a question. It's not even clear which quote comes from which source.

What I will say is that atmospheric physics is _hard_. For instance there's a strangely colder layer in the upper regions of Venus' atmosphere :
http://www.space.com/17850-venus-atmosphere-cold-layer.html
Like asteroid impacts, planetary atmospheres are a grey area between astronomy and geophysics. So I don't feel at all comfortable in trying to understand the details of whether Venus is in thermal equilibrium or not, or whether the instrumental measurements had significant errors and were adjusted for good reason or because of malpractice. My experience in observational astronomy suggests that errors are usually larger than people like to claim, but I know nothing of this specific instrument or research area.

Were Venus to have a negligible atmosphere like Mercury, I might try a simple calculation to work out how far in the past it would have been molten in order to get an estimate of when the catastrophe occurred. But the presence of that incredibly thick and complex atmosphere - which at the very least is going to provide a lot of insulation, runaway greenhouse or no, strongly suggests that this wouldn't give a meaningful result. Someone with better knowledge of radiation transport could probably model it.

So could we rule out cooling in some other way ? Hard to say, you should talk to a geologist or a meteorologist or a planetary scientists, or preferably all three. Depends what sort of catastrophe, I guess. The surface of Venus doesn't look much like Earth, or Mars, or Mercury - but then it's more massive than Mercury, much hotter than Earth or Mars, and has by far the thickest atmosphere. So probably it's not too surprising that it also has very different topography. The problem would be to differentiate between this naturally different morphology due to the different properties of the planet and the difference caused by whatever catastrophe is proposed. As far as I know, planetary models aren't sophisticated enough for us to predict things with this level of detail yet.


If Callisto and Hyperion were orbiting the Sun would they be considered to be planets ?
Q : Well, if Callisto (undifferentiated) and Hyperion (big and fluffy) were orbiting the sun, how would they be classified ?

A : [This question was in response to my proposed definition of "planet" to clear up the Pluto issue once and for all : http://astrorhysy.blogspot.cz/2015/07/its-planet-deal-with-it.html. My definition is quite simple - anything that's massive enough to be round (we can quantify exactly how round later) would be considered a planet while anything else wouldn't. But within that definition there would be many, many subcategories. For instance the current IAU list of "dwarf planets" wouldn't change, what are currently considered to be "planets" would become "giant planets", thus putting them on an equal linguistic footing. Further refinement could be used - for example, gas planets, rocky planets, icy planets, etc.]
If we take the definition I've proposed, Callisto would be considered a dwarf planet (or possibly a dwarf ice planet) since it's big and round. Hyperion isn't big or round so it would be considered an asteroid.


How can we detect planets around other stars when photos of planets in our own Solar System have such low resolution ?
Q : I got the impression that our science claims its ability to find earth-like planets from the distant of lightyears, yet failed to produce ultrahigh resolution pictures of moon, mars etc. At least to be openly shown to public. Earth-like, mars-like, venus-like, they have big differences in supporting life! Can you really tell how far those claims go for earth-like planets? i mean this kind of pict. Or does anyone have them in better resolution?
https://lh3.googleusercontent.com/xRgJ2oR05KDKQkgcTxzyZ_f4-p2Gho5ATBwt9eABj3bi0YVbw2__IyOOm2-4FWLKAYeHuPUjUQ

A : That Mars picture is certainly very low resolution but that looks like poor image compression by someone on the internet. Google searching, "Mars surface" or "Moon surface" will get you many, many vastly better images. But landers are extremely difficult : so far there have been successful landers on the Moon, Mars, Titan, a couple of asteroids and a comet, and that's all.

As for planets around other stars, we won't be getting high resolution images of them anytime soon. We can barely resolve even the nearest large stars, let alone planets which are hundreds of times smaller. Even detecting Earth-sized planets is very challenging, never mind imaging them. But we can make some determinations about what conditions must be like there if we know their orbit from their parent star, which will tell us roughly what the surface temperature is. If we could measure their spectra (i.e. colours), which is feasible in the next decade or so, we could work out the chemistry of their atmospheres. Pretty pictures, however, are a very long way off.


Is there a chance the IAU will adopt a new definition of "planet" that includes Pluto ?
Q : Since you're a current member of the IAU, is there actually any validity/possibility of this article's claim?
http://www.independent.co.uk/news/science/moon-100-new-planets-nasa-solar-system-a7588246.html
I understand Alan Stern has a thorn in his side about the 2006 ordeal, but it seems too ridiculous to reclassify the Moon, and moons, etc, planets.

A : The Independent seems to be going downhill lately; that article conveys almost no meaningful information. Here's a better one :
http://www.sciencealert.com/nasa-scientists-have-proposed-a-new-definition-for-planets-and-it-could-change-everything

"Stern and his colleagues have rewritten the definition of a planet, and are submitting it to the IAU for consideration. "We propose the following geophysical definition of a planet for use by educators, scientists, students, and the public," they write. "A planet is a sub-stellar mass body that has never undergone nuclear fusion and that has sufficient self-gravitation to assume a spheroidal shape adequately described by a triaxial ellipsoid regardless of its orbital parameters." If that's a little too jargony for you, their 'layman's version' is simply: "Round objects in space that are smaller than stars.""

And in their original proposal (which is very short):
http://www.hou.usra.edu/meetings/lpsc2017/pdf/1448.pdf
Thus, dwarf planets and moon planets such as Ceres, Pluto, Charon, and Earth’s Moon are “full fledged” planets.

Defining a planet based on what it is clearly isn't bonkers. However, I doubt anyone would be particularly happy to see the Moon labelled as a planet... not unless you also explicitly define what a moon is, which the IAU doesn't do. For that reason alone I doubt the IAU will adopt it. I've also heard somewhat unkind rumours that the whole "planet" definition was done just to take the American's sole Solar System planet away from them...

Solution : get rid of the IAU definition completely and go back to the lack of a system we had before. Then what you'll get is lots of convenient, working definitions of planets and minor planets and ice planets and so on, but none of them will be "official" and they'll be much more flexible (as they should be because there are many grey areas). If that doesn't stop people complaining about what is and isn't a planet nothing will. Heck, we already have plenty of other stupid conventions, like spectral classifications and the magnitude system, why bother pretending we know what a planet is ? :)


Where else in the Solar System do you think is most likely to harbour some form of life ?
Q : Quick Survey.. If you were to bet on which body in the Solar System has more chances of harbouring some form of organic life, which one would you bet on ?

A : I voted Europa as it seems to have the best chance of a permanent liquid ocean. I seem to recall that the water vapour from Enceladus is thought to be coming from much smaller reservoirs so different parts could be liquid at different times.


Can we see the Apollo landing sites in enough detail to see if the flags have turned white ?
Q : Cos there was this meme that said the flags have turned white and a picture of the flag being white.

A : The landing sites can't be seen in quite that detail, but it does appear that at least some of the flags are still there. As far as I know their colour can't be observed directly, but a flag left exposed on Earth for that length of time would be heavily faded, and one left on the Moon (with sunlight not reduced by any atmosphere) would be in much worse shape.
https://www.hq.nasa.gov/alsj/ApolloFlags-Condition.html


Is asteroid 2017 EA's near-miss a cause for horror ?
Q : Because this article was pretty alarmist. Doesn't composition matter more than size ?
http://www.mirror.co.uk/science/horror-nasa-reveals-asteroid-passed-9960548

A : Composition does matter, but the maximum damage is set by the energy, i.e. the speed of collision. Energy is proportional to mass, which depends on density. And that never varies by much more than a factor of a few. So if you know the asteroid's size, you've got its mass pretty accurately already. But energy does as speed squared, so if you double the speed you quadruple the energy - speed is much more important than mass for the energy content. It's true that composition has a role in how likely it is the asteroid will survive the atmospheric passage and collide with the Earth or explode in the atmosphere, but the upper damage limit is set by size and speed far more than anything else. Or to put it another way, if your asteroid doesn't have that much impact energy, it doesn't matter what it's made of at all. Of course if it does have a significant energy then it does matter.

In this case, based on its size, assuming a density equal to water (at most it will be a few times this), and its speed relative to Earth of 18 km/s, I calculate a maximum energy equivalent to about 1 kt of TNT. The Hiroshima atomic bomb is generally reckoned at around 15-25 kt. The Chelyabinsk meteor is reckoned to have been travelling at about the same speed but much larger, with an energy of ~500 kt - and that exploded in the atmosphere.

So it's cause for "horror" only in a very limited sense. In all probability it would have exploded in the atmosphere and caused little or no damage. Had it impacted, only about 1% of the Earth's surface is covered by cities. So there's a very limited chance it would have done any damage at all - admittedly, the worst-case scenario isn't good, but we're talking more like the equivalent of a powerful earthquake than a global disaster.

But if you really want something to keep you awake at night worrying about asteroids, try this... https://twitter.com/AsteroidMisses?ref_src=twsrc%5Egoogle%7Ctwcamp%5Eserp%7Ctwgr%5Eauthor


Is our Solar System expanding as the Universe expands ?
Q : So the Universe is expanding which unless I misunderstand, means our own solar system is also expanding. Would this not also reflect in our weather and tide changes? Not arguing the global warming issue just asking if this could also be impacting our global changes. What can we expect as we continue to move away from our own sun?


A : Our Solar System is effectively not expanding. The amount of expansion depends on the scale you're looking at : even on the scale of a galaxy it's tiny. The expansion is measured by the Hubble constant which is around 71 km/s/Mpc (1 Mpc = 1 million parsecs or about 3 million light years). So, on a scale of a million light years it's significant. But on smaller scales it becomes negligible. On a distance of the scale of the Milky Way, it's expanding by just 2 km/s. This is tiny compared to the motions of stars and gas within the galaxy, which are more like 200 km/s. On the scale of the Solar System (taking 10 billion km as the distance to Pluto) it's about 0.02 mm per second.

0.02 mm per second is about 1 km per year. If that was constant, then in a million years that's not such a small change - a million kilometres would certainly be a sizeable change in our orbit ! Certainly enough to change the Earth's climate much more rapidly than our Sun changes as it consumes its hydrogen fuel.

But there are two problems. First, the expansion of Earth's orbit will be smaller by a factor of 60 just because its orbit is (obviously) smaller than the edge of the Solar System. So that million kilometre expansion reduces down to just 15,000 km, which is nowhere near as significant. Second, and much more importantly, the rate of expansion is so slow that again gravity dominates. Orbital mechanics is complicated and often not intuitive, but the bottom line is that giving the Earth an bit of an extra push in its orbit wouldn't necessarily make that orbit expand by that speed.

So gravity completely overwhelms the expansion on small scales. Only when you start going to large scales - millions of light years - does it become really significant. But, it appears the expansion is accelerating. If this continues, then one way the Universe might end is in a "big rip", where the expansion gets larger and larger on smaller scales. First galaxy clusters would be torn apart. Then individual galaxies would start to explode. Eventually even Solar Systems, and, given enough time, even matter itself.
https://en.wikipedia.org/wiki/Big_Rip

Oh, and I should probably add that the above is a bit of a simplification. Motion through space is not quite the same as motion of space. The classic analogy is to pretend space is a rubber sheet. If you have a ball, you can roll it across the sheet - that's motion through space. But if you expand the sheet, the ball is still moving, even though it stays on the same place on the sheet. That's motion of space, and it gets tricky quite quickly. :)


Could you jump off Saturn's moon Pan ?
Q : Is Saturn's moon Pan so small that you could reach escape velocity just by jumping ?

A : Pan is an exceptionally bizarre-looking tiny moon of Saturn, about 30 km across and shaped like a flying saucer. Here's a nice little image that gives a sense of scale : https://plus.google.com/u/0/+CiroVilla/posts/ajDiiTE6VjA
And more images of this weird-looking thing can be seen here.

Such a bizarre-shaped object is only possible at all (how it formed is another matter) because of its very low gravity - 0.0002 g at most, far too low to pull it into a sphere. Standing on the surface it would take objects about 30 seconds to fall 1 m, and the escape velocity is just 6 m/s (13 mph - a fast run but not a flat-out sprint; Earth's escape velocity is 11,200 m/s or 25,000 mph).

Since I already had some simple code to calculate what would happen to an object moving in a gravitational field I did some simple calculations to see what being on the surface of Pan would be like. Could you jump off it ? Probably not - if your vertical take-off speed (from a standing start) is 3 m/s you're doing well. But you'll soar about 3.3 km straight up and take about 1hr 20min to return to the ground. Launch yourself at the same speed at 45 degrees and you'll spend an hour gracefully arcing over the surface, reaching a height of about 2 km and landing about 6 km from where you started.

The unofficial world record for a standing start high jump is 1.9m, meaning a starting velocity of 6.1 m/s - maybe just enough to be able to jump off Pan and never come back. But the high jumper's centre of mass doesn't necessarily clear the bar, so that could take things down to nearer 5.3 m/s. At that speed the jumper will spend 6 hours off the surface and reach about 21 km altitude.

Or if they angle the jump at 45 degrees they'll travel on a weird-shaped arc (not a parabola because gravity varies significantly with altitude) for 5 hrs 30 min and land about a third of the way around the moon. But, because gravity varies with altitude, jumping at 45 degrees is no longer the way to maximise distance. If they jump horizontally they can actually go straight into orbit, reaching a maximum height of about 7 km, taking a little under 7 hours to return to their jump-off point.

On the other hand because Pan has such a very weird shape, gravity will be weaker on that bizarre flange (since it's further from the centre), so it's quite possible that an Olympic athlete would be able to jump off Pan from a standing start on the equator.

Still, while only elite athletes may be able to reach 5 m/s from a standing start, just about anyone can reach Pan's escape velocity in a run. I'm not sure what would happen if you tired to sprint. Each step already propels you to orbital velocities, so you'll quickly end up going so fast that you start travelling on huge, slow arcs, with each step taking minutes or hours to complete and taking you a significant fraction of the way around the moon.

Peeing would also be interesting; by my reckoning it should be possible to achieve an arc 1 km in length with a maximum height of 300m...


What's the correct term for systems of planets around other stars ?
Q : I've recently been in a dispute about what the proper use of terminologies are, involving what other "solar systems" technically should be called. The opposition feels "solar systems" is properly defined. Adding articles from NASA, and ask a (Cambridge) Astrophysicist. Also stating "a" solar system is defined by planets, etc orbiting "a" sun, not star, or the Sun, but sun.

Myself, and another are on the page of if it's not the Solar System, it should correctly be referred to as a star system, stellar system, planetary systems, extrasolar planetary systems, or the stars' name, and system. like Solar System, comes from our stars' name of Sol. Of course we added articles with this language as well. Obviously, both terminologies are frequently used in explaining things to people, through multiple means, and degrees of communication. So, which is clearly the more astronomically correct?

A : If I had to choose I'd probably go with "planetary systems". It's sufficiently vague that it can include rogue planets with orbiting moons, but is well-defined enough that no-one would be surprised if it included a star. "Star/stellar systems" could be taken to mean any system of stars - binaries, clusters, etc. without necessarily referring to planets. "Extrasolar planetary systems" feels like overkill. Using the name of the star, e.g. the Rigellian system, works, but only if the star has a common name, which most don't. Similarly I think "star system" works well enough for individual cases but not for the whole class.

That said, I'd probably use "solar systems" for outreach purposes, just as I'd use "light year" instead of "parsec".


Is the Moon a giant asteroid ? [Daniel Phillips, age 7]
Q : Well, is it ?

A : It might be ! No-one knows for sure. The problem is that we've looked at lots of asteroids through telescopes, but we've only sent spaceships to a couple of them - and only one managed to bring anything back. We've sent people to the Moon six times, and they brought back quite a lot of Moon rocks to study. But the Moon is a very big place, and six different places isn't so many. So we don't know enough to say for sure.

But from what we do know, it seems that the Moon is very different to all the other asteroids we know about. It's much, much bigger than the next biggest asteroid :
https://upload.wikimedia.org/wikipedia/commons/1/16/Ceres_Earth_Moon_Comparison.png

Also, the Moon appears to be made of different stuff to the asteroids. Actually it seems to be much more similar to the rocks of the Earth !

So our best idea is that the Moon is something else. What we think might have happened is that billions of years ago, when the Solar System was still forming, another huge planet smashed into the Earth and sent up a massive cloud of molten rock which turned into a ring around the Earth, and eventually the Moon. That explains why the Moon is so much bigger than the other asteroids and made of similar stuff to the Earth.


What is the Sun ? [Daniel Phillips, age 7]
Q : Well, what is it ?

A : It's a huge ball of hot, thick glowing gas. It's almost a million miles wide - a hundred times as big as the Earth, and it weighs as much as 300,000 Earths.

The gas that the Sun is made of isn't like the air. It's a bit like honey compared to water : water is runny, honey is thick and goopy. The gas in the Sun is much, much thicker than air. When gas gets like this, it does something very strange. It gets VERY hot and stars to shine like a light bulb. It's much hotter than lava or any fire on Earth, but it isn't really burning or on fire - it's just glowing because it's so hot. We think it can keep going like this for another 5 billion years or so before it finally runs out of energy.


How many planets are in the Solar System ? [Daniel Phillips, age 7]
Q : Well, how many ?

A : Nine : Mercury, Venus, Earth, Mars, Jupiter, Saturn, Uranus, Neptune and Pluto. Some silly people may tell you that Pluto is not a planet, but they are wrong. Don't listen to them.

Well... actually we don't really know what a planet is. We used to think that a planet was anything that went around the Sun, except for the Moon. Then we found asteroids, most of which are small and lumpy but a few are big and round, just like planets - and so are some of the moons around the other planets. And now we know there are lots of huge, round, icy objects at the edge of the Solar System...

So we know there are lots of different things that go around the Sun. Some are big, some are small. Some are smooth and some are lumpy. Some are rocky, some are gassy, some are icy, and a few are made of metal ! Some of them go around each other (a few asteroids have their own tiny moons !) but others don't. They're all very different to each other and we don't really know how they formed. And we know of some planet-like objects that don't go around any stars at all ! That's why it's very hard to say what's a planet.


Could we use Lagrangian points to move around the Solar System ?
Q : Well when we are talking about the lagrangian points , is it possible to use them for space transportation that could also save fuel energy. Also is it possible that if any Lagrangian point was located on Earth , we would be able to go from one place to another on Earth as no effect of gravity would be there and we would be able to move with the rotation of Earth ??????

A : They're points where the gravity of two or more bodies balances out. You can use them to park spaceships in stable orbits (the Hershel and Planck telescopes both used L3), but not really for helping us get anywhere else.
http://www.space.com/30302-lagrange-points.html

If you wanted to create a Lagrangian point on Earth, you'd need to move something of comparable mass to the Earth to such a close distance that the tidal forces would rip it (and probably the Earth) apart.


Can we work out the impact speed of the meteor that killed the dinosaurs ?
Q : At least once in my life, I have heard that we are due for a huge meteor akin to what took out the dinosaurs. My question is can we gather the speed of the meteor which took out the dinosaurs? ?

A : We probably can't get the exact speed because there are many sources of error to consider : the composition of the rock it impacted and the asteroid itself, the angle of its trajectory, its mass and size. All these will play a factor in determining how much energy the impact released and how much damage it did. But we can be confident it was moving at several km/s relative to the Earth, because this is the typical speed Earth orbit-crossing asteroids typically move at.


Why is the Earth still spinning ?
Q : I still don't understand why Earth keeps rotating? Won't friction or other forces slow it down eventually ?

A : Although space is not truly a vacuum, it's so close as makes no difference. Friction is negligible even on time scales of trillions of years, by which point the Sun will have long since died.

But it can lose energy through tidal forces - one side of the Earth experiences a greater force towards the Moon (or the Sun) than the other. This causes it to bulge outwards slightly and heat up as it rotates (i.e. the bulge moves across different parts of the surface). The Moon is already tidally locked towards us, hence we only see one side of it. But it's still rotating - it's just rotating so as to keep the same face presented towards the Earth. The timescale for tidal locking to occur is somewhat complicated (https://en.wikipedia.org/wiki/Tidal_locking) but suffice it to say that the rate of energy loss is small. As far as I'm aware it would take longer than the lifetime of the Sun for the Earth to become tidally locked to the Moon or the Sun.

The short answer is that yes, forces will eventually slow the rotation down, but "eventually" is a very long time indeed since the energy loss rate is very small.


Could an asteroid make the Sun implode ?
Q : This has a planets ,Astroids a solor system coming through are solor system " RIGHT through are's...This is coming at us faster its sligshoting right at us..Could this cause are Sun to implode on us as well..???

A : I'm not entirely sure what you're asking, but we're in no immediate danger from any known asteroid. Based on their measured orbits, there's little or no chance of a collision in the next few centuries. We can't measure them accurately enough to say anything beyond that. There's no way any asteroid could cause the Sun to implode - the largest one is many millions of times less massive than the Sun. If an asteroid did collide with the Sun it would simply evaporate. Many comets have been known to do this.
https://svs.gsfc.nasa.gov/11307


Is gravity a constant and if so why were the Apollo astronauts able to bounce higher on the Moon ?
Q : Is gravity a "constant", or does it change with the size of the object under it's "sphere of influence"? The reason I ask, is that while the Apollo missions were going on, on the moon, the astronauts were able to "bounce", proven they weighed less on the moon, than on Earth... or that there was less gravity on the moon.

A : Weight is just a measure of how strongly something is pulled towards the surface. This depends on the mass of the object and the strength of gravity. If you go to the Moon you'll weigh less because its gravity is weaker, but your mass stays the same. Someone heavier than you on Earth will still be heavier than you on the Moon, but you'll both weigh less than you did on Earth. The gravity of the Moon is less than Earth because it has less mass than Earth.

To expand on that slightly, as I should have done when I originally answered the question, gravity from any object at any point is determined by three things : 1) The value of the "gravitational constant", G; 2) The mass of the object; 3) The distance from the object. "G" is indeed a constant, a fundamental property of the Universe. If its value were higher, gravity would be stronger everywhere. Likewise if the mass of an object is greater, then the strength of gravity at any distance increases. But if the distance increases, then gravity decreases. If you want to work out how strong gravity is, you have to have all three numbers.


Why is the Moon leaving Earth ?
Q : The other question has to do with "why is the Moon slowly moving away from the Earth and how is that possible after all these millions of years? I thought "once a satellite, always a satellite".

A : The Moon has been slowly moving away from Earth ever since it formed - it hasn't just started doing that recently. The reason for this is because gravity also depends on distance - one side of the Moon is quite a lot closer to the Earth than the other, so it feels more of the Earth's gravity than the other. And similarly, one side of the Earth is closer to the Moon than the other. This is why we get tides - because parts of the Moon and the Earth are attracted more strongly than the rest. This is also why the Moon is slowly moving away from the Earth, although the details are somewhat complicated :
http://curious.astro.cornell.edu/about-us/37-our-solar-system/the-moon/the-moon-and-the-earth/111-is-the-moon-moving-away-from-the-earth-when-was-this-discovered-intermediate

However, the Moon isn't leaving. It's just moving from one orbit to another. It'll settle down in about 50 billion years...
https://www.universetoday.com/112450/why-is-the-moon-leaving-us/


Does the Moon have a real name ?
Q : Yes, it's Luna. [This was on someone else's Q&A so I corrected it]

A : Luna isn't the real name of Earth's moon, it's simply, "the Moon". Luna doesn't have an official status, it's just the Latin term for "Moon".
https://futurism.com/the-moons-real-name-and-others-too/

Somewhat dishearteningly, my comment was first labelled as "criticism", then I was told that "correction is the same thing as criticism" and then my comment disappeared. So much for Carl Sagan's quote that "astronomy is a humbling and character-building experience."


What do you think of Planety McPlanetFace ?
Q : This wasn't asked directly, but it's been going around a lot lately. In particular this one study attracted a lot of attention which claims there's no longer any evidence for the so-called "Planet 9" lurking in the edge of our Solar System :
https://www.forbes.com/sites/startswithabang/2017/06/21/goodbye-planet-nine-new-and-better-data-strongly-disfavors-a-giant-world-beyond-neptune/#3706c46760a3

A : Personally, I'm not ready to call it yet. But I AM ready to remind y'all of what I said back in April last year (https://plus.google.com/u/0/+RhysTaylorRhysy/posts/JeFx13ZJ1BB), so I'll quote myself at length :
"I'm just going to go on the record and state that I think planet "9" is a silly thing, and a year or two from now no-one will care about it any more. It just seems to me that they are making incredibly strong claims from extremely flimsy evidence : “It's such a long history of people being basically wrong that standing up and saying we're right this time makes us almost look crazy," Dr Brown said. "Except I'm going to stand up and say we're actually right this time."
http://www.independent.co.uk/news/science/planet-9-what-we-know-about-this-new-mysterious-dark-world-a6826156.html

That sort of "we're definitely right" attitude may be fine if you're NGT but it's ridiculous if you're doing actual research. Coupled with Brown's self-labelling as a Pluto killer, the word, "grandstanding" leaps to mind like a kangaroo on a trampoline. The current definition of planet is bloomin' daft anyway. This smacks of glory hunting.
http://www.bbc.co.uk/news/science-environment-35365323
http://astrorhysy.blogspot.cz/2015/07/its-planet-deal-with-it.html

Then there's the evidence itself. Six objects. Six ! From which they infer a clustering probability of 0.007% by chance. Yeah.... really ? Seems to me that that's a crazy-small population of objects to draw any firm conclusions from. There could be any number of as-yet undiscovered objects out there which don't show the same clustering. The idea of a giant planet as the only explanation feels like building a house of cards.
http://www.scpr.org/news/2016/01/20/56964/caltech-researchers-answer-skeptics-questions-abou/

#NotImpressed"

To resume, this latest study is being greeted with some bizarre double-standards towards those of us who view Planety McPlanteface in a less than favourable light. Apparently, saying, "Goodbye Planet Nine" is definitive chest-thumping and insulting, whereas calling yourself a "planet killer" or saying that the skeptics are "babies" is just fine because it's all light-hearted fun if you take the time to look into it properly. You have to very carefully check the original studies and authors, but the doubters can be dismissed out of hand because they're just obviously biased. And it's apparently actually a GOOD thing to deliberately provoke people when there's no real need to get all hot and bothered, because goodness knows there's not enough anger in the world already. It's fine to wade into a peaceful discussion calling people "hacks" and "calling BS" and expect everyone to treat you with much more respect than you've shown them, apparently, although why exactly this should be the case is something which has quite slipped my mind for the moment.

Sometimes I just don't get people. But this is a good opportunity to remind everyone of just how good a model can be and still be wrong :
http://astrorhysy.blogspot.cz/2016/04/perfectly-wrong-or-necessary-but-not.html


What makes the Northern Lights appear green ?
Q : What makes Northern Lights appear green? If seen from space will they still appear?

A : The lights are caused by charged particles from the solar wind interacting with atoms in the atmosphere. The chemical composition of the atmosphere varies, so at different altitudes the particles are more likely to interact with different chemicals. The light is produced when a particle excites an electron from one energy state in an atom to another. To return to the lower energy states the atom omits a photon, and the wavelength (colour) of the photon corresponds to the change in the energy - which is determined by the structure of the atom (i.e. the chemistry).
http://www.webexhibits.org/causesofcolor/4D.html

And yes, they are still visible from space :
https://www.nasa.gov/image-feature/flying-through-the-auroras-green-fog


How many habitable planets are known ?
Q : Hey, how many planets had we found that can support life and which ones are the closest? Just curious.

A : For extrasolar planets, there are at least a dozen known which are less than twice the size of Earth (so probably have a solid surface) in the habitable zone of their star :
https://www.nasa.gov/ames/kepler/a-keplers-dozen-small-habitable-zone-planets
https://en.wikipedia.org/wiki/List_of_potentially_habitable_exoplanets

But being in the habitable zone only means that their temperature would allow liquid water to exist, based on the distance from their star. Depending on the density and composition of their atmosphere, this estimate might be wrong - and/or the chemistry of their atmosphere could be such that it wouldn't support life as we know it.

There are also a few gas giants believed to exist in the habitable zones. While they might have exotic lifeforms in their atmospheres*, they could also have rocky moons that could support more familiar sorts of critters.
https://www.centauri-dreams.org/?p=26133

* Also, the temperature of gas giants varies internally in a complex way, so even ones outside the zone might have regions within them that are nicely temperate.

Finally, because moons in close proximity to giant planets can be "tidally heated", it's possible that they could have habitable regions even when the heat from the star isn't sufficient. Europa and Enceladus are the best-known examples of this, with a frozen icy surface but possible large volumes of liquid water beneath the ice.
https://solarsystem.nasa.gov/europa/overview.cfm


Could the asteroid belt be the remains of an old planet ?
Q : Why can't the Asteroid Belt not be the remnants of an old planet. We have not found another solar system with an Asteroid Belt. You would think that they would be common. They do not separate the gaseous bodies with the rocky bodies. And if it was a planet there that exploded, and you think that there wasn't enough asteroids in the belt to make a planet, who said there had to be? There would be craters on near by rocky planet and moons in this solar system from that planet, even earth.

A : Models of the formation of the Solar System are still very uncertain, but it would be difficult to form a planet in the belt - it would be torn apart by the gravity of the outer planets acting against the gravity of the Sun.

In fact we do have some evidence for asteroid belts around other stars :
https://spaceflightnow.com/news/n0106/05exoasteroids
But it's very difficult to be sure. Even the largest asteroid (Ceres) is thousands of times less massive than the Earth, so it's extremely difficult to detect them directly. We do know that dusty discs are common around young stars though.


The asteroid belt definitely WAS a planet, actually, because the aliens told me so.
Q : There are several possibilities to having the extinction of a planet. You have to completely open your mind. First of all the planet you are on and this solar system is millions of years old. Life here may have taken a bit to develop, but we are mighty close to the sun. If life barely need the sun, they might move away. If life was here in this solar system before man, thriving and accelerating on other planets in this solar system, resources and jealousy could have an effect on the haves and have nots. Takers could have had a long battle with another planet like Mars. Caused the big trench on Mars and they retaliated by blowing up their entire planet.

Or, the planet could have been too close to the orbit of the unknown moons of Jupiter, and had a major collision.

Or, a unknown large comet could have struck the planet, who knows.

Or, a previous civilization created a black hole and blew up the planet my smash atoms together. Where is your imagination.

A : [The rest of the discussion indicated that this person was a devout believer in aliens who refused to believe that the Valles Marineris feature on Mars could possibly be natural, or that observational evidence was actually quite important. Since evidence-based arguments seemed to have no effect, I decided to try another tack. This didn't work either, but oh well].

I was quite keen on the whole "Face on Mars" stuff back when I was a teenager. Of course we only had the old Viking images back in those bygone days, but one feature being advocated as evidence of a planetary war was that one hemisphere of the planet was much more heavily cratered than the other. It's a fun scenario to speculate about. And if that's all you want to do - speculate about interesting possibilities, imagine ways to make the scenario fit the evidence - then that's all fine, I'll not stop you.

But if you want to claim that it's actually true or a better explanation than the standard models, then you are not correct. I've largely covered this question before :
http://astrorhysy.blogspot.cz/p/q-a.html#recentasteroidbelt

I'd add that although it's true that the ideal method of measuring asteroid composition would be to send Bruce Willis to each and every one of them and dig down as deep as possible, we still do know the surface layer composition pretty well. And there isn't any obvious reason why an exploding planet should create asteroids with different surface layer compositions within the belt, so actually it doesn't matter that we don't know their complete composition.

"You have to completely open your mind... Where is your imagination."

Science is a creative process, but it's tempered by the observational evidence. You can invoke "aliens with advanced technology" to explain literally anything. "Where's my tea ? I'm sure I had it a moment ago... bloody aliens must have stolen it !" And for a while this is fun and interesting. Ancient aliens involved in a titanic battle for the Solar System, cool ! But what you mustn't do is to say, "I already know the answer, it can't be anything other than aliens". If you do that, you have denied yourself a whole bunch of other exciting possibilities - and you will limit your imagination, not enhance it. That, for instance, there could be entirely natural mechanisms for creating a canyon two thousand miles long, a hundred miles wide and four miles deep ! Can you imagine such a thing ? I call that extremely exciting !

Yes, you do have to throw yourself at the mercy of the observers, but that is a wonderful, exciting thing to do. To admit that you don't know how the Universe works but to try to find out, wherever that may take you and no matter how crazy the conclusion. Already claiming that you know the answer without having properly examined the evidence is boring. It just becomes aliens, aliens, aliens... and no-one ever finds them.
http://astrorhysy.blogspot.cz/2016/04/the-most-boring-thing-in-universe.html

Opening your mind should mean considering all possibilities, natural and aliens alike. But after that comes something even more interesting : actually being able to test your scenarios. Sure, you can stick with the speculation if you want. But it's so much more rewarding to say, "my idea predicts this, let's go and test it through observation". Then you realise that many of your ideas have to be rejected. The truly open-minded, rational person finds that their conclusions are forced upon them by the evidence. There is no inconsistency in being open-minded and saying, "actually, we tested that idea and found that it didn't work" - or more pertinently in this case, "actually, the alien hypothesis doesn't make any testable predictions, so I'm going to test all the natural possibilities until I'm satisfied that they can't work". If you invoke an untestable idea like aliens or magic wizards, then you've denied yourself the possibility to truly learn anything, because you have no way to verify it.

It's a cliché, but Arthur C Clarke said it best :
“Two possibilities exist: either we are alone in the Universe or we are not. Both are equally terrifying.”
Forget whether either of those is more likely for a moment and just entertain the second one as a hypothesis. Go look at the stars and consider for a moment that in all that great abyss, the only sentient creatures might be us, on this scrap of a planet in the outskirts of a typical galaxy. All of that might exist with nothing else in the Universe to comprehend it but us. All of that happening just because of the same basic physical processes that we can test right here in laboratories. I don't know if that's really the case, but good God it's a staggering, wild, crazy idea. Don't deny yourself that. You're missing out.


How can tidal locking of a planet to its parent star depend on the star's brightness ?
Q : From this article : https://phys.org/news/2017-08-earth-sized-planets-orbiting-nearest-sun-like.html

"they are not so faint that planets would be tidally locked" - I failed to understand the connection between tidal locking and faintness of the star. Anyone ?

A : I'm guessing that they really mean "massive stars" and "habitable planets". If it's an astronomer writing the article then brightness and mass are often used interchangeably. A less massive star has its habitable zone further in, where tidal locking will occur more quickly.


How does the composition of a planet affect its gravity ?
Q : I was thinking about planetary gravitation. How would planets of different composition, but having the same volume and mass have their gravities vary?

A : If they are all uniform density spheres then the gravity will be identical. Newton's Shell Theorem shows that the gravity inside a uniform sphere is given only by the mass interior to any position. Conversely, outside the sphere gravity depends only on the mass of the sphere and the distance from the sphere. http://www.sparknotes.com/physics/gravitation/potential/section3.rhtml

Of course in practise you can't really have this - if you change the composition, you have to change the density so you change the volume, i.e. the size of the sphere, so you change the strength of its gravity at any position. You'd have to vary the amount of empty space inside the sphere, meaning you change the geometry from a uniform sphere and the shell theorem breaks down. You'd have to know exactly how the mass was distributed to say what the effect of this would be.

Another (maybe simpler) way to think about this is to imagine if the Earth was crushed to the size of a pea, enough to turn it into a black hole. Then if you're placed at the same distance from the centre of the Earth as you are now (about 6371 km), the force you feel pulling you downwards will be identical, even though the composition and density of the Earth is completely different. But the thing is that as you fall closer and closer to the Earth, your acceleration keeps increasing - unlike the current situation, where if you fall down a hole your acceleration (but not your speed) actually decreases.


Does the gravity of a planet have anything to do with its spinning liquid core ?
Q : As far as I understand from this, is, that much of the mass is concentrated in a point wrt the Earth''s core, but Earth's gravity would only work as it does, as long as the core is liquid and spinning at current speed. In that state, gravity (the pull) is equally redistributed outside the sphere of the core. That 'outside' includes the Earth's mantle. There are slight deviations because of the construction of the rock around the core (to the best of my knowledge).

A : It's not directly dependent on whether the core is liquid or spinning. That will affect the magnetic field but not the gravity - with the caveat that it will have some small influence because it will control the precise distribution of matter. The Earth isn't a perfectly uniform sphere, and if the core didn't have a spinning liquid component it's safe to say that plate tectonics would work differently and it might not bulge quite as much in the middle as it does. So it affects those small deviations, but these need specialist equipment to be measurable. If the core was solid, you'd still feel 1g of acceleration.
https://en.wikipedia.org/wiki/Schiehallion_experiment


Does the spin of a planet's core affect its gravity ?
Q : My logic tends to lend more weight towards the need for the sphere to spin in addition to having enough mass.

A : Spin could be a factor in two ways. First, a spinning object will bulge outwards slightly, so the Earth is slightly wider at the equator than at the poles. But this causes only a small change of about 0.5% in strength of surface gravity. Under normal conditions, it's fair to say that spin has really nothing to do with gravity at all : a perfectly rigid sphere will have exactly the same gravitational field whether it's rotating or not.

... Well, almost. The second effect is only really noticeable in extreme circumstances like a black hole. Because gravity is really the curvature of spacetime (not a "force" in the normal sense), space itself can be deformed slightly differently by spinning masses compared to stationary ones. So in really extreme cases, spin can make a difference.
https://briankoberlein.com/2013/09/07/frame-dragging/


Is there a nice simple formula to work out the characteristics of a planet ?
Q : I was thinking about planets and got curious about if there is a way to find a common formula for their spin, size, distance from the sun, mass, other planets. So it would make sense for Mercury's location and spin etc in comparison with Jupiter's location and spin etc.

A : I don't think that's possible. Spin will depend on the size and composition of the planet as well as their distance from the Sun because of the effects of tidal braking (http://www.astronomy.ohio-state.edu/~pogge/Ast161/Unit4/tides.html). That depends on models of planetary formation, which are extremely complicated and can't be reduced to a simple formula.

While you can work out their orbital motion around the sun fairly easily : http://www.astronomynotes.com/gravappl/s8.htm
... what you can't do is work out their intrinsic parameters (mass, size, composition, spin, etc.). For that you have to do it the hard way and observe them.


Is this an asteroid from beyond the stars ?
Q : I don't think this is your area, but have you heard about this? Do you think it's likely an intersteller object?
http://www.foxnews.com/science/2017/10/27/mysterious-object-from-deep-space-has-entered-solar-system.html


A : [This question was asked very soon after the discovery. I had my doubts about whether it was BS or not, especially given the source of Fox News, but of course this turned out to be correct, and there are now umpteen numbers of articles on the unfortuntely difficult to pronounce interstellar object Oumuamua out there.]

It seems plausible enough to me. The Oort cloud is supposed to extend roughly halfway to the nearest star, so I don't see why we couldn't get the occasional comet from elsewhere. Actually I've never really understood how the Oort cloud is supposed to be stable, given its large size (I guess maybe the bulk of the cloud is found at smaller distances, but I don't really know). So yeah, I don't see why not.


Is this a nice photograph of Mars ?
Q : Planet Mars in the morning. Taken by NASA's HiRise Camera aboard the Mars Reconnaissance Orbiter.
https://plus.google.com/u/0/photos/photo/100282219702538007748/6488733633515297986?sqid=100276549585470687394&ssid=9377654b-33ad-41df-9973-22ecda52102c

A : Actually no. That's a CGI rendering by Kees Veenenbos - Mars' atmosphere isn't that thick.
http://rebloggy.com/post/art-space-mars-3d-science-nasa-fc-kees-veenenbos-terragen-jruck-us/94843891858


Do scientists know in which nebula the sun was born ?
Q : Well, do they or don't they ?

A : The Sun's about 4.5-5 billion years old and completed about 20 orbits of the centre of the galaxy. We can't measure its motion accurately enough to trace its exact location that long ago - we'd need to know the motions of all the other stars and material as well, which we don't. More fundamentally, energy injected by stellar winds of short-lived massive stars, as well as supernovae explosions from the most massive of all, tend to disintegrate nebuale on timescales ~10-100 million years. The galaxy's a very dynamic and changing place over the lifetime of a single star. Whatever nebula the Sun was born in is almost certainly long gone.


Do blue moons really look blue ?
Q : [In response to an article about some supermoon thingy featuring a picture of a blue moon]
A blue moon is the second full moon in a month. Its nothing to do with colour however the super moon will be a lunar eclipse.

A : Actually, that's only true in the US. In the UK a blue Moon refers to the very rare cases where unusual atmospheric phenomena do make the Moon appear blue. Having the Moon appear blue in the picture is a bit misleading, but the article does go on to describe the definition correctly.


Was Kepler's early model of a Solar System with circular planetary orbits a silly idea or was it sensible at the time ?
Q : [Response to a comment on this very nice article]
You have to take into considerations the "tools of the trade" he had available, during his lifetime. They didn't have high def imaging nor did they have space travel available to them, so they could not 'observe', the planet 447 yrs ago.

A : I think Einstein said it best : "Make things as simple as possible, but not simpler". Circular orbits are as simple as can be, and had been the prevailing wisdom for centuries. So Kepler and others tried as hard as they could to keep it as simple as possible, until finally the data just couldn't be bent to fit the theory any more and they were forced to add complexity... in a sense. It would be fair to say that ellipses are actually a lot simpler than the final hideously tortured system of circles and epicycles they came up with.

With hindsight, it's easy to judge how foolish they were for sticking so determinedly to circles, but it's interesting to consider what unconscious biases we have today that are making our theories more complicated than they need to be.


Would an asteroid impact at the Mariana Trench be worse than if it hit somewhere else ?
Q : Which would might come first: an asteroid hitting the Mariana Trench, causing a crack in the earth, causing an ELE as we know it on Earth, OR that same asteroid striking the Mariana Trench, causing a tsunami, which in turn would cause volcanoes on land and in the seas to activate, causing an ELE.

A : If an asteroid hits the Mariana Trench, I don't think it's going to make things particularly worse. At that point there's a whole six miles of water for it to penetrate, and a massive volume of water to absorb the impact. AFAIK the oceanic crust isn't particularly thin at deep ocean trenches (I may be wrong !), so if you want to crack the crust you're better off hitting somewhere with shallower water. Hitting dry land won't help though, because the continental crust is much thicker than the oceanic crust. And extends well out into the sea in some cases. So what you'd want to do is get the balance right, and find the place with the shallowest depth of water but over oceanic crust rather than on the continental shelf.

I would guess that it's sheer energy of the impact that's the main factor in triggering an extinction. Secondary effects like tsunamis and volcanoes won't help, but if you've got enough energy in the blast to trigger a volcano, my guess would be you're gonna cause a mass extinction anyway.

My naive guess would be that an asteroid probably could trigger a volcano (and definitely a tsunami). But to what extent I really have no idea. Asteroid impacts are that weird overlap between astronomy and geology. :)


Is anything big enough to be round a sort of planet ?
Q : [For context see this article : https://www.forbes.com/sites/startswithabang/2018/05/08/you-wont-like-the-consequences-of-making-pluto-a-planet-again/#34a6317d3422]
But does that mean, as Alan Stern and David Grinspoon contend, that we should call every object that can pull itself into a round shape a planet?

A : Hell yes, it does. But what we absolutely should not do is make the linguistically absurd step of calling some things "planets" and some things "dwarf planets", as though a dwarf version of a thing was somehow different from the thing itself.
The best solution would be to stop the IAU from making definitions altogether and let planetary scientists get on with things. The next best solution would be to define more categories of objects, e.g. giant planets, dwarf planets, terrestrial planets, gaseous planets, ice planets, etc. We could even have classical planets, recognising that the definition isn't all that well-understood and sometimes just made for historical reasons. The biggest challenge, I think, would be how to define moons as not being planets, though that too might be a prejudice we have to try and escape rather than accommodate.


Is this asteroid impact video accurate ?
Q : I mean this one : https://www.youtube.com/watch?v=bU1QPtOZQZU

A : Cool art. Not sure why the asteroid is covered in lava, or why the ejecta first moves on parabolic trajectories but then appears to stop.

By my calculations a 500 km diameter asteroid moving at the escape velocity of the Earth (it can't impact at less than this speed), with the density of water, has about 3E28 Joules of energy. According to the venerable Boom Table (http://www.projectrho.com/public_html/rocket/usefultables.php#id--The_Boom_Table) that's enough to melt the crust. The damage shown here is an understatement : there won't be any ruined cities or even anything left that resembles the current continents. There will just be a smooth, glowing sea of slowly-cooling lava.

It would take about a century to cool down by my quick and dirty reckoning :
http://hyperphysics.phy-astr.gsu.edu/hbase/thermo/cootime.html

I assumed the mass of the crust is 2.5E22 kg, that the crust is mainly basalt with an emissivity of 0.72, and initial temperature of 1300 K and a final temperature of 300 K. For the particle number I used the molar mass of silicon dioxide since that's most of what basalt is made of. All these numbers are found on the internet.


How hot does a pole get moving through the atmosphere ?
Q : [This was regarding this "What if ?" from xkcd, which is about sliding down a pole from the Moon to the Earth]
Wait; how hot does a pole get that's constantly moving through the atmosphere at mach 1? Does it have any climate effects that would make this story better or worse? Does the air get stirred up enough in the atmosphere that everything starts moving at the speed of the pole?

A : Google sez that Concorde reached a maximum temperature of 125 C, and that's at Mach 2. So hard to handle but not enough to do much else. To expand on the original answer, fireman's poles are about 10 cm wide (snigger), and about 5 mm thick. If they're made of stainless steel, as is common, then that gives then a mass of 115 g per cm of length. The mass of the pole within the 100 km or so depth of the atmosphere will be about a thousand tonnes, which is less than that of a large rocket at take-off. So I predict very minimal thermal effects to worry about.


Are there really only four planets in the Solar System ?
Q : I vaguely remember hearing about this in Cardiff where they occasionally study planet formation :
http://www.mpia.de/~beuther/pf_ws1516_rvb.pdf
From slide 7, the two main modes of planet formation are core accretion and gravitational instability, with the latter only being possible for gas giants. I don't think anyone would go so far as to (seriously) say that objects formed by core accretion aren't really planets. However, I have even more vague memories of a speaker claiming very strongly that anything formed by any other mechanisms wouldn't be a real planet (possibly he was talking about gravitational collapse and brown dwarfs but I'm not certain). It's always struck me as odd to define something by its formation mechanism.


STARS

Are the stars all dead ?
Q : When looking up to the stars it's intriguing to know that the light you see has taken millions of years to reach the Earth. As you are effectively looking back in time (because the light has taken so long to reach the Earth) how do we know what exists in space at this very moment? Is there another way of viewing objects in space that gives a more recent representation than waiting for the light to get here?

Probably a stupid question but it crossed my mind that there could literally be nothing in space anymore but yet what we see is what used to be there...millions of years ago.

A : It hasn't taken all that long to reach us. More like a few thousand years or less. Only 4 years in the case of Alpha Centauri.

https://www.youtube.com/watch?v=ELD_JvSk6xU

The most distant thing you can see with the naked eye is (pretty much) the Andromeda galaxy, from which light has been travelling for about 2 million years. But since it would take about 200 million years for the galaxy to rotate even once, there's just not enough time for there to have been any significant changes.

By measuring the velocities of stars and galaxies, in principle we could work out where everything is right now. This is difficult to do accurately, and there wouldn't be a lot of point to it. Since nothing can travel faster than light, we'd have no way to verify our findings other than waiting a long, long time.

Stars live for many billions of years, and the Universe is about 13 billion years old. New stars are being born all the time, so there's not much chance that all the lights have gone out yet. :)


Does dark matter burn ?
Q : Is it theoretically possible for there to be dark matter "stars" ?

A : Although dark matter is not thought to be able to undergo fusion like normal matter does, some models of it postulate that dark matter particles may sometimes annihilate each other when they collide. In that case, it might just be possible to get something like a dark matter star :
http://en.wikipedia.org/wiki/Dark_star_%28dark_matter%29
http://www.space.com/9302-dark-matter-stars-solve-cosmic-mystery.html
They're a pretty speculative concept at the moment at there's no evidence (to my knowledge) that they actually exist.


How can stars form ?
Q : If gravity is such a feeble force, how is it able to explain the current theories of stellar dust / gas coalescing together from tiny bits with almost non-existent gravity into bigger and bigger clunks ? Something is missing here and maybe gravity is not such an omnidirectional force as believed ?

A : Think bigger - it's not so much about the attraction of individual particles to one another, so much as the collapse of massive clouds. Once you form a cloud the mass of a star (this can happen through various instabilities, e.g. Jeans instability : http://en.wikipedia.org/wiki/Jeans_instability) it will collapse in a time dependent on the mass and size of the cloud. The mass of that cloud being many times the mass of a star, the collapse can easily happen on timescales of millions of years. As it does so, density increases, and the collapse gets faster.

Another way to think about it is that right now, the entire mass of the Earth is pulling you down. Similarly at the edge of a gas cloud, the entire mass of the cloud is acting to cause it to collapse. 

The cloud as a whole may have the mass of a star, but this is very, very different from saying that the center of a cloud has the mass of a star. Collapse of the cloud occurs because the total mass of the cloud is very large.


How does dust form ?
Q : Where's this dust coming from anyway? I thought that after 13 billion (or 40 billion) years there wasn't any left... there is still plenty of it so why is it not all gone yet and what regurgitates it somehow ?
[This question was part of a larger, unproductive discussion]

A : This not so well understood. Fusion inside stars can create heavier elements, which can then be ejected by winds and supernovae explosions. Exactly how much dust can be produced in this way is very controversial. However, what I think you're also referring to is not just the dust, but also the gas, which is a little bit better understood.

The reason there is still plenty of gas around is that star formation is more complicated than a simple collapse of a gas/dust cloud. The gas also has temperature (pressure) which resists the collapse. So first you need to cool the gas for it to start to collapse (exactly how this happens is complicated to say the least). It can also be supported by its rotation and (possibly) magnetic fields. Then, other stars disturb the gas by winds and supernovae (which also help replenish the interstellar gas - not all gas which forms a star is forever lost). So you don't have all gas collapsing at once - only a few, small regions where conditions are just right for the gas to collapse and stars to form.

See also star formation wars.


The lifecycle of stars
Q : Is there a sort of endless lifecycle or are we "blaming" again to be in a Universe just the right age that still have gas hanging around ?

A : Since we know of the existence of white dwarfs, neutron stars and black holes, the recycling isn't perfect - some material is lost. Eventually we will run out of gas (wiki sez this will take a few trillion years). Maybe it's interesting we live so close to the start of this very long era when the Universe has gas... that's almost a philosophical question. I don't know.


I am an evil supervillain, and I'm considering using a supernovae to destroy the Earth. Can you help ?
Q :If I wanted to destroy a planet, would a supernova do the job ? Can I use one to take out planets in neighbouring star systems ?

A : Certainly. Knowing the energy output of a supernova and the energy required to obliterate Earth (both from Atomic Rocket's boom table : http://www.projectrho.com/public_html/rocket/usefultables.php#id--The_Boom_Table), it's easy to calculate the maximum distance at which a planet would receive enough energy to be blown to bits moving at such extreme velocities that they could never recombine (i.e. escape velocity).

Depending on the energy of the supernova, this distance can be as low as 2.5 AU or as high as 250 AU (1 AU = 150 million kilometres, the average distance of the Earth from the Sun). But technically this is only the energy needed to move the entire Earth to its own escape velocity. Of course such an immense, sudden blast is going to do a lot more than give the whole thing a gentle shunt. The energy is equivalent to detonating a bomb made of TNT that's ten times more massive than the Earth.

The temperature of the mantle is typically well above the rock's melting temperature, so when it's exposed to space by the blast (releasing the enormous pressure keeping it as a semi-solid) it will melt of its own accord. A lot of it will even vaporise since it's even above its boiling temperature (iron boils at 2,800 C; Earth's core is at 5,400 C). So I'd speculate that what you'd get it a lot of gravel-sized grains rather than rather than mountain-sized fragments. Difficult to be certain though.

Supernovae are devastating for their own planetary systems, but not nearly powerful enough to actually blast planets apart in other star systems. At a distance of one light year, the energy received by an Earth-sized planet would be billions of times too low to actually destroy it. It might well still sterilise the planet from its radiation, but it wouldn't actually smash it into pieces.


Yeah but seriously, there's this planet I don't like and I don't want it to be there any more. I can use a supernova, right ?
Q : Can debris from supernovae destroy planets in other star systems ?

A : Sorry, but it's extremely unlikely. A large supernova will probably disintegrate just about every planet in its system (see above question). A less powerful one will merely destroy planets within a few AU. The fundamental problem is that you have to give a planet-sized body enough energy to destroy another planet-sized body without somehow destroying that same planet-sized body... tricky.

However, there's one other effect from a supernova that just might be able to do the job. If enough mass is ejected by the explosion, and it's not too powerful, distant rocky planets (say ~100 AU from the star) might just survive relatively intact as massive bodies. But now most of the mass of their parent star will have gone. They may now find themselves at the escape velocity from the stellar remnant, so effectively they get hurled off into interstellar space at several km/s (it's not likely to be much higher than this since orbital velocities are lower the further you are from the star - get closer and the velocities will be higher but you also get closer to the obliteration zone).

Assuming that things are setup just right - supernova just powerful enough, and planets just far enough away - that's potentially sent a swarm of hundreds of rocky bodies careering through the galaxy. Sounds deadly ? Only in the ludicrously unlikely event that they ever hit anything. My favourite analogy for the size of space is to shrink the Sun to the size of an aspirin - at that scale the nearest star is around 300 km away. Planets would be the size of specks of dust. And if you scatter a a few hundred dust grains, the chance that they will hit another dust grain 300 km away is basically nil. A probability of about 1E-22, by my reckoning.


How do we know how far away the stars are ?
Q : How can we know for certain that a star is as far away as we think it is ? I looked it up on Google and it said that for stars over 400 light years away we had to rely on how bright the star is. Is this a precise way to measure the distance from here to a star ?

A : It's true we can only directly measure distances for stars within a few hundred or few thousand light years, using the parallax method (http://www.eg.bucknell.edu/physics/astronomy/astr101/specials/parallax.html). Beyond that, we have to use other methods which are checked using stars for which we can get direct measurements. There's a variety of different methods used to reach further and further distances, the distance ladder : http://www.daviddarling.info/encyclopedia/C/cosmic_distance_ladder.html The further away the object is, the greater the measurement error.

The Gaia mission will greatly increase the number of stars for which we can get accurate distance measurements. As well as giving a more accurate picture of the Galaxy, it will allow us to get more accurate estimates of objects at greater distances : http://spectrum.ieee.org/aerospace/astrophysics/gaias-mission-to-map-the-milky-way


Are stars born in stellar nurseries all very different to each other ?
Q : Why do you think stars in the same nursery have different make-up ? The nursery is very large, a light year across or more, there could be different constituents in that large of a space. I think our star grew up with Barnard, Proxima Centauri, and 4 other siblings, 7 all told.

A : I don't think they do have dramatically different initial compositions. Most interstellar gas (about 70%) is hydrogen.

What certainly will have an effect on their later composition is their mass. Low mass stars slowly and steadily fuse hydrogen into helium for trillions of years. High mass stars have a much higher rate of fusion, quickly exhausting their supply of hydrogen. Then they begin fusing helium into carbon and then carbon into heavier elements.

We probably didn't grow up in the same cluster as our current neighbours. The Sun is about 5 billion years old, so it's orbited the galaxy probably twenty times. Stars have random motions as well so most likely our current neighbours were formed somewhere else.


Do supernovae form planets ?
Q : It's my understanding that depending on mass, several things can happen at the end of a star's life, one being supernova, one being implosion/creation of a dense black hole. Do supernovas not stable star nurseries called nebulae ? Or is that a spin off of the star life cycle, that one of the 2 can occur, star nursery or planetary formation ?

A : Stars are born inside nebulae. When they die a lot of their mass can be returned to the nebula and form more stars.

Really low-mass stars can live for trillions of years. Eventually they swell into red giants, but then collapse again. Not much of their mass ever gets back out.

Larger stars live for a few billion years but when they reach the red giant phase they may also shed their outer layers into a so-called planetary nebula. It's a historical name, it doesn't actually have anything to do with planets. A lot more mass gets re-inserted into the nebula from these stars.

Really massive stars (say > 10x the mass of the Sun) explode as supernova after a few million years. Being incredibly hot, they can also spend a long time shedding much of their outer layers. The surviving remnant of the explosion can form a neutron star or black hole depending on how massive it is. Most of the star ends up back in the nebula either from the winds from the star or from its supernova explosion.

So yes, stars can and do form nebulae, and nebulae can and do form stars. Planet formation, it's thought, happens alongside star formation. A small part of the nebula, being a bit denser than the rest, collapses. As it does so it forms a disc, since random motions within the nebula will have given it some spin. Most of the mass ends up in the centre as a star, but some parts of the disc can fragment and form planets in orbit around the star.

At least, that's the super-simplified version. Huge amounts of details are not well understood.


Can proton stars and neutron stars share the same space ?
Q : Can a proton star and a neutron star share the same space, or does there have to be some distance between the two ?

A : Neutron stars form because the star gets so dense than the protons and electrons can combine (neutrons being very slightly heavier than protons). You can't combine a neutron with anything to make a proton, so proton stars can't form. Anyway if it did happen it would have a massive positive charge and blast itself to bits.


When the massive UY Scuti explodes, will we feel the effects on Earth ? How bad will it be ?
Q : When UY Scuti goes nova, because light waves, shock waves/gravity waves, and sound waves all act differently... how soon after would those in our Solar system have knowledge of that, not by the Hubble or it's replacement, but physically ? Because UY Scuti is so distant, does that mean our descendants won't feel anything for centuries ? Or would not the light, gravity wave and sound waves travel that distance ?

A : The energy released from a type 2 supernovae (as UY Scuti will be) is massive : http://www.projectrho.com/public_html/rocket/usefultables.php. But the distance is also massive. If the explosion is roughly spherical, the energy received per square meter on Earth comes to around 3000 J (3E44 / r*pi*8.98E19^2). That's more than I was expecting, but still isn't very much - enough to heat up 1 kg of water by about 0.7 C (although not really because the energy release takes a few days/weeks, so the actual power, the rate of energy received, will be much lower). We won't even receive this much energy though, because the intervening gas and dust will absorb a lot of it. My guess is that like the supernova that created the Crab nebula (6,500 light years away - a little closer than the 9,500 light years of UY Scuti), it would be visible as bright star but nothing more than that : http://www.space.com/16989-crab-nebula-m1.html


If the Sun is expanding into a red giant, can we observe this process happening ?
Q : Have there been any type of measurements of the Sun taken to see how much it has grown over the centuries ? Also, since Mercury is so close to it, would our first sign of it growing and expanding by taking Mercury out. Sorry, I'm not sure how to word that last sentence and make sense.

A : Offhand I'd say the expansion process is so slow that we won't have been able to measure it over the course of a few centuries. Quick check : let's say it take about 1 billion years to double the Sun's diameter, that means it's growing by about 1 metre per year. But, the expansion is likely to be strongly non-linear, so probably it will be even slower until eventually it expands much more rapidly.
https://www.physicsforums.com/threads/the-time-of-becoming-a-red-giant.460062/
Mercury is a few tens of millions of kilometres from the Sun, so we would certainly be able to measure the Sun growing well before it starts to engulf it.


How often are new stars born ?
Q : How many stars are born in our galaxy every year, and how many are there in total ?

A : The number "1 new star every 18 days" is often quoted, though I'm not sure where it comes from. It's equivalent to a star formation rate of about 20 stars per year, which is extremely high. Other estimates go as low as about 1 star per year; some say 2-3, the highest I can find is 7.
http://arxiv.org/abs/1001.3672
https://ned.ipac.caltech.edu/level5/March15/Kennicutt/Kennicutt5.html
http://www.nasa.gov/centers/goddard/news/topstory/2006/milkyway_seven.html

A caveat is that star formation rate is typically estimated by mass of stars formed, not number of stars formed, and some stars are less massive than the Sun. But most of the links seem to suggest that this isn't that important.

As for the total number, probably 100-400 billion.
http://asd.gsfc.nasa.gov/blueshift/index.php/2015/07/22/how-many-stars-in-the-milky-way


Do stars explode when they're young or old ? What sets them off ?
Q : Is it just age that makes stars explode, or is there more to it than that ?

A : What happens to a star depends pretty much only on its mass. If a star is greater than a certain mass (>~5x the mass of the Sun), it will eventually explode after a few million (or perhaps a few hundred million at most) years. There are no massive stars of a different "type" that avoid this, except maybe for a few which just collapse directly into a black hole.
http://www.jpl.nasa.gov/infographics/uploads/infographics/full/10737.jpg

Also note that novae and supernovae are different phenomena. Supernovae are exploding stars that are unable to continue fusion. Novae are eruptions on the surfaces of white dwarfs (which are long-dead stars that no longer shine by fusion) which are accreting material from a nearby companion star.

So it depends on your definition of age. You could very well say that a star is "old" based on how far through its life cycle it is - that's a perfectly reasonable definition. On the other hand it's also reasonable to say that only young stars explode, in the sense that supernovae occur very soon compared to the death of low-mass stars, which live many times longer than massive ones.


What would happen if a star from one system went into another star system ?
Q : Would it be out of the realm of possibility for a rogue star to either join with or kick out an existing star, in a solar system ? I tried looking online for the answer to that question and all I got was Star Wars related links. So, I figured I'd come here and ask the question. I know stars have been kicked out of their orbits, but not sure how that happens.

A : A star passing through another solar system could certainly cause chaos. Stellar mergers are not impossible, but they are very rare because the distances between the stars are so massive.

Stars can scatter each other out of their original systems though. The gravity of two stars moving past each other can change the orbits of both stars around the center of the galaxy (just as the trajectories of space probes change due to the gravity of planets in the solar system).

Another way this can happen is if in a binary star system, one explodes - the other star no longer orbits anything, so it gets flung off into space.
https://en.wikipedia.org/wiki/Stellar_kinematics#High-velocity_stars

"Rogue stars" are known to exist which are not part of any galaxies. These are thought to form when galaxies collide and parts of them get torn off into intergalactic space.
https://en.wikipedia.org/wiki/Intergalactic_star


If two stars really like each other, can they merge ?
Q : If two stars interact and get close enough, can they merge to become a bigger star ?

A : Yes, but it would depend on the exact geometry of the encounter. Mergers are believed to happen inside star clusters, and are thought to be important in the formation of massive stars (at least some people think so) :
http://thedailyjournalist.com/scientia/two-stars-merging-into-a-super-massive-star-seen-by-astronomers/

That can happen in the case where the stars orbit each other and lose energy through tidal forces. It's not as simple as having the stars get close enough to each other, e.g. Phobos is moving closer to Mars whereas our Moon is moving further away from the Earth. The point is that it's a slow process which can take millions of years, so the stars gradually merge rather than slamming into each other.
http://www.universetoday.com/14258/phobos-might-only-have-10-million-years-to-live/

Direct head-on collisions are very rare because the distances between the stars is so much greater than the size of the stars. If it did happen, it's safe to say it would be a spectacular event. Because of the gravity of the Sun, any star which collided with it would be moving at at least 617 km/s by the time it hit. That would release as much energy as the Sun produces in ten million years.

KERBLAMO !


Are rogue stars dangerous ?
Q : A rogue star, primarily known as an intergalactic star, is a star that has escaped the gravitational pull of its home galaxy and is moving independently in or towards the intergalactic void. More loosely, any star in an unusual location or state of motion may be termed a rogue star. In other words........ IT WILL KILL OUR GALAXY IF IT WANDERS IN OUR GALAXY!!!!!!!!!!!!
[Yes, this quote is verbatim]

A : A rogue star is absolutely no more dangerous than any other star. The mass of a star is about one hundred billion times less than the mass of a galaxy. There are precisely ZERO ways in which a star, rogue or otherwise, can destroy a galaxy.

How one goes from "unusual location" to, "threatens the entire galaxy" is quite beyond me.


Were there larger stars in the past ?
Q : What is the likelihood that there is a star larger than UY Scuti closer to the beginning of the universe ?

A : Extremely high. In the early Universe there was nothing but hydrogen and helium, whereas today there are also a great deal of heavier elements. These can be important in gas cooling, which is what you need to get gas to collapse and form a star.

Since there were almost no heavy elements in the early Universe, cooling was not as efficient. To overcome the pressure from the heat of the gas (pushing it outwards) would have required much higher gas masses. Whereas UY Scuti is maybe 50 times more massive than the Sun, early stars may have been many hundreds or even thousands of times as massive as the Sun.
http://astronomy.swin.edu.au/cosmos/P/Population+III
http://www.universetoday.com/24776/what-were-the-first-stars/


How come I can see Polaris (the North Star) all year round ?
Q : Do I have a celestial stalker or something ? I mean, jeez, take a hint already !

A : The size of the Earth's orbit around the Sun is extremely small compared to the distance to even the nearest star, and the Earth doesn't change its orientation as it orbits. So the north pole of the Earth is always pointed towards Polaris.


Are we likely to see a supernova with the naked eye any time soon ?
Q : I want the Earth to have a second Sun, damnit !

A : It's not impossible, but don't get your hopes up.
http://www.iflscience.com/space/could-supernova-be-visible-earth-next-50-years


Will gravitational waves provide advance warning of supernovae ?
Q : Is that why NASA has sent out the gravitational wave "buoys", So that Earth could "at least have a chance", at preparation ?

A : Gravitational waves should travel at the speed of light (if the theory is correct) so they won't give any advance warning. But they may be useful in finding supernovae that are hidden by all the gas and dust that blocks our view of much of the Galaxy.

But mainly the point of looking for gravitational waves right now is to win a Nobel prize. :) They're on of the few (possibly the only) remaining major predictions of General Relativity that has yet to be observed. People have dedicated their lives to finding them, so the detections would be an end in itself. A non-detection would be even more exciting, because then we'd know that relativity has a serious flaw.


Would a supernova at the distance of Alpha Centauri be dangerous ?
Q : It's possible that I may have gone a little too far with my desire to see a second Sun.

A : Interesting question. Fortunately none of the stars in the Alpha Centauri system are big enough to go supernova. If they did, we definitely wouldn't get blasted to bits, so that's good news :
http://astrorhysy.blogspot.cz/p/q-a.html#supernovadamageplanet

However, by my very quick and rough calculation, the energy we'd receive from the explosion would be equivalent to covering the Earth with one 100W light bulb in every square metre, and plausibly 100x that. Factor in that much of this radiation will be dangerous UV and gamma rays, and it's safe to say that things would be bad.

There's also a very nice explanation here : https://hps.org/publicinformation/ate/q10108.html


Would we feel the blast from a supernova at the same time as we saw the explosion ?
Q : Now photons travel the speed of light (given) so some energies match that? Can particles travel a percentage of the speed of light ? Or would a sun going nova reach us en mass about the same time as its distance 20 light years away (20 years)? At 40 light years would the mass of the explosion take much longer and have less mass because of its expansion ?

A : Some particles will be accelerated to nearly the speed of light, but only a small fraction, and we're talking subatomic particles rather than large meteorites. Supernovae expansion velocities are generally measured at at most a few thousand kilometres per second (10% the speed of light), and usually less, so it would take decades or centuries after we see the explosion for the blast wave to hit us.
https://en.wikipedia.org/wiki/Supernova_remnant

Indeed the mass that hits us will be much less than the mass of the whole supernova, because the explosions are very roughly spherical. Even at 4 light years, the fraction of mass that hits us would be far, far less than a trillionth of a percent.


Could gamma ray bursters explain the Fermi paradox ?
Q : The Fermi paradox, for unfamiliar readers, is that "if they existed they would be here", or, "where is everybody ?". That is, intelligent life has arisen at least once in the Galaxy (us), so it should have done so many times in the past as well. There's been more than enough time to colonise the whole galaxy by now, so where is everybody ?

Fr a complete overview see : http://www.projectrho.com/public_html/rocket/aliens.php#id--Alien_Contact--The_Fermi_Paradox

A : On GRBs, I don't know so much about them but I'm not sure we really know enough about them to say what influence they might have had on the Galactic habitable zone (https://en.wikipedia.org/wiki/Galactic_habitable_zone).

My feeling is that the Fermi paradox is still a paradox. Although there are large stretches of geological time where change was slow and gradual, there also seem to have been a few sudden, important events. Multi-celluar life didn't really get going until the Cambrian Explosion, possibly triggered by the end of Snowball Earth. The extinction of the dinosaurs was due at least in large part to an impact event. The development of the human brain seems to have happened in a few million years or less. So, it seems to me that there should be a spread of development of lifeforms, from some planets which are still hosting unicelluar life to others which could have civilizations much more advanced than ours.

Given that, and that even at sub-light speeds (which could be done with today's technologies if we really, really wanted to - a massive Orion ship or a solar sail comes to mind) the Galaxy could be colonized in just a few million or tens of millions of years, the lack of a Galactic Empire seems surprising.

None of the proposed explanations seem convincing to me. Life just isn't common ? But it seems to have started on Earth very quickly. Intelligent life is rare ? Perhaps, but once it started on Earth it took off very quickly indeed. Difficult to reach other stars ? Yes, but surely not that difficult. A Great Filter (e.g. a huge GRB) that prevents life from evolving beyond a certain point ? Difficult to believe a galactic-spanning civilization wouldn't be able to do something about it (even if that just meant hiding).


What's the deal with the star with the alien megastructures ?
Q : Come on, it's aliens, isn't it ?

A : [I wasn't actually asked this directly, but I'v written quite a lot of responses to this so I thought I'd collate them]

A : Normally my advice is to wait a year (literally a year) to decide if a press release story is really interesting or not. It takes a long time to do a proper, independent analysis, and often stories depend on quite complicated reasoning.

Not in this case however. Here we have something where the observational data show that something seriously weird is afoot, and no amount of analysis is going to change that. Out of literally millions of stars observed over the years, this one is unique.

"Tabby's Star" is weird because its brightness is changing in not one but TWO ways that have never been seen before. First though, it's a main sequence star - we know this from very precise measurmenets of its colour. This means it's in the most boring part of a star's life where all it does is sit there fusing hydrogen into helium. OK, it might have the occasional flare and starspots, but that's about it. Major changes should take millions of years to become detectable. For comparison, the Sun (which is only a little bit smaller and cooler then Tabby's Star) varies in brightness by around 0.1% over the course of an 11-year cycle.

Tabby's Star is doing two incredibly weird things. The first is that its brightness occasionally drops dramatically - by as much as 20% in a few days. No other (main sequence) star does this. The second is that its overall brightness has been steadily decreasing over the last century - also by around 20% in total. Unfortunately there aren't any records further back than this. Assuming this rate to be constant, it would have been (just about) naked-eye visible around 5,000 years ago, but to be unmissable you'd have to go back 7,000 years. So there aren't any ancient star catalogues which could help us.

Thus far, no-one has a good explanation of either of these behaviours. Even the alien megastructures hypothesis isn't without problems, because the heat from the star ought to make them radiate in the infra-red, but this isn't seen. Comets could cause the rapid dips, but it requires an impluasibly large number of giant comets - and an outright insane number for the century-long dimming (it's not obvious to me why the number of comets should be increasing with time either), plus there's again no IR excess. No-one seems to hae suggested that it's just not a main sequence star, presumably because the data is good enough to rule this out.

For once, the over-used "scientists baffled" headline may be right on the money. If a mundane explanation isn't found - give it year - then my guess is we're looking at 5-10 years before we can say with confidence what the heck is going on here. What that will turn out to be I have no idea.


If we dropped small wormhole into a star, would it cause a supernova ?
Q : What happens if you drop a black hole/wormhole into a star? How does it vary depending on the star (mass, stage of life/remnant), and the hole size (stellar, primordial, varied intermediary sizes) and type (black hole, wormhole with empty space on the other size) ?

There is one relatively famous SF example where they drop a (6.7m diameter, disc-shaped) wormhole in a (possibly Sun-like) star (with a planet in decaying orbit around a black hole on the other side) to blow it up it up in a matter of hours (and those are the good guys).

A : I am familiar with the example of which you speak. It always struck me as an odd way to creating a supernova given that stars which are less massive don't explode. Maybe it's about disturbing the balance between pressure and gravity by rapidly removing a lot of mass.

It's easy to estimate how much mass can be removed, but it depends strongly on where in the star the Stargate is since temperature varies massively from the interior (~15 million K) to the "surface" (~5000 K). That means the particle speeds will vary from something like 6.5 km/s to 350 km/s.

The average density of the Sun is only slightly higher than that of water, so the following approximation will give us a handle on how fast the Stargate will drop :
http://www.thenakedscientists.com/forum/index.php?topic=49977.0

A terminal velocity of a few m/s means we're looking at several years for the Stargate to reach the core. Or maybe it never will, since the density of the Sun at the core is many times that of iron or even (probably) naquada, which isn't that heavy.

Anyway, near the surface let's assume the average density for the Sun - 1410 kg/m^3. The area of the wormhole is pi*(6.7/2)^2 = 35 sq m. Velocity of the plasma into the wormhole is 6.5 km/s. So in 1 second we get a volume of material equivalent to 35*6500 = 2.3E5 cubic metres = 3.2E8 kg. In 12 hours that's 1.4E13 kg. Alas, the mass of the Sun is 2E30 kg, so we've only decreased the mass by around 7E-16 %. This will achieve precisely diddly-squat.

If we allow the Stargate to have some technomagical propulsion so that it reaches the centre of the Sun, the total mass lost in 12 hours will be about 3E-16 %. Still useless. [Oops - I forgot to account for the greater density in the core, but since it's only 100x greater that's still nowhere near enough to be significant !]

You may be wondering about the black hole's gravity sucking in matter faster, as clearly shown in the original "oops, we've connected to a black hole" episode. Well, even at its most extreme that clearly never reached more than a few gs, which means the infall rate won't be significantly higher.

So the method in the show just doesn't work at all. As to whether removing a really large amount of mass would cause a supernova, I'm not sure. Maybe if you removed all of the outer material around the core, it would expand to reach a new equilibrium. I don't think it would explode though. To do that you'd need to increase the pressure drastically, and I can't see any way to do that by removing mass.

Stars being eaten by black holes have been observed. They don't explode, but they do do interesting stuff :
http://www.huffingtonpost.com/entry/black-hole-ate-star-burped_us_565cbaace4b079b2818b5b90


What if we dropped a large wormhole into a star, or into a neutron star ? Would that cause a supernova ?
Q : I want to cause a supernova using a wormhole. Can you help ?

A : Yes I can. if you threw a neutron star or white dwarf into a giant star, you could potentially trigger a supernova or at least a nova. Novae happen when material accumulates on the surface of a white dwarf to the point where there's a massive fusion explosion. I don't think this is possible in the case of a wormhole, where material is being drained.

Still, if you did remove enough of the star you could destroy a planetary system, albeit in a more stately manner. If you remove the entire fusing core of the Sun (which extends to about 0.25 solar radii) you'll have removed enough mass to send all the planets into highly elliptical orbits. Not quite enough to send them hurtling into intragalactic space, but close. Which is not terribly threatening to Apophis [Evil alien in Stargate whose ship is destroyed when our plucky heroes drop a womrhole into a star] though.

But then it hit me that the best way to destroy a star with a wormhole is obvious : don't link a star and a black hole, link two stars. Massive stars for preference. But if you really really want to make sure, link to a neutron star.

If you had a massive Stargate that could swallow the neutron star whole and spit it into the target star, you'll get a TZW object. That might be short-lived on astronomical timescales, but probably won't instantly explode. Probably the increased density of material around the neutron star causes an increased temperature, preventing material from quickly infalling.

However, if you have a regular sized gate (assuming it can survive), then things get much more spectacular. Pulled by the neutron star's tremendous gravity, the gate will slice through it at nearly the speed of light. As it does so it will spew neutron-degenerate matter into the target star. Without the gravity of the neutron star to hold it together, this will be bad. Like, seriously mega-hyper-total bad.
https://www.youtube.com/watch?v=o_EBqZPCZdw
http://io9.gizmodo.com/5805244/what-would-a-teaspoonful-of-neutron-star-do-to-you


If a star turned into a black hole would it suck in all its planets ?
Q : You know how planets orbit the sun because of its intense gravity? The planets around this black hole are sucked inwards because of the intense gravity.

A : No, for two reasons. Firstly, most stars that form black holes first explode as supernovae (it's only the surviving core which collapses to a black hole) which most likely will destroy any planets which were in orbit. Secondly, the gravity of the black hole at any given distance is no stronger than that of the star.

The mass of the black hole is no larger than that of the star that created it - and usually a lot less as most of the star is blasted away in the supernova. So, if any planets did survive, they'd actually move further away from the star since the gravity at that distance is now weaker than it was before.

The gravity only reaches extremely high levels very close to the black hole. With the star, the strongest gravity will be at the surface of the star. If you were to move inside the star, the gravity actually gets weaker. Inside a sphere, it's only the mass beneath you that pulls you down gravitationally - and the deeper inside the star you are, the less mass beneath you, so the weaker the gravitational pull.

Now, with a black hole, things are different. All the mass is now contained within a much smaller region (technically an infinitely small point in standard GR). Let's say the star was 1 million kilometres across. At 1 million kilometres (or more) from the black hole, you experience exactly the same gravity as if the star was still there. The difference is that now, if you move closer, you experience higher gravity, because the all the mass of the star is still beneath you. So, if you were just 1 kilometre from the center of the star you'd experience basically no gravity at all (because almost all the mass is above you, so effectively doesn't count), whereas if you were 1 kilometre from the black hole you'd be experiencing massive levels of gravity since all the mass is still beneath you and now you're effectively much closer to it.


If there was, like, a teaspoonful of white dwarf material somewhere, would it destroy the Earth ?
Q : If you had a tablespoon of a white dwarf and you put it into your coffee, it would go through the cup, and travel through all the layers of the earth until it reaches the centre. Then, the earth would probably collapse...

A : The density of a white dwarf is incredibly high, but a teaspoon of white dwarf matter would only weigh a few tonnes - not enough to cause the Earth to collapse.
http://www.popsci.com/technology/article/2009-10/what-would-happen-if-i-ate-teaspoonful-white-dwarf-star


If there is no oxygen in space, Then, how the sun and other stars are burning ?
Q : Yer fancy book-learnin' don't scare me nuffin !

A : Stars are not like giant lumps of coal. They shine by nuclear fusion, where light elements combine to make heavier elements. In the process, a very small amount of mass is converted into energy. But honestly, googling "how does the Sun shine" will get you dozens of good website explaining this. Here's one :
http://earthsky.org/space/why-does-the-sun-shine



Are the stars we see in the night sky just in a small part of the Milky Way ?
Q : Is this meme about all the stars you can see being within this yellow circle accurate ?
https://pbs.twimg.com/media/ByaUZASCcAAan9I.jpg

A : Yes it is ! Well, within reason. Of course we can also see other galaxies, e.g. the Andromeda Galaxy, which is about 2 million light years away :
https://en.wikipedia.org/wiki/Andromeda_Galaxy

However, to me the word "stars" in this context suggests individual stars, not star clusters or galaxies. And it suggests stars we actually do see routinely, not by deliberately looking for a particular star at a particular distance. So I downloaded the image and measured the size of the circle, and by my reckoning it's around 3,300 light years in radius. That's pretty much on the money as far as seeing individual stars with the naked eye goes.
http://www.armaghplanet.com/blog/how-far-away-is-the-farthest-star.html
(If you Google this you'll find that there are maybe one or two stars outside this circle but I think this is case of unnecessary pedentary. Probably > 99% of stars are within this circle, which in my view is good enough)

On the other hand, once you allow, say, double stars and other multiples, things quickly get much, much fuzzier. If you count supernovae, then they can be seen at very much greater distances indeed. But obviously we don't normally actually see them, and the meme seems to be about what we usually see.
http://physics.stackexchange.com/questions/45759/what-is-the-farthest-away-star-visible-to-the-naked-eye


If the distances between the stars are so large, why do they look so close together in photographs of other galaxies ?
Q : Well, why ?

A : There are two reasons for that. First, the stars illuminate gas and dust within the galaxies, so as well as the light that comes directly from the stars, you're also seeing light reflected off nebulae within the galaxy - which are much larger than the stars.

Secondly, and more importantly, there's the effect of resolution. Your eye (or a telescope) can only perceive details down to a certain size - but features smaller than that can still be seen. The size they appear to be is determined by the point spread function, which depends on the size of the lens (among other things) :
https://en.wikipedia.org/wiki/Point_spread_function

Simpler explanation : Imagine someone holds up a shiny coin which reflects sunlight toward you. If they're standing 10 metres way, it'll look like a bright shiny coin. 100 metres away and you'll have no idea what they're holding, but you'll still see a bright point of finite size. 1000 metres away and it will look exactly the same, just dimmer. The important thing is that beyond a certain point, the coin won't look any smaller.

It's the same with stars through a telescope. If you have high enough resolution, what appear to be diffuse clouds can be seen as individual stars. Hubble has been able to do this for nearby galaxies, but beyond a certain distance even its resolution isn't high enough - everything looks fuzzy.


If a star dies , does it still have the same mass ?
Q : Well, does it ?

A : Mostly no, but it depends on how massive it was to begin with.

Really massive stars don't live very long (a few million years or less) and emit strong winds. More than half of their initial mass can be ejected into space before they finally explode as a supernova. The final remnant core could either be a neutron star or black hole, but is usually only a small fraction of the initial mass of the star.

Less massive stars like our Sun live a lot longer (a few billion years) but don't explode. Their winds are much less violent. However when they start running out of hydrogen fuel they expand into red giants. They can then shed their outer layers to produce so-called "planetary" nebulae (which are nothing to do with planets), leaving behind just a small white dwarf which again has just a small fraction of the star's initial mass.

But very low mass stars have even less violent histories and can shine for trillions of years. They never shed much of their mass or explode. So the final white dwarf is probably much closer to the star's initial mass than for larger stars. Not sure of the numbers though.

[Someone pointed out that the stars convert mass into energy, with the Sun losing 5 million tonnes per second. This is true but irrelevant.]

5 million tonnes per second is actually not significant. Over the ~5 billion years of the Sun's existence that comes to a grand total of just 0.04% of the Sun's total mass.


If a star becomes a black hole does its mass increase ? Does its size depend on its density ?
Q : If a star becomes a black hole, its mass will be some fraction of the mass of the star. But if there is surrounding material, the mass of that black hole can then increase over time. For example most stars are binaries, so the black hole might grow by accreting material from the second star. This is how stellar mass black holes are detected.
http://blackholes.stardate.org/objects/factsheet.php?p=cygnus-x-1

It's not really correct to say that the more the density, the bigger the black hole. Technically the density of the singularity is infinite - which is a big problem for relativity. For a non-rotating black hole the size depends only on its mass, not the density :
http://astronomy.swin.edu.au/cosmos/S/Schwarzschild+Radius


What's it called when you live in a binary star system and one star occults the other? Is it still an eclipse ?
Q : Apart from "brrzzzkkglblob", or whatever the alien is for "eclipse", obviously.

A : I think it would still be called an eclipse, since the systems where this is observed are called eclipsing binaries.
http://csep10.phys.utk.edu/astr162/lect/binaries/eclipsing.html


How close can two stars get without colliding ?
Q : I read that there is a planet which orbits two suns. I don't remember the name, but what is the closest two suns can be together without colliding? I imagine it depends on the size too..?

A : The absolute distance will depend on the sizes and masses of the stars. But there isn't really a strict limit, since stars can actually be in contact with each other yet still orbiting each other for many millions of years without merging.
http://www2.astro.psu.edu/users/rbc/a1/lec16n.html


What's going on with Tabby's Star ?
Q : Can somebody explain this to me ? More and more scientists are claiming that the star dropped to 20% in terms of light emission. Some are even now hypothetically stating that it is a Dyson Sphere. Is it still possible that gas or cloud deposits enclose a star ? Wouldn't that destroy rocks or anything close to it due to heat ? (sorry not an astronomer)

A : It's probably not a Dyson sphere. Anything that blocks the light should re-radiate it in the infra-red as it heats up, but no infra-red excess has been observed.

That said, none of the other proposed explanations (and there aren't many) do much better. It could just possibly be a swarm of giant comets, but there would need to be an awful lot of them and while they might explain the sudden dips, they wouldn't explain the slow dimming.

Since no other star shows behaviour quite like this, right now we just don't know. We need more data. Fortunately there are plans and funding in place to get it :
https://www.kickstarter.com/projects/608159144/the-most-mysterious-star-in-the-galaxy/description

Paul Carr has compiled by far the best summary of the situation :
https://disownedsky.blogspot.cz/2016/02/tabbys-star-for-perplexed.html


Shouldn't a Dyson sphere be heat resistant and therefore hard to detect ?
Q : Are the materials resistant to heat ? Wouldn't it be logical for engineers of the "dyson sphere" to construct it using materials or elements that does not harbour any heat to prevent damage or catastrophic failure ? And doesn't a Dyson Sphere supposed to absorb energy and heat not reject or radiate it?

A : Being resistant to heat (i.e. maintaining its shape and rigidity at a high temperature) is one thing, but absorbing but not radiating heat is quite another. That would be entirely new physics. No such material is known to exist - in any case, you'd probably want it to radiate heat otherwise its temperature would only increase.

Although Dyson spheres are supposed to absorb energy (in order for the inhabitants to make use of it), there's no known way they can absorb energy without re-radiating some of it. Just as, for example, solar panels absorb a lot of the Sun's energy and convert it to useful electricity, but they still get hot and radiate waste heat. Physics says this must happen. Actually, people have done deliberate searches looking for Dyson spheres based on the waste heat they should be re-radiating.


Could the dimming of Tabby's Star be caused by evaporating comets or sunspots ?
Q : If a star is surrounded by comets circling it, wouldn't there be possibilities of some of that ice melting and causing a reflection of the star...similar to the wavelengths of light reflecting off a swimming pool...uneven and off-focus light reflections ? I was thinking along the lines that the water is in the comet like a thick viscosity...light would dim if more 'viscos mass' was in direct path? Doing such could also make the 'comet/ice planet' a perfectly placed magnifying lens towards us and its dimming because the refraction is no longer towards us?
Another thought was sun-spots...but those would have to be huge, really huge?

I did all ?'s because a . makes it look like I know, but I'm just throwing ideas...I'm sure if we keep an eye on it we'll figure it out.

A : It won't melt exactly. Because the pressure is so low, it will sublimate directly from ice to water vapour.
http://astronomy.swin.edu.au/cosmos/s/sublimation

Comets could possibly explain the sudden dips in the emission, but not the slow, steady dimming that's been observed. However, I've been wondering if something like huge cometary tails blocking the light could be responsible, rather than the comets themselves. I'm not sure. The dips seem to occur very quickly, within a matter of hours, then lasting for a few days or weeks. My guess would be that if it was comet vapour causing the dimming, the time to reach the lowest brightness and the total time of the dip should be roughly the same, since comet tails are generally pretty diffuse things.

I don't think it can be sunspots. The rotation period of the star is known to be about 1 day, so if part of the star got significantly darker, we should see the brightness varying rapidly every day as the bright/dark parts moved in and out of view. In principle a really large band-like sunspot stretching around the entire star would avoid this, but this would be a super-weird discovery, much larger than anything ever seen on the Sun.

Then again since this discovery is unique (not to mention the long-term dimming), whatever explanation is correct is going to be interesting. In this case, extraordinary evidence may require extraordinary explanations. The difficulty is that it's easy to come up with a really weird explanation for a really weird event, but very much harder to determine if it's correct. Maybe it's a combination of factors. Maybe all the speculation so far is completely wrong. I doubt we'll know for sure until we get more data.


Could stellar winds prevent dark matter from entering star systems ?
Q : Could the "Solar winds" be preventing the DM from entering (for the most part) any solar system ? If DM has very little weight, the closer it gets to a radiating object the stronger it is pushed back and clumps up.

A : There could be some dark matter in the solar system. Not very much, but a little. If dark matter is the diffuse, collisionless substance we think it is, then it fills the entire galaxy more or less uniformly.
http://scienceblogs.com/startswithabang/2013/07/03/does-dark-matter-affect-the-motion-of-the-solar-system/

I think that in order for dark matter to be affected by the solar wind, it would have to interact with normal matter or photons. If it absorbed light, we'd be able to see regions where the stars (or whole galaxies) looked darker due to the dark matter blocking the view. Such regions do exist, however the evidence is extremely strong that these are caused by dust. Models predict that the dust should cause not just absorption of the light but also reddening, as observed. Dust also re-emits in the infra-red, which is also observed. As far as I know all absorption regions are consistent with being due to dust rather than dark matter (e.g. http://www.esa.int/Our_Activities/Space_Science/Herschel/Herschel_completes_largest_survey_of_cosmic_dust_in_local_Universe).

If dark matter interacted with the particles in the solar wind it would have a clumpy structure in the galaxy and this would then affect the motions of stars. It's hard to say how significant this would be, but my guess is we'd be able to see different motions of the stars in regions of different stellar densities. On the larger scales, the standard model of dark matter being totally collisionless seems to work pretty well. The most famous example is the Bullet Cluster, where two galaxy clusters have collided. The gas in the clusters gets stuck in the middle, while the galaxies and the dark matter keeps going - exactly as the model predicts.
http://chandra.harvard.edu/press/06_releases/press_082106.html


Why does a star's gravity get stronger when it dies ?
Q : Why the gravitational pull gets so strong when the star dies and its size and mass compress, why is it not that strong when the nuclear fusion is going on inside the hole and the star is of a huge size and mass ?

A : That's because of the density of the star is much lower before it collapses. If you were able to travel through the Sun (or the Earth, or any other spherical object), the strongest gravity you'd experience would be at the surface. It turns out that as you go deeper into an object, the only gravity you feel is from the mass below you (closer to the centre). At the very centre you feel no gravity at all because all the mass is above you.

Imagine being inside a hollow shell. Strength of gravity depends on mass and distance. It turns out that the gravity from the smaller mass of the part of shell closest to you is exactly balanced by the larger mass that's further away. The maths-heavy version is here :
http://www.sparknotes.com/physics/gravitation/potential/section3.rhtml

Since a sphere is the same as a series of shells, only the mass closer to the centre matters.

When a star stops fusing, its core collapses. Some of the star can be blasted away, but not much, so let's ignore that. Say you were in a spaceship orbiting just above the surface of the star, and you keep your ship at that exact same distance from the centre. As the star shrinks, the gravity you experience doesn't change - the mass below you is the same, and you're the same distance from the centre. So if the Sun were to shrink, we won't notice any difference - the planets would stay in their same orbits. But now you can fly your spaceship lower than you could before. The mass pulling you down is the same... but you're closer to it. So the gravity can be much, much stronger. Acceleration due to gravity is given by a = GM / r^2, where G is the gravitational constant, M is the mass below you, and r is the distance away. If you keep the mass constant but halve r, the acceleration increases by a factor of four. Make r 10% of the initial value and the acceleration becomes a hundred times stronger.


If we increase the density of gas around a white dwarf, could we help cool it down by convection ?
Q : Well, could we ?

A : [This and the next several questions were part of a longer discussion which you can read here; I've edited my answers here so they are more self-contained.]
Well, that was my idiotic suggestion to the question of how we could cool a white dwarf star into a black dwarf faster than usual. Normally this takes billions of years, if not very much longer (https://en.wikipedia.org/wiki/Black_dwarf), because the white dwarf has a huge reservoir of thermal energy but only radiates it very slowly (not, as some articles on the internet claim, because it's very small, but because its outer surface is rather cool - but we'll get back to that in another question). And the only way it can lose energy is through radiation. But, if we blew some cold gas over it, perhaps we could get convection to transport the heat away much faster ?

It wouldn't be a matter of simply dropping the gas on and letting it heat up. For starters the white dwarf wouldn't be able to heat the gas enough for it to escape its own immense gravity. So you'd have to send the gas on a trajectory at a tremendous velocity where it would scrape the surface, get heated by the dwarf and continue on its merry way. Except that won't work because, as a certain Isaac Kuo pointed put, the energy of the gas at this velocity (or indeed if you simply drop it onto the dwarf from a great height) will be far, far higher than the thermal energy it will receive from the dwarf. So you actually end up heating the dwarf instead of cooling it.

You also can't build anything on the surface of the dwarf, nor fly any spacecraft close to it. Acceleration is something like half a million g on the surface, and that's... well, unpleasant. Which means there's no way of delivering any coolant to the surface more serenely.


Could we cool down a white dwarf by increasing its internal convection ?
Q : Since the outer layers of a white dwarf insulate its much hotter inner core and keep it warm, would increasing the convection and mixing the two layers (or removing the outer one) help it cool down faster ?

A : In principle, yes. White dwarf cooling is a very complicated process (http://www.physics.upatras.gr/UploadedFiles/course_149_4311.pdf) but the main reason they take a very long time to cool is their structure. Most of their mass is incredibly hot, probably millions to billions of Kelvin, consisting of highly conductive "electron degenerate" matter that's been squished so hard it's only the pressure of trying to force electrons together that prevents it from collapsing even further. But this incredibly hot core is surrounded by a thin layer of highly insulating non-degenerate material. While the core may be at 100,000,000 K, the outer layers can be just 10,000 K. It's the outer layers that radiate heat away, and since they're not very hot the dwarf doesn't lose energy very quickly.

A really simplified calculation shows what a difference this makes (but you shouldn't take the actual values too seriously). Knowing the total thermal energy content of the dwarf based on its mass and core temperature (http://www.ucolick.org/~mountain/AAA/aaawiki/doku.php?id=how_long_does_it_take_for_a_white_dwarf_to_cool) and how quickly it radiates energy based on its surface temperature (http://cronodon.com/SpaceTech/WhiteDwarf.html) it's easy to estimate how long it should stay hot for. Assuming a white dwarf of 1 solar mass with an internal temperature of 100,000,000 K and a surface temperature of 10,000 K, it will take about 1 billion years to lose its entire store of thermal energy.

If we could somehow get that hotter material up to the surface (or expose the core), things would be MUCH faster. Luminoisty is very strongly dependent on temperature : double the temperature and the rate at which energy is emitted goes up by a factor of 16. Increase it from 10,000 K to 100,000,000 K and the rate goes up by a factor of a quadrillion. So in principle our white dwarf could cool down in just a few hours.

Both of these are overly-simplified calculations. In reality the surface temperature will never be constant because it's always losing thermal energy : the more it loses, the colder it gets and the slower it radiates. Hence real values of white dwarf cooling range from a sedate 10 quadrillion years (70,000 times the current age of the Universe) up to a truly insane 10 37 years. Watching paint dry would seem like the height of hedonistic debauchery compared to this.

Still, in principle it seems we could cool down a white dwarf on more human timescales if we could only get that hot inner material up to the surface. You're probably sensing a "but" here. And you're right. Continue reading...


Okay, why can't we we cool down a white dwarf by increasing its internal convection ?
Q : You said in the last question that there was a but. Explain yourself.

A : Because unfortunately increasing internal convection will involve adding more energy into the system than it will radiate. There are two basic ways we could do this. Either we try and scrape off the outer layers in some way, or we somehow move material from the inner core up to the surface.

The total thermal energy stored in the white dwarf is huge. Each kg contains about 3.5 billion Joules - about the same as a cruise missile or what it would take to break every atomic bond in the human body (http://www.projectrho.com/public_html/rocket/usefultables.php#id--The_Boom_Table). And yet that's nothing : it's got about ten thousand times as much gravitational potential energy (the energy it could potentially release if it was falling, or, conversely, the energy it would take to raise it from the centre to its actual height). Getting each kilo from the centre of the core to the surface is going to take the energy of 3 kt TNT - a sizeable nuclear explosion - just to overcome gravity alone. Even at the surface, raising a kilo of matter by just 1 km will take about as much energy as it's got in thermal energy. And you'll need to raise it by at least 10 km to get out of that insulating outer layer - it's not a matter of just moving the material sideways, any more than you can dig a hole without piling up material somewhere.

This process won't be efficient. As well as overcoming gravity you also have to move the material through the surrounding, incredibly dense matter. You can imagine how much more energy it takes to dig a shaft from excavating material than just from moving the material upwards : well, white dwarf matter is about half a million times denser than ordinary iron. So in actual fact you'll have to inject a staggeringly large amount of energy to do this - vastly more, per kg, than the white dwarf stores as heat. You'll unavoidably make the white dwarf much hotter than when you started, not cooler. To make things worse, it doesn't matter how you do this - it's a fundamental limit of the white dwarf itself.


Okay fine, but what if we shoot a black hole through a white dwarf to increase convection and make it cool down faster ?
Q : I mean really there must be a way to do this.

A : It's a clever idea, because black holes swallow material and drag it around through their intense gravity. And the black hole can be shot straight through the white dwarf and keep going out the other side, so in that sense it's not a normal collision. Does this avoid all the problems of mixing the hot core with the surface material we've discussed ?

Not really. In fact it will make things spectacularly, cataclysmically worse.

First a short digression about stellar evolution. Stars about as massive as our Sun end their lives by (relatively) gently shedding their outer layers and leaving behind a slowly-cooling denser core : a white dwarf, typically 5,000 km across. This has quite a strict upper mass limit of about 1.44 solar masses. Add any more mass than this and the electron degeneracy pressure can no longer support it against its own massive gravity, and it collapses into an even smaller, denser neutron star (around 20 km across). So more massive stars tend to form neutron stars rather than white dwarfs. Only the most massive stars of all have (in their final explosive deaths) densities high enough that even neutron stars can't from and a runway gravitational collapse leaves behind a black hole. Typical stellar-mass black holes are at least 5x the mass of the Sun.

The black hole's large mass means that the white dwarf experiences significant effects even at large distances. Even at around 4 million km the white dwarf will feel 1 g towards the hole, meaning that every second its speed towards the hole increases by 10 m/s (ignoring the mass of the dwarf itself). Now, in one sense this is good. In order to any chance of mixing cold and hot material and not just swallowing the white dwarf whole, the black hole must be moving at at least escape velocity - 6000km/s. And since the white dwarf is only around 5000 km across, this means the collision could last for just one second - but oh, it will be one hell of a second. Or actually much less than 1 second, because by the time the black hole and white dwarf "collide" they will be moving towards one another at very nearly the speed of light.

But long before that happens the white dwarf will be experiencing higher and higher accelerations from the black hole. By the time the two are 11,000 km apart, the acceleration from the black hole is equal to the white dwarf's surface gravity. And that's about half a million g. It's now going to be at the level where it may be torn apart. Or, as this immense gravity begins to compress its material, it might collapse (or start to collapse) into a neutron star.

If it avoids this fate, it might just survive but good lord it's going to suffer the equivalent of being slashed with a lightsabre nearly as powerful as a supernova. Probably the most meaningful definition of size for a black hole is its event horizon : get closer than this and there's no escape. For a 5 solar mass black hole that's about 15 km, much smaller than the white dwarf. So this tiny hole screams through the white dwarf, sucking in material towards it at almost the speed of light, all the while itself travelling at near lightspeed. Based on the size of the event horizon, the black hole will intersect and devour at least 3x1025 kg of degenrate matter - about five times the mass of the Earth.

Now you may be thinking that this is not very significant compared to the mass of the white dwarf or the black hole, and you'd be right. The problem is that material falling onto the black hole at nearly lightspeed will radiate some fraction of this enormous kinetic energy away as heat (https://www.ucl.ac.uk/mssl/research-degrees/Lectures_and_Seminars/LecturesArchive1516/MatPage). Accretion is actually the second most efficient form of power in the Universe - only a matter-antimatter reaction is more potent. For a black hole, around 1% of the rest mass energy (the famous E= mc<2) will be liberated as heat, as a conservative estimate. In our scenario, this is an massive amount. Not nearly enough to blast the star apart - it's about a few thousand times too low for that - but enough to give it about ten times more thermal energy than it started with ! So you have this relativistic black hole searing through the white dwarf releasing almost a small supernova's worth of energy which is all the while confined by this immense gravitational pressure... yeah, the dwarf is having a bad day at the very least.

If the tidal force from the black hole does trigger a collapse into a neutron star, then our dwarf is absolutely doomed. A neutron star is only about 20 km across, barely larger than the black hole's event horizon. What you'll have is a white dwarf hurtling towards the event horizon, collapsing into neutronium and being torn to shreds before disappearing forever into oblivion - but not before it releases a full supernova's worth of energy in the process. It is, in short, about as far up the proverbial creek as it's possible to be.

And using a smaller black hole won't help. Remember, dragging material around inside the white dwarf inevitably means you have to inject more energy in than you want it to radiate away. This idea cannot work.

Bugger.


What if we put a black hole in orbit of a white dwarf in order to cancel out the surface gravity ?
Q : That way we could build stuff on it without getting squished, right ?

A : You're going to have to cancel out pretty much all of the half-million g of the surface gravity. And if you do that the dwarf is going to experience severe tidal effects as the gravity will vary drastically from one part to another. I'm pretty sure this would mean it was within the Roche limit of the black hole, the point beyond which everything held together by gravity gets torn apart. Never mind that without the high pressure from its own gravity, the entire dwarf is likely to just plain explode.
http://www.popsci.com/technology/article/2009-10/what-would-happen-if-i-ate-teaspoonful-white-dwarf-star


Ok fine, how about using LOTS of orbiting black holes to cancel out the white dwarf's surface gravity ?
Q : That way we could build stuff on it without getting squished, right ? And we could stop the decaying orbits by giving the black holes kinetic energy with lasers and particle beams !

A : I'm not 100% certain, but I suspect that any attempt to cancel out the white dwarf's surface gravity is doomed. The thing has immense outward pressure counterbalanced by half a million g. You'd have to balance that with immense precision in order to cancel out the gravity to a point where you could think about building anything on it. Simply adding another black hole on the opposite side won't help because then you have the white dwarf being pulled in two directions. A ring of black holes will leave the poles exposed. A swarm of black holes might just do it, but good luck getting that to be stable.

I am not sure how lasers or particle beams would give kinetic energy to a black hole, though I suppose if you fired particle beams of very dense matter (i.e. more black holes) in the right trajectories you could help speed up the black hole as their mutual gravity accelerates them.

But in any case, cancelling out the white dwarf's gravity even on a local scale is probably not a good idea. I imagine you'll end with with something like this, although not quite as powerful :
https://www.youtube.com/watch?v=o_EBqZPCZdw

So now you have this swarm of black holes, very carefully managed by beams of smaller black holes (and if they're too small their Hawking radiation will NOT be negligible : http://xaonon.dyndns.org/hawking/), orbiting a white dwarf that is likely about to explode.

Leave the dwarfs alone !


Okaaay.... how about this one, we fill the white dwarf with teeny-tiny wormholes.
Q : Like, really really little ones that are so small only photons can escape. That would extract the thermal energy of the white dwarf without changing its mass.

A : I suppose this would work if you accept that wormholes are a possibility (which is by no means certain), and if you could remove only photons that would be a very effective way to cool it down. You're going to have problems keeping them open in the dwarf's massive gravitational field though.

Another trick you could try with a wormhole is to build a time machine. Keep one aperture fixed and accelerate the other on a long trip at highly relativistic speeds. The moving aperture travels into the far future to an era when white dwarves have cooled naturally. Then you just send one of the black dwarfs from the future back through the aperture to the present and hope like hell you don't cause any paradoxes.


What if we built a giant ball of iron and dropped hydrogen on it ?
Q : I need something similar to a black dwarf and apparently cooling down a white dwarf just isn't going to work. So could we mimic one by building a giant ball of iron (which won't undergo fusion) of around 1 solar mass, and start a thermonuclear reaction by dropping hydrogen on it ?

A : I suppose it depends on how the iron (or whatever) is assembled. If you just pile up iron until you have a solar mass of it, you'll have a ball with a radius of 320,000 km and a surface gravity of 75g. Well, not necessarily. As the mass gradually builds up the pressure will increase and so, perhaps, will the density. I am not too sure about the materials physics to comment on exactly what would happen, but my guess would be that you'll end up with something much more interesting than a simple ball of solid, cold iron - but probably not electron-degenerate matter.

This giant iron sphere wouldn't be suitable for fusion unless its density was much higher. But before that it's worth a brief digression, because maybe we can get you something even better. See, if you have a massive, compact object you can use it to release energy by accretion, which can be far more efficient than fusion.

If you drop something from high enough above the Earth, the fastest it can possibly impact the ground is 11 km/s (ignoring the atmosphere and any other motion the object had). Which is pretty darn fast - meteorites contain a lot of energy, which they release on impact. But if you have a white dwarf, you have an object about the same size as the Earth but 330,000 times denser. The impact speed will now be 6,000 km/s and the object will contain almost 300,000 times as much energy. For a neutron star, which is much smaller and denser again, the impact speed would be a significant fraction of the speed of light and the energy released can be hundreds of millions of times greater than on a white dwarf. You can also get accretion energy around a black hole, as even though they don't have solid surfaces any material falling towards them can end up orbiting in accretion discs. https://www.ucl.ac.uk/mssl/research-degrees/Lectures_and_Seminars/LecturesArchive1516/MatPage

If you are specifically interested in fusion, however, then yes you can get this to happen on white dwarfs (or similar objects) in the right circumstances. It's not so much the impact velocity heating the material and starting fusion : it's more that the gravity prevents the hydrogen from expanding despite its ferociously high temperature. Deposit enough hydrogen and at 20 million K, runaway fusion causes a huge explosion : a nova.
https://en.wikipedia.org/wiki/Nova


My sci-fi scenario needs an object that's hard to detect at 1 LY unless something impacts it. Since black dwarfs are apparently impossible to produce even for really advanced aliens, have you got anything else ?
Q : Pretty please ?

A : Since a black dwarf is just a cold white dwarf, it's perectly possibly that accreted material could cause a thermonuclear explosion just like a nova (https://en.wikipedia.org/wiki/Nova). But white dwarfs take longer than the age of the Universe to cool down, so black dwarfs don't yet exist. However a non-rotating neutron star might fit the bill. It will certainly release a lot of energy if something impacts it (see https://www.ucl.ac.uk/mssl/research-degrees/Lectures_and_Seminars/LecturesArchive1516/MatPage). Unlike a white dwarf, which has an insulating outer layer, neutron stars can reach quite low temperatures in just a few gigayears (http://astronomy.stackexchange.com/questions/14387/what-happens-over-time-as-a-neutron-star-cools). Since they're also much smaller even than white dwarfs (20 km across), their thermal emission would only be a billionth of that of the Sun. So unless they're rotating (in which case you get a pulsar) they should be very hard to detect and would fit the bill pretty well.


How many stars are in the sky ? [Daniel Phillips, age 7]
Q : Well, how many ?

A : No-one knows exactly. On a dark night, far away from any street lights, you can probably see around 2,500 stars. That doesn't sound very many, does it ? If you could start counting one number every second, you could count to 2,500 in less than an hour.

But those are only the individual stars you can see. Sometimes, on a clear night, you can also see a very faint white band across the sky. If you look at that with binoculars, you can see that it's made of stars. LOTS of stars. I mean it. LOTS !!!

We can only see the very nearest stars as points of light. Imagine looking at a crowd of people. If they're close, it's easy to see and count each one. But if they're far away, they all blur together. That's what happens with stars. Using powerful telescopes, we can get a "closer look" at those distant stars and try and count them. We think there are around 100 BILLION stars in this "Milky Way". This is a bit of a guess, though, because no-one's bored enough to sit there counting each and every one. It'd take too long. If you counted one a second, it'd take you about 30 YEARS to finish counting them all.

But that's just the start. The Milky Way, we think, is a huge disc of stars called a galaxy, and our Solar System is somewhere inside it. Using telescopes, we can see other, very distant galaxies... and there are about 100 billion of them. So 100 billion galaxies each with 100 billion stars... that's an ENORMOUS number of ten sextillion stars in total ! It looks like this :
10,000,000,000,000,000,000,000.
If you started counting at one per second, it'd take you 300 trillion years to finish.

That's a lot of stars.


What are the stars ? [Daniel Phillips, age 7]
Q : Well, what are they ?

A : The stars are other Suns ! They look like tiny points of light because they are VERY far away. The nearest one is about 25 trillion (25,000,000,000,000) miles away. Our fastest rocket would take about 20,000 years to reach it.

Stars are not all exactly like the Sun. If you look carefully, you can see that some of them are redder and others are bluer, and some are fainter and some are brighter. Some stars are much smaller than the Sun, but others are much, much larger. Small stars tend to be very cool and red, and live a long time - much longer than the Sun. Bigger stars can be very hot and blue, or cool and red, but they usually don't live as long as the Sun and explode when they run out of fuel. Most stars - we think - also have planets going around them just like our own Solar System.


If the Universe is expanding then how come the constellations don't change ?
Q : WHY: "if" the universe is expanding (moving) are the stars in our hemisphere in the same location as mapped thousands of years ago ? (please explain)?

A : The expansion rate depends on distance : the further away something is, the faster it's moving away from us. Within our own Galaxy, the rate is so small that it's dominated by gravity - the minuscule "push" from the expansion is much smaller than the "pull" being felt by the stars towards other stars. Actually we can only even detect the expansion by looking at other galaxies, which are much further away.

However, the stars do move within the Galaxy quite a lot faster - we can even see this from old photographs. Have a look here :
https://plus.google.com/u/0/+RhysTaylorRhysy/posts/aXBJPbtzCai
Look closely at the bright star in the top right. The motion isn't due to the expansion of the Universe, but simply because the stars are moving through the Galaxy anyway.

But whether any of the bright stars of the major constellations are moving enough that the changes would be visible over a few thousand years is another matter. Early star maps were nowhere near as accurate as modern ones, so there would have to be a very big change so that we could be sure the change was real and not just due to the ancient astronomer making an error. Still, we can now measure the changes very accurately and predict what the constellations would have looked like in the past as well as the future (https://www.wired.com/2015/03/gifs-show-constellations-transforming-150000-years/). I imagine someone must have done a study of whether ancient records agree with this, but I can't find any references right now.


How are these two stars moving on a W-shaped path ?
Q : I know you are not concerned with trivial astronomical objects that are closer that a couple of megaparsecs, but if you ever have the time or inclination can you explain the track of Luhman 16AB to me?
Specifically why is the 3 year track shaped like a curved letter "W" instead of an open loop like I would expect? I do not understand why there is a sharp point in the middle of the track instead of loop. I mean, they are orbiting around their barycenter, right?

A : My first thought was that these two objects are orbiting a third unseen companion at a similar distance and phase in their orbit. Since the system as a whole is moving across the sky, that would easily explain the w-pinch shape (like a very simple spirograph pattern).
https://plus.google.com/u/0/photos/photo/103860147871348309151/6429911123851104530?icm=true

.. however, they say explicitly that this is not the case. With elliptical orbits around a common centre, the pinch-shape can be reproduced quite easily. However, since the objects must be on opposite sides of the centre of mass, it's no obvious to me how they could appear to be at a similar phase of the orbit while reproducing the pinch.
https://plus.google.com/u/0/photos/photo/103860147871348309151/6429913778705368882?icm=true

On some more google searching, I think the answer might be parallax. Just as the outer planets display retrograde motion at opposition , so the same could be happening here :
http://action.larouchepac.com/solar_motion-1


Could ancient massive stars have created supermassive black holes ?
Q : I watched this video and it said, the first stars were 50 times bigger than the Sun, and and their deaths, and the distribution of metals, gave rise to the first galaxies and the second generation of stars. Makes sense to me. So here is a crazy thought. maybe an astrophysicist can run with the idea. When stars that large supernova, sometimes they create black holes. Would it be possible that the SMBH in the centers of galaxies are the remnants of the First Stars going SuperNova and giving birth to galaxies and second generation stars?

A : Yes, it's possible. It's actually quite an old idea. Here's a nice write-up about one of the recent explorations :
https://astrobites.org/2014/03/21/a-new-way-to-die-what-happens-to-supermassive-stars/

The problem is that directly observing these population III stars is damn hard. It's thought that originally the chemical composition of the Universe was little more than hydrogen and helium, with only a trace amount of heavier elements. That would change how stars form (this may be mentioned in the documentary but I don't have time to watch it).

Gas in the modern Universe has significant amounts of heavier elements like carbon, nitrogen, oxygen, etc. As the gas collapses under gravity, it heats up due to compression. This causes a pressure which partially balances out the gravity driving the collapse. But those heavy elements radiate heat much more effectively than hydrogen or helium, so the collapse of the gas isn't resisted very much.

In the early Universe, without these heavy elements there wouldn't be much to stop the gas heating up, making it much more resistant to collapse. That means early stars would have been far more massive. But unfortunately that means they would also have been very short lived. Since they're also extremely far away, that makes them extremely challenging to observe.
http://astronomy.swin.edu.au/cosmos/P/Population+III


This article about a star that keeps exploding rules out dark matter, right ?
Q : I mean this one : https://www.9news.com.au/technology/2017/11/09/13/17/star-that-wont-die-baffles-astronomers

A : It's got absolutely nothing to do with dark matter.
[I got no other response so I have no idea why anyone would see a connection to dark matter here. The article is nice, there's nothing wrong with it, but it's about a completely different topic !]


You're lying about the size of Betelgeuse.
Q : [Regarding my video The Light of Other Suns]
This is a lie. Betelguese will head up to uranus and neptune, pluto only

A : Pretty sure Betelgeuse isn't that large. It does depend quite strongly on how you measure size (stars this large don't have a distinct surface - unfortunately I had to compromise the video's accuracy because it's extremely difficult and time consuming to render diffuse objects), but I couldn't find anything claiming it Betelgeuse would extend much further than Jupiter.
https://www.eso.org/public/unitedkingdom/images/potw1726b/
https://en.wikipedia.org/wiki/Betelgeuse#Diameter
https://www.space.com/35084-betelgeuse-red-giant-star-cannibal.html
http://www.gb.nrao.edu/~rmaddale/Education/OrionTourCenter/betelgeuse.html


"DrRhysy uy scuti is the size of the solar system"

I mean what can you say to that ? As a response it makes no sense. Why would anyone say that ? Sigh.


What would we see if we travelled further and further away from a star ?
Q : Let's consider a hypothetical universe
Where there is only one star
Compulsive
And a spacecraft which can detect the intensity of light emitted

What will be the observation by the spacecraft regarding the intensity of radiation when it will have moved very very very far away ??
I mean will it continuously decrease, increase, remains constant or sth else ?
Answer with proper explanation :)

A : Interesting and complicated question. :)

So, there are three components : the star, the ship, and space. I'm gonna assume that space is empty, flat, Euclidian, and not expanding.

On a basic level, the size of the star as seen to someone on board will continuously decrease. Eventually it will become a point source, too small to resolve any structures. The crew of the ship won't see much except for how the brightness of this point source changes over time.

I assume that if the ship is going very very very far away then it will take a very very very long time to get there. This means the mass of the star (also its chemical composition and rotation but let's keep things simple) is going to be very important here, because that determines the fate of the star.
- Very low mass stars (red dwarves) will essentially just fade, very very gradually, eventually going dark over trillions of years. And that's it.
- Stars about as massive as the Sun last a few billion years, and towards the end they increase in brightness before shedding their outer layers as a planetary nebula, leaving behind a white dwarf that cools even more slowly than a red dwarf (https://en.wikipedia.org/wiki/Black_dwarf).
- Stars much more massive than the Sun explode as supernova, with the remnant becoming either a neutron star (likely a pulsar for a good long while) or a black hole.
http://chandra.si.edu/resources/flash/stellar_evolution.html

And throughout its life, especially if it's rapidly rotating, the star will be ejecting material into space. So even if space starts as empty, it will gradually start to become just a little less empty. That material, spread out over a large area, can reflect the star's light but will also itself glow at other wavelengths, as well as obscuring the star. This will be especially important for very massive stars.
https://en.wikipedia.org/wiki/Wolf%E2%80%93Rayet_star

So the star itself is going to vary in brightness over time. Some stages of stellar evolution produce variable stars, where the brightness can change substantially on timescales of days or years :
http://www.atnf.csiro.au/outreach/education/senior/astrophysics/variable_types.html
Unless the star is a red dwarf, it's going to continuously brighten over the course of its lifetime, very briefly increasing dramatically at the end when it explodes only to rapidly decrease and then fade slowly over the rest of eternity. That's what the ship would see if it kept a fixed distance from the star : a general increase in brightness, possibly followed by a variable phase, a huge increase and then a steady decline forever after.

But our ship is continuously moving. It's going to need some sensors to detect the star and its ever-expanding gassy aura. It would be a good idea to take as many as possible to cover the full wavelength range, since the star will be evolving over time. And as well as the distance, the speed of the ship will be important. The faster it goes, the more redshifted the light from the star will appear. There's no limit on this, so if the ship is travelling very close to the speed of light, it will barely be able to see the star at optical wavelengths at all - it'll need a radio telescope.
https://imagine.gsfc.nasa.gov/features/yba/M31_velocity/spectrum/doppler_more.html
(It might also be able to detect neutrinos and high-energy particles from the star, which require very different sorts of detectors)

If the ship is accelerating, then the redshift will change, so it will have to gradually alter which sensors it uses to detect the star. If it's powered acceleration, then the peak radiation will keep shifting to longer and longer wavelengths. If the powered acceleration stops, then no matter how far away it is, the star's gravity will cause the ship to decelerate - so the peak wavelength will get shorter again. I'll assume that the ship is moving so fast that it will never start moving back towards the star though.

Even ignoring the movement of the ship, the peak wavelength of the star will change. Initially it will be brightest at optical wavelengths, whereas with pulsars the peak might be brighter at X-ray or radio wavelengths. Even black holes could be, in principle, detectable by Hawking radiation. Give them long enough - and we're talking truly insane amounts of time here - and massive black holes evaporate, ending in a final, dramatic burst of gamma rays. And then nothing. The ship will detect the final burst of Hawking radiation, and then that's it.

... well, not quite. There will still be the faint shine from the residual material ejected by the star earlier in its life. Eventually, even this - it's thought, no-one is certain though - eventually decay into pure radiation, by which time so will the ship, so there will be no-one left to observe anything.


GALAXIES

Are "rogue stars" part of dark matter galaxies ?
Q : Every now and then "rogue" wandering stars are discovered, and I am curious if they might, in fact, be luminous matter that are part of a predominantly dark matter galaxy.

A : Galaxies which are mostly dark matter have been proposed as a solution to the "missing satellites" or "dwarf galaxy" problem. Simulations of the evolution of the Universe have predicted about ten times as many dwarf galaxies as are actually observed.

Until very recently, these simulations have only included the dark matter since the calculations of the gas and stars are much more complicated. Theorists try and work out what the simulations really predict by using simple scaling relations from observations : e.g. a dark matter halo of this mass will normally contain this much gas and this many stars.

It has now become possible to include the gas and stars in the simulations directly. One preliminary, non-peer reviewed suggests that dark galaxies are still important :
http://adsabs.harvard.edu/abs/2014arXiv1412.2748S

Dark galaxies are viewed as intensely controversial. There is not yet a consensus as to whether a dark matter halo could accumulate a detectable amount of gas without forming any stars. My own research involves looking for dark galaxies by searching for hydrogen clouds without stellar counterparts. There are a few very interesting candidates, but none so far that are entirely convincing.

However, could these rogue stars be parts of largely dark galaxies ? Tricky. Star formation theory isn't all that well understood. The first massive stars which formed might eject the remaining gas through stars and supernovae explosions, however, it's difficult to believe that just one (regular-sized) star would survive. Additionally, rogue stars can be explained by galaxy collisions or possibly by one star in a binary system exploding :
http://www.universetoday.com/118881/lopsided-supernova-could-be-responsible-for-rogue-hypervelocity-stars/

In short, the answer is "probably not".

EDIT : Update ! Over the last year, so-called "ultra diffuse galaxies" have become an incredibly hot topic in extragalactic astronomy. Currently we know that there are large numbers of very faint galaxies and we think they're extremely dark-matter dominated. Recently some researchers have claimed that three isolated stars could indeed be part of a dark galaxy. I haven't read the paper but I'm rather skeptical based on the press release. You can't get kinematics from just three stars, so without gas measurements there's no way to know if the stars are in a dark matter halo or not (except maybe through gravitational lensing). I stand by the original answer of "probably not", but I thought this new information should be added.


If the Universe is expanding, how can galaxies evolve through merging ?
Q : Do you have an explanation for the counter-intuitive theory of galaxy evolution through merging when dark energy is (possibly) separating galaxies apart ?
How about something a bit science-fictiony like an invisible thread that from the central black hole keeps each star in the galaxy together in whatever shape it is and then travels through the Universe as a whole based on its needs to keep everything in balance ?

A : Dark energy is supposedly causing the acceleration of the expansion, but was much weaker in the past. So mergers were far more likely in the past. Moreover, the expansion is only really important on very large scales - get galaxies close enough and their gravity easily overcomes the expansion.

... at this point, the discussion should have ended, but it didn't. I include the following an an example of someone behaving as though their ignorance trumps genuine science.

Questioner : Rhys, how convenient: apply a different set of rules each time: very small, quantum; regular, gravity; very large, dark energy. it looks like we'll never find a unifying theory this way.

Me : Ummm, no, that's not what I said. I said one force dominates over another at different distances. The expansion of the Universe still happens on very small scales, it's just not enough to overcome gravity. An analogy would be that you're currently feeling a larger attraction to the Earth than the Moon - it doesn't mean the Moon's gravity is having no effect at all, only that the Earth's is much stronger. It's simply the case that one number is greater than another, that's all. This is not at all the same as the conflict between quantum mechanics and general relativity, which are fundamentally different.

The discussion continued unproductively. As far as I'm concerned my answer fully satisfies the question.


Can radio signals travel faster than light ?
Q : If a radio signal from a nearby galaxy can be gauged at 4x's the speed of light, does that potentially mean that either there is a more advanced life force out there, or does it mean that a planet or star went boom and its remnants could be headed our direction ? Would it not be a wise idea to plan for a "shower" of some sort ?
http://www.dailygalaxy.com/my_weblog/2012/02/the-unidentified-object-of-galaxy-m82-todays-most-popular.html

A : This is called "apparent superluminal motion". If you were to watch, say, a blob of gas in a fast-moving jet move across the sky, and assumed it was only moving across the sky, the calculation could show that it's moving faster than light. In reality, the jet may be pointed toward us. If so, the blob gets closer to us, so the light now has less distance to travel to reach us than it did originally. If the jet is pointing almost directly towards us and moving very fast (close to the speed of light) it can appear that it's moving faster than light.

Basically, the signal arrives unexpectedly early, but only because it was emitted from somewhere that's now significantly closer to us.
http://math.ucr.edu/home/baez/physics/Relativity/SpeedOfLight/Superluminal/superluminal.html

I don't know how exactly the inclination angle of the jet is determined, but this effect was predicted before it was discovered (1966 by Martin Rees).  It isn't really faster-than-light travel, it's a projection effect. Here's a better explanation than mine :
http://briankoberlein.com/2013/10/28/superluminal/


Do we know exactly when we'll collide with Andromeda ? Are we measuring things correctly ?
Q : If Andromeda is the closest galaxy to the Milky Way galaxy, how do they measure the distance, from center to center or from the outer edge to the outer edge ? The reason I ask is because if it is from center to center, the two galaxies would have collided long before the two centers even kiss each other... Is that a fair assessment ?

A : In terms of estimating when the collision occurs, it makes little difference. Andromeda is about 2 million light years away, while Andromeda and the Milky Way are about 100,000 light years across. The errors in determining the distance are about the same as the size of the galaxies involved. So at the moment, it really doesn't matter if the distances are center-to-center or edge-to-edge, because the uncertainties are too large.


Will the gravitational pull of Andromeda cause it to collide with us as faster-than-light speeds, or at least get faster when it gets near us ?
Q :  Even though Andromeda is 2 million miles away, currently; as far as I know; galaxies don't have a speed limit... unlike humans. So I was thinking that that 2 million light year gap could lessen depending on the energy and gravitational pull or conversely repulsion of the two galaxies. Does that make sense?

A : Well, maybe. :)

We've been able to measure the speed of Andromeda both across the sky and towards or away from us. That means we know its true 3D motion through space. Based on that, it appears to be heading straight for us, and will collide in a few billion years.

The collision will be long, drawn-out, and above all messy. Tidal forces will stretch the galaxies as they get close together - the edges of the galaxies will be more strongly attracted than their centres, due to the difference in the gravitational forces. We can model how that will happen to get some idea of what it will look like :
https://plus.google.com/106944536119733342041/posts/RM95A7XdB7g

The only "speed limit" galaxies have is the speed of light, but neither the Milky Way nor Andromeda will ever move anywhere near that quickly relative to one another. The speed of light is an absolute limit because accelerating something to that speed requires infinite energy.


Can the debris of an exploding star damage our galaxy and/or travel faster than light ?
Q : I was thinking that external forces, like the eventual implosion of Betelgeuse would cause waves of movements of other galaxies. Meaning that, the way I am interpreting things, once Betelgeuse explodes (even if it already did), it would cause inertial waves of other galaxies (due to its immense size, is how I was thinking), speed would then increase over the speed of light.

A : Nothing can accelerate faster than the speed of light because that requires infinite energy.

Betelgeuse, or any other star which explodes, is just one of ~100 billion in our galaxy. It's only a few times the mass of our Sun - its "immense size" is, to be blunt, completely pathetic compared to the Galaxy. Supernovae are powerful, but they're not that powerful... the explosion will re-distribute most of the star's mass throughout the galaxy, but it won't actually change the overall mass of the galaxy at all.

I am not sure what you mean by "inertial waves of other galaxies". The mass of the galaxies is not affected by supernovae (OK, maybe a very little, negligible loss due to gas that's ejected by the explosions); if the explosion is perfectly symmetric then a supernovae doesn't even emit gravitational waves. The gas within the Milky Way will absorb the blast wave from any supernova - it won't propagate beyond our galaxy.

The interstellar medium - the space between the stars - is not empty. It's extremely thin - actually it's closer to empty than the most perfect vacuum ever created on Earth, but it's not empty. If we could see the hydrogen gas that fills the Galaxy, the night sky would look far more spectacular than what we actually see (shameless plug
http://astrorhysy.blogspot.cz/2015/03/under-hydrogen-sky.html)

When a star explodes, its matter is flung outwards, sweeping up all this interstellar gas, and so the blast wave gets heavier. Conservation of momentum means that it slows down and eventually stops. I don't remember the exact number off the top of my head, but the total mass of gas in our galaxy is similar to the total mass of stars. So, the matter from one exploding star is very, very much smaller than the mass of gas it would have to sweep out of the way to leave the galaxy.

The momentum of the explosion also cannot change the motion of our galaxy. If it's spherical, then the total momentum is zero, because everything is moving in opposite directions. If, somehow, the explosion was not symmetrical, and the whole mass of the star just decided to rush off in one direction, then the Galaxy would have a small momentum change. But remember the mass of one star is still far less than a billionth the mass of the whole galaxy - a supernova just isn't enough to really affect the galaxy.


What happens to a galaxy after its central supermassive black hole evaporates ?
Q : If black hole shrinks and vanishes then what occupies the space of black hole and how when there is no strong gravitional force ? And galaxy having one black hole in center continues its way or is dispersed ? If nothing occupies the space after black hole evaporates and its gravity becomes weaker then what provides centripetal force to galaxy ?

A : Even supermassive black holes are much less massive than their host galaxies, so nothing much will happen to the galaxy when the black hole evaporates. Mind you, their evaporation time is so long that all the stars would have died long before this.

Supermassive black holes have masses typically around a few million solar masses, whereas galaxies have more like a hundred billion solar masses in stars alone (plus a similar amount in gas and maybe around ten times more in dark matter). This is the source of the gravity holding them together, not the black hole. All together, the supermassive black hole makes up less than 1% of the mass of its host galaxy.

The spin (centripetal force) of a galaxy arises from very early in its formation as clouds of gas move past each other. This won't be affected by the loss of a black hole, which is just not large enough to have much effect on the galaxy (except for stars which are very close to it).


Could galaxies be rotating too quickly due to electical forces rather than dark matter ?
Q : Why can't an electrical or static charge be involved for dark matter they way they do on the space station with a statically charged knitting needle ?

A : Several reasons. Since the Universe seems to be electrically neutral overall, it would be hard to get any object to have a strong charge : it would tend to attract opposite charges and become neutral again.

Most of the gas in galaxies seems to be neutral atomic hydrogen, so it's not obvious how an electrical field could give it a strong motion, or where that field would be generated so far from the center of the galaxies.

Gravitational lensing measurements of clusters of galaxies indicate there is missing mass, which agrees with the strong motions of the galaxies. While this doesn't necessarily rule out modified gravity, I think it does rule out electrical forces. AFAIK the electrical force cannot deflect light.

The Bullet Cluster (and others like it) provide particularly strong evidence. Here the gas (hot, ionised gas, not neutral this time) and stars have become separated. Gravitational lensing measurements indicate that the dark matter has followed the stars and become separated from the gas. No lensing effects are seen from the gas. So, even if an electrical effect could deflect light (which AFAIK is not possible), it would be strange that it is only associated with the stars.


Will the Milky Way collide with the Andromeda galaxy in the future ?
Q : Is there real evidence that Milkyway and Andromeda will collide in the future ?

A : Measuring its velocity along our line of sight is easy. Measuring its velocity across the sky is much more difficult, but the results say it's basically negligible. It's pretty much headed straight for us. https://en.wikipedia.org/wiki/Andromeda%E2%80%93Milky_Way_collision#Certainty
http://www.universetoday.com/95557/its-inevitable-milky-way-andromeda-galaxy-heading-for-collision/


How can there be hot gas in deep space if space is so cold ?
Q : Gas cooling in space ? I was under the impression that space was a very cold place until one was close enough to a star then in the stars radiant field things could be heated ? How in the vastness of space can gas be hot, warm or cooling ?

A : The short answer is that the density of gas in space is very, very low - about 30,000,000,000,000,000,000 times less than air at sea level, and about 100,000 times thinner than the most perfect vacuum ever created in a laboratory. This low-density gas doesn't need much energy to heat it up. It's so thin that it tends to stay hot because collisions between particles are very rare (roughly, every few million years or so IIRC).

Because the density is so slow, if you were floating unprotected you'd freeze to death in this hot gas. The amount of energy you'd lose by radiation would be far larger than the amount you'd receive from the gas.

It's not true of all gas - when its density becomes higher, collisions increase so it loses energy much more quickly. Then it can become very much colder, sometimes only a few degrees above absolute zero.


Why study dwarf galaxies ?
Q : What is it about small dwarf satellite galaxies that makes them worth studying ?

A : In this context (http://arxiv.org/abs/1510.08715), it's all about the numbers. There are ~30 known dwarf satellites of the Milky Way, but more isolated galaxies have only about one companion each. Models of galaxy formation typically predict many hundreds of dwarf companions.

One possibility is that some of the dwarfs never accrete enough gas to form stars. Maybe star formation is triggered in the Milky Way satellites by the other galaxies in the Local Group (Andromeda and Triangulum). These isolated galaxies don't have any giant neighbours, so maybe their dwarf companions haven't been detected by optical searches because their gas has just been sitting there all this time doing bugger all.

It turns out that this is not the case - the isolated galaxies not only lack optically bright companions, but they have hardly any gas-rich, optically dark companions too. So it seems that galaxy formation really works differently in different environments, although as to exactly what the differences are and why they occur is anyone's guess.


How can we see other galaxies with all those pesky stars in the way ?
Q : I mean, there are like 100 billion stars in the Milky Way. How can we possibly see outside our own galaxy ?

A : It's not quite so difficult because our galaxy is relatively thin. The disc is about 100,000 light years wide but only 1,000 light years thick. That means it has roughly the same proportions as a CD.

It is true that in certain directions it's impossible to see through all the stars and gas. For that, other techniques are needed. But in most directions the density of stars is not so awful.
http://www.universetoday.com/88091/the-hidden-galaxy-in-the-zone-of-avoidance/


What will happen when our galaxy and Andromeda collide ?
Q : What will happen when Andromeda and the Milky Way collide ? Will there be more stellar mergers, and what will happen to the Sun ?

A : It's believed that the two galaxies will merge to form an elliptical galaxy - a giant ball of stars. My guess would be that the merger rate would increase simply because the density of stars will increase, but given that this rate is incredibly low to begin with this isn't something to worry about.
http://www.space.com/15944-milkyway-galaxy-crash-andromeda-artist-pictures.html

It's impossible to predict exactly what will happen to the Sun - we don't know the orbit of the Sun or the other stars precisely enough to make predictions that far into the future (about two billion years). It could end up inside the new elliptical galaxy, or get thrown off into intergalactic space to become a rogue star.


Will the collision of the Milky Way and Andromeda increase the amount of dangerous gravitational waves ?
Q : With the Milky Way and Andromeda Galaxies colliding, would that mean an increase in tidal waves in space? I figured I'd ask, just to make sure I was understanding the whole picture. Also, would that possible increase of tidal waves in space, would that also correlate to an increase in tidal waves on a water packed planet, like Earth ? Should we be moving away from the oceans, just in case ?

A : The terms "gravity waves", "gravitational waves", "tidal waves" and "tidal forces" all refer to very different things, but have annoyingly similar names. See http://astrorhysy.blogspot.cz/p/q-a.html#gravitytides. In short, gravity waves and tidal waves are, in space, not a thing.

What you do get in space are gravitational waves. These are so incredibly weak they can't even be detected without some of the most sensitive equipment ever devised. No amount of collisions with other galaxies will ever increase these to a level where they would ever have any effect on us whatsoever.

But what you also get are tidal forces, e.g. the gravity on one side of (say) the Earth is different than on the other. These are far stronger than gravitational waves, and depend on the mass and distance of the object causing the tides. In the case of the Earth this is due to the Moon and also the Sun to a lesser extent, because although it's bigger than the Moon it's also much further away (the planets also have a very small effect).

Very roughly, the number of stars in Andromeda is about the same as in the Milky Way. So during the collision the number of stars in the merging galaxies will have approximately doubled. This will increase the tidal forces experienced by any star or planet, since there are now more massive objects around, but not to a degree where anyone would notice. The reason is that the distance between the stars is so vast that the chances of any star passing close to another is still extremely small.

In short, what the collision will do is bring in a lot more massive stars, but these won't cause any serious tidal effects for us because they'll still be too far away.


How big is the error when trying to estimate the mass of a galaxy ?
Q : It's fairly easy to see how the mass of a galaxy is estimated as Newton's Gravitational Force being equal to the Centripetal Force. Easily enough found http://www.gb.nrao.edu/~rmaddale/Education/Wvsta'98/mass1.html but I have a few further questions. Such as, what is the general error involved in the estimate ?

A : A surprisingly complicated question.

We can estimate the rotational velocity pretty well, to within a few km/s or better from raw measurements. The complication here is correctly for the inclination angle of a galaxy - if we see it face on, we can't measure the rotation angle at all since we can only measure line of sight velocities. If it's perfectly edge on we can measure the rotation width directly and all is well. Most galaxies are somewhere in between so we have to try and correct for that, making assumptions about how thick the disc is based on statistical studies of large numbers of galaxies. Still, this correction is usually small, though for galaxies at small inclination angles (<30 degrees) we would probably chuck out the estimate because the correction starts to be really difficult to do accurately.
https://web.archive.org/web/20150303051728/http://eo.ucar.edu/staff/dward/sao/spirals/methods.htm

A bigger difficulty arises because calculating the radius of a galaxy depends on its distance, which is much, much harder to estimate. This is a huge topic which I won't go into here, but estimates can easily vary by a factor of two.

The worst complication of all comes from the dark matter. Since we don't really know how far into the dark matter halo the visible matter extends, all our estimates are lower limits.


Would it make much difference if one used Einstein's equations for gravity instead of Newton's when estimating the mass of a galaxy ?
Q : The usual formula uses Newton's Law of Gravity instead of General Relativity, obviously for simplification. Have estimates been done using GR? What would be/is the difference in estimates ?

A : If I recall correctly the difference becomes negligible for the low accelerations involved from the gravitational field of a galaxy. So if anyone has tried this they result they'd get wouldn't be much different from the Newtonian approximation.


How is the mass measured for a group or cluster of galaxies ?
Q : Is it as simple as adding up galactic mass estimates (seems doubtful to me) or are other methods used ?

A : Indeed, you can't simply add up the masses of the individual galaxies because of that pesky dark matter. In groups and especially clusters, galaxies are moving faster than expected based on the masses of the galaxies (just as individual galaxies are rotating too quickly based on the masses of their visible matter). You have to use the position and speed of the galaxies from the approximate cluster centre, just as when using the rotation of an individual galaxy to estimate its mass. Since you can only do this for a few galaxies, and since clusters are chaotic places, this means the error is a lot larger than for individual galaxies. Cluster (unlike groups) of galaxies also contain hot X-ray emitting gas, which can also be used to estimate the dark matter content - one can estimate the mass of the gas based on its brightness, and then estimate how much mass there would need to be in the cluster to prevent it evaporating.

Another technique is gravitational lensing. This is free of the problems of accurately estimating the inclination angles and rotation velocities, but has its own problems which I am not at all expert in. From what I remember, errors from this technique are also of order a factor of a few.
http://www.cfhtlens.org/public/what-gravitational-lensing


What are you planning to research for the rest of the year ?
Q : Something cool, I hope.

A : Right now I'm working on some optically dark hydrogen clouds I found in the Virgo cluster for my PhD. They're relatively isolated from the known galaxies in that region, they have rotation widths as large as giant galaxies, and they don't show any sign of being part of extended streams. Details here : http://adsabs.harvard.edu/abs/2012MNRAS.423..787T
http://adsabs.harvard.edu/abs/2013MNRAS.428..459T

For the last few months I've been working on two projects to try and understand these things better. The first has been to construct a catalogue of similar features. It turns out that these particular clouds have some of the highest velocity widths of any other known starless clouds. Similar features are often attributed to being just hand-waving "tidal debris", though there are only two papers showing that this is possible. One of these, frankly, just isn't very good. The other models a very specific system which has quite different properties to these particular clouds.

Hence the second, bigger project to try and simulate their formation. We know gas is being lost in the Virgo cluster galaxies, but long gas streams are very rare despite their being predicted by simulations. So, maybe the streams get torn apart by galaxies interacting with them in the cluster. I drop a gas stream into a simulated Virgo cluster and watch the ensuing chaos. Thus far, the results are pretty definitive that this process cannot reproduce the observed clouds. This part of the project is almost done and the paper should be submitted soon. The major task for the rest of the year will be to add in the hot gas of the cluster itself, which may change the results dramatically.

The reason these little gas clouds may be significant is that if they're not tidal debris, the other obvious explanation is that they're galaxies which have never formed stars. That has major implications for the missing satellite problem, particularly as these are rather large. Galaxies this large really shouldn't be able to avoid forming stars.


Should we get the sausages ready for when the Milky Way collides with Andromeda ?
Q : That is, I've heard it's going to be a spectacular event and the galaxies will merge to become an elliptical, and all the gas will get burned up. So should we get a barbecue ready ?

A : That is the commonly-held view, but I'm not so sure.

It will definitely be a spectacular event. Stars are going to be flung about all over the place, that much is certain. The common wisdom is that there will also be a massive surge of star formation which will quickly use up all the gas and leave the galaxy as an emaciated wreck - a monstrous "red and dead" elliptical galaxy.

Elliptical galaxies are deceptively simple things. They look like big red featureless balls of stars, and simulations show that they could be formed by the mergers of spiral galaxies. The problem is that sometimes they contain a lot of gas but no dust, or dust but no gas, sometimes both, and sometimes neither (especially in galaxy clusters). Could this just be explained by mergers ?

I have my doubts - for one thing most spirals possess both gas and dust, so the presence of one or the other in ellipticals is not so easily explained. Also, in complete contrast to what's expected intuitively, gas during mergers does not appear to be apidly consumed by star formation. The gas content might even increase slightly, which only goes to show that one should be very careful indeed about drawing "obvious" conclusions. The Universe isn't a simple place and doesn't always do what you think it should.
http://astrorhysy.blogspot.co.uk/2015/10/the-very-interesting-gas-that-doesnt-do.html

Moreover, spirals are dominated by dark matter but ellipticals appear to contain very little. How can this be if they're formed by the merging of spiral galaxies ? Personally I think the current view of galaxy evolution is woefully incomplete at best.
http://astrobites.org/2013/04/04/do-elliptical-galaxies-have-dark-matter-halos/


If the black hole at the centre of the Milky Way was a quasar, could we see it with the naked eye ?
Q : So I am watching "How The Universe Works" and they are talking about Quasars at the moment. Suppose our Milky Way's SMBH would still be a Quasar, what would that look like from Earth ? Would we see be able to see the north and south beam, or would that still be too far away ? They did say Quasars outshine their whole galaxy, so I am assuming it would look like an extremely bright star in the sky.

And in such a Galaxy, noting the awesome power the Quasar puts out, would life on a planet like ours still be possible ? Or would the fact it is outshining it's whole galaxy be a problem in other forms, for planets such as ours ? That is, in the form of radiation/particles perhaps, not being thrown our by the SMBH north and south poles but in perhaps other directions ?

A : It's actually not so difficult to estimate exactly how bright a quasar would be at the distance of Sag A*.

3C 273 is the brightest quasar in the sky with an apparent magnitude of 12.8, or an absolute magnitude of -26.1 at optical wavelengths (v band, presumably). This is a measure of how much energy the thing is putting out, accounting for its distance (apparent magnitude is how bright it actually appears in the sky).

Given the actual distance to 3C 273 (2.4 billion light years or 750 megaparsecs) and its absolute magnitude we can then work out how bright it would appear if it was as close as Sag A* (26,000 light years or 8,000 pc) using the distance modulus equation :
https://en.wikipedia.org/wiki/Distance_modulus

That gives an apparent magnitude of -11.5. Not as bright as the full Moon, but brighter than anything else in the sky. Maybe as bright as a cresent Moon, but I'm not going to work out which phase precisely.

... but, there's a lot of gas and dust in between us and the centre of the Galaxy. That will dim things dramatically. According to NED (https://ned.ipac.caltech.edu/cgi-bin/objsearch?objname=Sag+A*&extend=no&hconst=73&omegam=0.27&omegav=0.73&corr_z=1&out_csys=Equatorial&out_equinox=J2000.0&obj_sort=RA+or+Longitude&of=pre_text&zv_breaker=30000.0&list_limit=5&img_stamp=YES) this will be by over 200 magnitudes ! It probably won't really be as much as that since that calculation includes dust on the other side of the galaxy as well, but it will definitely be enough to make the quasar invisible.

Therefore, the effects of the quasar would only become important on a long timescale - after hundreds of millions of years, all that gas and dust would have been consumed/destroyed, allowing us to see it. Another bright light in the night sky would affect animal hunting behaviour, migrations, breeding, etc. I'm not too sure if the nastier X-ray radiation would pose a more sinister threat or not, but I doubt it.

As for the visibility of the jets, they can be surprisingly bright at optical wavelengths - you can even see them with a large amateur telescope (http://www.astronomy-mall.com/Adventures.In.Deep.Space/m87jet.htm). What I can't find is the surface brightness of the jets, which would determine if we could see them with the naked eye. I suspect not.


Will gravitational waves be significant for studying galaxy evolution ?
Q : I mean, you must be super excited now that all your problems are gonna be solved, right ?

A : Probably not. The only sources which can be detected at the moment are black holes and neutron stars, which are just too small to have any effect on entire galaxies. Even though the energy transmitted in gravitational waves can be enormous, their effects are minuscule unless you're very close to the source.

The merger detected by LIGO saw 3 solar masses converted into gravitational wave energy : since the energy itself basically doesn't affect anything, the only real effect will be from the mass loss. That's tiny compared to a whole galaxy. So in terms of studying galaxy evolution, gravitational wave measurements probably won't be useful anytime soon. Maybe when we get to detecting waves from supermassive black holes, or from the very early Universe when galaxies were first assembling, they'll reveal new information we couldn't have gained otherwise. But that's years away.

Make no mistake : gravitational waves are exciting and will likely open whole new fields of study - but let's not go nuts. They aren't going to help answer every problem in astronomy.


If the Sun moves out of the plane of the Galaxy, wouldn't it be pulled towards the centre more strongly and so spiral inwards ?
Q : We know the Sun is moving out of the plane of the Galaxy but how can it be on a stable orbit if that's the case ?

A : We are attracted towards the centre but we're also rotating around it. The further away from the center we get, the weaker the gravitational attraction but our rotation velocity remains constant - so if anything we would spiral outwards. But as we get further away, there's now more mass pulling us back in - hence the galaxy is stable.


How do we know about the Sun's motion through the Galactic plane ?
Q : Does it involve strange rituals including chanting and a lizard ?

A : As I understand it (and I'm no expert in this), by measuring the motions of nearby stars relative to us. By subtracting the systematic (average) velocity by which they all appear to be moving out of the plane we can work out the Sun's true motion out of the plane. But this isn't a simple problem and I don't want to give an impression of knowing more about it than I really do !
http://www.astro.utu.fi/~cflynn/galdyn/lecture7.html


Why does every galaxy have a black hole at the centre ?
Q : Well, why ?

A : They don't. Above a certain mass (I forget the number) almost every galaxy has a supermassive black hole, but below this they're much rarer. No-one understands yet why this is the case.


Are rogue stars and planets created by galaxy collisions ?
Q : What examples of recent history, can show how planets might react when a galaxy the size of Andromeda collides with a galaxy the size of the Milky way. Is this how rogue planets and stars are "born"... meaning after a collision ?

A : Rogue stars I'd say definitely yes. It's not the only way stars can get ejected from their galaxies (they can also get too close to a black hole or another star - unless they head straight for it they can get flung out at tremendous speed), but it's probably the main mechanism at work. https://en.wikipedia.org/wiki/Intergalactic_star
http://astroweb.case.edu/hos/Virgo/


Rogue planets I'd say (cautiously) probably not. Very roughly, the number of stars in Andromeda and the Milky Way is about equal. That means that on average the collision will roughly double the density of stars, which isn't enough to have much of an effect on individual star systems.

To rip a planet out of a star system you'd have to have a very strong difference in gravity over the scale of the system. While the collision of the galaxies will generate forces which are very strong over large distances - so whole star systems can be chucked out - it most likely won't generate significantly different forces on the much smaller scale of individual star systems. So probably the rogue stars which are formed will carry their planets along with them into the deep void of intergalactic space. Any entities living on said worlds will have a nice view for a few billion years but end up feeling incredibly lonely.


Is there an equivalent of a Hertsprung-Russel diagram for galaxies, showing how they evolve ?
Q : More generally, as we know with stellar classification evolution is obviously strongly taken into account with the inclusion of the "main sequence", is there a similar scale for Galaxies? Put it more plainly; is there are a "main sequence" in galactic classification for those galaxies that are in the more "steady" phase of their evolution OR is galactic evolution not really much comparable to stellar at all ?

A : It's complicated. There are the well-known red sequence and blue cloud, which is the equivalent of a galaxy HR diagram :
http://www.txhughes.com/images/sequences.jpg
But if and how galaxies move from one to the other is not known. Stellar evolution appears to be dominated entirely by mass. For galaxies it also depends on mergers (which are at least common - many people would say even the dominate evolutionary mechanism), tidal encounters (which depends on the density of nearby galaxies) as well as ram-pressure gas stripping and other gas-only effects (which depends on the surrounding gas outside the galaxy), and possibly the initial mass function (how many massive stars are formed that can explode/have strong stellar winds which can ionize/blow out the gas completely, which is in turn dependent on the chemical composition of the gas (more metals = more cooling).

And that's the simplified version. Some people think galaxies can move more-or-less freely from the blue cloud to red sequence (by running out of gas for star formation) and even back again (by merging with a gas-rich galaxy or re-accreting material from the intergalactic medium). Others think that movement between the two sequences is a rare and not significant process. The bottom line is that there's no widely-accepted "typical" process of galaxy evolution.


Could these newly-discovered ultra diffuse galaxies be ordinary galaxies at a different stage of their evolution ?
Q : [Asked in relation to this post : https://plus.google.com/+RhysTaylorRhysy/posts/5epAqQWLAAt]
One element, I assume being considered, but that I'd like to know more about, is consideration regarding the possibility that those galaxies are in a non "main sequence" stage of their evolution, such as in the process of formation, merging, or tidal disruption caused by other galaxies or dark matter influences.

A : For these UDGs, one idea is that they formed all of there stars early on in a relatively short burst, blowing out all of their gas. That would explain why they're so faint - they just never had the chance to form many stars. The difficulty with this is that they're so dark matter dominated it's not yet clear if the stars/supernovae would have had enough energy to remove their gas. It also wouldn't explain why the number of detected galaxies is so much lower than cosmological models.

But it seems even less likely that these are unstable, transient objects changing from one type of galaxy to another. They have very simple, smooth, spheroidal structures and their globular cluster distribution appears to be quite symmetrical. If they were experiencing significant disruption, they should be messier. An exotic possibility suggested for similar objects in the Local Group was that these aren't stable galaxies at all, but are just "tidal debris" that's evaporating - we happen to see it at the stage where it's still barely detectable. Given the new discoveries of large numbers of similar objects in cluster, that now looks to be very unlikely.

The short answer is that we don't know.


Could Unruh radiation explain the direction of galactic rotation ?
Q : [Unruh radiation was recently proposed as an explanation to the current darling of the pseudoscience world, the EmDrive. The claim is that this device can produce thrust without expelling any material or direct physical contact with anything - in Clarke parlance it is a "space drive" or "star drive". Conventional physics says this would violate conservation of momentum, a principle so well-established that breaking it is basically saying that "a wizard did it.
A sort of perverse hope recently appeared with a paper claiming that another well-known effect called Unruh radiation could explain what was going on without violating conservation of momentum after all. Unfortunately it also claimed that this could also explain the acceleration of the Universe without dark energy and the rotation of galaxies without dark matter. That's a hell of a claim based on barely measurable effects. The scent of "too good to be true" is wafting through the air like the fetid aroma of a decaying whale carcass.]

"...the idea behind it can also explain galaxy rotation ..."

Hmmm. Would that make the rotation direction (CW/CCW) predictable in galaxy formation ? Or is CW/CCW just an "above or below view" issue ? How could you know ?

A : As you say, it's an above or below issue. There's no standard against which to measure the rotation direction. Actually some galaxies have components which rotate in opposite directions :
https://medium.com/starts-with-a-bang/messier-monday-an-elliptical-rotating-wrongly-m59-3228197387ac#.2h0no5i1v

As far as I can tell, this wouldn't necessarily cause any problems for this idea (or indeed most other ideas of gravity/inertia). It only predicts the speed of the rotation, not the direction. It's not obvious to me if this model has any fatal flaws from a galactic perspective, I have to think about it some more.
Fortunately I did not have to think about it because John Baez already explained in some detail that although Unruh radiation is a thing, it is weaker than Piers Morgan's claims to journalistic integrity. It's about as good an expalanation as saying, "because sausages".
https://plus.google.com/+WinchellChung/posts/HUFF2tVKwQh



How many gas clouds are there around M33 ?
Q : I've been reading your new paper about the HI clouds you and your colleagues observed around M33 (https://arxiv.org/abs/1605.01628). I wonder if you can clarify one small point for me. In the results section it states that you found 32 clouds of which 11 are newly observed, but later on it states that you found 22 discreet clouds which matches well with the (25) predictions of simulations. So how many clouds did you find ?

A : It's not quite as simple as that. Previous observations discovered a population of clouds around M33, nice and straightforward. But our deeper observations revealed that the disc of M33 is more extended and some of the "clouds" are actually just extensions of the disc - hence we "found" 32 "clouds", but some of these aren't really clouds at all. Since the simulations predict the number of dark matter subhalos (i.e. galaxies) we discount these disc-extensions when comparing the number of clouds with simulations. So we discovered all the previous clouds and more besides, but it turns out that the number of true clouds is only 22. See also :
http://astrorhysy.blogspot.cz/2015/11/keenans-ring.html
And the original paper :
http://arxiv.org/abs/1511.02710


Is Keenan's giant gassy Ring really all that giant ?
Q : The article (https://arxiv.org/abs/1605.01628) says "If the cloud [a.k.a. Keenan's Ring, a giant gas cloud near the M33 galaxy] is at the distance of M33, it is larger in size than the galaxy." This seems to indicate that the determination of distance is weak. But the rest seems to assume that the cloud is at the distance of M33... ?

A : Indeed, there's no way to directly measure the distance to the Ring. It's very unlikely to be much further away than M33, otherwise it would be both enormous and with no plausible origin. It's also very unlikely to be within our own Galaxy since the redshift is all wrong. If it's as close as M33, it's still as large as M33 itself (very few other starless hydrogen clouds this large are known) and it's difficult to see how it could have originated from M33, but at least it would be a possible source of the gas. If it's closer, it could be part of the much larger Magellanic Stream, but it's quite far away from the rest of the stream. In any case, there's no good reason why it should be ring-shaped at all. It's a strange one.
http://astrorhysy.blogspot.co.uk/2015/11/keenans-ring.html


What do you think of this press release about dust ?
Q : Have you seen the civilian press reporting http://www.upi.com/Science_News/2016/05/06/Study-Distant-glowing-dust-hides-several-galaxies/3021462555614/ on this paper http://arxiv.org/abs/1605.00006 ? I've always been interested in star and planet formation. How significantly does this change star formation models ?

A : First off, I thought the outreach version must have suffered a garbled translation. But then I read the original (http://www.sussex.ac.uk/newsandevents/index?id=35487) and it's scarcely any better. They have NOT solved one of astrophysics great conundrums. They haven't "solved" anything, for starters, and I'd hardly call it a great conundrum anyway. What they've done is formulate a statistical model to see how likely it is galaxies detected in Herschel, which has a pretty low resolution, are actually composed of several galaxies. They find that this could mean that galaxies are forming stars about 30% less rapidly than previously thought. Is it important ? Sure, if you're studying those sorts of distant dusty starbust galaxies. But I'm left with a lingering confusion as to why they did a press release. 30% is not exactly a revolution : a galaxy which was thought to be forming 1,000 stars per year is now "only" forming ~700 stars per year. That's even assuming their model is correct, and that this stage it's very much in the "only a model" stage.

As for how this might change star formation models in general, it's hard to say. Starburst galaxies are pretty extreme systems, and despite the claims in the paper, I have a tough time accepting that a 30% drop really makes them much less extreme. Plausibly this makes no difference at all at the level of individual star formation - it could just mean that conditions throughout the galaxy are a bit less conducive to star formation. Really the correct approach is to wait and see how other experts in the field (and that's not me) respond to it.


What do you think about this silly meme about a void in space ?
Q : What do you think of this meme ? Is this the same region that was used for the Hubble Deep Field ?
http://static.damnlol.com/i/7efe22d0ad4b926f9a6f324ee8a0b248.jpg

A : This is not a void in space. Voids in space as described in the caption do exist, but do not appear this empty because you can still see the galaxies behind them. The image is in fact of a Bok globule, a (much smaller !) cloud of gas and dust which absorb radiation, hence appears dark.
http://www.daviddarling.info/encyclopedia/G/globule.html

This particular Bok globule is also nowhere near the Hubble Deep Field, nor was the HDF selected to target any kind of galactic void - just a region where there was nothing especially interesting.
https://en.wikipedia.org/wiki/Hubble_Deep_Field#Target_selection


Why don't turbulent extragalactic gas clouds look like smoke ?
Q : https://plus.google.com/u/0/+RhysTaylorRhysy/posts/gxcD3bNis5k
I'm in slightly over my head over here, but I have to ask: why is it 'spiky' ?
If we considered it as two gasses of different temperatures and densities, then wouldn't they curl up like smoke-balls and like ?

A : Well, that's complicated. The structure will depend on the velocity gradient of the turbulence - if the velocities vary strongly over small distances, you'll get a different structure than if the gradient is shallower. Plus all the gas has significant self-gravity : it wants to collapse. Simultaneously it's being heated by by the surrounding hotter gas, which makes it expand. Unlike fire though, there's no source of energy to maintain the turbulence, so its continuously being resisted - only the initial energy allows it to keep moving. Also unlike fire-borne smoke, the dense gas here starts off stationary with respect to the intracluster gas - if we put a wind in, things might change significantly. Finally, the structures seen depend to some extent on the resolution of the simulations, which here are rather low to keep things fast.

The short answer is that there are really too many variables to intuitively predict the shape of the gas.


I expect Rhys Taylor knows about this press release on spiral arm formation.
Q : [I assume that this was really asking, "what do you think about this ?" rather than a generic declaration to the world that I'm probably aware of it, so...] What do you think of the idea that spiral arms in galaxies are density waves, as in this press release ?
http://phys.org/news/2016-08-astrophysicists-mechanism-spiral-arm-formation-disk.html

A : He does, but he's not terribly impressed by the press release. I have to mention that the very first galaxy simulation showed spiral arms as density waves back in 1941 (not in the '60's as the article claims)... but what's more interesting is that since computers weren't a thing then, the guy used light bulbs instead (one of those seriously crazy experiments that deserves to be mentioned at every opportunity).
https://ned.ipac.caltech.edu/level5/Biviano2/Biviano5_1.html

Modern simulations routinely show spiral arms as density waves (e.g. shameless plug https://plus.google.com/u/0/+RhysTaylorRhysy/posts/EUV3Uyc29ss) so it's a bit odd to call this a discovery. Also a bit strange to mention pitch angle because I'm pretty sure variation in pitch angle was already well-known. There was even a paper showing a near correlation between pitch angle and mass of the central supermassive black hole (http://iopscience.iop.org/article/10.1086/588727/fulltext/fg1.html).

Maybe the paper itself is better, I'm confused by the press release but it wouldn't be the first time the original article got garbled in translation.

Later, I read the paper completely if not very carefully. Seems like a decent bit of research to me, but it's about a rather obscure (though important) detail of the accepted theory of spiral arm formation. Specifically they compare the position of the spiral arms in different wavelengths. The theory being that the density waves should move at different speeds at different distances from the galactic centre, and the resulting stars which form should then appear at different positions relative to the density wave (in the gas) itself. They look at different wavelengths which trace the gas, young stars and dust and find that it agrees very well with the density wave theory.

Which is all well and good but I am left wondering why they did a press release on this. They barely mention the alternative theory and anyway the density wave model is already widely-accepted. Essentially, the validate a model that everyone already believes is true. Well, I guess one has to satisfy the paymasters somehow...


If Andromeda were to go nova instead of colliding with the Milky Way, what would happen ?
Q : If Andromeda were to go nova, instead of colliding with the Milky Way galaxy, how would such an event affect the Milky Way galaxy, since they are almost touching on the edges ?

A : Andromeda won't go nova - a nova is an eruption on the surface of a white dwarf star. Like a much less powerful supernova, it's something that can only happen to an individual star.
http://curious.astro.cornell.edu/physics/78-the-universe/stars-and-star-clusters/general-questions/352-what-is-a-nova-beginner

Novae are common in our own galaxy, estimated at a rate of about 40 per year. They don't have any direct effect on us - again, like a supernova, they'd have to be really close to do that. There's really no chance of that happening - 40 eruptions per year is not so many in a galaxy of ~400 billion stars. So a nova in Andromeda would have essentially no effect on us at all.
https://en.wikipedia.org/wiki/List_of_novae_in_the_Milky_Way_galaxy

But on a much longer timescale (millions of years or more), novae in our own Galaxy probably have at least some role in heating the interstellar medium and injecting heavier elements, which is important in star formation. Imagine being in a room with a small fire burning, slowly heating it up and filling the air with soot. Now imagine that every so often someone chucks in a grenade for good measure. The interstellar medium is a bit like that, full of constant winds being blown out by hot stars but also punctuated by very violent novae and supernovae explosions - there are shocks, bubbles, filaments, and lots of fun stuff we don't really understand yet.
http://www.centauri-dreams.org/?p=9671

As for this very extended gas around Andromeda and the Milky Way (see http://hubblesite.org/newscenter/archive/releases/2015/2015/15/full/), my guess would be that at least some of it comes from supernovae and novae explosions blasting material out of the galaxies, with the rest being leftover material that hasn't yet fallen into the galaxy's discs. This gas is very thin but also very hot, which means its particles are moving very quickly in random directions. So if there was an explosion in/near this gas, it would send a shockwave through it but it would be quickly smoothed out because of the gas' high temperature.

I found a lot of press releases about this extended gas claiming that the Milky Way and Andromeda are going to collide sooner than expected. This is only sort-of true. It's true in that they have these much larger clouds of gas which may already be interacting. But this gas is very, very thin, much thinner than the gas found in the discs (by a quick glance at the original paper I estimate it to be around a million times thinner than the normal gas in galaxies). So it's not likely to have much of an effect on the collision, and certainly the stellar discs still won't collide for several billion years.


What do you think of this article on Dragonfly 44 being mostly made of dark matter ?
Q : is it row sombrero galaxy ?
http://www.scientificamerican.com/article/this-weird-galaxy-is-99-99-percent-dark-matter/

A : What ? I don't know what that means, but here are my thoughts on the article.

Why they are using the Sombrero Galaxy in the image I have no idea. Worse, they label it as an "ultra diffuse galaxy", which it most definitely is not ! Also :

Van Dokkum and his team later realized that there was something very odd about Dragonfly 44: a galaxy that big couldn't possibly hold itself together with so few stars. There wouldn't be enough gravity, and the stars would drift apart. They suspected that dark matter was responsible for holding the galaxy together, and this particular galaxy seemed like it contained a ton of it, so they set out to determine exactly how much.

That is badly misleading. There's nothing odd about galaxies which require dark matter to hold them together, that is normal. It's the very reason why dark matter was proposed to exist !
https://www.youtube.com/watch?v=YA-boPQtSIc

There are two unusual things about galaxies like this :
1) They're incredibly faint for their size, 100-1000 times fainter than normal galaxies. They hint that there could be even fainter galaxies, possibly with no stars at all. That would solve the "missing satellite problem", the fact that cosmological models predict far more galaxies than we see.
2) They are (it seems) extremely massive for their brightness, so much so that they fall off the normal relations for galaxies. Standard models do in fact predict this, but until now no-one had detected any galaxies like this. The leading alternative to dark matter, modified gravity, does not predict galaxies like this AFAIK, so this may rule that model out.

On the other hand, no-one has a good idea why some dark matter halos of equal mass may form a few stars while others form a thousand times as many. That one's going to be tricky.
http://astrorhysy.blogspot.cz/2015/10/moaaaarr-galaxies.html


Could Dragonfly 44 be full of Dyson spheres ?
Q : [In relation to this article, about one of many newly-discovered "ultra diffuse" galaxies - very faint but massive objects]
I'm waiting for someone to opine that all the visible stars in that galaxy are shrouded by Dyson spheres. I'll do my ::eyeroll:: in advance.

A : Wait no longer !
http://adsabs.harvard.edu/abs/2015ApJ...810...23Z

Basically the Tully-Fisher relation is how bright a galaxy is compared to its rotation velocity. Like the galaxies the authors search for in that paper, this galaxy is rotating much faster than most galaxies of the same brightness... which "could" be attributed to most of the stars being surrounded by Dyson spheres. :)

(Of course Dyson spheres should cause an infra-red excess, which so far as is known is not the case.)


Are you very skeptical about Dragonfly 44 ?
Q : Didn't I read something by +Rhys Taylor being very skeptical about this ?
http://phys.org/news/2016-08-scientists-dark-milky-massive-galaxy.html

A : I would describe my position as "cautious" rather than very skeptical. I'll have to read the paper again but IIRC they extrapolate the large mass (much larger than directly measured) based on dark matter profiles from numerical simulations. This might be a very sensible thing to do, I'm not sure. I'm concerned because we're using rules established from pure dark matter simulations that we aren't all that sure work very well to explain normal galaxies to also explain these new things, which are a thousand times fainter. I'd be a lot happier with some independent mass estimate.
https://plus.google.com/u/0/+RhysTaylorRhysy/posts/efNVzzChWwN


What do you think about Dragonfly 44 ?
Q : Tell us how you really feel.

A : OK, that's enough with the Dragonfly 44 questions, people. Just read my blog post with the executive summary here :
https://plus.google.com/+RhysTaylorRhysy/posts/d1mphYuN3mx


What would happen if dark matter started absorbing light ?
Q : When normal matter absorbs light, it re-radiates it another wavelength. Absorbing light heats matter up, and the wavelength it radiates at depends on its temperature. The higher the temperature, the shorter the wavelength. The Sun, stars and incandescent light bulbs all radiate at visible wavelengths because their temperature is a few thousand Kelvin. Interstellar dust tends to be rather cooler and radiates in the infra-red. So dust lanes in the galaxy appear as dark, opaque streaks in normal light but show up brightly in the infra-red.

Dark matter is different. We don't even know if it has a temperature : it seems that it only interacts with normal matter through gravity. But, if it did absorb light, it would have to make everything we see darker. Normal matter both absorbs and scatters light in such a way that the shorter blue wavelengths are more affected, so as well as darkening any stars behind the dust it also causes them to appear redder. Whether this would also happen with dark matter would depend on the detailed physics of what it actually is.

The effects this would have on galaxy evolution would also be strongly dependent on what happens to the dark matter and exactly how it absorbs the light. For example it might cause the dust and gas in galaxies to receive less heat from the stars so they would cool, causing them to collapse and so form stars more rapidly. But if it also re-radiated the energy as normal matter does, it might not have much of an effect. My guess would be this could be very important if dark matter had absorbed light in the early universe, when stars and galaxies were still forming, but probably wouldn't have such a dramatic effect today(except on very long timescales) since galaxy formation is largely over.


Is dark matter involved in ram pressure stripping ?
Q : What about this press release, eh ?
http://www.businessinsider.com/ram-pressue-stripping-killing-11000-nearby-galaxies-2017-1

A : It's wrong.

"The answer paints a picture of these galaxies falling through their larger dark-matter halos, having their star-forming gas removed in a fast-acting process called ram-pressure stripping."

To be fair, the rest of the article correctly points out that it's not the dark matter halos which are doing the stripping at all - it's the hot gas inhabiting the halos. At high speeds the force that builds up from the pressure of the external gas exceeds the force holding the gas in the galactic disc and it gets blown away. The fact that there's a large dark matter halo as well is somewhat incidental, and definitely shouldn't be stated like this in the last sentence !


Once all the stars go out, will all the hot gas around galaxies start collapsing and form more stars ?
Q : So I guess on the very longest scales this is a cyclic process, since all that keeps the gas found outside galaxies hot is ionization by starlight, and that's all that keeps it from eventually recollapsing. So once the quenched galaxies go out, the gas will fall back into them and a new (much more stretched-out) burst of star formation might begin ?

A : That's controversial. Heating of the intracluster gas can happen as galaxies and galaxy groups are accreted and lose gravitational energy, as well as through active galactic nuclei (a nice size comparison : it's like a marble heating up the whole Earth). Hot young stars can ionize gas directly, but not on very large scales. They can also heat the gas through stellar winds and more significantly through supernovae, but these are again mainly on galactic scales rather than cluster scales.

Its temperature is indeed what prevents it from recollapsing. But if and when it does re-collapse is not well understood. Some people think that gas streams seen around some galaxies are evidence for cooling flows, but it's very difficult to prove that this gas hasn't been stripped out of the galaxies instead. Elliptical galaxies on the edge of clusters sometimes contain gas whereas in the cluster interior they're barren, which is sometimes interpreted as evidence for the gas cooling. But elliptical galaxies outside clusters often contain lots of gas anyway, so again it's hard to prove the link. In principle though, the cooling gas could indeed eventually lead to more star formation on longer timescales.


Could some dark matter halos just be made of normal matter ?
Q : Sorry for a silly question, but what is the reason to exclude the possibility of (some) dark matter halos around galaxies being just ordinary unlit and non-glowing matter that would interact with the galaxy's matter following the laws and customs of ordinary matter?

A : Complicated question. I shall divide the answer into two parts - first you must understand why we think these dark matter halos exist at all, and then I'll look at ordinary matter that remains optically dark.

First, your question implies that we already know of such dark matter halos and they need explaining. In fact we don't know of any such electromagnetically-dark halos from direct observations, but they are inferred from models. We only know of dark matter at all thanks (largely) to the rotation and motions of galaxies - plenty of evidence for it inside optically bright galaxies, but little or no evidence for pure dark matter halos.

"Dark matter" has become synonymous with non-baryonic (i.e. a new type of hitherto unobserved particle) matter that doesn't even interact with normal matter except through gravity. This is very computationally cheap to simulate since you just need a bunch of particles with gravity, with none of the pesky gas dynamics and other complicated physics associated with ordinary matter.

Numerical simulations of the Universe have therefore traditionally used only this particular non-baryonic dark matter. The number of detectable galaxies is computed based on how much ordinary matter should fall in to each halo. This, unsurprisingly, does not give the right answer - but the consistency of the method in being wrong by a factor 10 or more is so large it's hard to dismiss as being due to the method of calculation.

Hard, but not impossible. Actually more recent simulations, with computers now (just about) powerful enough to handle the ordinary and dark matter at the same time, are finding that some dark matter halos never accrete enough normal matter to form any stars. Still, the reason these "dark galaxies" exist in the simulations at all is because of the non-baryonic dark matter, so they're not the purely optically-dark clouds of normal matter you're suggesting. Such clouds would not help explain the missing galaxies problem (since it's really the dark matter that's missing), so in that sense there's no particular reason to assume they exist at all.

The second aspect to the question is whether dark but otherwise normal matter could exist at all. To that the answer is "yes with a but". As well as missing galaxies there's also the "missing baryon" problem, where models of Big Bang nucelosynthesis predict around 5x as much ordinary matter as is detected. It's thought that most of this missing normal matter ends up in the hot, very low density halos of galaxies and galaxy clusters, where it's very difficult to detect. There are some hints that this is the case (e.g. https://www.scientificamerican.com/article/the-milky-way-s-missing-mass-partially-found/) but it's very tentative at this stage.

This hot matter would not be particularly clumpy, so you wouldn't get it forming structures on galaxy scales until it cools. On the other hand, probably not all of this missing normal matter is hot ionized gas. Some of it could be very cold molecular (or perhaps atomic) hydrogen, which is hard to detect indirectly and nearly impossible to detect directly. That has the potential to form dark galaxy-like objects, and is similar to the stuff you're suggesting.

How much there could be is controversial. Big Bang nucelosynthesis allows there to be just enough missing ordinary matter to explain the dark matter inferred from galaxy rotation curves (which is an interesting coincidence), though not the much greater amounts implied by the motions of galaxies in clusters. So it's still just about at the edge of respectability to propose that significant amount of "dark matter" is just hard to detect normal matter.

But even when it's not illuminated (you can never get zero illumination mind you - all normal matter has a temperature and radiates), normal matter would still make its effects felt in other ways. Microlensing studies have more-or-less ruled out objects like black holes, planets and other small bodies. Vast amounts of cold, undetected gas could explain away the non-baryonic dark matter, but at the expense of totally smashing every aspect of star formation theory and maybe gravity as well - it doesn't seem possible that such gas could exist without collapsing and forming stars at a far higher rate than observed.

So the short answer is "no". We don't know how many dark matter halos are really out there, and we know of no form of ordinary matter that could create such a structure that's compatible with observations of dark matter where we're confident it exists.


What's your take on this press release about a new law of nature ?
Q : I mean this one : https://phys.org/news/2017-02-team-radial-common-galaxies.html

A : Oh, no, not this one again...

The basic claim started in an earlier paper that there's a very tight relation between the acceleration observed in galaxies and their density of normal matter. Since dark matter is supposed to account for 90% of the mass of galaxies, it's not immediately obvious why the density of the normal matter should correlate so well with its acceleration. That ought to be dominated by the dark matter, so there shouldn't be much of a relation between normal matter density and acceleration.It suggests that maybe dark matter is not such a good interpretation after all.

This observation was in fact already known before but the paper measured it much more precisely; there had always been some doubts over how strong the relation was. No-one had really paid it that much attention and it was seen as a curiosity. But now the authors went to the opposite extreme, proclaiming it as "tantamount to a new law of nature". And it really does seem to be the authors who said that, not some crazy press release officer trying to generate clickbait.

To be blunt, I find this to be an absolutely crazy statement. Yes, it's interesting - but to make such an incredibly bold claim without the community reviewing it first is extremely disappointing. That's the level of claim you can only make after years and years of deliberation, NOT in one paper just because you think so.

Since then several papers have shown quite clearly that the standard model can explain this result just fine, thank you so very much; I review a couple of them here and here but there have been others. So there doesn't seem to be much interesting about this at all. That the latest findings support the earlier claims is not the least surprising; that they are still making these overblown claims is just aaaargh. This, in my view, is one of the surest ways to turn people off science and lead them to mistrust it. Not by merely claiming radically different ideas or theories from the standard model - that's absolutely right and proper. No, it's the extreme, unwarranted level of expressed confidence that's the issue here, not the claim itself. Keep saying you're (effectively) so certain of these things you have no right to be certain about, only to find, inevitably, that there's also a much more mundane explanation, and all you do is make everyone look incompetent. The sad thing is that I've met the lead author several times. I think he's a good researcher who seems to be genuinely open-minded, which makes it all the harder to understand what's going on here.


Do irregular galaxies have black holes in their centres ?
Q : irregular galaxies don't have black holes in the center

A : Well, there are a few irregular galaxies with candidate black holes in their centres :
http://www.space.com/31755-hubble-spies-on-a-messed-up-galaxy.html
http://www.dailygalaxy.com/my_weblog/2015/04/a-supermassive-black-hole-exists-where-its-not-supposed-to-be-inside-a-blob-galaxy.html
But in general it does seem that only massive galaxies (which tend to be spirals and irregulars) host central supermassive black holes, though not all !
https://en.wikipedia.org/wiki/M60-UCD1

We don't really know why this should be. We don't yet have anywhere near a good enough understanding of how supermassive black holes form to say why they should prefer larger galaxies. Black holes are tiny compared to their host galaxies (that last example aside), but their mass seems to correlate with even the large-scale properties of its host (e.g. https://arxiv.org/abs/1304.4937). And the status of intermediate-mass black holes (tens to hundreds of thousands of times the mass of the Sun) is also poorly known. Supermassive black holes are almost uniqutious in massive galaxies, but whether this extends to smaller black holes in smaller galaxies isn't yet known. Galaxy formation theory still has more questions than answers !


Are there any advances in astronomy that you're excited about ?
Q : Well, are there ?

A : [I put this one in the "Galaxies" section because I'm hopelessly biased]
Yes, lots ! I'll limit myself to the three main aspects of astronomy : instruments, observational discoveries, and theory.

The instruments I'm most excited about are the SKA pathfinder telescopes ASKAP and MeerKAT in Australia and South Africa. Although designed "only" as tests for the much larger Square Kilometer Array, they'll be extremely powerful instruments in their own right. The volume of data from these will be much greater than anything produced by other instruments at similar frequencies, giving us not only huge numbers of detected galaxies but also imaging those galaxies at high resolution.
http://www.atnf.csiro.au/projects/askap/index.html
http://www.ska.ac.za/gallery/meerkat/

The discovery I'm most excited by is the detection of hundreds of extremely faint, extended galaxies in clusters that seem to require huge amounts of dark matter to hold themselves together. No-one predicted these. In some ways they make sense : if galaxies form when gas collects into dark matter halos, we should expect some halos to gain less gas than others, so galaxies of equal total masses should have a wide range of stellar masses. But simulations have so far predicted that galaxies of a certain mass should ALWAYS form stars, whereas these results imply that many of them do not. It's too early to say if these results will help or hinder our current models - either way, it's an exciting development !
http://astrorhysy.blogspot.cz/2015/10/moaaaarr-galaxies.html

The theoretical developments I'm most excited by are the prospects of solving the so-called "missing satellite" problem, where far fewer galaxies are detected than simulations predict for the last 20 years or so. Papers proposing solutions have been around for a while but have never been very convincing. Now they're starting to make more sense, as more sophisticated simulations show that some galaxies just wouldn't be detectable (e.g. https://plus.google.com/u/0/collection/ociYX). Similarly there's this observation that satellite galaxies orbit in planes around their larger parent galaxies, whereas simulations have predicted they should be roughly spherical swarms. A number of papers have come out in the last couple of years proposing different solutions and questioning just how common these planes really are, and although we certainly don't have the definitive answer yet, in my opinion it's looking more and more likely that the standard model will be vindicated. Not that there aren't plenty of other mysteries left to solve... :)


Is this black hole in a distant galaxy heading right for us and if not why are the media making such a fuss about it ?
Q : Anyway did they said that it's heading towards earth particularly if not then there is no reason of making any hype and secondly 8 billion light years from earth by that earth will loss it's atmosphere like mars or we will be somewhere else so again no need to make a fuss about it
http://www.independent.co.uk/news/science/nasa-supermassive-black-hole-discovery-a7650656.html

A : I don't know where the above idea that it's heading right for us came from. The expansion of the Universe means that nothing eight billion light years away is heading towards us. The reason to report it is not because it's dangerous or will ever directly affect the Earth in any way, but simply because it's interesting.


Can black holes move through space ?
Q : Black holes do not travel. Despite that for the black hole to travel to Earth it would take it millions of years depending on it velocity and distance.
http://www.independent.co.uk/news/science/nasa-supermassive-black-hole-discovery-a7650656.html

A : In all probability the black hole isn't even moving in the direction of Earth (I have no idea why people keep suggesting this). It's 8 billion light years away, meaning it would take at least 8 billion years to get here at the speed of light, which cannot be reached. In reality it would actually take vastly longer because of the expansion of the Universe, which is carrying distant galaxies away from us. By the time it got "here", "here" would have moved and the Sun would have long since died.

What is known, however, is that it is definitely moving. Although black holes don't emit light themselves, matter falling into them can. This is how the black hole was detected. We can measure the "redshift" of this light (http://astronomy.swin.edu.au/cosmos/C/cosmological+redshift) to determine how fast it's moving along our line of sight. This doesn't tell us how fast it's moving across the sky though, which is much harder to measure. In this case the movement is inferred because the black hole is not where it should normally be, at the centre of its host galaxy (and from the difference in velocity of the black hole and the rest of the galaxy). In any case, black holes certainly do travel !


Are these ionised gas clouds evidence of galaxy destruction ?
Q : Is it possible that these ionisized gas clouds give an extended evidence of a a process of destruction. https://phys.org/news/2017-04-ionized-gas-clouds-abell-cluster.html
About Abel 85 and Abel 1367 I read a scientific referat here in pdf-format that evidence had be given to these acts. I give the link: (some facts ar dated in the Year 2001) eso.org - www.eso.org/sci/meetings/2005/NGG/presentations/Thursday/Bravo_Alfaro.pdf

A : This is a really interesting paper. I'm going to have to read through it a few times, but I think it's a nice example of how little we understand about the cluster environment. They have some extremely interesting objects, far from the nearest galaxies and without any obvious origin. It's also apparently not so easy to explain what's ionising the gas so far away from the galaxies (though this is in stark contrast to the common wisdom that all gas should be ionised below a certain density).

It's worth looking at the original paper (https://arxiv.org/pdf/1703.10301.pdf) if only for the figures beginning on page 16. They have a wide variety of different objects in this sample, even though they have just a few objects. Some are fairly obviously gas that's being stripped out of galaxies in nice, simple linear streams but others are more complex wakes. A few show hints of helices and others are isolated clouds.

Most likely most EIG features do result from relatively smooth, continuous interactions that transform rather than destroy. But not all. Models show that a few really strong interactions are expected to occur in clusters that can tear a galaxy apart. And some objects are seriously weird (my favourite is figure 2 here : http://goldmine.mib.infn.it/papers/preprocessing.html).

In short, while most of these H-alpha features are probably the result of relatively gentle disturbances, at least a few are likely to be from much more violent encounters - quite possibly involving full-on galaxy destruction (at least for dwarf galaxies if not giants).


Are some galaxy encounters more important than others ?
Q : So granted we are probably talking massive interactions that take place during the course of millions or possibly billions of years, would it be fair to assert that certain gravitational gradients or densities "superseed" others and take over the interaction which will ultimately determine where the gas or matter will condense the most, thus dictating the overall shape of the galaxies, gas and clusters?
https://phys.org/news/2017-04-ionized-gas-clouds-abell-cluster.html

A : Sort of. There are two main processes at work in clusters : galaxy-galaxy encounters and ram pressure stripping. Ram pressure stripping occurs as the galaxy moves through the hot but very thin gas found throughout the cluster. How strong this effect is depends on the density of the intracluster gas, the gas in the galaxy, the gravitational field of the galaxy (from the combination of its stars, gas and dark matter) and strongly on how fast it's moving through the cluster. The density of the intracluster gas can also vary surprisingly sharply, so in some cases ram pressure can only be effective for a very short period (millions or tens of millions of years) as the galaxy moves through the densest gas, while in others it might last much longer (hundreds of millions of years, perhaps a billion or so). This is thought to be the dominant mechanism for gas removal in clusters.

Galaxy-galaxy encounters can be similarly varied. This is something I looked at in detail for my last two papers, and in short it's bloody hard - maybe even impossible - to identify which interaction is dominant. Since gravity does not have a limited range, a galaxy in a cluster feels the effects of every other galaxy to varying degrees based on their mass and distance. Relative speed is also important, since a weak interaction that lasts a long time may be more damaging than a strong one that lasts only briefly (here the interaction timescale is also more varied, with no upper limit).

In general, it's not the case that a single or a few interactions are responsible for the overall effects on either the galaxy or the stripped gas. It does happen in a few cases, where a single giant galaxy can be readily identified as the likely cause, but this is rare. Most of the time it's impossible to say if a single interaction was responsible. In some cases it really appears as though only the cumulative effect of all the other galaxies was the cause - take away any individual galaxy, and the results wouldn't change much.

I suppose a ship on a stormy sea might be a reasonable analogy. Sometimes you can identify a single freak wave that sinks the ship, other times it's simply the endless battering that finally ends it.


Do galaxies move ? What about galaxy superclusters ? ?
Q : Well, do they ?

A : Yes they do. We can measure how fast they're moving towards or away from us fairly easily (see, for example, Cosmic Flows : https://www.ipnl.in2p3.fr/projet/cosmicflows/)

Measuring motion across the sky is much harder, because galaxies are much further away than stars so they appear to move more slowly. It can be done, but only for the nearest galaxies.
https://briankoberlein.com/2014/09/22/movin-right-along/


Where are you looking for Ultra Diffuse Galaxies ?
Q : [This was in relation to my project to look for molecular gas in UDGs using the IRAM 30 m telescope in Granada, which I posted a lot about on social media]
Much talk about dark matter in these ultra diffuse galaxies comes bubbling up in the search engines. Where will your team be looking?

A : We're looking in the Pegasus group/cluster. We have a deep HI cube for this region, which I catalogued earlier in the year because I was sick of running simulations. By happy coincidence, a completely separate group happen to have run a UDG-finding code over the same region, and some of our sources match :
http://adsabs.harvard.edu/abs/2017ApJ...846...26S

And more on UDGs : http://astrorhysy.blogspot.cz/2017/07/ultra-diffuse-galaxies-revenge-of-ghosts.html

Does this paper make the standard model more likely ?
Q : [This was in response to a summary of this paper that I posted here]
Well, does it ?
A : I do like the "issing satellites problem" of the paper's abstract. But I'm very skeptical of this claim. First, they claim that the anisotropy of the Milky Way's satellites is only a hint. This is not so :
https://blogger.googleusercontent.com/img/b/R29vZ2xl/AVvXsEgSRQPTVmlnTSWDFD7D_gloNs61OggZxiYI4UMdzoMiUoNmd2z6cSGgCKQz6c4OXBEnIEAxRXw3P9HFD_I8Xex2vSUMZvmA9YpGRAxKQngkHr8gdwf1pCxpNw1ttGUE4_eFFQLnx4UGfhA/s1600/MWPlane.jpg There it is, big as life. You can't just pretend it might not exist. And corrections for the incompleteness of surveys and the zone of avoidance (where the Milky Way blocks the view) have already been made and found to be minor. Also, while I think the work of Kroupa, Ibata, Pawlowski et al. is just wrong in many regards for the claims of other satellite planes around other galaxies, not citing them at all doesn't seem right.

Their caim about being able to solve the missing satellites problem problem is nothing new : this has been known since Simon & Geha 2007. You CAN solve the problem, with enough complexity, i.e. if you throw enough extra physics at it and tune everything just right. The difficult part is not showing that any solution exists (which is what I think they've done here), but that any particular solution is the correct one. You need to show that the parameters required for the explanation are the ones which are actually true in reality.

The other big problem with this is that they're only considering satellites. But now it seems as though the problem is much more general : there are fewer isolated galaxies than expected as well, in many different environments. So removing the detectable gas and stars by tidal encounters (which they say is the main correction to the standard model) can't be the general solution : it will work for the Milky Way satellites, but not for isolated galaxies that have nothing else to interact with.

That said, being able to show that CDM can work is still better than showing that it can't work, even if it doesn't show that the result is actually correct. But again, that's been known for a decade. What's new here, and may or may not be more useful generally (I'm not going to go through them and check), is that they produce analytic equations to account for the corrections due to different processes. Which is nice, but a very far cry from being able to claim, "mystery solved".


Can we measure the distances to gas clouds in our galaxy ?
Q : Come on, make with the answer you lazy bum.

A : Stars can have direct distance measurements, but gas clouds are more difficult. It's very hard to get a parallax of a diffuse, extended object. But with some ugly maths, it's possible to use velocity information (which can be measured directly) to get distance information. It works well, but it has limitations that direct distance measurements don't, such as not being able to map things too close to the galactic centre on the sky.
http://ircamera.as.arizona.edu/astr_250/Lectures/Lec_22sml.htm


Do the satellite galaxies of Centaurus A orbit it in a plane ?
Q : [This was in response to the following summary and gif I posted here : https://plus.google.com/u/0/+RhysTaylorRhysy/posts/NwikyMfHrS3]
There's one point in the animation (well, two) where I can cover all of the points with one finger, whereas 90 degrees later it's more like three fingers wide. Does a 3:1 ratio constitute a plane? Seems pretty similar to what I recall of the Milky Way's version, but I didn't do the same test for that one.

A : If you include the top two galaxies, them it does show a hint of an extension. Take them away and it really looks - at best - like a slightly squashed cloud. It's a heck of a leap to say this small squashed cloud poses any kind of challenge to CDM. This is something which requires a full blog post for, but if you want you can skip to section 2 in this (very long) one :
http://astrorhysy.blogspot.cz/2017/11/the-dark-side-of-galaxy-evolution-iv.html


What are you doing with the IRAM 30 m telescope ?
Q : So if I understand your Secret Mission, you're trying to work out something with gas-poor galaxies. What does "weak detection" mean?

"For comparison, we detected it with Arecibo in 5 minutes at the neutral hydrogen line, but only weakly. Looking for molecular hydrogen (using the much weaker CO line)."

I'm looking at this..
http://www.cv.nrao.edu/course/astr534/MolecularSpectra.html

A : The targets are ultra diffuse galaxies. Forget the gas for the moment : they're about the same size as the Milky Way, but optically 100-1,000 times fainter. Here's a lengthy summary (feel free to skip) :
http://astrorhysy.blogspot.com.es/2017/07/ultra-diffuse-galaxies-revenge-of-ghosts.html

We selected these particular targets by a complete fluke. By coincidence, a Chinese team (who I've never heard of before) did a search for UDGs in an area of the Pegasus group/cluster and published their catalogue... which just so happened to coincide with part of the AGES survey of atomic hydrogen gas. Three of our hydrogen detections are unambiguous matches to their catalogued UDGs, and that's what we're looking at with IRAM.

The amount of atomic hydrogen in these things isn't small. In fact it's comparable with (maybe even slightly larger than) the Milky Way. They are weak detections only because they're quite distant. About a hundred or so objects like this have been detected via 21 cm emission.

But the amount of molecular hydrogen is as yet completely unknown. An increasing amount of theoretical and observational evidence suggests that it's the (much colder) molecular gas which actually forms stars - the generally warmer atomic gas has too much thermal energy to collapse (this is simplifying hugely, but is basically correct).

That's where IRAM comes in. Because H2 doesn't emit directly, we use the CO line as a tracer. That requires higher frequencies, which necessitates a smoother, more accurate surface of the reflector. Hence telescopes operating at these mm wavelengths have to be much smaller and therefore less sensitive, plus (IIRC) the line is intrinsically weaker anyway. So whereas we can get a halfway-decent, though not brilliant, detection with Arecibo at 21 cm in 5 minutes, with IRAM at ~1mm we're spending ~5 hours on source. And so far neither myself nor the other team have got any detections. Just how significant this is really needs proper calibration to place a mass-sensitivity limit on the observations.

Provisionally, it looks like these objects have for some reason done a lousy job of converting their atomic gas into molecular gas. If this is correct (and it might not be !) then we have to work out why these objects manage to form a few stars, whereas other objects of similar total and atomic mass are so much more efficient. It's fun stuff, but complicated.


How do they know a Fast Radio Burst was five billion light years away ?
Q : Well, how ?

A : As I understand it, they get the distance by measuring the distance to the galaxy the burst appears to reside in :
https://public.nrao.edu/news/fast-radio-burst/
I suppose in principle the FRB could be a chance alignment with the galaxy and actually closer or further away. More data on different sources could answer this : if they all line up with galaxies, it would be very unlikely that they're all chance alignments.

There are different ways of measuring distances to galaxies, but at this range they probably did it by measuring the redshift of the galaxy. That gives how fast it's moving away from us, which in turns gives its distance by Hubble's Law :
http://astronomy.swin.edu.au/cosmos/H/Hubble+Law


How did this little gas cloud get so many metals ?
Q : [See this post for details : https://plus.google.com/u/0/+RhysTaylorRhysy/posts/WZqSfeXP1R3?cfem=1. In short, there's a mysterious gas cloud in the Virgo cluster, far from any galaxies but sharing their chemical composition. It also lacks an old stellar population but has plenty of young "O"-type stars.]

A : The idea here is that the gas cloud was stripped from a galaxy which has already been cycling gas into stars and back into gas for billions of years. So it begins its lonely wanderings through the ICM already enriched in metals. Much later, for no apparent reason, it suddenly decides to start forming stars. Those will also enrich the gas with metals, but by a comparatively negligible amount because there just isn't enough time.

The act of stripping itself could certainly have triggered some star formation. But this isn't necessary to increase the metal content, because gas within galaxies has already been cycled through several generations of stars so it's got plenty of metals already. The main problem is that billion year gap between when stripping started and the current episode of star formation. However, the freefall time (for gravitational collapse) of such clouds is highly sensitive to their initial radius - it could be as much as 1 Gyr if they were just a little bit bigger initially (I know because I already did this calculation for very similar clouds - http://adsabs.harvard.edu/abs/2016MNRAS.461.3001T section 3.2.1). So maybe we just happen to be witnessing the cloud at that particular moment.

I'd be surprised if the cloud turned out not to have any old stellar population at all. But the really weird thing for me is where the rest of the stripped material has gone - simulations produce clouds like this by gas stripping in different ways, but only as part of very extended structures. They haven't yet demonstrated any way to either rip out a compact cloud without a much larger gas feature, or produce some particular feature which is much denser and/or brighter than the rest.


Why didn't this simulated gas cloud collapse ?
Q : [See this post : https://plus.google.com/u/0/+RhysTaylorRhysy/posts/Buvd5QH9NR3?cfem=1]
Why didn't it collapse gravitationally and how did it start off as a sphere? Sound waves might play a role as well.

A : Locally, parts of the gas do collapse and form those little dense blobs that fly off. But the sphere as a whole has too much energy - each section of the cloud is moving at greater than the overall escape velocity, so it's doomed to explode. Since the velocity field is complicated, parts of the cloud collide with each other, forming locally gravitationally bound substructures.
Pressure waves are probably important for the small structures, but probably not for the overall cloud. Sound speed is equivalent to a crossing time of about 200 Myr, a bit longer than what's shown here.

I'll take this opportunity to answer the other part of the question I forgot about : it starts off as a sphere because we tell it to. The claim is that such clouds could remain stable, which assumes them to be spherical. We haven't yet investigated how such structures could form in the first place, but given the rapid disintegration it's a safe assumption that the initial geometry won't change anything.


Does this simulation assume the existence of a supermassive black hole ?
Q : [See this post : https://plus.google.com/u/0/+RhysTaylorRhysy/posts/HSMLj5J2Zr4?cfem=1]
Well, does it ?

A : Nope - on these scales, the mass of any supermassive black hole would be too small to have any impact.

Actually, I'm going to expand on my original answer to clear this up. It's a common misconception that supermassive black holes are incredibly important for their host galaxies. This is true, but only very provisionally. If the SMBH is feeding, it can generate so much energy that it may significantly affect star formation throughout the galaxy. But if it isn't, chances are it won't matter much. SMBH masses are typically very much smaller than their host galaxy masses, so they might affect the orbits of the stars nearest to them, but those further out won't notice at all.

There's an interesting exception to this : at least one "ultra compact" galaxy is thought to harbour a SMBH. In this case, the mass of the black hole is a significant fraction of the total mass of its host galaxy. One idea is that objects like this can form during galaxy interactions, where the central SMBH and some of the surrounding stars can be ejected into extragalactic space.
https://futurism.com/smallest-known-galaxy-supermassive-black-hole/


What's neat about this galaxy without any dark matter ?
Q : Why is this interesting ? Aren't there lots of ways to form galaxies without dark matter, e.g. by tidal encounters ?

A : This requires a bit of context. This cool galaxy was discovered that seemed to be lacking any dark matter, normally important in preventing galaxies from exploding. There's nothing in principle that makes this impossible in standard physics : as long as the mass density distribution and motions of the stars are correct, you can certainly get galaxy-size objects without dark matter. In this case the galaxy doesn't have that many stars and they're not moving very fast. Most galaxies this size have much faster stellar motions, suggesting an unseen component holds them together.

Ironically this becomes a bigger problem for theories which try to replace dark matter with modified gravity. Those would predict much faster stellar motions, in stark conflict with the observations.

There are ways of forming objects like this with standard physics, especially with tidal encounters (e.g. this paper). What's very interesting about this object in particular is that it's very extended but low mass. It should be much more vulnerable to tidal interactions than normal, dark matter dominated galaxies, but it doesn't show any signs of disruption. The stellar motions are also so low that there hasn't been much time for them to have completed more than one or two orbits, so it's surprising how spherical it is.

Naturally such an interesting and downright weird object was subject to intense controversy and criticism. In my opinion most of the early critiques just didn't wash (e.g. this one, with detailed refutals here and here). However the latest claim is that the distance estimate is wrong, and this is a very detailed, careful piece of work. In my opinion this object is no longer very interesting.


A galaxy surely can't have more than one black hole !
Q : [Regarding this article on the potential discovery of thousands of black holes in the Galactic center. I haven't read the original paper but I gather it's about stellar mass black holes and not directly related to the much more massive beastie, Sag A*]
It contains only a single but huge black hole. The galaxy would have been unstable if we had thousands of black violating basic theory of origin !

A : I haven't read the original paper, but I also don't see any fundamental problem with having multiple black holes. We know there are multiple stellar mass black holes within our Galaxy, for instance. The point at which the density of the black holes, given their motions, becomes so high that you would expect mergers to happen is one that demands a quantitative analysis, but it doesn't sound like the density here is much greater than a regular star cluster.

There's a few things to remember : - You only get all the weird relativistic effects when you're very close to a stellar mass black hole indeed (http://astrorhysy.blogspot.co.uk/p/q-a.html#blackholessuckohmygodthatsoundsdodgy).
- The distances between the stars (or black holes, whatever) is huge. My favourite analogy is that if the Sun is the size of a 2cm coin, the nearest star is a few hundred kilometres away. OK, in some cases the density is thousands of times higher, but it's still so low the chance of a collision or merger isn't increased all that much.
- Getting black holes to merge has actually been a theoretical challenge known as the final parsec problem (https://physics.stackexchange.com/questions/221111/the-final-parsec-problem).


Could you make a 3D version of this video, please ?
Q : [The video in question is this one with an explanatory blog post here]. Basically it's a fly-through of galaxies detected by a hydrogen survey in the nearby volume of the Universe, which I make as the survey catalogue is released. The latest, 2D version with the 100% complete catalogue can be found here. I'm working on a VR version but this is technically demanding for the software.]

A : Yes. Yes I can.
https://www.youtube.com/watch?v=YkR8E6x_2mo


Is this a picture of the Andromeda galaxy ?
Q : Specifically this one : https://plus.google.com/u/0/+franciscomartinez/posts/hNmEysbP5En?cfem=1

A : No, it's clearly not. It's a completely different galaxy, NGC 3990 (https://en.wikipedia.org/wiki/NGC_3190). This unfortunate mistake has been widely reproduced on various picture-sharing websites, but trust me, this isn't M31 - I once did a study of environmental region of this object.


What happens to turbulent clouds in a vaccum ?
Q : [Regarding my paper on turbulent spheres, for which I'm still working on the full blog post. We've got these dark clouds of hydrogen and we simulate their evolution under the assumption that their otherwise puzzlingly high velocity dispersions are due to internal turbulent motions]
Isn't pressure negligible in a vacuum? The exception being at "solid" celestial objects such as stars and planets. Also, is it wrong to essentially consider intergalactic space an open system? If it can be considered an open system, then the laws of thermodynamics do not apply.

A : The model we considered intrinsically requires that the clouds are not in a vacuum. Galaxy clusters are known from X-ray observations to contain an extremely hot but low density gas - this is the intracluster medium. The model we tested was based on the idea that the internal gas pressure of the clouds themselves (acting to cause them to expand) was in balance with the external pressure from the intracluster medium (acting to cause them to collapse). So there's no vacuum anywhere in this model, just regions of different densities and pressures (albeit very low densities in some parts).
You can watch what happens to one of the clouds here :
https://plus.google.com/u/0/+RhysTaylorRhysy/posts/Buvd5QH9NR3


What's the data set used in this pretty movie ?
Q : Eerily reminiscent of a uniquely catastrophic hangover I once endured... I mean this one : https://plus.google.com/u/0/photos/photo/103860147871348309151/6557934896540978098

A : Here's one I made earlier, displayed in a slightly more intuitive way :
https://plus.google.com/u/0/+RhysTaylorRhysy/posts/aNvtgAuPNwD
See also comments therein. Basically this is all-sky HI data displayed as a spherical volume, with some camera trickery. In this one I did some more complex processing to generate the colours rather than just basing them on the intensity of the data as is usual. The camera motion is a simple linear trajectory right through the centre, but it has an extremely wide field of view to give the heavy distortions near the edges. Much more of this can be seen here, with the goal being to get people to think about why we choose to view data in certain ways and how our perception alters our understanding.


Can you shroud a galaxy ?
Q : [This was the question asked - and partially solved ! - on the following blog post which describes a program to calculate how galaxies would appear if their stars were masked by Dyson spheres : http://104.197.48.230/sunscreen/]

A : I'd be amiss not to mention this paper, which has already tried searching for "shrouded" galaxies by means of the Tully-Fisher relation. The TFR is an observed relation between the rotation speed of a galaxy and its total baryonic mass... which until not so long ago was a very good proxy for optical brightness. The relation has a very small scatter. If a galaxy's stars were largely enshrouded, this wouldn't affect measurements of its rotation speed (which are normally done by measuring the gas content) but it would affect the wavelengths at which they radiate, shifting them to the infra-red. So galaxies ought to be shifted off the standard TFR. Of the ~1400 searched galaxies, no good candidates were found.
https://arxiv.org/abs/1508.02406

But this has become much more complicated in the last few years with the discovery of very large numbers of ultra-diffuse galaxies, which are comparable in size to the Milky Way but very much fainter. Few measurements of their rotation exist because they're so faint (and most don't seem to have much gas), but of those that do, there's some evidence that they deviate from the TFR.

Bottom line is it could potentially be very hard to differentiate enshrouded galaxies from naturally-formed objects. We just don't know all that much about the galaxy population yet (of course I suppose all these UDGs could be enshrouded galaxies, but that's the classic "aliens explain everything" syndrome).


So what's the difference between "enormous dwarf" and "small regular sized"?
Q : [This was prompted by this article on the discovery on Antila 2, a dwarf galaxy in the Local Group : https://phys.org/news/2018-11-enormous-dwarf-satellite-galaxy-milky.html]

A : Whether a galaxy is a dwarf or giant is typically defined by its total brightness. The historical reason for this is that total amount of light emitted is relatively easy to measure compared to physical size, and most galaxies seemed to have roughly the same number of stars in any given extent. So if you had the total brightness, you could automatically get a pretty accurate estimate of the size.

Over the years, many more faint galaxies (such as this one) have been discovered that don't fit this relation : they are very large, but have very few stars. They are "enormous" in the sense of how extended they are, but "dwarfs" in terms of mass.

This prompted a follow-up question :
Q : Small in mass? Or small in luminosity?

A : For the stars, mass and luminosity are basically equivalent. So few stars = low mass = faint.

Where it gets more complicated is for the galaxy as a whole. A rough working definition of galaxy would be a system of stars and/or gas bound together by dark matter, which is usually much more massive than the visible stuff. The stars are usually just the tip of the iceberg.

There's an ongoing controversy for really faint, extended galaxies as to just how much dark matter they have. Most people seem to think it's not all that much. So they are faint, don't have many stars or much stellar mass, and don't have much total mass. They're just very spatially extended, huge dwarfs.

But a few people think that at least some of these "ultra diffuse" galaxies have as much dark matter as giant galaxies. They'd be very faint, with few stars and not much stellar mass, with their stars very spread out, but with a very high total mass - sometimes they're called "crouching giants". So the usual definitions of "dwarf" and "giant" wouldn't work for these at all.

Antila 2 is so small that it's likely a dwarf galaxy - ultra diffuse galaxies tend to be about the same size as the Milky Way, which is much larger than Antila 2.


COSMOLOGY

What is dark matter ?
Q : I want to ask is there any dark matter or dark energy in the universe because nobody had ever seen this or seen colliding this with any matter in the space ? Is all the darkness in the space dark matter ?

A : No, dark matter isn't the darkness in space. That's just an absence of light. Dark matter is believed to be completely transparent. However, dark matter is thought to fill much of the space between the stars. The exact nature of dark matter is still a mystery. It is believed to be some as-yet unidentified particle which a) doesn't interact with normal matter at all except through gravity and b) is probably collisionless, meaning it doesn't even interact with itself.

Some models of dark matter suggest that dark matter particles may, occasionally, interact with each other and produce a burst of energy. If that's the case, we might be able to detect gamma rays where the density of dark matter is particularly high. No convincing evidence of this has been found so far.

Lastly, there are still alternative theories of gravity which could do away with dark matter entirely. In my opinion, these aren't very likely and the evidence for dark matter's existence is pretty strong. However, we won't be certain until we detect a particle directly. There are several experiments underway that are trying to do just that.

More details here :
http://astrorhysy.blogspot.cz/2015/02/seven-misconceptions-about-dark-matter.html


Does dark matter exist ?
Q : How can we prove dark matter exists ?

A : Right now, we can't prove it exists. We can only say that all our observations are consistent with dark matter : e.g., galaxies are rotating too fast so should fly apart without dark matter; galaxies in clusters are moving so fast the clusters should fly apart without dark matter; gravitational lensing (light being distorted by massive objects) is consistent with the idea that there's matter we can't see (in amounts consistent with that predicted by the other methods).

The only way to prove its existence once and for all would be to directly detect a particle with the properties that could explain all the observations. This is very hard to do since dark matter only interacts with normal matter very rarely (except through gravity), if it ever interacts at all. Nonetheless, several experiments are underway to try and do just that, e.g. http://lux.brown.edu/LUX_dark_matter/Experiment.html

The main alternative to dark matter is to modify our theory of gravity to fit the observations. However, finding a theory which works in every situation (e.g. from masses of planets right up to those of galaxies; from nice stable orbits to the chaos that is interacting galaxies) is proving to be very difficult. In my opinion, dark matter is a much more likely explanation.
http://astrorhysy.blogspot.cz/2015/02/seven-misconceptions-about-dark-matter.html


Can we use better telescopes to see the beginning of the Universe ?
Q : Will there come a time, when we could see all the way back to the beginning of the universe with much more powerful telescopes than the current Hubble telescope. Or are we pushing the bounds of technology as we know it, now ?

A: Complicated topic, to say the least. So, bear with me. :) It's true that we can't see galaxies beyond a certain distance because of the expansion of the Universe. However, because the Universe is so large, we're also looking back in time.

This isn't such a big deal when looking at nearby galaxies like Andromeda which are "only" a few million light years away, i.e. we're seeing them as they were a few million years ago. That's just not enough time for anything really significant to have happened to them (large spiral galaxies take a few hundred million years to rotate even just once).

But we can see galaxies very much further away than that - billions of light years away, or to put it another way, galaxies as they were billions of years ago. And that is long enough to see significant changes. But more than that, we're seeing them when the Universe was significantly smaller than it is now.

Go to high enough distances and you start seeing the Universe when it was so young that stars did not exist. At some point you reach the fireball of the Big Bang itself. The most distant thing we can see is called the "surface of last scattering" - the time when the Universe cooled enough that it became transparent and light could move unhindered through it. This is thought to have been at around 300,000 years after the Big Bang.

Ordinary telescopes will never be able to see beyond that point because the entire Universe was a hot, opaque fireball until that point. Gravitational waves (which we haven't detected yet, but we do keep trying) would allow us to "see" past that point, since they're not light waves and wouldn't be affected by the high temperature.


Is our Solar System expanding along with the rest of the Universe ?
Q... well, OK, comment : Our solar system is expanding and moving away from the Sun, while at the same time the Sun is moving towards Andromeda. Too bad, we won't be around to see the "union".

A : No, the Solar System is not expanding... at least not yet :
http://www.universetoday.com/107142/is-everything-in-the-universe-expanding/

On small scales, yes, our perspective is changing because galaxies can be  gravitationally attracted to one another, so their motions aren't what you'd expect if the expansion was perfectly smooth. However, on the very large scale, these"peculiar motions" are negligible.


Dark matter - wrong name ?
Q : Could or should we rename dark matter as augmented gravity or accentuated gravity?

A : No, because dark matter is a specific idea that there is really extra mass that we cannot detect directly. MOND (Modified Newtonian Dynamics) is the most popular theory proposed to modify gravity in such a way as to mimic the effects of dark matter.


Why didn't the young Universe collapse ?
Q :  If dark matter was prevalent in an earlier Universe what has prevented a general collapse of the entirety of matter ? Equilibrium ? Why did structures form ?

A : The early Universe was hot, so there was a pressure pushing outwards. I don't know why it was hot - I'm very much hoping that someone can give me a good explanation of that - except that if it hadn't been hot, it would indeed have collapsed, and we wouldn't be here to discuss it.

If the Universe had started off with exactly the same density of matter everywhere, then nothing much would happen. As it expands, the density would just drop (or if the mass was high enough, re-collapse). But that appears not to be the case. All matter had some random initial motions, so the collapse wasn't the same everywhere. While there problems in the details, simulations have done an excellent job of reproducing the observed structures (networks of filaments and voids) just through slight variations in the initial density and motion of the dark matter particles.

Dark matter particles are thought to be collisionless, so particles just move through each other. This means you don't get a runaway collapse of the dark matter - it re-expands a bit after the collapse. Gas, however, does collide with itself - but the other complex processes at work prevent star formation from occurring in one massive burst.


Was gravity always present ?
Q : If it all came from a very small object (in size and time) what force contrasted gravity on such a dense / compact initial "object" or was gravity not present at the beginning and only came into existence at a later stage, much like we imagine dark matter and dark energy to have had different "levels" at different times.

A : Gravity was and always will be present*. I am not quite sure what you're asking, but the basic view of the early Universe is something like this : the very hot, small early Universe expanded extremely rapidly. Gravity continuously acts to slow the expansion down - like throwing a ball into the air. For a long time it was very uncertain if the expansion speed or gravity would win, but current observations strongly favour that expansion wins. There was simply too much expansion speed for gravity to ever bring it to a halt (like throwing the ball upwards at escape velocity).

However, the early Universe wasn't quite uniform. Density variations mean there is still some localised collapse into what became the filaments of galaxies we see today.

* Except possibly in the first few nanonseconds of the Universe's existence, when all forces were unified.


It all done just don't make no darn sense !
Part of a larger, unproductive discussion. I include this here as an example of someone who thinks their ignorance trumps all of modern science.
Q : Rhys, so "expansion" is a force that was, is and will be stronger than gravity, also thanks to dark energy, and gravity only acts whenever one need to explain a more common object like a galaxy or a (extra)solar system [I had clearly explained to the inquirer that this is nonsense]. Nebulas are oddly-shaped but still produce round objects [I explained that one too]. Dark matter greatly increases gravity / attraction but not enough to counter expansion [Yes, and that]. Galactic black holes don't grow over half percent of the total mass, although it's projected that they will engulf the whole galaxy [Sigh...]. It all so totally don't make any sense :-)  Let's revisit this whole concept when we meet some ET !

A : Nope. Sorry, but astronomy is not a soft science. Thousands of people have run thousands of simulations to explain the structures of nebulae, galaxies, and the Universe as a whole. If you do not understand the principles behind these, let alone the physics and the numbers involved, you are no more entitled to pronounce judgement upon them than I am to comment on, say, neuroscience. Mathematics matters. You can't ignore it because you don't agree with the end result.


Is the expansion of the Universe really accelerating ?
Q : How firm is the data indicating that the expansion of the universe is accelerating? Is this something that with new data or better analysis might be found not to be the case 20 or 30 years from now ?

A : I'd say it's pretty solid. Maybe not 100% certain, but good enough for government work. :)

The evidence that the expansion comes from supernovae explosions - specifically, type Ia supernovae. These happen when a white dwarf gains enough mass from a companion star that it re-start fusion. The point at which this happens is thought to only depend on mass so the resulting explosion should always be of the same energy. Knowing that energy, we can work out the distance to the supernova pretty accurately.

The benefit of using supernovae is that we can measure both their redshift and distance even in distant galaxies. Because light travels at a finite speed, the further away a galaxy is, the younger (and therefore smaller) the Universe was when light left that galaxy. After accounting for this, the supernovae data indicate that the acceleration of the Universe is increasing - which is something nobody was expecting.
http://stardate.org/astro-guide/btss/cosmology/dark_energy

There was an alternative interpretation of the supernovae data : we live in a void, a region of the Universe which happens to have much less matter than the rest. Since there's less matter inside the void, there's less gravity to slow down its expansion, which would look like acceleration.

This led me to a rather interesting paper-chase. Here's an original press release from 2009 categorically stating that we don't live in such a void :
http://phys.org/news152374990.html
And here, on the exact same day, is another wesbite (mis)interpreting this to mean that we DO live in a void and dark energy is wrong !
http://archive.news.softpedia.com/news/Void-Theory-to-Replace-Dark-Energy-Hypothesis-103226.shtml

More recent, independent evidence futher supports the notion that we don't live in a void. As far as we can tell, the expansion really is accelerating.

http://www.pbs.org/wgbh/nova/blogs/physics/2012/04/whos-afraid-of-the-dark-alternatives-to-dark-energy/


Eddie's in the space-time continuum, cap'n !
Q :  If space is the thing that is moving, why is it the Milky Way and Andromeda galaxies can not simply go under or over one another. Why is it that in the next billion or so years, the two will collide ?

If space moves, than that would mean that there are "currents" in space. Why can't a current go over another one, like air does ? There are different levels of clouds, at various levels, each moving in their own direction, separate from each other, but also together, to form the cloud formations we see.

Why can't space be the same way ? Or is it ? Where there are currents in space guiding the various solar systems and galaxies around.

A :  The standard view is that space is expanding in all directions equally. This lets us predict how galaxies should be moving. When we see differences, we assume it's because the galaxies are moving independently of the expansion of space, usually because the gravity of other galaxies is affecting them. That's a bit like a boat moving against a current.

But, the idea that space itself could be flowing in different directions is super interesting, and really it's above my pay grade. But I'll try anyway. :P

Gravity is curved space. Here's a very nice demo (first 2-3 minutes are enough, but it's worth watching in its entirety) :
https://www.youtube.com/watch?v=MTY1Kje0yLg

So, if space was severely distorted enough to carry galaxies along like a flow, that would basically be the same as gravity, and we'd see all kinds of funky gravitational lensing effects :
http://imagine.gsfc.nasa.gov/news/grav_lens.html

I really don't know if it's theoretically possible - or even makes sense - to talk of curved space without gravity. But it's going to be pretty tough for Andromeda and the Milky Way to avoid each other. Like two sumo wrestlers rolling around on a trampoline, things are gonna get ugly...

Both galaxies have mass. so both have gravity. Lots of gravity (or, big dips in the spandex sheet if you prefer). Since it seems they're heading more or less directly toward one another, their gravity is only going to make things worse - they'll get faster and faster until they collide.


Q : A very simple question from all of you, is there any end of space or it is infinite ?​

A : SIMPLE ?!?!

Nobody knows. However, it cannot be both infinite in size and have existed forever, because the sky is dark at night. See this exceptionally wonderful 4min video :
https://www.youtube.com/watch?v=gxJ4M7tyLRE
In principle, it could be infinite in space but not in time. Which has some crazy implications - keep going long enough in any one direction, and you'll eventually encounter exact copies of yourself :
https://www.youtube.com/watch?v=Al9EyNoCsRI
You might wonder if the Universe isn't infinite, does it have to have an edge ? Not necessarily. Space could be curved on a very large scale, so that if you fly off in one direction for long enough, you end up back where you started (like flying around the world).
https://www.youtube.com/watch?v=_k3_B9Eq7eM
Personally I like this idea because I can't imagine how a Universe becomes infinitely large in a finite amount of time, or what the edge of the Universe would be like if it was finite in size. In this view, the Universe is both finite in size (but still really frickin' large) and time and has no edge.

However, it still means that at some point in the distance past the Universe went from a state of complete non-existence to existence, so in that more woolly philosophical sense, it doesn't necessarily avoid all the infinities.

UPDATE : I give a much more complete explanation here : http://astrorhysy.blogspot.cz/2015/04/the-most-astounding-fact.html


Q : What's the latest progress in the recognition of dark matter ? [April 2015]

A : There have been a few hints of direct detections of dark matter, most notably from an instrument on the ISS. The latest claim was as recent as September 2014 :
http://news.discovery.com/space/mystery-signal-could-be-dark-matter-hint-in-iss-detector-140918.htm
However, this was a very tentative "result" and even the press release states, "We have not found the definitive proof of dark matter." A similar claim was made in April 2013 : http://www.space.com/20490-dark-matter-discovery-space-experiment.html
I can't find any follow-up to this; it's pretty much fallen by the wayside so I assume the result was never confirmed.

Even more recently, the Fermi gamma ray observatory discovered gamma rays from a dwarf galaxy. This is also a possible signature of dark matter particles :
http://www.astronomy.com/news/2015/03/unexpected-gamma-ray-emission-from-dwarf-galaxy-may-point-to-new-understanding-of-dark-matter
There was also another tentative claim of detecting dark matter in the Milky Way in the same way :
http://arstechnica.com/science/2014/04/fermi-telescope-detects-signal-that-could-be-annihilating-dark-matter/

The problem is that all of these are only claims that the result is compatible with dark matter. They're intriguing, but no-one is convinced by them. The search continues.


Does the Universe even have an age ?
Q : Why do people try to give an age to the universe when time is an entirely relative expanded fabrication? The age of a black hole is not the same age as the rest of the universe because the passage of time is not the same and you can't use expanded time as a valid measure of time anyway.

A : Time does vary depending on speed and the gravitational field you're in. However, these effects are very small unless you're going close to the speed of light or near (as in within a few kilometres) the event horizon of a black hole.

In the standard model, space expands uniformly. Away from any stars or galaxies, time passes at the same rate everywhere. You get some weird local deviations around black holes, but that's about it. You also get deviations wherever you have any mass at all, but so small that it's basically negligible. It doesn't really make much difference to the age of the Universe if time is passing 0.0000001% more slowly near a star than in deep space.

If you go back far enough, the Universe was so small that it was filled with dense matter, but here the gravitational field would have been more-or-less uniform everywhere. So time was still passing at about the same rate everywhere, because gravity would have been strong everywhere. This is "proper time" in relativity : time time experienced by an observer accounting for their velocity and gravitational field. Since everywhere was experiencing (pretty much) the same gravitational field, proper time didn't vary throughout the Universe. So it absolutely makes sense to define an age of the Universe.

Granted, black holes do cause problems. So does the singularity in the standard model at the Big Bang. Light is also weird, since the proper time for a light beam is always zero no matter how far it travels. To a photon, everything happens at once. And I have no idea what the philosophical implications are of that.


Does the Universe even have an age if space and time are expanding ?
[Part of a larger, unproductive discussion] Q : If space is expanding, then time must be expanding as well. So how can we even say that the Universe has an age ?

A : I am no expert in relativity, but here is my naive, non-mathematical, simplified interpretation. Short version : the cosmological principle states that space is and looks the same everywhere. So whatever passage of time we've experienced, the rest of the Universe has too.

Imagine that early in the Universe you scatter a bunch of stopclocks at random positions. Some of them will end up in completely empty space. Some will end up inside galaxies. A very, very few will be unlucky and fall into a black hole.

Let's further imagine an observer with the magical ability to travel instantaneously from one clock to another. We'll have them spend 14 billion years just watching one of the clocks which is in empty space, because we'll also imagine that they are seriously dedicated to looking at clocks and immortal. We shall also give our observer an enormously powerful telescope with which they may examine more distant clocks.

Our observer spends 14 billion years just watching this one clock in empty space, far from any galaxies. Finally tiring of this, they use their telescope to look at a clock inside the nearest galaxy. They will see that that clock does not quite read the same as their clock, since the galaxy has gravity and probably some motion relative to the observer (but not very much, since it's the nearest galaxy). But the difference will be small, no more than a few months after 14 billion years.

Then they look at a clock that's falling into a black hole. Here they will see that this clock reads quite different to their own clock, because of the black hole's gravity. But they realise that there are hardly any black holes, which only significantly influence time over a very small region. So they conclude that black holes haven't affected the age of the Universe at all, just very very small parts of it.

Things get much more interesting if they look at clocks in very distant galaxies. One obvious reason is that light takes a long time to travel across the Universe, so distant clocks viewed by our observer will report that less time has passed than our observer's clock. Of course, that doesn't mean that less time has really passed for the other galaxies, just that our observer can't see what's happening in those galaxies right now.

More interesting will be what happens if the observer watches one of those distant clocks for a little while. Because of the strong apparent motion of the galaxy with respect to the observer (near light speed), they will see the seconds tick by more slowly than on their own clock. This effect has actually been observed using supernovae. http://www.astro.ucla.edu/~wright/cosmology_faq.html#TD

But, what if our observer now travels instantly to one of those distant clocks ? If they are clever, they will know the Universe is expanding and so compensate for this. They will find that all of the distant clocks - except those very few which are near black holes - also read about 14 billion years. When they look back to the original clock, they will also see that time appears to be ticking away more slowly. All observers think the other clocks are ticking more slowly than their own, because of the apparent motion due to expansion of the Universe. But this is just a choice of reference frame. It is not that there has really been a difference in elapsed time for each galaxy and/or that different parts of the Universe have aged different amounts (they can't all be slower than each other, after all - that would make no sense at all !). It is different to the Twin Paradox, where twins travelling on spaceships moving at different speeds really do age differently.

The standard model is that the Universe has expanded uniformly everywhere. So all observers experience (almost, allowing for small local deviations) exactly the same amount of time. It's only that they perceive other observers to have experienced a different passage of time. That space is expanding doesn't make it wrong to estimate the age of the Universe anymore than it means that time hasn't elapsed.

Another point is that the expansion is negligible on small scales like star systems, and only important on very large scales like distant galaxies. That's what (I think) it makes more sense to think of this type of time dilation as being a perceived time dilation rather than a real one.


Can the speed of the acceleration of the Universe exceed the speed of light ?
Q : If space is expanding at an increasing rate, will it eventually grow to be faster than light ?

A : Unfortunately, it's more complicated than that. My highly simplified answer would be, "no, but you can think of it that way." In one sense, expansion is already happening faster than the speed of light, but in another, it isn't a speed at all - it's a speed per distance. Or not even that, since distance itself is changing - thinking of it as speed may be fundamentally flawed. Motion through space is not the same as motion of space. It's "travelling without moving", as they say in Dune. Perhaps it's best to think of the expansion as an apparent motion rather than a real motion.

A couple of links which provide more complete answers :
http://www.askamathematician.com/2014/03/q-how-can-the-universe-expand-faster-than-the-speed-of-light/
http://curious.astro.cornell.edu/about-us/104-the-universe/cosmology-and-the-big-bang/expansion-of-the-universe/616-is-the-universe-expanding-faster-than-the-speed-of-light-intermediate?


When an object falls past the event horizon of a black hole, how do we see it ?
Q : Around a black hole is an imaginary boundary called the event horizon. Now even light cant escape after crossing that boundary and if an object passes through it the object seems to be not in motion for an observer. Now because light cant come back how is it that we see an object passing the event horizon ?

A : The simple answer is that we can't. Once it reaches the event horizon, it's gone forever - we won't be able to see it any more. The much more complicated answer that deals with time dilation and whatnot is here :
http://math.ucr.edu/home/baez/physics/Relativity/BlackHoles/fall_in.html?


If space is flat, if we keep travelling will we end up in another dimension ?
Q : If space is flat, would we be able to go straight, in any direction and escape this dimension of space and occupy another dimension ?

A : https://www.youtube.com/watch?v=_k3_B9Eq7eM? - that's the best answer I know to that.

Further discussion raised the closely-related question of whether the Universe must have an edge, given that it is flat.

All the observations show that the Universe is perfectly flat. In the Big Bang model, this means the density of the Universe had to be exactly the right density of matter to begin with. A tiny amount of too much matter would deform space so much we would be able to observe its curvature today, if it hadn't caused to already re-collapse. A tiny amount too little and it would have strongly negative curvature, and everything would have expanded so fast that stars would never have been able to form.
http://archive.ncsa.illinois.edu/Cyberia/Cosmos/FlatnessProblem.html

Personally I'm not sure this is really a problem. If a specific density is required to explain the observations today, then darn it that means it had that density. But the most popular explanation is inflation - the Universe was initially strongly curved, but expanded very rapidly. So on really, really large scales - much larger than we can observe - the Universe is indeed curved. It's like walking down the street : you don't notice the Earth is curved, but you would if you walked all the way around it.

IIRC a torus-shaped Universe would also appear to be flat since triangles would still have angles that add up to 180 degrees. So that would get you a "flat" Universe without an edge (I think), which is nice. I don't know how we'd test for this possibility. [Thanks to Ethan Siegal for confirming that this is the case. We can't test for this possibility except by travelling right around the torus - i.e. we'll never know if this is case or not. Bummer.]
Or heck, maybe the Universe does have an edge. A detailed simulation has been run exploring what that would look like : http://www.futurama-madhouse.net/fashion/315cowboy.jpg?


Do we have to point telescopes away from the Galaxy towards the direction of the Big Bang to measure the Cosmic Microwave Background ?
Q : I do have a question about when a telescope is trying to measure the CMB : is the telescope pointing away from the galaxy ? Because there would be some distortion if it was pointing towards our sun, but the telescope can orbit the sun to look "behind it" But it cannot orbit the galaxy to avoid the distortion it would create to try to look through it to the other side of it, did we just get lucky that where we think the Big Bang originated from is easily seen from our place in one of the arms of the Milky Way ?

A : Planck measured the CMB across the entire sky. It's very difficult to get useful cosmological information from the "zone of avoidance" - the bit where the light from our Galaxy dominates over more distant sources. It can be done, but you have to be very careful. I assume that most of the CMB analysis is done on regions looking out of the plane of the Galaxy, where the sky is much less crowded.

The Big Bang originated everywhere, it was not an explosion occurring at one point in space. Rather it was the expansion of all of space, simultaneously. So we can see the CMB in all directions.

The advantage of being in a spiral disc galaxy is that although the sky is awful for cosmology in some directions (looking through the disc), it's much better looking out of the plane. If we were in an elliptical galaxy we'd be screwed because there'd be a lot of foreground stars in all directions (though much less gas and dust).


Can you really rip a hole in space and time ?
Q : In science fiction, you read about "ripping a hole in space-time"... Yet, is there anything realistic in the idea that space-time can be "ripped" or "torn" ?

A : I remember in Brian Greene's The Elegant Universe he concludes (after a very long and complicated discussion) that spacetime can tear according to string theory.

I suppose you could regard a singularity (the center of a black hole, where spacetime has infinite curvature... whatever that means) as a rip. Most people regard them as a problem for general relativity since infinities cause all kinds of horrendous problems for the equations. The suspicion is that the're not really infinitely dense, although AFAIK no-one has a good justification for this besides "infinity sucks".

The answer is a definite maybe : http://www.askamathematician.com/2009/11/q-if-black-holes-are-rips-in-the-fabric-of-our-universe-does-it-mean-they-lead-to-other-universes-if-so-then-did-time-begin-in-that-universe-at-the-inception-of-the-black-hole-could-we-be-in/


Could dark matter just be normal matter that exists in another Universe which only interacts with ours through gravity ?
Q : I've got some reasons from the Q'ran to believe this, apparently.

A : No. Firstly, it's very unlkely to just be normal, baryonic matter. Microlensing experiments have ruled out dark matter as being ordinary planets and/or black holes (see http://astrorhysy.blogspot.cz/2015/02/seven-misconceptions-about-dark-matter.html point no.4.).

Having the dark matter be part of another universe is a very drastic solution to a problem that we can solve just by postulating the existence of a new, albeit weird, particle. Neutrinos are pretty close to what we need the dark matter particle to do, so invoking a new particle is a lot simpler than invoking a whole new universe.

Also, baryonic matter in another universe would have all the problems that baryonic matter would have in our Universe. When galaxy clusters collide, their gas gets stuck in the middle but the stars and dark matter keep going, exactly as models predict. If the dark matter was actually ordinary gas, it should still get stuck in the middle - it being in another universe wouldn't prevent that.


How fast are we moving away from the Big Bang ?
Q : I take it the matter of our galaxy is still moving away from the origin of the Big Bang. Does anyone know how fast ?

A : The Big Bang wasn't an explosion exactly, it was the rapid expansion of space. ALL of space - it occurred in all places simultaneously. In that sense it's still happening now. It didn't happen in one spot we can move away from. Our motion relative to the point of the Big Bang is zero, but because all of space is expanding, our motion to distant galaxies is enormous.

There's a complication in that because galaxies have gravity they pull each other in different directions, away from this uniform expansion. We can measure how fast we're moving away from what we'd expect if this didn't happen, and the answer is 627 km/s. To simplify slightly, this is how fast we're moving away from where the Milky Way first formed - but since the Big Bang happened everywhere, there's no getting away from it.
https://en.wikipedia.org/wiki/Cosmic_microwave_background#CMBR_dipole_anisotropy


Can we see the beginning of the Universe, or at least close to it ?
Q : I thought scientists could already see back to the first trillionth of a second after the Big Bang ?

A : Nope. See http://astrorhysy.blogspot.cz/p/q-a.html#seebb, and also http://www.space.com/20330-cosmic-microwave-background-explained-infographic.html. So far as I know we currently have no direct observations of anything that came before the CMB -"only" theory. But that's theory in the strict scientific sense of the word : that is, models which are consistent with many other observations of what came next.


Does the inertia of the galaxies affect the cosmological expansion ?
Q : So the expansion of the Universe seems to be slowing down (or accelerating nowadays, but the result is the same). If we take the balloon analogy, we have pebbles on the surface, but the surface is expanding at a variable rate, so the inertia of the pebbles should them bend the balloon surface inward (now outward), giving us back the nice trampoline surface analogy for gravity.

So why is this theory of gravity not working? Because if it was, someone else would have thought about it in a century. It seems to my older self that it wouldn't take all of gravity into account, nor that inertia works that way, but I'm not really sure.

A : Like all analogies, the balloon model of the expanding Universe is very good in some ways, but not perfect. When you want to make a prediction, it's better to rely on the maths.

To answer the question, that might work if you inflate the balloon very rapidly and then stop. A slightly simpler analogy (which can be easily demonstrated) would be a set of keys (or any small, heavyish object) on a towel. If you raise the towel slowly, the keys just sit there, deforming the surface. If you raise it quickly and then stop, the keys will shoot up before landing and settling back down in equilibrium.

The analogy with spacetime only works (as I understand it) in the slowly moving, maintaining equilibrium mode. You don't get any extra deformation due to inertia - it's only mass that causes deformation. If all of spacetime were to stop expanding right now, we wouldn't get any sort of negative curvature developing.

I would cautiously suggest that a better analogy for this purpose might be to imagine stretching a flat rubber sheet (keeping it flat on the ground) rather than a balloon, with a pebble glued to the very centre. In this case it doesn't matter how fast you stretch the sheet or how rapidly you vary the motion - the pebble will always remain in equilibrium.


Does ordinary matter become repulsive at high relative velocities ?
Q : Maybe this could be a way of explaining why the expansion of the Universe is accelerating.

A : Not as far as I know - mass ALWAYS acts to slow things down. How much it does this is debated. There are many alternative theories of gravity to the standard Newton/Einstein models, which have gravity decreasing in different ways. Such models have been proposed as alternatives to the standard model of dark matter. I'm not aware of any theories which say it should become repulsive, but it wouldn't surprise me if someone had tried this.


Is there any way to see back to the Big Bang ?
Q : Is it just a matter of inventing better telescopes and technology ? Could we ever get a video of it ?

A : Unfortunately, for the foreseeable future this is not possible.

When the Universe was young it was very hot and very dense, - an opaque soup of gas that no light could penetrate. This is called the surface of last scattering, or the Cosmic Microwave Background. Frustratingly, it stops us from seeing anything that happened earlier than about 300,000 years after the Big Bang.
http://www.space.com/20330-cosmic-microwave-background-explained-infographic.html Seeing past it using electromagnetic radiation is literally impossible. There are two known exotic technologies that could see past this irritating barrier : neutrinos and gravitational waves.

Neutrinos are almost massless particles which pass through just about everything. They can be detected, but it's friggin' hard. Like, seriously friggin' hard. The IceCube telescope in Antarctica uses a cubic kilometre of ice and only detected 37 of the little buggers in three years, they're that stealthy.
https://icecube.wisc.edu/about/overview

Gravitational waves would also be a possibility but these haven't been detected at all yet. If I recall correctly, the resolution of gravitational wave detectors will be very low, so making images isn't really a possibility. Video is not likely in the foreseeable future. Science is a harsh mistress.


As the Universe expands and cools will we see changes in its chemistry that could be significant on a large scale ?
Q : As the universe continues to expand and cool, I wonder if there could be more global phase changes yet to come, such as molecular hydrogen condensing into ice, or even something far more fundamental which might promote contraction of the universe in (another episodic) big bounce.

A : Probably not. Molecular hydrogen is generally much warmer (up to ~1500 K though can be ~100 K) than the CMB (3 K). So making the background temperature at most 3K cooler won't have much effect (though you do get weird quantum effects near absolute zero). Also, the expansion of the Universe doesn't have much effect within galaxies. It could make a difference on larger scales but it would cause the density to decrease, making it more difficult for any further condensation to occur.


Do you have a graph showing the size of the Universe over time ?
Q : Can you put your hands on a graph that shows the relative universe size (as a function of "z") over time? I.e., how long ago was z = 1, z = 2, z = 3 and z = 6 ?

A : I was all set to re-learn the Friedman equations when I stumbled on this - which seems to be exactly what you were asking for :
http://oneminuteastronomer.com/6911/how-big-is-theuniverse/
A more detailed version, emphasising that we don't know how the size changed during its first moments very well because of inflation :
http://www.forbes.com/sites/startswithabang/2015/12/26/ask-ethan-how-big-was-the-universe-when-it-was-first-born/

To see what a redshift of 1 means in real numbers, use the cosmology calculator :
http://www.astro.ucla.edu/~wright/CosmoCalc.html


Is there any evidence for the expansion of the Universe other than redshifts and Cosmic Microwave Background radiation ?
Q : Well, is there ?

A : This is a big question so I decided to give it a pretty thorough answer. The very short answer is "yes". The long answer is also "yes", but with many interesting caveats.

First, it's worth noting that when the Square Kilometre Array is constructed, it may be capable of measuring how the redshifts change over time. In principle you could watch a single galaxy and see its redshift change as the Universe expands. In practise because galaxies aren't just moving with the overall expansion, you need to measure a huge number of galaxies with very high precision over about ten years to see a meaningful change. This is probably about 30 years away, but would be as close to absolute proof of expansion as is possible to get.
http://arxiv.org/abs/1501.03822

Existing supporting evidence that the Universe is expanding includes the Tolman test, the time dilation of supernovae light curves, the anguluar diameter test, and a few others. For a review, see : http://arxiv.org/pdf/1501.01487v1.pdf

The two main arguments are the Tolman test and the time dilation of supernovae. The Tolman test says that if the Universe is really expanding, the surface brightness (brightness per unit area) of galaxies should vary with redshift - if galaxies have the same distribution of stars at different redshifts. The problem is that we don't expect this to be the case since galaxies interact and merge and star formation rate varies strongly over time. So it's difficult to account for this, and when people try they get different results. It's probably fair to say that so far the results are not inconsistent with expansion, and they certainly don't rule it out.
https://en.wikipedia.org/wiki/Tolman_surface_brightness_test
https://briankoberlein.com/2014/05/24/selection-bias/

Secondly, if the Universe wasn't expanding, the brightness of a supernova should take about the same time to decrease in the distant Universe as it does nearby. That's not the case. Very distant supernovae take about twice as long to dim as nearby ones. And we know from other observations that these particular supernovae are type Ia (https://en.wikipedia.org/wiki/Type_Ia_supernova), which occur when a white dwarf accretes material from another star and eventualy becomes so massive that a huge fusion explosion is set off. The mass at which this happens should be fixed and not depend on any other parameters, so the peak brightness and the rate it decreases shouldn't vary.

Cosmological redshift/time dilation provides a very natural explanation of this. Just as the wavelength of light decreases from a source that's moving away from us, so should the time between events. Just as with time dilation due to motion through space, an observer closer to the supernova should see it take, say, 20 days to dim, but an observer far away should see it take longer.
http://burro.astr.cwru.edu/Academics/Astr328/Notes/Redshift/redshift.html

It turns out that this effect has been observed and is consistent with the model of expansion based on redshift. This is pretty powerful, but not conclusive, evidence that the Universe really is expanding.
http://www.astro.ucla.edu/~wright/cosmology_faq.html#TD

... but, it should be noted that quasars and gamma-ray bursters do NOT show the same effect ! Oh noes ! The reasons for this are still not understood. For quasars, one explanation may be that they evolve over time. We know that galaxies change over time, so quasar activity might also change as their fuel supply varies. Unlike with the critical mass for a type Ia supernova, this might be strongly related to the redshift, in a way that could exactly cancel the predicted time dilation :
https://www.reddit.com/r/askscience/comments/idvqv/why_do_quasars_seem_to_not_be_beholden_to_time/

As for gamma ray bursters, I am even less of an expert about these than I am about quasars. My guess would be that since the mechanism for the explosion is still not well-understood, it's much harder to say if the lack of time dilation is a serious problem given the scatter in the duration. I found a few references claiming that it's not such a big deal, but I wouldn't like to speculate too much.
http://www.mpe.mpg.de/events/GRB2012/pdfs/talks/GRB2012_Kocevski.pdf


Could the Steady State model be made to work ?
Q : Is there any other scientific explanation for redshifts and the CMB that could bring back a static, steady state model to explain our cosmological order ?

A : In my opinion, the Steady State model is decisively ruled out by other observations. We know that galaxies in the distant Universe are different to ones nearby, in a variety of different ways. For example, star formation rate varies strongly with redshift. The numbers on this "Madau plot" vary with different observations, but the characteristic hump-shape is pretty much an observational certainty :
https://ned.ipac.caltech.edu/level5/Illingworth/Ill5.html

Then there's the Butcher-Oemler effect. Consistent with star formation rate increasing with redshift, more distant galaxy clusters contain more galaxies with bluer colours (short-lived stars are bluer, so this is consistent with star formation rate being higher in the past).
http://astronomy.swin.edu.au/cosmos/B/Butcher-Oemler+Effect

Quasars are also much more common at higher redshifts :
http://csep10.phys.utk.edu/astr162/lect/active/quasars.html

There's also the fact that galaxy morphology (shape) varies with redshift. There are more peculiar galaxies and fewer spirals and ellipticals at higher redshifts :
https://ned.ipac.caltech.edu/level5/March04/Conselice/Conselice3.html A nice visual example of this is the Spiderweb Galaxy. To my knowledge, nothing like this is known in the nearby Universe :
http://www.esa.int/Our_Activities/Space_Science/Flies_in_a_spider_s_web_galaxy_caught_in_the_making

Taken together, in my opinion a Steady State model is now impossible. While redshift is not, strictly speaking, the same as distance, we know the correlation is good enough to say with certainty that more distant galaxies are different to local galaxies. This is in direct conflict with the Steady State idea. See also my article on why the sky is dark at night, which is damned hard to explain in an infinite eternal Universe, but straightforward if the Universe hasn't been around forever.
http://astrorhysy.blogspot.co.uk/2015/04/the-most-astounding-fact.html

But I haven't actually answered the question yet. On redshifts, the most well-known alternative idea is that galaxies could have an "intrinsic" redshift. The evidence for this was that some galaxies which look like they're interacting with each other apparently have quite different redshifts. I don't know what the statistics are on this, but I'd be willing to bet that these are a small minority compared to the number which are interacting and at similar redshifts. There's also no known mechanism by which redshift could be altered in this way.
https://en.wikipedia.org/wiki/Non-standard_cosmology#Redshift_periodicity_and_intrinsic_redshifts

There's also the idea that maybe photons lose energy as they travel, so that redshift could indicate distance but not necessarily motion (i.e. expansion). This "tired light" idea cannot explain the supernova time dilation, and would predict that distant objects appear more blurry than they actually do. It also doesn't explain the CMB.
http://www.astro.ucla.edu/~wright/tiredlit.htm

The CMB is explained extremely well by the Big Bang model, but cannot be made to work in a Steady State universe. One idea is that's actually just redshifted starlight, but the details show that this simply doesn't work.
http://www.astro.ucla.edu/~wright/stars_vs_cmb.html

Another idea is that supernovae/quasars create and eject "iron whiskers" which could transform the radiation of the youngest stars into the observed CMB. But this is highly contrived explanation. A friend of mine once did some research about this and found that they don't work; alas I'm not an expert in this so I can't comment further.
http://web.missouri.edu/~lia/papers/needles.pdf
http://rationalwiki.org/wiki/Alternative_cosmology#Quasi-Steady_State_Cosmology

I would also add that I find a infinite/eternal Universe philosophically unsatisfying. Probability doesn't mean anything everything happens an infinite number of times, so if something extremely unlikely is observed, one can always say, "that's because of infinity" rather than searching for a real explanation. It's tantamount to magic.


Why are distances in space so enormous ?
Q : One thing that has been on my mind is why is there such great distance between planets and the Sun ? And on that same track, why is there such a great distance between the various stars and galaxies ?

A : That's either a really easy or deeply profound question depending on how you look at it. :)

The really easy version is that it's due to the strength of gravity. Given how fast everything is moving, if gravity was stronger everything would be closer together. In Newton's model of gravity there is a fundamental constant in the equation, G - make this value bigger and the force of gravity would be stronger. In Einstein's model it would mean that space is more curved by the same mass. Either way, it's all due to the strength of gravity.

The deeply profound version is : why should gravity take this particular strength and not something else ? That's not something I think anyone has a good answer for. Sometimes people invoke the anthropic principle : if it was different, we would not exist. That causes all manner of problems and doesn't actually explain anything unless you think the sole purpose of the Universe is life. Which I do not.
http://astrorhysy.blogspot.co.uk/2015/03/the-absurdly-anthropic.html


What's your opinion on the detection of gravitational waves ?
Q : Well, what is it ?

A : I was more impressed with the press conference than I thought I would be.
https://www.ligo.caltech.edu/news/ligo20160211

The signal seems like a very convincing detection. It doesn't look as strong as I thought it might, but it appeared in both detectors and matches the predicted signal so precisely it's hard to believe it could be anything else. Plus it was (mostly) under wraps for months. This is much better than the all too common tactic of doing science by press release and hoping no mistakes have been made.

I am less convinced of the importance of gravitational wave astronomy, however. Yes it's super super important for testing relativity, and yes, maybe it will tell us lots of interesting things about cosmology.... and yes, no doubt there will be some major surprises in store. For all these reasons it is a genuinely very exciting discovery and a historic moment. It deserves the Nobel prize it will inevitably get.

And yet... the spatial resolution is abysmal. We know a surprising amount of detail about happened, but the constraint on where it happened is so large you might as well wave your hand toward the sky at random and say, "THAT WAY !". Literally that - try it ! It isn't going to get much better anytime soon, with detectors coming online in the next few years only narrowing it down to a few square degrees. That's scarcely any more helpful if the source is a billion light years away. So if we don't even know which galaxy these events are happening in, I remain cautious as to how much we'll actually learn from these regarding how the Universe works.


I'd like to know more detail about deciphering the age, direction and source of the gravitational waves.
Q : Come on then. You said ask anything.

A : I did indeed, but you probably want to ask Jonah Miller rather than me ! https://plus.google.com/u/0/+JonahMiller/posts

As I understand it, direction comes from the time delay between the signal received at one antenna and the other. Since there are only two antennas, this doesn't really constrain the direction very much at all. It would be a lot better with 3 detectors, but still (IIRC) this will only constrain the location to within ~10 square degrees.

The distance (age) and masses of the sources are both inferred from the frequency of the signal. Through computer models of the interaction, the frequency tells you the mass. The mass determines the energy emitted, and since you can measure the strength of the signal and know the total amount of energy emitted, you can calculate the distance.
http://www.astro.cardiff.ac.uk/research/gravity/tutorial/?page=4blackholecollisions


Since energy and mass are equivalent, are dark matter and dark energy really just the same thing ?
Q : Well, are they ?

A : No. To simplify, dark matter is largely thought to exist because galaxies are spinning too fast. It is thought to be an as-yet unknown form of matter, but still definitely matter in the regular sense - it has gravity.

Dark energy is an unknown force or energy field that is causing the expansion of the Universe to accelerate. It is not thought to be a form of matter, as this would have the opposite effect. No mainstream theory AFAIK postulates any connection between dark matter and dark energy. The similarity of the names doesn't mean anything.


Since matter can never be created or destroyed, are you the same age as the Universe ?
Q : Come on then. Make with the answer, clever clogs.

A : A small amount of matter is converted into energy during fusion inside stars. AFAIK the reverse process doesn't happen to any significant extent.More significantly, fusion joins atoms together into new atoms. So it is not correct to say your atoms are as old as the Universe, since many of them didn't exist until they were created inside a star.

I suppose you could say your matter is as old as the Universe, but I would also not say that you're even as old as your matter. You're not 'you' until your atoms are assembled.


What do you think of the idea that Universe had no initial singularity ?
Q : Specifically, as described in this article : http://phys.org/news/2015-02-big-quantum-equation-universe.html

A : There's a nice summary here :
https://briankoberlein.com/2015/02/10/beginning/

Personally, I'm not sure it helps. Either the Universe has existed forever, or for a finite time. The trouble is that if it's existed forever that just replaces one infinity (the singularity in standard cosmology) with another. Probability becomes meaningless in an infinite universe since everything happens an infinite number of times - this is the "measure problem" (https://en.wikipedia.org/wiki/Measure_problem_(cosmology)). It's even weirder in this particular model - why should the Universe have got along for an infinite amount of time before suddenly deciding it wanted to explode ? Does "now" even mean anything in an infinite universe ? Seems to me that these are pretty deep philosophical issues our monkey brains are ill-equipped to answer.


Does space have limits ?
Q : Our observable universe is limited because of metric expansion of space over distances beyond gravitational bounds. Is there any evidence to suggest that the universe may have other limits beyond what we can observe? Is it reasonable to assume that there are no limits other than what we have observed, or should we accept the possibility of limits that have never been observed just because we can imagine such possibility ?

A : Tricky. It depends what you mean by "limits". If you mean, "is space infinite ?" then see this earlier answer :
http://astrorhysy.blogspot.cz/p/q-a.html#sizeofspace

I would add a few points :
First, I dislike the notion of an infinite universe because it means that everything possible happens somewhere. That means that science is basically knocked on the head - there's no point trying to explain anything because everything that can happen, does. Probability is meaningless.
Of course the Universe doesn't care what I think, so it could be infinite. But since we can't test this observationally, we've no choice but to resort to philosophy.

Second, the observable universe is a tricksy concept. It's not like on Earth where if you want to see over the horizon you just go for a long walk. Their are, according to models, parts of the Universe so distant that they will never, ever affect us. To go beyond this and report back to everyone left on Earth we'd have to travel faster than the speed of light.
https://en.wikipedia.org/wiki/Event_horizon#Cosmic_event_horizon

Third, if you meant, "does the Universe have an edge ?" then I dislike this idea too. It's difficult to imagine what an edge to space would look like, but it would also violate the Copernican principle. Observers close to the edge would see a very different Universe to ones near the middle. Of course, the Universe could just be vastly larger than the visible Universe, so that most observers are never aware of the edge.

I prefer the notion of a torus-shaped Universe, which solves all of these annoyances. Flat, finite and unbounded, that's the way to go.
http://astrorhysy.blogspot.cz/p/q-a.html#ieatsevenimpossiblethingsbeforebreakfast


Why do black holes bend space ?
Q : I have a question, why do black holes bend space ? How is it work actually ?

A : All mass bends space. According to general relativity, gravity isn't exactly a force - it's the curvature of space. Don't think of gravity as a force or somehow separate from the bending of space. Gravity is the bending of space. Every single atom bends space a tiny little bit. A pretty nice explanation can be seen here (it really is easier to show this rather than trying to describe it) :
https://www.youtube.com/watch?v=MTY1Kje0yLg

Black holes just bend space more than other objects because they have a lot of mass in a very small volume. Technically they bend space by an infinite amount according to relativity. Most people think this indicates that relativity isn't a complete description of what's going on, because infinities cause all kinds of horrendous mathematical problems.


If space was curved enough could I see the back of my own head ?
Q : Scientists said 'we can see the back of our head', that means, the matter is orbit the black holes, right ? That's why, we can see the matter behind the black holes, even in a picture.

A : Well I am not sure about the specific example of a black hole, but yes - if space was sufficiently curved it would be possible to see the back of your own head. We can see slightly less dramatic examples of this all the time. For example, stars near the Sun have their position slightly altered because of its gravity.
http://light2015blog.org/2015/03/19/what-do-we-talk-about-when-we-talk-about-a-scientific-theory/


How do we know space is curved ?
Q : If space is curved, how do you know it is? You live on the curve itself. The distance you measure from the mass is the distance along the curve. And that as the mass is far, far away. That means all you can measure is a very, very small curvature, regardless of how close you get to the mass. If fact, the closer you get to a mass, the flatter the curve becomes.

Space is not curved and time is not a dimension.

A : [Oh God hear we go...]

Space is indeed curved and time is indeed a dimension, albeit not a spatial one.

There's a wealth of evidence that space is curved. You can measure this because the path of a light ray follows the curvature, making it appear as though distant objects are in a different position. For instance, the position of stars near the Sun are observed to shift in exact agreement with the predictions of relativity :
http://light2015blog.org/2015/03/19/what-do-we-talk-about-when-we-talk-about-a-scientific-theory/

Gravitational lensing also produces distorted images of distant galaxies, again in agreement with the predictions of relativity. In some cases the effect of this is extremely clear - you can measure a very large curvature without having to be close to the mass at all :
http://www.cfhtlens.org/public/what-gravitational-lensing

[The OP decided to ignore what I wrote and continued to advocate their intuitive opinion over rigorously tested mathematics. Oh dear.]

Q : If you can measure the curvature, you are not in our universe.

This mystical curvature changes the radial distances from the mass but not the circumferential ones. An object travelling in a circle with its centre at the mass will have no changes in the distance it travels. This mystical curvature has no effect on the circumferential distances.

But when a photon passes by a mass, its path has its greatest curvature, which is the exact opposite of this mystical curvature.

You have just proved space is not curved.

A : That's not the case. See in particular the first link I posted. The rubber sheet analogy is a pretty good one. Here's another, slightly better diagram :
http://www.physicsoftheuniverse.com/images/relativity_light_bending.jpg
I don't really know how else to describe it. Curvature just doesn't work in the way the OP thinks it does.

[Still not enough. Come on, people, you can test how curvature works for yourself using a blanket and a heavy book. Einstein's maths may be difficult, but the concept of curvature really isn't.]
Q : Why people insist That Einstein never made a mistake on Relativity and had it published ?

A : We insist that Einstein had relativity published because, in fact, he did. No-one insists that Einstein never made a mistake, but everyone insists that the predictions of relativity have come true because... well, because they have.


Is space globally curved ?
Q : [This was in the context of the curvature of ALL of space, as opposed to bits of it in an earlier question (http://astrorhysy.blogspot.cz/p/q-a.html#einsteinmusthavebeenverystupid). In a sense, "does space curve ?" and "is space curved ?" are different questions/]

A : We can state with close to certainty that space has local curvature, as described.

Global curvature is another matter. Observations say no, it's flat :
http://m.teachastronomy.com/astropedia/article/Measuring-Space-Curvature
(Caveat : the surface of a sphere is curved, but the surface of a torus is flat. We could in principle detect if the universe was curved like a sphere, but not like a torus.)

But theory says that it shouldn't be flat. If it's even slightly curved, it should either collapse before any stars have time to form or expand so rapidly that the density never gets high enough for star formation. Hence the idea of inflation : the early Universe expanded very rapidly, so that the observable Universe today is just a very small part of the whole. So globally it might be curved after all, just by such a small amount that it's not detectable.
http://www.astronomynotes.com/cosmolgy/s12.htm
(According to some there's no problem here at all and it's just a coordinate issue, however, this goes against absolutely everything I've been taught so I'm rather skeptical :
https://www.researchgate.net/post/How_do_cosmologists_determine_the_curvature_of_the_universe_based_on_astronomical_data)

Apparently inflation does produce distinct signatures which could be detected, which resulted in the BICEP2 debacle last year. But no-one understands what caused inflation, and furthermore theoretical estimates of how large it should be based on known physics are ~10^120 times greater than observed - probably the most wrong answer anyone's ever come up with.

Personally I've never liked inflation much. If the Universe had to have precise conditions to start off flat, then dang it it must have had those conditions. The fact that the Universe is homogeneous is often cited as evidence that there must have been some period when regions of the Universe which are now distant must have been in contact with each other - i.e. inflation caused their separation. I've never understood why anyone would assume that the Universe just wasn't homogeneous everywhere to begin with anyway.


Is the Universe shaped like a doughnut ?
Q : If a torus-shaped Universe is flat, then could the Universe be torus-shaped and smaller than we think ? Could it actually be smaller than the observable Universe, allowing us to see a younger Milky Way whose light would have circled (torused ?) the Universe ? And if not, of course, how do we know/what did I get wrong ?

A : In principle yes, the Universe could be smaller than it appears. What appear to be very distant galaxies could actually be light emitted by a close galaxy that's traversed the Universe several times. Since the Universe is continuously expanding, we could even get the appearance of galaxies in a continuous distribution of distances, not just at multiples of the true size of the Universe.
https://www.khanacademy.org/science/cosmology-and-astronomy/universe-scale-topic/big-bang-expansion-topic/v/a-universe-smaller-than-the-observable

I think in that case, though, we would inevitably see the same structures if we looked in opposite directions - the same large-scale structures of galaxies or the CMB, but mirror-flipped (though it would be more complicated than that since there would be multiple paths the light could take to reach us). As far as I know, no such repeating structures have ever been observed. That's easily avoided by having a larger-than-observable Universe (doesn't preclude it from bein a torus or other geomtery) but I don't see a way around this in a smaller-than-observable case.
http://abyss.uoregon.edu/~js/cosmo/lectures/lec15.html
But much better, watch this :
https://www.youtube.com/watch?v=_k3_B9Eq7eM



Yeah but really, could Steady State be the answer after all ?
Q : [Amalgam of responses from this thread : https://plus.google.com/u/0/103548473693845391486/posts/NS2N5SettnX] Don't forget Tectonic Plate Theory would get you fired until it was undeniable. As much time that has been spent on the Big Bang Theory we should know it all....wait we just found gravitational waves. By your thinking we no longer need to put another dime into the big bang Theory.

I personally see more common sense in the steady state Theory. Whose to say that quasars aren't the baby stages of gravitational waves? There are just too many unanswered questions with the steady state Theory to ditch it because we have seen more of the Big Bang Theory. Maybe the steady Theory finishes the the Bang Theory. You don't just stop the research. I believe in the future scientists will be researching the steady Theory again. It didn't pop up for no reason. Someone just hasn't found the link. :-)

A : As for plate tectonics, I'm well aware of this. Don't go there or it will go the worse for you.
http://astrorhysy.blogspot.cz/2013/02/i-wanna-be-pseudoscientist.html

Steady State is pretty much dead since more distant galaxies are clearly different from nearby ones, quasars are more common at greater distances, the CMB, etc. _Un_steady State is, I guess, still possible. The idea that the Universe isn't really expanding is, I suppose, just about barely tenable, but not at all likely.
http://astrorhysy.blogspot.cz/p/q-a.html#redshiftsjustarentenoughforme
http://astrorhysy.blogspot.cz/p/q-a.html#bringbackthesteadystate

Honestly, Steady State is done. If you don't close some avenues of inquiry, you'll never make any progress.? Which I explain in great detail here : http://astrorhysy.blogspot.cz/2015/10/not-so-open.html

To say that this line of reasoning means that we wouldn't have investigated the Big Bang model is wrong. On the contrary, that is precisely the opposite of what I'm saying ! It is exactly because the Big Bang theory works very well that we need to keep testing it. Same as with any theory that works - you have to keep testing it to find out where it stops working. That's the only way you learn. At the same time, you don't keep investigating ideas when you know they don't work.

Testing theories is not done as an intellectual chest-thumping exercise. It is done in order to learn more. This article has the right of it :
http://www.bbc.com/news/science-environment-31162725
"The data so far has confirmed that our theory is really really good, which is frustrating because we know it's not!" Prof Shears says. "We know it can't explain a lot of the Universe. So instead of trying to test the truth of this theory, what we really want to do now is break it - to show where it stops reflecting reality. That's the only way we're going to make progress."

Classical Steady State theory really is as dead as the dodo. The whole point of it was that the Universe is not evolving with time. This was a valid idea in the 1950's, but we now know with 100% certainty that that isn't the case. No observations will ever disprove this. It's like saying that one day we will find the edge of the Earth if we just keep looking a bit harder.

Where there is considerably more wiggle-room is the idea that the Universe hasn't always been as we see it today, but is still infinitely old (and perhaps infinitely large as well). It's possible that the Universe is in some way cyclic, with many Big Bangs. That's a much harder idea to prove/disprove. At present there's no reason to think this is likely to be the case. But this is very different from the original Steady State theory.


How does the expansion of space cause resdshift ? Where does the energy of each photon go ?
Q : If the temperature of the universe at the epoch of recombination was in the visible light range, then has the expansion of the universe stretched the actual photons into microwaves, or are we viewing visible light photons that are being instantaneously stretched at the point of our receptors into microwaves because we're moving away from the source at nearly light speed ?

In a universe in which energy is conserved, where does red shifted energy loss of the photons go ? Is the energy loss gradual over time, or instantaneous at the point of absorption ?

Expanding gas cools because the molecules are continually doing work on a piston, or what ever, but the last scattering surface of the CMB photons was at the epoch of recombination, so they've done no work on anything. So is there any intrinsic sense in which CMB photons still carry the very same unshifted frequency that they were emitted with ? I.e., does a gamma ray know it's a gamma ray ?

A : [This question was, for some unfathomable reason, asked on a joke thread that reported on Saidq Khan's election to Mayor of London by playing on his surname with many a reference to Star Trek. This seems a bizarre place to ask me a question, so I responded in kind].

KHAAAAAN !!!!

As I understand it, YOU KLINGON BASTARDS YOU'VE MURDERED MY SON ! both redshift due to motion through space and of space (cosmological redshift) OF ALL THE SOULS I HAVE ENCOUNTERED ON MY TRAVELS cause an instantaneous change of the photon HIS WAS THE MOST.... HUMAN !. Cosmological expansion is acting continuously, so the wavelength changes continuously over time. But the wavelength you perceive the photon will also depend on your motion, I WAS AND EVER SHALL BE, YOUR FRIEND so it's complicated.
http://astronomy.swin.edu.au/cosmos/c/cosmological+redshift

About energy conservation, my hand-waving response would be I'LL CHASE HIM ROUND THE MOONS OF NIBIA that the energy density goes down, but the total volume goes up, so the total energy AND ROUND THE ANTARES MAELSTROM is conserved. And energy conservation AND ROUND PERDITION'S FLAMES is somewhat... subtle in general relativity BEFORE I GIVE HIM UP !.
http://www.preposterousuniverse.com/blog/2010/02/22/energy-is-not-conserved/

I'm not entirely satisfied with HOW WE DEAL WITH DEATH IS AT LEAST AS IMPORTANT AS HOW WE DEAL WITH LIFE that answer. I suppose you could think of the photons as WE'RE TALKING ABOUT UNIVERSAL ARMAGEDDON ! doing work against the expansion of space, just as they do work escaping from a gravitational field for gravitational redshift. Space pulls the photons apart, so the lost energy goes into the expansion of space. DAMMIT JIM, WHAT'S THE MATTER WITH YOU ? but that's just my very naive interpretation. My suspicion is that THE NEEDS OF THE MANY it's the expansion of space which OUTWEIGH THE NEEDS OF THE FEW causes the loss of energy, but the total energy is still not really OR THE ONE conserved.

My relativity knowledge is not what it once was.

LIVE LONG AND PROSPER


Are we sure the Tired Light Theory was wrong, then ?
Q : Pretty darn sure. Like any alternative to the expanding evolving universe, there are just too many things it can't explain - the fact that surface brightness of galaxies decreases with distance, time dilation of distant supernova, the spectrum of the CMB, the greater numbers of quasars at greater distances, the different morphologies of distant galaxies, etc.
https://en.wikipedia.org/wiki/Tired_light
http://www.astro.ucla.edu/~wright/tiredlit.htm
http://scienceblogs.com/startswithabang/2010/10/08/big-bang-alternatives-could-my/



Who can explain to me why Einstein was trying to find the Gravitational constant of the universe ?
Q : Well, what's it for ?

A : [The questioner seemed repeatedly confused about the difference between G, g and lambda, then eventually the conversation degenerated into a case of, "I admit I don't know this very basic fact, but I'm pretty sure I understand all these much more complicated aspects of the theory." Here's my answer anyway.].

There's the gravitational constant G, which as far as I know Einstein never did much research on, and the cosmological constant Λ (Greek letter lambda), which is what famously gave Einstein a headache. I assume that's what you're asking, though you keep referring to the gravitational constant. That's something entirely different (https://en.wikipedia.org/wiki/Gravitational_constant). As far as I know, it isn't possible to predict the value of G by theory (not relativity anyway), it has to be measured. Which Einstein didn't do, since he was a theorist.

As for lambda, that's another story. When Einstein came up with general relativity, he realised the equations implied that a static universe was impossible. Given enough time, it had to collapse to a point. This meant the Universe could not be infinitely old and static, which was the prevailing wisdom of the time. So Einstein added the cosmological constant as a way to make his equations agree with this widespread prejudice. That's the short answer to your question : because he thought the Universe was not expanding.

The other way to save his equations was to assume the Universe was expanding, and by direct implication it had once been smaller in the past - i.e. it was finite in age. That would at least prevent a rapid collapse, and if the expansion was rapid enough it would mean the Universe would never re-collapse. There was no particular reason to assume this was the case at the time, as there was little or no evidence for an expanding Universe. So adding the cosmological constant made sense, as a way for his theory to agree with what everyone thought the Universe was doing. I would be rather surprised if he never calculated a value for the constant though.

Of course, it wasn't long afterwards that the Universe was discovered to indeed be expanding - so if Einstein hadn't introduced the term, he would have made an astonishing prediction. Then again, by the end of the century it was found that the expansion of the Universe was accelerating, which also needs a cosmological constant (or something similar). So he may have be right after all, but for the wrong reasons.

http://www.space.com/31055-removing-cosmological-constant-was-the-blunder.html
https://en.wikipedia.org/wiki/Cosmological_constant#History


Isn't G related to the total amount of gravity in the Universe ?
Q : I thought G was the sum total of all the gravity in the universe. Regardless of the fluctuations of force.

A : The way I see it, G is just a number to make the equation work correctly using our choice of units. E.g. we generally use kilograms for mass, not pounds or stones. The value of G (given some choice of measurement units) determines how strong gravity is for any given amount of mass. If it were larger, all gravity would be stronger : everything would be more attracted to everything else. It's not related to individual planets or other celestial objects.


So the force that holds the planets in orbit and keeps the moons orbiting the planets is the weakest force ?
Q : Not to mention it faster than light or the lights momentum would be able to escape its grasp considering how weak it is.

A : Yes, gravity is the weakest force, and yes, light does escape - very easily ! The only thing light cannot escape from is a black hole. There's a nice explanation of how weak gravity is here :
http://physics.stackexchange.com/questions/4243/what-does-it-mean-to-say-gravity-is-the-weakest-of-the-forces


The Universe has expanded so much that some parts of it are now unobservable. But could we explore those parts if we had a faster-than-light drive ?
Q : Out of curiosity, wouldn't an advanced civilization capable of warp speed travel be able to explore the unobservable universe ?

A : Yes they would. There's nothing special about any particular region; if they had a hypothetical FTL drive I'm not seeing any kind of cosmological reason which would prevent them from exploring the unobservable regions. To an observer in any part of the Universe, more and more of it becomes unobservable as time passes, but their particular region looks normal. Unless, that is, you take the extreme case of the Big Rip, where matter itself is eventually ripped apart by the expansion.
https://en.wikipedia.org/wiki/Big_Rip


How do you know that dark matter is diffuse ?
Q : Well, how ?

A : Because of the rotation curves of galaxies and the velocity dispersion of galaxies in clusters. Highly concentrated dark matter couldn't cause a flat rotation curve at large distances - they imply that the total amount of mass keeps increasing as you go further away from the galaxy.
http://bustard.phys.nd.edu/Phys171/lectures/dm.html
Basically you equate F = GMm / r^2 with the equation for circular motion, F = mv^2/r

Dark matter is thought to be distributed in roughly spherical halos. In principle it could be part of the disc, but that should cause observable changes in the vertical motions of the stars. As for why dark matter makes things move fast in the first place, imagine if the Earth was ten times heavier. Anyone falling over would hit the ground much faster. Similarly, baryonic matter falling into a dark matter halo moves faster because there's more mass.


How small was the fraction of deuterium created during Big Bang Nucelosynthesis ?
Q : There's one part of cosmology which has always baffled me as a civilian: Recombination
https://en.wikipedia.org/wiki/Recombination_(cosmology)

I kinda get it, but know better than to think I fully understand what's going on. Here's where it kinda meshes with your H1 / H2 problem :
https://en.wikipedia.org/wiki/Big_Bang_nucleosynthesis#Deuterium

Only a finite amount of deuterium was created at BBN. But how small a fraction is it?

A : I've always thought recombination should just be called "combination", it's not as if the electrons and protons were previously combined... I'm no expert in BBN, so don't take this as more than an educated guess. But as I understand it, something close to 100% of the observed deuterium should be primordial because there's no other known way of producing it. Models of BBN predict the amount of deuterium, which depends on the baryon density of the early Universe. As long as the amount of deuterium hasn't been significantly altered by any other processes (which seems to be the case), observation of deuterium can constrain the baryon density of the early Universe.
http://www.astronomynotes.com/cosmolgy/s6.htm

Other constraints on the age of the Universe and the baryon density should allow you to falsify the model - if they implied the model predicted much more or less deuterium than is actually observed, there'd be a big problem. There's a complication in that chemical evolution isn't simple - even if the deuterium amount doesn't change, the amount of ordinary hydrogen can (e.g. by accretion). The deuterium abundance in the Milky Way is actually slightly less than predicted (so if there's any non-primordial deuterium is must be negligible), but, apparently, chemical processes can explain this quite well :
http://hyperphysics.phy-astr.gsu.edu/hbase/astro/deuabund.html


Could the Big Bang be a white hole ?
Q : Would it be possible that the Big Bang was/is a white hole (sorry if you hear this a lot) ?

A : For once the short flippant answer and the "more detailed" response are going to be the same : sure, why not ? In terms of what caused the Big Bang to happen I think we're very much still in the "here be dragons" stage. Something has to provide that source of energy and mass. Could be a magical deity, could be our own Universe in the future connecting to itself through a black/white hole, could be something to do with quantum, who knows ? Not me. ;)


How can we see light from the Big Bang, shouldn't it have overshot us ?
Q : If we can see light from the edge of the big bang ,and that's were we came from, why did that light not pass us ? or move with us ? How can we see back to a beginning that began us ? ...maybe this is a physics question?...?

A : There are two parts to that question. First, we can't see directly back to the Big Bang itself. Initially the Universe was nothing but a plasma of protons and electrons which scatter photons of light, so it was opaque. Eventually the Universe expanded and cooled enough to allow the protons and electrons to combine into neutral gas, which is transparent. So from that point on any photons were free to travel, preserving a snapshot of the Universe at that moment (which we see today as the Cosmic Microwave Background). This is known as the "surface of last scattering" and is generally reckoned to have happened 300-400,000 years after the Big Bang.
https://webcache.googleusercontent.com/search?q=cache:GI3aFGBQJVIJ:https://ned.ipac.caltech.edu/level5/Glossary/Essay_lss.html+&cd=2&hl=en&ct=clnk&gl=us

The second part of the question is that the Big Bang didn't happen at a fixed point, it happened everywhere : all of space is expanding. So our Galaxy has been observing photons from the CMB for billions of years, because the CMB was formed throughout the whole volume of the Universe. We've absorbed some photons but we're still receiving others from the most distant part of the Universe.


Does the energy of the Universe really add up to zero ?
Q : So, when Michio Kaku refers to it all adding up to zero energy, is there any "creative accounting" going on ?

A : As far as I'm aware it's not known for certain if the total amount of energy really equals zero. Yet even if it does, I'm not sure that's particularly helpful either. A rock falling in the gravitational field of a planet can have zero net energy : positive kinetic energy but negative gravitational potential energy. But clearly the rock and the planets are things which do exist, but the total energy of the system hasn't helped explain their existence. It's not clear to me how this would help explain the existence of the Universe either.


How can the Universe be expanding if it's infinite ?
Q : So firstly the universe is ever expanding, yet it's infinite. Those both contradict one another are they separate theories ?

A : The evidence that it's expanding is very strong, but there's not really anything to indicate that it's infinite. It's finite in time, so it could be finite in space as well.
http://astrorhysy.blogspot.cz/2015/04/the-most-astounding-fact.html


Could frame dragging explain dark matter ?
Q : [This was in relation to a recent cat fight claiming that dark matter has problems explaining small variations in the rotation curves of galaxies. In fact, as has now been shown by many independent authors, that is simply not true at all. https://plus.google.com/u/0/+RhysTaylorRhysy/posts/6MXdAjNPnNv?cfem=1]

What about frame dragging for the galaxy ? Is gravity an emergent force ? Could the underlying principles that give rise to emergent gravity be responsible ?

A : I doubt frame dragging or any other relativistic effect could be responsible - someone would have noticed this from Einstein's equations by now ! The point is rather than the earlier claims were that this would be very difficult to explain through standard models, this is now not the case. Therefore this observation cannot be used to test whether modified gravity theories are any better than standard models.

Though there is now yet another alternative model of gravity claiming that "emergent gravity" is responsible after all : https://astronomynow.com/2016/11/08/new-theory-of-gravity-might-explain-dark-matter/

Call me skeptical : people love trying to "explain" dark matter because if they succeeded they'd be hailed as the next Einstein. Yet the model has survived many alternatives and always come out on top. For sure there's a scientific revolution awaiting the unification of general relativity and quantum mechanics, but that doesn't mean dark matter doesn't exist.


Do most models of dark matter say it should interact with itself ?
Q : Don't most particle physics models of possible ways dark matter could exist also suggest that it interacts with itself and with ordinary matter via one of the three known non-gravitational forces on extremely rare occasions?

A : Yeah, there are a bunch of models of self-annihilating dark matter that could emit gamma rays. In the extreme case there are models showing that you could get star-like objects from dense concentrations of neutralinos : http://www.space.com/9302-dark-matter-stars-solve-cosmic-mystery.html As I understand it (but the particle physics is outside my specialist area, which is galaxy evolution) the dark matter in these cases would only radiate when it self-annihilates, and the signal would only be strong enough to be detectable in extreme cases like the galactic centre :
https://www.nasa.gov/content/goddard/fermi-data-tantalize-with-new-clues-to-dark-matter
I'm not sure what the current status of this idea is though. In galaxy evolution models this effect is typically ignored as it's not thought to be significant in the grand scheme of things, but it could be an important way to detect it.


If dark matter is self-interacting then shouldn't it behave just like a normal gas ?
Q : Shouldn't the dark matter particles in the models follow gas dynamics when interacting with each other?

A : It depends on the nature of the interaction. If they're each other's own anti-particle, then probably not : each particle will annihilate. If they were interacting in some other way, such as via electrical forces as normal matter does (but somehow limiting this interaction only to the dark matter), then yes.

But this (along with baryonic dark matter) is pretty convincingly ruled out by the Bullet Cluster, in which two galaxy clusters have passed through each other. The galaxies themselves have kept going and are largely unaffected; stars are too widely separated for collisions to be frequent. However, the gas fills a much larger volume : there's no way it should be able to avoid a collision. And it hasn't - it's very clearly got stuck in the middle. The dark matter in the clusters appears to have behaved very much like the collisionless stars and kept going, with gravitational lensing observations indicating that the clusters still possess huge amounts of unseen matter, with little or none of this ending up in between the clusters. So the standard model of dark matter as a collisionless particle appears to be holding true.
http://chandra.harvard.edu/press/06_releases/press_082106.html


What's your take on Verlinde's new theory of gravity ?
Q : Well, what is it ?

A : Erik Verlinde is a respected, "definitely not a crackpot" kind of physicist who's come up with a new model of gravity. It apparently predicts things similar to the more well-known "MOND" (Modified Newtonian Gravity") theory that's been a thorn in the side of standard-model advocates for the last 30-odd years. The neat thing about it is that whereas MOND was created as a mathematical solution to avoid the need for dark matter, Verlinde's is a more "bottom-up" approach which predicts MOND from basic physical principles.

However, that's about the extent of my knowledge of the material, which is far too mathematically advanced for the likes of me. So take this response with that in mind. Even so, my first reaction to any alternative theory of gravity is more or less this :
https://xkcd.com/1758/

I am not really qualified to judge Verlinde's specific idea. One thing that modified gravity theories in general suffer from is a lack of simulations, usually because it's not as simple as changing the strength of the force law. In MOND, which apparently Verlinde's idea is very similar to, acceleration depends on the distribution of matter in a completely different way to Newtonian gravity. Analytic predictions only work well in very limited scenarios and it's impossible to say what would happen in a full-on simulation in anger, i.e. a large-scale cosmological simulation or a smaller scale version with baryonic physics. These modified gravity ideas might work much better than the standard model... or they might have entirely new problems that the analytic models can't predict. We just don't know yet.


Does the bullet cluster interaction data rule out some of the extremely weakly interacting forms of dark matter I was talking about previously ?
Q : Well, does it ?

A : For context the type of dark matter referred to was the self-annihilating kind. Some models predict that dark matter particles can occasionally interact and annihilate each other, producing a gamma-ray photon we could detect. The signal would be strongest where the dark matter density is greatest, like in the centres of galaxies and galaxy clusters.

Actually I was going to say that the density of dark matter in the centres of clusters probably wouldn't be high enough to detect a signal, but I stumbled on this...
https://arxiv.org/abs/1607.07420
I only read the abstract, but it seems they do believe it's possible to just about detect such self-interacting dark matter in a nearby galaxy cluster. The Bullet Cluster is (from some quick Googling) 15x further away and 3x less massive, so the expected signal would be hundreds of times weaker (since the interaction is weak, the density during the collision couldn't really be more than double its usual value since it would be effectively just overlaying two normal-density clusters). So it doesn't give us any evidence either way for this kind of self-interacting dark matter.


Is gravity caused by protons and electrons spinning through space ?
Q : Gravity - is it caused by protons or electrons spinning through space as within the earth thus attracting protons or electrons from within matter ?

A : Current theory is that it's due to ANY mass. There are places known in the Universe where there don't seem to be any protons and electrons (i.e. gas) present at all but we can still detect mass - its gravity deflects light, making distant background objects look distorted.
https://www.nasa.gov/vision/universe/starsgalaxies/dark_matter_proven.html


Is gravity caused by protons and electrons spinning through space creating a magnetic field ?
Q : Gravity - is it caused by protons or electrons spinning through space as within the earth thus attracting protons or electrons from within matter?? So if all the protons and electrons inside the spinning earth are creating a kind of magnetic field that attract protons and electrons from within matter that cause it the to fall towards the earth. The same theory could be applied to galaxies when a positive side faces a negative side they attract each other and collide, but when a positive side faces a positive side it pushes apart like two magnets, thus causing an expansion and eliminating the need for dark energy. Maybe this theory could eliminate dark matter and dark energy even though it was extremely poorly explained etc.

A : Magnetic fields don't affect all matter equally - there's no way they could explain why everything falls towards the Earth with the same acceleration regardless of mass or chemical composition. Non-metallic objects aren't accelerated by magnetic fields (except in extreme cases). The expansion of the Universe appears to be occurring at the same rate everywhere (albeit accelerating with time) and there's no way that effect would be seen with a magnetic galaxy repulsion model; if you have randomly-oriented galaxies there's no reason to suppose that more distant galaxies would be more likely to be repelled from one another. Indeed, if you've ever tried to push one bar magnet away from another, you'll have seen that the slightest imbalance causes one to flip over so they end up attracted and collide. As for explaining dark matter, it's actually possible to measure magnetic fields in galaxies and it seems they're not able to explain the observations.


What's beyond the Universe expanding ?
Q : Well, what ? Original question in Portuguese : Então, o que há além do universo que se expande ?

A : I think by far the best answer to that can be found here :
http://www.universetoday.com/1455/podcast-what-is-the-universe-expanding-into/
All I can add is that our everyday experiences don't really prepare us well for questions like this. When we see a balloon expanding or cake expanding in the oven, they're still expanding into something. But that's not the case for the Universe - the mathematics predicts something we're not really able to understand very well.

Another approach would be to imagine an infinite universe filled with galaxies. Now imagine those galaxies are all flying apart. Easy, right ? But that infinite universe can't be expanding into anything, because it's infinite. :)


Is that new theory that does away with dark matter a good idea or is it bunk ?
Q : There was a much hyped story a while ago about an "alternative to dark matter" relying on only gravity. Is that bunk or under evaluation?

A : As per my earlier answer, the maths is beyond me. People who do seem to understand it think it's not crazy, but it's still very early days. There are a whole bunch of reasons both observational and theoretical in support of dark matter. Showing that an alternative theory can do better in one or two aspects is easy; showing which is better overall is a huge task. MOND's been struggling with this for 30 years. None of it's bunk, but I wouldn't hold your breath.


Is dark matter any better than alternative theories ?
Q : Let's face it, Dark Matter is getting harder and harder to defend with the current explanations. Dark Matter is as good an explanation as any, at this point, by my (uninformed) guess.

A : Good lord, no ! It isn't even close. Dark matter very successfully reproduces the large-scale structure of the Universe on the correct timescales - no other model does that. Its basic results are confirmed by observations of rotation curves, galaxy motions in clusters, measurements of the CMB, and gravitational lensing (most especially but not limited to the Bullet Cluster). To do all this at the same time is no small achievement, one which no other theory comes in any way close to matching.

And yet...
http://astrorhysy.blogspot.cz/2015/08/seven-very-good-reasons-to-be-little.html

In the last year or so there have been some extremely interesting developments. Point 2 now appears possible to explain just fine in the standard model. There are tantalising hints that point 3 may also be solved - at least some of these new "ultra diffuse galaxies" appear to lie significantly off the Tully-Fisher relation, so it might just have been a selection effect. Point 5 is certainly by no means done, but there is an increasingly large opinion that many of the missing galaxies simply don't form so many stars (UDGs seen to vindicate this) and the other simulation/observation discrepancies have at least some possible explanations which are far less radical than chucking out dark matter (https://plus.google.com/u/0/+RhysTaylorRhysy/posts/9FvpHoU4v9q). Point 6 : well, I heard a talk by Marcel Pawlowski (one of the leading figures in the "planes of galaxies" area) a few months back. He convinced everyone of the existence of a plane around the Milky Way, but unwittingly had the opposite effect for other supposed planes. To my mind it now looks very plausible that the plane around the Milky Way is just a bit unusual with the other planes nothing more than dodgy statistics. Point 7 still stands : yes, it's weird that the Bullet Cluster exists, but not impossible - and no other theory can yet explain those observations.

I said I was 80:20 in favour of dark matter. Despite the progress I still hold to that figure. We don't yet know enough about the dynamical masses of the UDGs - some, perhaps many, could just be dwarfs and so might not explain the TFR after all. Explanations for the missing satellites still rely on models with many free parameters, so there's plenty of scope for errors there. And the planes-of-satellites observations remains tricky - it's difficult to detect really low-mass galaxies outside the Local Group, so the statistics are rubbish.

So while only a fool would claim dark matter as a certainty, it definitely isn't on an equal footing with its competitors.


Can modifying gravity explain away dark matter ?
Q : There are some new ideas that the apparent existence of dark matter points to flaws in our current theory/understanding of gravity. (The pull of gravity decreases with square of distance from center of mass, so outer planets in a given solar system orbit more slowly than inner ones, but, due to an as yet unknown error in gravitational theory, stars in outer regions of galaxies orbit the center at the same speed as the inner stars do.) And the answer may not be known until we have a fully workable, self-consistent quantum gravitational theory.

A : Well, there's MOND (Modified Newtonian Dynamics), which has been around for about 30 years. It's had some successes (it does a great job of explaining "small" things like galaxy rotation) but some serious problems too (motions of galaxies in clusters). Which I go on about at length here...
http://astrorhysy.blogspot.cz/2015/08/seven-very-good-reasons-to-be-little.html

The thing about MOND is that it's just an empirical fit to explain galaxy rotation curves - it doesn't have a physical motivation (in a sense, neither does dark matter since no such particle was predicted by the standard model of particle physics). And it's very hard to reconcile MOND with general relativity, let along quantum mechanics. There have been some hints that relativistic versions of MOND could explain other indicators of dark matter like gravitational lensing, but only hints - no-one really knows what it predicts.

The latest development is so-called Verlinde gravity, which is an independent attempt at a quantum theory of gravity that intriguingly reproduces some of the findings of MOND. However, it's fiendishly complicated and it will be some time before we know how successful it really is.
https://www.quantamagazine.org/20161129-verlinde-gravity-dark-matter/


How can you get something from nothing, as in the Big Bang ?
Q : How can you get nothing out of nothing.. before the bang.

A : [I presume the question means something from nothing since nothing from nothing is easy.]
If I knew the answer to that one, I'd be enjoying a well-deserved Nobel Prize instead of writing a blog. :P

Oh, sure, I could harp on about fundamental particles theoretically popping into existence from a pure vacuum, but what on Earth would be the point ? You can't imagine what pure nothingness is like. What does it mean to say that time didn't exist ? Why are the laws of physics the way they are and how can they exist if nothing else exists ? These are questions for philosophers and/or a large bottle of whisky. It might be nice to pretend they're science, but they're not.


How come gravity is stronger on smaller objects ?
Q : For instance, a neutron star is fairly small, in comparison to our Sun. Yet, isn't it's gravity pull of objects towards it much stronger than the Sun's gravity pull on the Earth? Or Is there something I'm not taking into consideration?
[This related to an earlier question about why gravity is weaker on the smaller but less massive Moon]

A : The gravity of the neutron star is indeed much stronger than that of the Sun, but only near its surface.

Say we have a star that's about a million kilometres across which starts shrinking. Anyone in orbit of that star isn't going to notice - any planets that it has will stay orbiting at exactly the same distance. The strength of gravity depends on how much mass there is beneath you and how far away you are from the centre of that mass.

But to anyone standing on the surface of the star (if that were possible and they were stupid enough to try it) things are very different. As the star shrinks, the mass beneath them doesn't change, but their distance from the centre gets much smaller. Half the distance and gravity gets four times stronger. Quarter the distance and it's sixteen times stronger. By the time the star's only a few thousand kilometres across, its surface gravity is about half a million times stronger than gravity on the surface of the Earth. When it's just a few kilometres across its surface gravity can be trillions of times stronger. But all the while, anyone at the original million-kilometre position of the surface or further away will notice no change in gravity at all. Any planets just keep orbiting exactly the same as they have for billions of years, except now everyone wonders why it's getting dark.

So a neutron star has insanely high levels of gravity close to its surface, but by the time you reach just a million kilometres or so, it's no worse than the gravity from any other star.


Could dark matter just be due to EM forces ?
Q : Many scientists make the assumption that the universe is finite. This is quite likely an errant assumption. That aside, I think it is electromagnetic force that keeps galaxies from flying apart as opposed to gravity. As I've said many times, electromagnetic force is many magnitudes stronger than gravity and also has infinite range. Also, as the article states, dark matter could very well exist in dimensions that we simply don't have the technology to detect. I think that we see effects of dark matter in areas where the barrier between dimensions is thinner or, perhaps, where the other dimensions are more in phase with our three dimensional reality.

A : Given enough mass, you can get as much strength as you want from gravity. The problem with electrical forces is charge induction : get a strong charge and it will attract opposite charges and neutralise itself. Gravity doesn't have that problem.

To my mind it doesn't really make sense to postulate the existence of truly enormous electromagnetic fields in space - where would they come from ? How would they affect the motions of material which is, overall, electrically neutral ? An EM field that moves a thin plasma in some turbulent motions, that I can understand, but not one that moves stars and gas in a coherent, ordered way on the scale of a galaxy.

Or, for that matter, a galaxy cluster. Gravitational lensing around clusters clearly indicates more mass than is visible, in agreement with the high speeds of the galaxies. But in particular, the Bullet Cluster (and several others like it), where the gravitational lensing of clusters that have collided shows that the dark matter resides with the galaxies, and not with the cluster gas that gets displaced by the collision :
https://www.forbes.com/sites/startswithabang/2017/11/09/the-bullet-cluster-proves-dark-matter-exists-but-not-for-the-reason-most-physicists-think/#756149621738

Just for the sake of argument, suppose you could get a deflection of light due to EM forces and explain the lensing that way. In that case you'd have to set that up just so that the EM forces associated with the galaxies produced the deflection, and not by anything in the hot, massive intracluster medium. The interaction of the clusters would also have to have left this lensing/EM forces unaffected and still in agreement with the motions of the galaxies in the cluster. I really find that very hard to imagine.


Could electromagnetic forces be underestimated ?
Q : [This was a follow-on to the previous question]

Thanks for your input. I wasn't trying to say that gravity has no role in the motion and interaction of celestial objects. I just think that electromagnetic force plays a larger, perhaps less understood, role as well. Scientists have seen gigantic blobs of plasma in space. Obviously, these blobs have an effect on objects around them. Perhaps dark matter plays a role too. I don't discount that possibility.

A : For sure, EM fields play a strong role in some situations. And I don't doubt they're not fully understood - magnetic fields are horribly complicated things, and no-one likes them very much. To what extent I'm not sure, but I suspect we're talking more on the scales of star formation activity rather than galaxy dynamics.

That said, dark matter worries me because it's not at all clear when we could draw a line and say the search has failed. At least with gravitational waves we could calculate a definite sensitivity limit beyond which we'd be able to confidently say, "nope, nothing there". Can't really do that with dark matter since there aren't really any constraints on the particles.


Do quasars prove that distant galaxies aren't old so the Universe can't be expanding ?
Q : [Regarding this article : https://www.livescience.com/62377-big-bang-evidence.html]
In this section they admit that we are seeing the galaxies as they were millions or even billions of years ago depending on their distance. How convenient! What they fail to explain though is that an observer in a distant galaxy would notice the same thing with regard to the Milky Way and other nearby galaxies. In other words, the observer would see quasars. Lots of them. So, assuming that the most distant galaxies are the oldest is a faulty assumption. In fact, many (but not nearly all) galaxies are close to the same age regardless of their distance. We have witnessed this with the maturity of many of the most distant galaxies.

A : Quasars appear more frequently in distant galaxies in massive galaxies. It's thought that every large galaxy goes through a quasar phase during its early history when it was denser, since there was more material available to fuel the black hole. This is extremely strong evidence for an evolving, changing Universe. It is absolutely true that an observer in a distant part of the Universe today would see "our region" dominated by quasars, but only because they'd be seeing it at a much earlier phase of its evolution.


Do blueshifted galaxies prove that the Universe can't be expanding ?
Q : I've already debunked their assertion that the universe is getting bigger. Many galaxies exhibit blueshift rather than redshift meaning that they are moving towards us; not away.

A : A very few (about a hundred : http://curious.astro.cornell.edu/ask-a-question/97-the-universe/galaxies/cosmology/539-why-are-there-blue-shifted-galaxies-intermediate) galaxies do show blueshift. This is simply because of peculiar motions; all of these are relatively nearby, and most are in the Virgo cluster. A massive system like this is absolutely expected to accelerate galaxies to high velocities so this is not in the least bit surprising, much less inconsistent with an expanding universe.


If it's supposed to be smooth, then why doesn't the Cosmic Microwave Background look homogeneous ?
Q : The assertion in this article that microwave background radiation is uniform is just plain wrong. It is fractal just as it should be. You can see "filaments". It's far from a homogeneous wall of energy. Does this look homogeneous to you?
https://upload.wikimedia.org/wikipedia/commons/thumb/3/3c/Ilc_9yr_moll4096.png/800px-Ilc_9yr_moll4096.png

A : The CMB is actually extremely uniform - that image simply uses a colour stretch to bring out the details. Variation across the CMB are of the order of 18 microkelvin, or 1 part in 100,000. It's incredibly uniform.
https://en.wikipedia.org/wiki/Cosmic_microwave_background


Why doesn't distance affect surface brightness of galaxies ?
Q : I would like you to explain one thing though and that is how distance is not a factor when it comes to brightness per unit area.

A : The flux from a whole source decreases in proportional to 1/d^2, where d is the distance. If the source is much smaller than a resolution element (the smallest thing any given observation can resolve), then of course they do look dimmer. More distant individual stars look fainter, no question about that.

But suppose the source is much bigger than a resolution element - let's stick with the floodlight viewed through a small (but finite) hole in a fence example from earlier. Imagine that initially the floodlight is initially placed very close to fence : only light from some small area of the floodlight will make it through the hole; the observer can only see a small part of the light.

Now suppose we can (somehow) chop up the floodlight into little bits, and we take away everything apart from that little bit of the surface we can initially see. If we now move this further away, it gets dimmer, just as we'd naively expect, regardless of whether we view it through a hole or not.

But suppose we keep the floodlight intact and move it further away. Now the view through the hole does not intercept only a small fraction of the floodlight - it covers a larger area, which is emitting a greater total amount of light. This larger area will necessarily also vary in proportion to d^2, exactly cancelling the loss of flux due to distance. Of course if we keep going, eventually the hole will subtend an angle larger than the entire floodlight, and at that point moving it any further away will again give a decrease in apparent brightness.

Although galaxies are composed of stars, which are point sources (except for one or two of the closest with the highest resolutions available), the effect is the same as for an extended source : each resolution element now intercepts more and more point sources with distance. So each pixel in your survey contains more sources, cancelling out the dimming due to distance.


But surely surface brightness does vary with distance, even if the equations say it doesn't ?
Q : But it seems obvious that this is not what we observe. Thus, debunking Olber's paradox. That is unless you are stating that the apparent uniformity, depending on what is being measured, of CMB is this effect in action. If this is not the case, then it seems to be an observation that only occurs under the right circumstances. Real world visible light observation not being one of those.

A : Ah, well, that first answer was the intro. In reality there are... interesting subtleties :). Most of this is in my original article on Olber's Paradox though - the links it contains are not idly chosen.

With regards to things on the scale of everyday life up to reasonably nearby galaxies, it's entirely correct to say that surface brightness doesn't vary with distance - with the trivial corrections for atmospheric extinctions and natural variation in surface brightness of individual galaxies. But for very distant galaxies it is not true, for several reasons.

First, galaxies evolve over time. Not only is the quasar frequency a function of redshift (i.e. distance and lookback time), but so is star formation rate. It varies significantly with redshift, by at least an order of magnitude, maybe a bit more (https://ned.ipac.caltech.edu/level5/Illingworth/Ill5.html). And since the most luminous stars have the shortest lifepsans, so a galaxy experiencing a high rate of star formation (and thus having a high number of bright stars burning briefly) will have a higher surface brightness.

Second, the assumption of constant surface brightness does not hold for an expanding universe - but the way in which this will affect things can be shown and gives a measurable, testable prediction. It turns out that surface brightness should vary as 1/(1+z)^4 (http://www.astro.caltech.edu/~george/ay127/Ay127_CosmoTests.pdf), where z is redshift (so you can see that if z is small, the correction is small, but if it's large, it becomes very important). This is the Tolman test, which was conceived in the 1930's - it's old hat. Once the corrections for varying star formation rate is made, it gives results completely consistent with an expanding universe that also rule out other theories (https://arxiv.org/abs/astro-ph/0106566; https://arxiv.org/abs/0905.3199).

But all of this of course only applies to fully formed, observable galaxies. The CMB might be different - starlight from a realm so distant that the galaxies themselves are indeed unobservable. However, this too can be shown not to be the case : the spectrum of the CMB is completely different to what that idea predicts.
http://www.astro.ucla.edu/~wright/stars_vs_cmb.html

While sometimes people do indeed describe the predictions of Big Bang nucelosynthesis in a circular way, the CMB itself is not as bad as all that. It is both consistent with the predictions of the Big Bang and inconsistent with other models. It's the main reason that virtually everyone abandoned the old Steady State model, because it just can't be made to work. Also the fact that distant galaxies have different morphologies, quasar fractions varying with redshift etc., that strongly indicate an expanding, evolving universe.

One really, really strong test for expansion that will eventually be done is to monitor the change in redshift of galaxies over time. The models predict that the recessional velocity of a galaxy should change as it gets further away. Instruments coming online in the next decade or so should have the sensitivity to actually measure this effect. But honestly, the evidence for an expanding universe is pretty damn well overwhelming already.


Could star formation rates be wrong because of time dilation ?
Q : Regarding star formation rate versus redshift, could there be another explanation for what apparent decrease in activity for higher redshift. Could the fact that the galaxies that exhibit higher redshift and, thus, are moving away faster, be creating an illusion of a slower rate because the light from new stars is taking longer to reach us? OK, the Tolman test provides a tentative answer for this. It's interesting that the function is similar to that of an inverted 4th order Julia; f(z) = z^4 + c. Or inverted as f(z) = 1/(z^4 + c). Since c = 1, the difference would be miniscule unless z is small.

A : Well, I'm concentrating on this aspect since galaxies are my thing, whereas with the CMB I'm going more out on a limb. However, as far as star formation rate goes, I think the effects of redshift are not directly applicable. Star formation rate is measured statistically through its secondary effects on the galaxy, e.g. changing its colour, rather than by watching individual stars turn on. Spectral line measurements give you the redshift, which allow you to correct its measured overall colour. The point is that you can (indeed, must) correct for the effects of redshift when doing this analysis - the effect is known and included, and accounts for much more complexity than the light arriving more slowly. Star formation rate estimates are complicated, but aren't done by watching individual stars light up.

Personally I think a finite, expanding universe is by far and away the most natural explanation of the data so far. The old model of a Steady State is utterly debunked. However, something of an Unsteady State, in which the Universe in infinite (in time and/or space) but experiences dramatic changes, might be possible. My objections to that one are more philosophical.


Does the Radial Acceleration Relation provide a good way to distinguish between different models of gravity ?
Q : So, there's a paper out by many of the usual suspects, arguing that the Satellite Planes of galaxies work they're doing is going to be the definitive test of dark matter. As if! I know you have thoughts on them and I'd love to hear them if it was possible...
https://arxiv.org/abs/1712.04448

A : RAR !

The sound made by dinosaurs and also the Radial Acceleration Relation, which I sometimes incorrectly call the MDAR (Mass Discrepancy Acceleration Relation). Whatever you call it, it's a relation between the expected and measured acceleration of stars and gas in galaxies. There's a really neat correlation between the two, which is thought to be odd because the dark matter ought to be dominant and not correlated with or controlled by the visible matter (which is only a small fraction of the total mass).

I did a pretty thorough write-up of this when it was really in vogue :
http://astrorhysy.blogspot.com/2017/08/this-isnt-law-youre-looking-for.html You'll want to read that one for the details I'll reference below, otherwise I'll just give a very superficial overview here. Skipping to the final sentence :

"What I suspect will happen is that we'll see a few more papers on this over the next year before everyone becomes horribly disillusioned, gives up and goes home."

Which is pretty much exactly what happened. The "discovery" of this "law" was hailed with some strong rhetoric and met with some vehement arguments because only the "discoverers" thought it was significant, pointing, they thought, to a flaw in the standard model of gravity. There are of course a bunch of disagreements in the field of galaxy dynamics, in part because comparing observations and simulations is bloody complicated. The neat thing about this relation was that it reduced things as much as possible back to basic physics, so that ought to avoid some of the uncertainties.

But within days of the first paper, others showed that this already happens in standard simulations. The initial claims that this result would be very difficult to reproduce with dark matter (especially by strong advocates of MOdified Newtonian Dynamics, who seemed to think it was impossible) were swiftly and decisively refuted. Later, other groups were able to explain exactly why this relationship arises in the standard model, showing that this is indeed entirely natural. The idea that the dark matter and normal matter should be uncorrelated is too naive : in essence, it's not that the normal matter controls the dark matter, but it's the other way around. Hence there's a correlation between the two. That's grossly oversimplifying, mind you.

The authors of this new paper appear to have conceded that essential point. Initially the relationship itself was taken as evidence in favour of MOND and against CDM, but the authors quote the scaling relations shown that can reproduce the RAR without much in the way of criticism, except for a few token statements that not all simulations show this. By and large, I commend their change of stance on this (but see below).

However, now the focus of attention shifts. While massive galaxies all lie on a very narrow RAR with low scatter, that's not the case for the dwarfs. It's already been demonstrated that dwarfs have much more scatter and possibly follow a slightly different RAR than massive objects. The scatter is noticeably worse for objects with lower quality observations, so some of that is undoubtedly just due to measurement errors (lower mass galaxies have fewer stars so it's harder to get exact measurements, and also they're not as dominated by rotation as more massive disc galaxies - hence they have more intrinsic variation of motions). But for the dwarfs which do have really good data, the stronger scatter and deviation seems to be real, albeit weaker than the faintest dwarfs with the poorer data quality.

Unfortunately for the MOND crew, simulations have already shown that this too happens in standard CDM simulations. They reference the work showing this ([31], see also blog post) but don't seem to comment on this aspect of the result. So here they appear to be trying to use this deviation as a way to test between MOND and CDM without acknowledging that this has already been shown not to be the case. They use their own CDM simulations and show there's an increase in scatter but not a change in the relation, claiming that the observed change in the slope points to an important discrepancy between theory and observation. Well, it might, potentially. But again, they don't comment on the earlier result which did show a change in slope, and there's not really much acknowledgement that the worst of the difference appears to be due to observational limitations. There's very little (if anything) that's new here, and as usual, the statement, "This result could hint towards a scenario in which there is no DM and the law of gravity needs to be modified along the lines of MOND" is technically correct, woefully misleading, and even this weak statement doesn't seem well-justified by the evidence.

"Villains who twirl their moustaches..." and all that. Though there are some fair points here, personally I've become convinced that MOND is just silly. It's true that a lot of CDM people are also biased, but the MONDian attempt to present a narrative of an oppressed group of dissenters is fundamentally flawed. You might not see this from any one paper, but overall that's exactly what they're doing. This isn't the only example of MONDers hearing but ignoring arguments. I know this for a fact because I inadvertently gave a lecture explaining why planes of satellites aren't real with one of the researchers in that field in attendance. His subsequent lecture notes ignore my arguments completely (admittedly I was rather rude about the whole thing, so - credit where credit is due - kudos to him for remaining civil).

In short, this deviation doesn't appear to be at all useful as any kind of test for MOND versus CDM. Although it's often stated that MOND predicts this relation, it'd sure be nice if some MOND group would please actually show this on the main plot - which again is not done here. Anyway there are just too main observational uncertainties.

It's an interesting question as to what would constitute a really good way to differentiate between the two theories. This approach might work if we had very high quality data of the 3D motions of the stars in satellite galaxies. But we also need a version of MOND (or equivalent) that's compatible with GR and can do all the other necessary predications. Despite decades of effort, we don't have such a theory. So for now I stand by labelling it as just silly. There just doesn't seem to be any need for the damn thing.


What do you think of this paper on missing matter ?
Q : This paper claims to have found some of the missing matter. What do you think ?
https://www.nature.com/articles/s41586-018-0204-1

A : The basic issue is that the amount of normal matter predicted by the standard model of cosmology is in disagreement with observations. Not only are there large amounts numbers (whatever) of missing galaxies, there's also a large quantity (hah ! take that, grammar Nazis !) of missing matter in general. Where's it gone ? Is it down the back of the sofa ? Under the mattress ? Did the dog eat it ?

One intriguing idea not discussed all that much these days is that maybe it lives in perfectly normal galaxies but is hard to detect. Could be the good ol' MACHOs (MAssive Compact Halo Objects) - asteroids, dead stars, black holes and the like. More radically it could be cold gas, which doesn't radiate all that much. What would be neat about this is that the amount of missing matter is just about enough to explain the flat rotation curves of galaxies, negating the need for dark matter on these scales (but not on the scale of galaxy clusters). Well, I suppose that's neat if you think of dark matter as a problem to be solved rather than an interesting entity to investigate, anyway.

But these ideas have been largely ruled out : gravitational microlensing surveys have turned up negative results for the former, while the latter would require an unknown mechanism to prevent the cold gas from forming stars (https://arxiv.org/abs/1204.4649). Fortunately, though cold material is hard to detect, hot material is also bloomin' difficult (I suppose normal visible matter is in a sort of Goldilocks Detectability Zone) - hot material quickly disperses to very low densities. The two leading candidates for reservoirs of hot material that could account for the missing mass are galactic halos and filaments. Halos are spheroidalish clouds of material that are bound to individual galaxies. Filaments are long streams of material linking many different galaxies in the so-called "cosmic web".

This paper presents evidence of gas found in halos. While it's damned hard to detect this gas by the radiation it emits, it's easier if it blocks the view of a bright background source like a quasar. In that case some of the light can be absorbed, if the material and the source are correct. As I learned in a workshop last week, this depends on some pretty complex physics. For example, X-rays might not directly be absorbed by the gas but can give rise to UV radiation, which can. I'm not going to pretend to even be qualified to attempt a summary of the processes at work, so I'm going to take the author's word for it that they've done everything correctly.

The point though is that the significance of this result seems a bit overstated. They've found pretty good evidence of the missing material, and rule out other explanations like self-absorption because that ought to vary on short timescales, which it didn't. But this is only for a single quasar, i.e. in a very, very small part of the Universe. It's cool and all, but it doesn't really say much about the overall problem.

I found this detection of hot gas in filaments to be much more convincing :
https://plus.google.com/u/0/+RhysTaylorRhysy/posts/FZXbAWmoHgY
For once the statistical nature of that detection is an advantage, because it means that filaments are present around large numbers of galaxies. That makes it much more convincing as a solution to the problem, or at least a big part of one. While there are a handful of other such claims for halo detection (see above link), I'd like to see much greater numbers before concluding that these too are an important part of the solution. What's particularly weird is that this paper doesn't cite the claims of filamentary hot gas discussed above.

I'd bet money on the problem being solved by a combination of filaments and halos. While the halos do need further scrutiny and much better statistics, in my opinion this issue has pretty much been resolved.


SPACE EXPLORATION

Why are space probes so frickin' SLOW ?
Q : What is preventing humans from making a spacecraft that go almost to the speed of light ? The fastest I have seen the Voyager I and II go in free space is around 33,000 kph... That is 40 yr old technology, and the "New Horizon" spacecraft is going to be going about the same speed... Why can't we make spacecraft that slice through space like a hot knife through butter ? Zoom !!! I'm ready to go, are you ?

A : The energy requirement to reach a certain speed scales as speed squared. Want to go twice as fast ? You need four times the energy. Ten times as fast ? A hundred times the energy. And that's at low speeds - as you get closer to the speed of light, things change... reaching light speed requires infinite energy. Even speeds just below that require outrageously insane amounts of energy.

Then there's fuel - you need a lot of it. To get a hundred tonnes to escape velocity, you need nine hundred tonnes of fuel. To go ten times faster, your thousand tonne rocket will now need to be more like ten billion tonnes.
http://www.wolframalpha.com/input/?i=Tsiolkovsky+rocket+equation

In principle you could go a lot faster if your rocket had a higher exhaust velocity. Problem is that that is very, very hard to do. See :
http://www.projectrho.com/public_html/rocket/index.php 


Are there any plans to make faster spaceships ?
Q : Are there potentially other, innovative or 'off the wall' ideas that can possibly help lessen the amount of energy and fuel we need for really high speed space travel ?

A : Yes there are ! A solar sail uses no propellant at all, so is not subject to the so-called tyranny of the rocket equation. It has to be very large to ever reach a high velocity with a reasonable payload, though (many hundreds of square kilometers, I think).

A similar idea is to use a powerful laser to push the sail. In that case the sail doesn't have to be so large because the laser provides additional pressure to that coming from the Sun. You'd need another laser at the other end to slow down, mind you.

There are other technologies that can provide higher exhaust velocities (ion engines) but less thrust. So they can reach higher final velocities but they take a long time to accelerate. Those are already in use for deep space probes, e.g. Dawn (http://en.wikipedia.org/wiki/Dawn_%28spacecraft%29#Propulsion_background).

Whether they scale up to larger masses, I don't know. I don't see why they wouldn't. For manned probes, you're still going to need a stonkin' great big ship because of the difficulties of providing life support for a long duration.

Then there's the Orion drive, which uses nuclear bombs to blast a ship forward. That would have both outrageous levels of thrust and final velocity.
http://www.rhysy.net/project-orion.html

Less crazily, there's the under-development VASIMIR engine (http://en.wikipedia.org/wiki/Variable_Specific_Impulse_Magnetoplasma_Rocket).

Probably the most exotic solution is the Alcubierre "warp" drive, but this relies on very exotic physics and should be considered as interesting speculation more than a practical solution.
http://jalopnik.com/the-painful-truth-about-nasas-warp-drive-spaceship-from-1590330763

That's just a few off the top of my head. For more, see this : http://www.projectrho.com/public_html/rocket/enginelist.php


Can we use the expansion of space to travel really really fast ?
Q :  Couldn't we use space's energy to our advantage ? After all, isn't space the thing that is moving the fastest ? Instead of fighting against space forces, shouldn't we somehow consider harnessing that very energy and use it to our advantage?

A : Unfortunately, space is expanding rapidly only on very, very large scales - as in between us and other galaxies. It can't be used in any useful way, since everything is moving away from everything else.

However, similar to a warp drive, we might just possibly eventually create a wormhole by folding space. In principle, there's no upper limit as to how fast you can deform space, so this could overcome even the limit of light speed. But this has similar problems to the warp drive - you need amounts of energy which are utterly impossible with today's technology, and, worse negative mass, which isn't yet even known to exist. It might not be impossible in the strictest sense, but it's going to take a complete revolution in physics and engineering to make it remotely practical.


Can a volcano launch a small moon into space ?
Q : Something that I have been thinking about was the massive explosion of Mt. St Helen in Washington state in 1980. What I have been wondering is what a 1/3 of its mass gone into the atmosphere, could some of it have escaped to form what might be another moon.

I realize that it is a far fetched idea, but I was wondering what the likelihood of another moon's influence on the planet Earth.

A : To send material into space you have to get it to around 11 km/s, otherwise it will just fall back down. Volcanic plumes can be extremely high (~50 km), but reaching that altitude can be achieved with a) much smaller velocities, more like 1 km/s - which requires 100x less energy or b) the material is hot, so it rises, like a hot air balloon, relatively slowly through the atmosphere.
http://www.reddit.com/r/askscience/comments/1gkiz3/could_a_volcano_eruption_theoretically_be/

Even if a volcano did eject some material into space, it's pretty much impossible that it could eject so much that it would become a gravitationally-bound structure, i.e. a Moon. The material would disperse pretty quickly just because of the random motions the explosion would have given it.

You'd also need a lot more mass than that of a mountain before there'd be any significant effects on the Earth - the mass of the Moon is many millions of times greater than that of any mountain.

However, strong tidal forces are certainly important elsewhere in the Solar System.
http://solarsystem.nasa.gov/planets/profile.cfm?Object=Jup_Io

Q : Earth's escape velocity is 11.2 km/s. The escape velocity doesn't depend on the mass. And the thrust created by a volcanic eruption is much much greater than the thrust required to launch a rocket. So if a rocket can achieve 11.2 km/s why can't a small particle from a volcano ?

A :  Because of the atmosphere. A rocket is continually accelerating - it is continually working against atmospheric drag. For a particle ejected from a volcano, it will have to be almost instantly accelerated up to at least 11 km/s, and in fact even more than this because of drag. The smaller the particle, the larger its surface area for its mass, so the faster it needs to go to overcome drag. And if it goes too fast, heating by air compression will cause it to burn up.

Q : If they can send monkeys, dogs and older men (former Sen John Glenn) into space, why not send younger people (12-21yrs old) into space and see how space can affect them. Not long term, but for say maybe two weeks. That way they can get an idea of what weightlessness can do to developing bodies and also give the young people an idea of why space exploration is so important and get their input on how they think things should proceed.

A : If we're serious about colonising space, then long term this may be unavoidable. However, we're certainly not there yet. Given that the effects of weightlessness are known to be pretty nasty (bone loss, muscle deterioration), sending people who aren't fully grown into space - willingly or not - would be highly unethical. 

There'd be a chance they'd experience problems years down the line - but to what gain ? There's nothing preventing them from becoming adult astronauts who can make a genuine contribution to the mission, rather than being an incredibly expensive, potentially dangerous (12 year olds on a space station... what could possibly go wrong...) payload.

For now we should continue to investigate the effects of weightlessness on animals and develop treatments for its effects on adult humans before we even think about risking development of children. By the time we ready to start thinking about kids in space, it's entirely possible we'll have centrifugal ships to mimic gravity, thus making the experiments unnecessary. In which case we're risking their health for nothing.

To my mind the consent of an 80 year old astronaut in full command of their mental faculties does not remotely compare to the consent of a 12 year old. Children simply do not have as much knowledge as adults (intelligence is not the same as knowledge), therefore their decision cannot be as informed as an adults. 

Space travel is inherently risky, but to me this does not seem like a risk worth taking. If, in the future, we come up with effective means to prevent the debilitating effects of weightlessness, then maybe. But it seems more likely we'll develop rotating spaceships by that point.

Q : Could someone explain to me why; on one of NASA's website pages they say Voyager's speed is 37,000 mph. Why would it not be able to go faster, due to the gravitational pull of the next star system ? Wouldn't it not speed up, with the pull towards that star system ?

A : The analogy I like for imagining the distances to the stars is to shrink the Sun to the size of an aspirin. At that size, the nearest star is about 270 km (170 miles) away, while Voyager 1 is just 140 metres away.

Q : Seriously, I need to make an urgent trip to Alpha Centarui next week.

A : Put it like this... Newton realised that if you fired a cannonball fast enough it could go into orbit. It took 270 years to make this a reality. And that was "just" to make the engineering work - for FTL travel we don't even know if it's physically possible, and indeed pretty much everything right now suggests that it isn't.

But, here's an interesting consequence of relativity : a spaceship travelling at close enough to the speed of light can fly across the Universe in a few seconds. That is, for the crew of the ship a few seconds will pass. For everyone on Earth, billions of years will elapse. Einstein not only spoiled Newton's clockwork Universe but he replaced it with one which is frickin' weird.

Current science does not say FTL is definitely impossible, just really damn difficult at best, and likely impossible. Wormholes and warp drives might work, but they are on the very edge of our understanding of physics. http://www.projectrho.com/public_html/rocket/fasterlight.php#id--Existing_Drives
"As a very rough approximation, you would need the energy the sun produces over 100 million years to make a wormhole about the size of a grapefruit." http://www.bbc.com/future/story/20140326-will-we-ever-travel-in-wormholes


Do astronauts age differently ?

Q : I'm thinking that because the people on the ISS are in orbit around the Earth, that they would age a bit differently than if they were further away, going away from the Sun.

A : Yes they do, by about a tenth of a second over the course of a decade. Time can pass at different rates depending on speed and gravitational field. For the ISS it's speed which dominates since the strength of gravity is about the same up there as it is down here. Fortunately, someone else has already done the maths :
http://ideonexus.com/2009/02/17/how-much-does-time-dilate-for-the-iss-astronauts/


Could we find new - possibly dangerous - elements on distant planets ?
Q : On Earth, we have the "Periodic Table of Elements". What happens if we go to another world and find new and different elements that have vastly different properties than those on Earth ? Do we mine then and bring them back to Earth, not knowing the consequences or do we leave them there ?

A : We can detect different elements by studying light. If you pass ordinary "white" light through a prim you can split it into its component colours (a rainbow). Using the proper instruments it's possible to see dark lines in the spectrum caused by different elements. This is how helium was discovered.
http://education.jlab.org/itselemental/ele002.html

I am not aware of there being any currently unidentified spectral lines that could be a similar discovery. AFAIK, theory predicts that it should be almost impossible to produce any heavier elements than those currently known by any natural process.

However, more exotic forms of matter are certainly possible. For example matter inside a neutron star is radically different from matter on earth. I'm not sure you could even label it as an "element" since it doesn't have any protons. You certainly wouldn't want to bring any of that stuff back to Earth.
http://io9.com/5805244/what-would-a-teaspoonful-of-neutron-star-do-to-you

Even more exotic - and uncertain if they exist or not - are strangelets, lumps of pure quarks. They're definitely not elements since they don't even have neutrons. And you really, really, really wouldn't want to bring any of them back to Earth.
http://www.wisegeek.com/what-is-a-strangelet.htm


Is NASA still using radioactive materials to power space probes ? Is there something else we could use instead ?
Q : Is NASA still using radioactive materials for it's "long range" space probes, like the New Horizons, or, is NASA switching over to a different type of engine and fuel system. Yeah, I know dilithium crystals are out, but I was thinking of some other form of propulsion...Maybe the use of strong magnets (http://science.howstuffworks.com/electromagnetic-propulsion.htm) to form some sort of energy source...Clearly, I'm guessing.

A : AFAIK nuclear power sources are the only thing we've got that are small and light enough for deep space missions. Beyond Jupiter sunlight is too weak to make solar panels practical.

I'd be very wary of the magnetic propulsion idea in the link idea. Getting something to "vibrate" only in one direction smacks of violating the conservation of momentum to me (which is basically magic). Acceleration by a magnetic launcher is a perfectly sound idea, but you wouldn't be able to use this to boost the spacecraft's initial speed too much. You don't want to reach orbital velocities low in the atmosphere, because heating from air compression would be bad. Useful on the Moon though.

Here's another idea that's maybe a more plausible use of magnetic fields for propulsion : https://en.wikipedia.org/wiki/Magnetic_sail


How fast would you need to go to reach Mars in 15 minutes ?
Q : Well, how fast ?

A : Speed = distance / time.
Time = 15 minutes = 15*60 = 900 seconds.
At 36,000,000 miles (the closest Mars gets to Earth), that means speed = 36,000,000 / 900 = 40,000 miles per second. At 245,000,000 miles (the furthest Mars gets from Earth), speed = 245,000,000 / 900 = 272,222 miles per second, which is faster than light.

Bonus : Oh. I was thinking that once the craft escaped the gravity of the Earth, it would go faster.

Bonus answer : Actually, if you only fire the rocket during take off, it will always continue to slow down (unless it goes into orbit of something). Even at higher than escape velocities it will always get slower and slower. Its speed will decrease less and less rapidly the further it is from Earth since gravity is less, but there's always some gravity acting to slow it, and nothing else pushing it forward.

Since the speed decreases less rapidly, at escape velocity (or higher) it would take an infinite amount of time for it to reach zero speed. At just below escape velocity it could take billions of years to stop and fall back, but it will happen eventually.

In reality things are more complicated because there's also the gravity of the other planets. If a rocket gets close enough to one of them then its speed away from Earth can indeed increase. With proper timing this can be extremely useful. http://gizmodo.com/how-did-we-get-to-pluto-so-fast-1716533356


When Voyager I reaches the heliopause, will we lose communications with it ?
Q : Well, will we ?

A : That will happen in about five years. I don't think reaching the heliopause will cause any communications problems itself, but unfortunately it's touch and go as to whether the power supplies will last that long.
http://voyager.jpl.nasa.gov/mission/interstellar.html
http://www.popularmechanics.com/space/deep-space/a8029/how-much-longer-will-we-talk-to-the-voyagers-11479518/


Could we live on a planet with slightly more helium in the atmosphere ?
Q : If we found a planet close to us and decided to ship half the worlds population to that planet, for them to colonize, but it had a slightly higher concentration of helium, in its atmosphere; what would the ramifications be ?

A : Helium is one of the most stable, unreactive substances there is. Having a bit more helium wouldn't cause any problems except that the concentration of oxygen would be a bit lower, and that wouldn't be good. According to this website :
http://classroom.synonym.com/minimum-oxygen-concentration-human-breathing-15546.html
... the oxygen content of the air can't be more than a few percent lower before problems start occurring (though, as someone else pointed out, that's not necessarily the case if the overall density is higher)


How do rockets work without an atmosphere to push against ?
Q : Surely rockets cannot work in space because there's no air for them to push against, right ?

A : Rockets don't work by pushing against the air, they work by the property of conservation of momentum.

Try the following experiment. Sit on a wheelie chair with your legs off the floor. Now try pushing against the air with your arms, keeping everything else as still as you can. What happened ? Nothing. You can't push against the air with your arms because it's not very dense and just moves out of the way.

Now try sitting on the chair and throwing a bowling ball at someone you don't like. This time you'll go backwards. Obviously the bowling ball isn't any better at pushing against the air than you are. If you want the over-simplified explanation, you are pushing against the bowling ball. Rockets, in a sense, push against their own exhaust - not the atmosphere. The exhaust is generated by their own massive amounts of propellant that they have to carry and doesn't need an atmosphere.


How tall does a building need to be for someone on the top of it to be weightless ?
Q : My intuitive guess is the height of geosynchronous orbit, since the orbital period at that distance is the same as Earth's rotational period. According to Google, that's over 26 thousand miles.

A : Geosynchronous orbit is correct. At that height, it would be impossible to fall off the building because you'd already be in orbit. But you'd still in freefall, just like onboard the ISS. Gravity at that height would otherwise still be noticeable, but much weaker than at the surface (about 0.02g, so it would take around 3 seconds to fall 1m, if you weren't orbiting).

Interestingly, if you fell off the building at any point below the top, you'd fall ahead of the building and go into an elliptical orbit around the Earth... unless you were too low, in which case you'd just crash and burn, or in fact the other way around.
http://astrorhysy.blogspot.cz/2015/08/twelve-mile-tall-tales.html

Technically you can never become weightless (without being in orbit) at any distance. Gravity just gets weaker and weaker the further away you go, but it never reaches zero.


Are "They" suppressing technologies much better than our primitive rockets ?
Q : Is it possible that we have made incredible progress in propulsion and space craft technology and it's just not in use by NASA and such yet? If so, why ? This audio documentary explains the probable reasons. It's a real awakening.

https://www.youtube.com/watch?time_continue=1969&v=8N0itbm5eQs

A : Original thread : https://plus.google.com/u/0/+DavidLazarus/posts/fSbzRFfND22?cfem=1 [Readers please note while I make every effort to answer all questions, if you want me to comment on an hour long documentary or read a lengthy scientific publication, I do so when time permits and entirely at my discretion]

OK, I watched the whole thing. For the benefit of readers the main idea is that secret near light-speed and FTL transdimensional technologies have been developed many decades ago but have been suppressed by higher powers. They are not quite impossible according to current physics but they would require stupendous energies and negative mass, and we're not sure if that's a thing.

I have to be brutally honest here : it doesn't make any sense to me at all. It's just the standard conspiracy theory fare : everyone is lying, only this group of people know the truth. Lots of claims about how people have come forward with radical claims but they have to be kept anonymous for fear of their lives. Oh how terribly convenient.
Fortunately I have a standard go-to response for this :
https://plus.google.com/+RhysTaylorRhysy/posts/XtsyKHd32Wy

In this particular case there are a number of things (leaving aside UFOs which is too large a topic to deal with here) which I do not find remotely credible :
-The claim that these technologies are being suppressed by those who are greedy and in power. Ummm, why ? With the resources of advanced space travel they could potentially have access to unlimited wealth and power. There's no advantage to holding them back.
-The claim that these technologies are being suppressed with ruthless efficiency on the mainstream media but apparently people on the internet are allowed to say whatever they want. It's not that difficult to take down a YouTube video.
-The idea that these technologies were discovered and made to work many decades ago. If that's so, it's extremely difficult to believe that no-one else has ever stumbled upon either the theory or the technology in all this time. If you can derive the principles of this stuff from physics that was known a century ago, it's not credible to suggest that no-one else has ever found it. Instant suppression, by any researcher in any institute anywhere on the planet ? No. Sorry, but just no. That is simply not how academia works. It certainly isn't perfect but it isn't anywhere close to that level either.
- "There's only one mind in the Universe and it's connected to everything". This is a classic case of replacing God with aliens.
- The unraised question : what are these few people who do have this technology actually doing with it ?

In short I don't think this presents any sensible ideas as to how or why such ideas have been suppressed. It presents a scenario in which there is an apparent short-cut to far superior technologies that either everyone else has missed for decades, or is ruthlessly suppressed except for a few plucky guys on the internet. Riiight.


What's the best way to see the curve of the Earth using current technology ?
Q : Can you help me understand what determines the escape velocity needed for a specific instance. Is it the speed of something, the angle and speed, of something ? Or does it have to do with propulsion ?

The reason I ask, is I have seen some GoPro camera videos, where they get to see the arc of the Earth and then fall back down to Earth. Bummer. What I was wondering is how to create something that might make it beyond the arc and keep going, to get a different prospective of the Earth and nearby surroundings. Does size have more to do with it than I am thinking ? Or is it the propulsion of the item that needs to be mathematically determined ?

A : The short answer is speed. If you reach 11 km/s in any direction then nothing else matters, you're leaving the planet whether you like it or not and you're never coming back. Unless you're heading straight down, in which case you're just leaving existence.

The long answer is that you'll need to go a bit faster than this because air resistance will slow you down (size will be important here). However, if you just want to go very high to see the curve of the Earth, you can - in principle - travel at any speed you want provided you keep supplying energy. This is difficult once you reach a certain altitude because the air is too thin to use to provide either lift or speed (jet engines propel themselves by pushing the air backwards).

When the air becomes too thin the only option is a rocket, which needs a lot of propellant. Unfortunately you can't simply slow down and take longer to reach the higher altitude. That turns out to need even more propellant, and so a much bigger, more expensive rocket. A rocket with a faster exhaust would need less fuel, but this isn't possible with today's technology.

If your budget is such that you'd opt for a GoPro camera, there probably aren't any good options. Possibly a small rocket launched from a high-altitude balloon or aircraft, to avoid as much of the atmosphere as possible.

If you have a few spare hundred million, you could try and develop something like the Skylon system :
http://www.reactionengines.co.uk/space_skylon.html
This a hybrid jet-rocket engine. It uses the air in the atmosphere (when available) to generate as much speed as possible, then when the air becomes too thin it switches to a normal rocket mode. In principle, this should give you the best of both worlds.


What if aliens make contact and they have a different name for planet Earth ?
Q : What happens when we come upon an older civilization than what is on Earth and they have a totally different name for Earth and our Solar system and the planets ?

A : If they're advanced enough to have found us, we bloody well us their names instead.


If I started firing off relativistic projectiles into the Universe and random, should I worry about hitting something ?
Q : If a projectile was fired at, let's say, 92% of light speed in a random direction (from Sol system or not), what would be the chances for it to hit something before the heat death of the Universe ? This includes both objects from the galaxy and beyond it. Or, put another way, how recklessly irresponsible can I be with relativistic projectile guns before seriously risking ruining someone's day at some point in the deep future ?
While the calculations for the galaxy seems relatively simple (though I may still get something wrong), I'm not sure how to proceed for beyond that.

A : On whether it's likely to hit anything, the answer that is a definitive, "no". The mean free path (http://hyperphysics.phy-astr.gsu.edu/hbase/kinetic/menfre.html) given the size and density of stars in the galaxy (for a point-source projectile) is by my calculations about 165 Gpc - much larger than the size of the observable Universe ! So even if the entire Universe had the stellar density of the Milky Way, you still shouldn't worry about your projectiles hitting anything by accident. Since the real Universe is even more empty than that (by many orders of magnitude), and expanding, the chances of any individual projectile ever hitting anything are extremely close to zero (the calculation outside the Galaxy is actually easy since there's bugger all there to hit - the problem reduces to, "how many galaxies would it have to pass through to stand a reasonable chance of hitting something ?").

This is neglecting gravity but that's OK because the projectile is moving so fast it will be essentially unaffected by everything it encounters. Literally the only thing strong enough to affect it would be a black hole and it would have to pass within a few km of the event horizon.

I made a crude estimate of what the actual probability of a collision would be. Assume the stars in the Galaxy are all distributed in a thin band of length 50 kpc (approximate circumference of the Galaxy) and height 0.6 kpc (real height of the stellar disc). Assuming the stars all have the radius of the Sun, the fractional area of this band covered by stars will be about 2.38E-11. Or, taking the reciprocal, the projectile would have to pass through about 42 billion galaxies before a collision should be expected - comparable to the total number of galaxies in the Universe. This will be a severe underestimate since it doesn't account for stars blocking other stars - in other words, you could fire at the very least hundreds of billions of projectiles before you should start to worry.


Would a relativistic projectile be stopped, fragmented and/or destroyed by interstellar/intergalactic drag ?
Q : Assume a spherical mass of 1000 kg.

A : That projectile, for interests' sake, has an energy equivalent to about 43 billion tonnes of TNT, or about 2 million times the energy of the Hiroshima bomb - more than the largest earthquake ever recorded, but still about 1000 times smaller than the asteroid that killed the dinosaurs. It would be an extremely unpleasant thing to encounter, but it wouldn't be a planet-killer.

On the drag, let's start with some ballparks.

Clearly the object will slow down significantly if it sweeps up its own mass through particles in the ISM. Let's approximate the cross-sectional area of the sphere to be 1 sq m just to make the maths a bit easier. The density of the interstellar medium is typically about 1 atom per cc, or 1 million atoms per cubic metre. Most of this is hydrogen, so that's a mass of 1.67E-27*1E6 = 1.67E-21 kg/m^3. Therefore to sweep up 1000 kg of mass the sphere will have to travel 5.988E23m = about 20 Mpc.

Or we could try Newton's impact forumla which considers how far a dense projectile will travel through a thin medium :
https://en.wikipedia.org/wiki/Impact_depth
For this case the result is about 150 Mpc, within an order of magnitude of the first estimate so that bodes well.

A more accurate treatment is provided here :
http://www.projectrho.com/public_html/rocket/slowerlight.php#id--Go_Fast--Bussard_Ramjet

However this still seems to only use classical (not relativistic) momentum. But it also shows (importantly) that velocity will never reach zero since the momentum of each particle will decrease as the ship slows down. Super super rough approximation : it will take about 15 billion years to reach 500 km/s, the escape velocity of the Milky Way at the solar distance.

So although there are a lot of crude approximations here, I think it's safe to say that drag will not be important.


Would nuking a dangerous asteroid at the last minute just make things worse ?
Q : This wasn't actually asked as a question but I found this video on YouTube and thought it needed some comments. You can read them in full in the link, here's the even shorter version.
https://plus.google.com/+RhysTaylorRhysy/posts/4scektvXpKX

A : Of course, if a dangerous asteroid threatens the Earth you'd want to stop it as soon as possible with whatever method of deflection or destruction you use. But if you've got no choice - the asteroid isn't detected soon enough, or your first efforts fail, then you might have to resort to firing a nuke at it at the last minute.

The video claims that actually no, don't do that, it will only make things even worse. I disagree. If you only manage to blast the asteroid into a few large pieces, then sure, this is not going to help. But I don't agree with the claim that if you do manage to blast it into smithereens (i.e. gravel-sized grains or smaller) then you're still doomed.

I don't think it makes sense to say that since the net KE of the debris cloud is still 2E23 J, the heat received by the Earth will be 2E23 J. Even if that were true, given the mass and heat capacity of the atmosphere it would only raise the temperature by around 40 C, not 1000 C as in the original video or even 100 C as a commenter on YouTube pointed out. I think he also calculates the power received by the Earth incorrectly - he's using the area of the surface of the Earth, but I would have thought the cross-section of the Earth to the debris would have been what you'd want. If so, he's underestimating the flux by a factor of two. Never mind that detonating just a few days earlier will mean that the >95% of the debris cloud will miss us.

But more importantly, this is waaaay too simplistic a way to estimate the change in temperature. First, gravel-sized pellets are going to burn up at >10 km altitude, so it's going to take a while for that heat to penetrate downwards by conduction. Secondly the glowing meteors are going to radiate isotropically, so ~half their energy is going to escape into space. Then, only a fraction of that energy is actually going to be absorbed by the atmosphere and raise its temperature. According to :
http://earthobservatory.nasa.gov/Features/EnergyBalance/page4.php

About 30% is going to be reflected back into space, 23% absorbed and 48% absorbed by the Earth itself. So given the isotropically radiating meteor, that means only about 0.5x0.2 = 10% of the energy emitted is going to raise the temperature of the atmosphere. Which means we're looking at more like a 4 C rise in temperature. That's certainly going to cause weather chaos, but it's hardly as bad as if the asteroid actually impacted.

Anyway my calculations are also likely similarly naive and simplistic. I really just want to point out that you can't always get away with approximations - the "details" can change the results dramatically. To say nothing as to at what point you could use the nuke to deflect the asteroid instead of blowing it to bits...

For comparison, the Pinatubo volcanic eruption injected about 5 cubic km of material into the atmosphere and it caused a global cooling :
https://en.wikipedia.org/wiki/Mount_Pinatubo#Global_environmental_effects

This asteroid would inject about 100x more material.

So I agree, a last-minute nuke is not what you want to do to avoid an asteroid - but not necessarily because it the asteroid would still heat up the planet. But we are talking about orders of magnitude difference here depending on if you blow up the asteroid a day in advance, a week or a month. So don't throw out the nukes juuuust yet.


Why does the Cygnus spacecraft need to do a de-orbit burn ?
Q : Can someone explain to me why Cygnus needs to do a de-orbit burn so it can burn up on reentry? Is it because if it came in too fast, it wouldn't have as much time to burn up ? Or a positioning thing, or...?

A : Since the obvious answer that it needs to get out of orbit somehow had already been raised and yet not deemed satisfactory...
It's more speed thing I was thinking. Why not just let it come in fast and hot. Why even do a deorbit burn. The thing is burning and breaking up over the ocean anyways.
... I assumed this confusion was about why the spacecraft needs to do such a careful, fuel-intensive maneuver. I'm guessing, but I bet when they say "deorbit burn" they don't really mean it's going to lose all of its orbital velocity - that would just waste fuel. Probably they just mean it loses enough velocity to send it heading downwards, it's still coming in hot.

Seems simple and clear enough to me - it's using just enough fuel to send it crashing to Earth so it will burn up, but not nearly enough to land safely. It DOES come in fast and hot, but it needs a de-orbit burn to do that.

Oh how wrong I was. Not about the de-orbit, but the state of mind of the questioner.

I'm talking about the Cygnus resupply cargo ship. If the thing is breaking up and crashing into the ocean, why would anyone care about saving fuel ?

Which other spacecraft he thought I was talking about I have no idea. Nonetheless I proceeded :

Because fuel has mass and transporting mass to space is what makes the thing so expensive. So on the way down you don't want to have any more fuel than you need - just enough to push it into the atmosphere and burn up in a selected area. Hence the deorbit burn probably uses the absolute minimum of fuel, so it's not the same as the much more carefully controlled deorbit of the space shuttle.

But apparently this answer makes me an asshole.

you obviously didn't read anything I said or have any clue what I'm talking about. I don't give two shits about the space shuttle. I'm asking about the Cygnus resupply spacecraft that burns up and crashes into the ocean once it undocks from the space station.

Can't really argue with logic like that. I can only assume he thinks the ship would instantly crash into Earth as soon as it undocks, for some reason.


Is there any research into wormholes and negative energy going on ?
Q : In a issue of BBC knowledge magazine whose topic was how to travel faster than light ... An article was there that talked about worm holes. In it was written that on putting negative energy into worm holes they can widen because they are microscopic. I knew that negative energy is a hypothetical idea but is there any research going on ?

A : Yes, there's definitely research going on into all kinds of far-out topics. Right now the concept of negative mass/energy is purely theoretical so although there are speculative concepts of what we could use it for, it's still an open question whether it's even possible for it to exist.
https://medium.com/the-physics-arxiv-blog/cosmologists-prove-negative-mass-can-exist-in-our-universe-250a980320a7#.legb0uat2

Research into theoretical and observational cosmology often seems like radically different things. Theoretical cosmologists deal with wormholes, multiple dimensions, and all kinds of other exotic physics. Observational cosmologists deal with measuring the temperature variations in the cosmic microwave background, the expansion rate of the Universe, and generally measuring things to greater precision. Most likely at least some of the more exotic possibilities the theoreticians are considering will turn out to be correct, but right now we don't really know all that much about some aspects of the Universe.


Can you spin a ball to make artificial gravity ?
Q : Can anyone tell me how i can spin a ball shape to make anything thats on it gets forced into it as if to replicate graverty ?

A : You can't really do it with a sphere, except very inefficiently. There would be gravity at the "equator" but not at the poles. So most of the interior surface wouldn't be useful. You'd have much better luck with a cylinder. The definitive, all-encompassing article is here :
http://www.projectrho.com/public_html/rocket/artificialgrav.php


Why do Space X love boats ?
Q : [On SpaceX's first succesful landing on a barge] someone explain to me (in words I can understand ) why they are doing this -- landing on a bobbing up and down barge (about the size of someone's back yard) in a gale-force wind blowing on the ocean.

A : There's a nice summary in this video :
http://www.theverge.com/2016/4/8/11392138/spacex-landing-success-falcon-9-rocket-barge-at-sea

Basically ground landings involve the first stage having to reverse direction to fly back to land. For launches that need a higher delta-v, this uses too much fuel. Also it's probably safer to keep the first stage over water (for everyone except the first stage, that is).


Should astronauts really be running marathons on the International Space Station ?
Q : These guys have nothing better to do ?!?

A : Actually no, there's nothing more important than exercise in space. If you don't do this, bones and muscles deteriorate which can cause very serious health problems.
http://www.nasa.gov/audience/forstudents/5-8/features/F_Your_Body_in_Space.html

In any case it was all of 3 hours out of a six month mission (which is barely longer than the amount of exercise required in space anyway), so I don't see why he shouldn't do this if he wants to.


Why is Martin Rees calling for an end to human space missions ?
Q : Just interests me as to why? What is the thought behind this?!? Or is it simply a major lack of thought and just simply sheer stupidity ?
http://edition.cnn.com/2016/06/16/europe/human-space-missions

A : He's not a stupid man at all, but he does tend to love the media spotlight. For example he's famous for stating that the human species doesn't have much chance of surviving the next century, which is pretty much guaranteed to grab attention coming from the Astronomer Royal, but doesn't have much basis in fact. Why would it ? He's an astronomer, not a politician.

Unfortunately the media have a tendency to weight scientific statements in a way that makes little or no sense. You almost never hear anything directly from climate scientists but almost everything the deniers say. Yet when Martin Rees or Stephen Hawking say things which are well outside their specialist area (Rees expertise is cosmology; he's a pretty good writer, but he's never done anything with human spaceflight) they're treated as all-knowing. Not that they shouldn't have their opinions or express them, just that the media give preference to people who are seen as experts when in fact they're really not.

What he's probably thinking is that in terms of pure cost and science, robots do better. And they do, to an extent. But in my opinion that just means we should invest more into making human spaceflight cheaper, because if the mission cost the same, just about anyone would like to have a human on board. They're a lot more versatile than robots, never mind that science isn't the be-all and end-all of space exploration : colonisation and asteroid mining aren't going to happen without humans.

Then again government-funded human space exploration hasn't delivered what everyone was expecting, but don't even get me started on that. :)


Could we build a space battle cruiser if we really, really needed to ?
Q : If we as a species had desperate need to put up some kind of space battle cruiser right now but no time to R&D many new technologies, what might we manage with a united humanity, real desperation, and almost unlimited budget allocation? Timeframe from R&D to battling humanity's enemies - three to five years. Three is safe, four risky, five is really pushing it. [Original thread with many interesting responses here : https://plus.google.com/u/0/+BenjaminBaugh/posts/C2qrVzYe69F?cfem=1]

A : If you've got a maximum of five years, I say go hell for leather chemical. You need a solution that works and works now, and chemical's all we've got. I'm not convinced either a VASIMIR or Orion drive could be made to work inside that time frame. Possible, yes, likely, no. Good enough for sci-fi ? Absolutely. But if we actually really needed a space battleship, I say throw every man woman and child into building something comparable to the Saturn V en masse. I assume that a threat from space requiring such a response is an existential threat, therefore, we may consider measures that would normally be considered abominable. The entire global economy needs to be re-worked to facilitate building as many rockets as possible. The only non-rocket building task will be to keep the rocket builders alive and able to build rockets.

For starters, let's assume you can take the whole of the US military budget and re-distribute it to NASA, SpaceX, etc. That's enough for about 190 Saturn V's per year (https://en.wikipedia.org/wiki/Saturn_V#Cost), which could surely be improved by economies of scale, assembly lines and improved manufacturing techniques and materials. So you're talking about a fleet of, say, 500-1500 Saturn V's in 3 years. Plausibly several times that since if if the species is facing an existential threat, you can also use other government spending as well as the economies and industries of Europe and Asia. So, guestimating, let's say of the order 5,000 Saturn V class rockets, or a payload to LEO of 700,000 tonnes. Call it a million; you can maybe get more payload with larger numbers of smaller rockets. Depends what it is you need to actually launch. Your real limit might not be from money but simply the number of engineers and industrial facilities suitable for rocket production. The goal is to turn the planet into one gigantic rocket factory. Everything else can get screwed.
https://blogger.googleusercontent.com/img/b/R29vZ2xl/AVvXsEjJBqyYMXcY6Qf24ooLDfp3oxvZ-2tlJiul9dLj6ysT3dC67PWQJxXejIGjfwug-5tSjlV2Lwtye2cv2WEn1LuHldX-ZKTAKIg3Vivy3YO8uQmWHe7oiQbGbZETtDcZa9PNY8hFndNUiw/s1600/Doctor+Who+-+Toclafane+rockets.jpg

Assuming this threat to be alien, I say fight them in LEO. If they're coming from another star system, their ships will hopelessly outclass anything we have and we won't stand a chance at intercepting them at long range. Our only hope would be to literally throw everything we can at them in a massive, last-second, all-out nuclear assault. In fact forget the battleship completely - just launch nothing but nothing but missiles. No space heroics, no fancy tech, just sheer unmitigated energy delivered to the target. At point-blank range no technology will allow them to doge/destroy every missile. Don't outsmart them, overwhelm them. As well as the real armed missiles, launch tens of thousands more lightweight decoys. I don't care how good your point defence is, you aren't going to deal with 50,000 rockets heading toward you with 5 minutes to spare.

There, that should ensure I never have to suffer political office.


Is the Orion drive as stupid as it looks ?
Q : [Comment was regarding this old project.]
That's some terrible efficiency. You should only have to detonate once. Even if you were on your way after the initial explosion the entire vessel would crush from mechanical fatigue. I mean come on man the front of the ship would be ejected from the rest of the ship like a friggin pinball and the crew turned into chunky salsa. That is one fucked up design.

A : If you'll read the accompanying blog post in the video description and also this earlier one (http://astrorhysy.blogspot.cz/2010/09/project-orion-how-to-nuke-spaceship.html), you'll see that I do in fact object to Orion actually being used, but not for the reasons you describe. So, first let me deal with your objections and then I'll summarise why I think Orion is a bad idea.

First, it's not terrible efficiency. Saying Orion is inefficient is like saying solar power is inefficient because it doesn't capture the entire energy output of the Sun. You have such insane amounts of energy to play with that it doesn't matter if you loose a lot.

Using a single (necessarily larger) detonation is impossible for practical reasons. First, you'd need a much bigger ship to survive the detonation because the acceleration would be so high - otherwise it would indeed be crushed. Second, you'd be relying on getting that first detonation exactly right - if your ship is pointing even slightly off-course, you won't end up going where you want to go. Multiple, smaller detonations completely avoids both of these issues.

With multiple detonations the entire vessel would NOT be crushed, that's what the shock absorbers are for. The initial acceleration of the pusher plate is indeed terrifying (hundreds or even thousands of g) but this is transferred to the rest of the ship much more gradually via the pistons. The acceleration would be no worse than a conventional rocket (a few g or less) so there's no reason the front part would somehow fly off. I can take you through the calculations to show this if you like, it can be done with nothing more than high school math.

However, one of the reasons I object is because it's not obvious whether or not the pusher plate could survive the detonation. Although various experiments indicate that it probably wouldn't be ablated by the heat of the explosion, AFAIK no research was done to see if it would buckle under the pressure. Possibly the gas bags connecting it to the pistons would be sufficient to prevent this, but it isn't really known. Buckling could lead to the plasma stagnating against the plate for much longer than it was designed for, leading to ablation. Also, in the event of the explosion being slightly off-axis, it's not clear to me what would happen to the pistons - would they be snapped off ? No-one seems to have addressed this.

Then there's the case of a failed detonation. If a bomb malfunctions and the chain reaction doesn't occur, the explosive within the bomb will still send shards of metal into the pusher plate - which could actually be more damaging that the smooth, brief fireball if it went nuclear. For a space-launched Orion this will only be a problem for the crew. For a ground launched version, it's not at all clear what happens in the even of a failure. With ~2000 bombs per ship, each with a mass of one tonne and launched at 200 mph via a gas-powered cannon, there must be some failure rate. If one detonates too close to the ship, what happens to the remaining nuclear material on board - does it trigger a single large nuclear explosion high in the atmosphere (which would not be so bad) or does it spread all that fissile material into the atmosphere ? Similarly if the ship is launched via rockets (as it my earlier video), what happens if one of them malfunctions - if it explodes, same question about the nukes, if the ship goes off-course, can it be aborted safely ? Then there are the political concerns, which are almost certainly going to be far more difficult to overcome than the engineering difficulties.

Orion is dead. It never had much of a chance anyway. But it's fun to speculate about it even so.


Could the ISS catch an astronaut if they fell off ?
Q : If an accident where to happen on the ISS and one of the astronauts tether broke, how fast would the astronaut be traveling and would the ISS be able to catch them in a "swing around" maneuver? I was just curious after watching a show and the astronaut went tumbling out of reach of the other crew outside. I didn't think the astronaut would move so quickly out of range. One of the risks, I guess.

A : Since spacewalks tend to be very slow, careful affairs, they probably wouldn't be moving at much more than a few cm/s. Maybe a few metres per second (a few mph - walking pace) at the very most. The risk of a tether breaking is very low - to my knowledge, an astronaut's tether has never broken. However an experimental satellite tether has broken, so it's not impossible.

In principle the ISS could be moved to catch an unlucky astronaut - it can move in three dimensions at about 1 m/s. But it isn't designed to be moved in a sudden emergency - it's a big, complex piece of equipment, and it's rarely moved without 24 hours notice. It's also controlled from the ground, not by the astronauts on board, so quickly moving the ISS in an emergency with the precision needed probably isn't possible.

Fortunately there isn't really any need to move the whole station. Astronauts are equipped with a small jeptack-style propulsion system called SAFER. This is designed to allow them to return to the station, or if their own system fails, for their colleague (solo spacewalks, if I understand correctly, don't happen) to rescue them. The risk of an astronaut floating off into the void isn't zero, but it's probably as low as it can be.


How can we improve communications with robotic spacecraft to help with exploration ?
Q : NASA is putting up a $1M challenge to guide a virtual robot on Mars. Hell, the R5 robot should have already been sent for deep space exploration to determine its capability of controlling equipment as a human would. Why doesn't NASA entertain this idea? All that is sent are spaceships which are controlled back on earth. The real challenge for deep space travel is communications. Somehow relay station's that boost signal strength need to be planted in space the same way we do on earth with satellites and transmission towers. Any ideas from all on deep space travel and the communication challenge? ?

A : Hell, the R5 robot should have already been sent for deep space exploration to determine its capability of controlling equipment as a human would. Why doesn't NASA entertain this idea?
But that is exactly what the competition is designed around - testing the robot before it's eventually sent on space missions. No point in sending a multi-million dollar piece of equipment on an irretrievable journey unless you're pretty sure it's going to work first !
https://www.nasa.gov/feature/nasa-looks-to-university-robotics-groups-to-advance-latest-humanoid-robot

Somehow relay station's that boost signal strength need to be planted in space the same way we do on earth with satellites and transmission towers.
As I understand it, there's no reason this couldn't be done except money. Every dedicated communications satellite launched to Mars is a satellite that could have been doing science. Since funding is limited and the number of Mars missions is uncertain, it's more cost-effective to have them use their own less powerful systems. Which is not such a big problem since a large radio telescope is capable of detecting a mobile phone on Mars. Of course it would be better to have a more powerful satellite transmitter, but it's a matter of priorities.
http://spaceflightnow.com/news/n1407/27marstelecom/#.V7gBOZh96hc

All that is sent are spaceships which are controlled back on earth. Not entirely. The current path is semi-autonomy. Rovers are directed towards certain areas by scientists on the ground but their onboard computers can decide the best way to get there.
http://www.extremetech.com/extreme/143884-how-nasa-drives-mars-rover-curiosity


How fast could an Orion-powered spacecraft get us to Mars ?
Q : I have read that this kind of propulsion could send a spacecraft to alpha centauri within 80 years, how long would it take to Mars ?

A : It depends very strongly on the the initial mass of the spacecraft, how much mass you want to send to Mars, and how much you want to get back to Earth. For Alpha Centauri in ~100 years you need a ship of several hundred million tonnes to start a colony, but if you just wanted to send a probe on a flyby you could use something a lot smaller.

For Mars, if you used a minimum energy transfer orbit a starting mass of 4,000 tonnes would get you 800 tonnes into Mars orbit and back to Earth, rising to 5,000 tonnes if the starting mass is 10,000 tonnes. That would take about 9 months. You could use a different orbit and get there faster, but the mass you'll be able to send will drop - maybe by a factor of a few. So you can either send 5,000 tonnes to Mars in a few months, or around 500 tonnes in a few weeks. It's probably better to go for the first option, because with that much mass to play with you don't really need to worry about the journey time - the astronauts can live quite comfortably.


What's your take on this article about generational starships being unworkable ?
Q : I mean this one : https://boingboing.net/2015/11/16/our-generation-ships-will-sink.html

A : Provocative, but defeatistly pessimistic. The article seems to me to have a whiff of, "if I point out enough arguments why this won't work, everyone will assume it's impossible to refute all of them, regardless of the strength of each individual argument". Personally, I find pretty much all of the arguments to be somewhat weak, and the combination of lots of weak arguments is not necessarily anything stronger. For example ,ost of the objections on grounds of physics seem at the extreme pessimistic end of the spectrum - only 10% c ? I think we can do better. Few are going to opt for a journey that long. A couple more points as examples :

"So in these areas of reproduction and work, generally regarded as basic to human meaning and political freedom, the society in the starship will have to rigidly control themselves. No matter their methods for achieving this control, they will end up living in some version of a totalitarian state. The spaceship will be their state, and to keep the spaceship functioning, the state will rule."

Pffft. Humans never have total freedom anyway. It can't be done. A spaceship will be a more extreme version of that but there's no reason that automatically equates to an unbearable life.

"The planet or moon they hope to inhabit will be either alive or dead. It will either harbor indigenous life, or it won’t. Both possibilities represent terrible problems for the settlers."

If they have the capacity to construct a multi-generational starship, they will most certainly be able to determine the nature of their destination before they leave ! I won't go through the rest of the article point by point; suffice to say that I think it's pretty much all like that.


Isn't it just too much effort for anyone to ever bother building a slow, generational starship ?
Q : Even if the problem is really super hard rather than impossible, the question we would then have to ask is, why spend our energy doing that ? Lots of space and mass in the solar system and it's a more easily knowable place. Why leave and create a place that can barely communicate let alone usefully trade with Earth ? Who's going to invest this much in a solution to an (existential) problem (e.g. surviving the death of the Sun) like that while it doesn't exist? Unless it's imminent of course. In which case sure, all kinds of desperate things are going to happen.

A : No-one's going to invest significant resources into a multi-generational ship unless there's an absolute need to do so. However, what about in the future when the species commands the resources of the asteroid belt and other worlds of the Solar System ? In that case, the resources necessary become far less significant. And then someone will, for their own inscrutable reasons, most certainly do it.


Will the expansion of the Universe limit our exploration of it ?
Q : In relation to the following video : https://www.youtube.com/watch?v=ZL4yYHdDSWs Even if the human race manages to travel at the speed that nearby clusters are moving away from us, the clusters are constantly moving away meaning it will take impossibly long to reach them now. How far do you think our 'pocket' will be when/if we manage to reach/exceed these speeds. Too far...

A : I really like this video, I think it does a very nearly perfect job. However I do have a problem with its major conclusion that human expansion is fundamentally limited to our own Local Group of galaxies. By "fundamentally" I mean that it claims the laws of physics themselves do not permit us to go any further, given the expansion of the Universe and that the speed of light appears to be a very strict upper limit. I'm not talking about the practical issues of travelling millions of light years, which are immense.

Call me hopelessly optimistic, but I don't think speeds of 2,000 km/s (the speed at which the Virgo cluster is receding from us) are so high we can't ever hope to reach them. That's less than 1% the speed of light ! Not easy, but not impossible by any means.

Rounding the numbers to make things easier, the Virgo cluster is 50 million light years away and moving at 1% the speed of light. So if it takes us a million years to develop relativistic spacecraft (pessimistic - we already know of some technologies that could do this in principle, e..g. solar sails and the Orion drive) by that point it will be just 10,000 light years further away. If we're travelling at a significant fraction of the speed of light we will only have to add an extra 500,000 light years to our journey, i.e. it will be 1% longer than we planned.

But the expansion of the Universe is known to be accelerating. However, a million years is completely insignificant compared to the lifetime of the Universe, so it's extremely unlikely that the acceleration would cause a large change on such a scale. Our measurements of the acceleration come from supernovae billions of light years away, so the timescale on which the expansion varies should be billions of years, not millions.

A much more detailed analysis can be found here : http://physics.stackexchange.com/questions/136056/how-does-the-hubble-parameter-change-with-the-age-of-the-universe The Hubble constant described how fast the Universe is expanding, while the Hubble time is roughly the age of the Universe - so 1 million years is about 0.007 Hubble times. You can see from the graphs in the link that the expansion doesn't accelerate significantly at all during that time. So, for certain, the effects of the acceleration are negligible on 1 or even 50 million year timescales. The video is absolutely right about us eventually not being able to reach or even see other galaxies, but I think it's simply wrong to say we'll forever be confined to the Local Group.


How can we build a gravity rocket ?
Q : Photon and neutrino rockets are too mainstream. How do I build a gravity rocket?

A : I would cautiously suggest this is not such a totally crazy idea for a civilisation capable of moving supermassive black holes (although it would not be efficient). Tidal forces aren't that strong even near the event horizon, so there might be a spot in which you can safely orbit the hole... you'll need to be going as fast as the recoil anyway, though, or you have no chance of staying in orbit. And you'd need precise timing to make sure you don't get hit by those beamed gravitational waves. And all this would only get you a maximum speed (from some quick Googling) of around 5,000 km/s. Couple of papers on this for the enthusiast (the maths is way, way beyond me) :
https://arxiv.org/pdf/gr-qc/9702005.pdf
https://arxiv.org/ftp/physics/papers/0110/0110042.pdf



Let's send some bacteria to Mars, yes ?
Q : And why it's not possible that we can also send a unicellular in mars to see how a new colony can develop there....

A : There's absolutely no reason we couldn't send a colony of microbes to Mars to see how they survive if we wanted to. But we don't want to do that until we're certain there's no life on Mars already, otherwise when we go searching for it it would be much harder to tell the difference between the native life and what we sent. However there have been experiments performed to see if microbes could survive exposure to space :
https://www.nasa.gov/mission_pages/station/research/news/eu_tef/


Could Star Wars have saved a lot of needless trouble if there'd been an astronomer aboard the Millennium Falcon ?
Q : This article :
http://www.syfy.com/syfywire/star-warss-40th-anniversary-thats-no-moon-what-if-it-were
... says, "It worked out well in the end, but if they had had an astronomer on board, a lot of effort could have been saved." Would you like to comment on that ?

A : I can see George Lucas being mad enough to release yet another version, this time with Chewie having a hitherto unknown degree in astronomy. A 30 minute diversion occurs in which Chewie writes and submits an observing proposal. Initially rejected due to missing the deadline, he tries again and is succesfull. He then proceeds to take photometric observations of the Death Star from multiple angles to determine its shape, probable distance and composition. He writes up his findings in a 13-page paper and submits it to a major journal. The reviewer takes a dim view of his "alien megastructure" theory and insists he first considers other, more natural explanations. A protracted argument ensues, which only ends when Chewie writes a rude letter threatening to rip the editor's arms off unless he's given another reviewer. The second reviewer quickly concedes and Chewie celebrates publication success. He's then awarded time on an interferometer and planetary radar system to more reliably determine the structure of the Death Star. This necessitates a lengthy training montage as Chewie's never used radar before and has to learn about delay-Doppler imaging and shape modelling. Even so, he still can't properly determine the composition of the Death Star so he has to submit yet another proposal for spectroscopic observations, finally concluding that this large, oddly-shaped metallic object is indeed "no small moon". All this happens in Wookie without subtitles, and it's still better than the Holiday Special.


Yeah but really, isn't using nuclear bombs as propulsion just plain silly ?
Q : Really, nuclear bombs used as propulsion? Yes I remember the old NASA film concept of Orion. I could just see how another civilization would greet us if a ship like this ever venture into their star system. Reminds me of an episode of SPACE 1999 called; Voyager's Return. The Alphans encounter the Voyager space probe that has been wandering the cosmos propelled by the Queller drive- a faulty nuclear engine system. Following the probe is a fleet of alien ships from a world devastated by the drive.

A : Oh, it's a stupid idea alright, but it's not a planet killer. That is a physical impossibility. The total yield of the pulse units is less than 1% of what was actually detonated in the atmosphere back when above-ground testing was still allowed. And this wasn't a good thing, but it didn't cause planetary devastation.

It's a stupid drive because it demands extremely high levels of reliability with a much higher risk factor if anything goes wrong. Planet killer ? Not a chance. City killer ? Absolutely, if it goes off-course and cannot be aborted.


Have we imaged the event horizon of a black hole yet ?
Q : This article claims we have :
http://sciencecosmolgy.blogspot.cz/2017/09/watchfinally-black-hole-original-imaged.html

A : This is extremely misleading. Leaving aside the technical details of how the image is constructed (which is very different from ordinary photography), the data from the first observations are still undergoing processing. I was at a talk by the lead scientists of the European section of the EHT earlier today, and it was stated very clearly that they don't have an image yet.


Does FAST have a design flaw ?
Q : I've been following the development of FAST. It's a bit of a mess. I do not understand why PRC decided to go with these enormous triangular reflector segments, crippling its ability to work at shorter wavelengths. FAST should have been designed by astronomers, not engineers.

A : The limit on wavelength range might not be directly due to dish design so much as the design of the whole telescope. FAST's design relies on being able to deform the dish to form a parabola to point it at different parts of the sky. This gives it a great advantage over Arecibo in that it will be able to survey about twice the sky. But it also takes a serious penalty : Arecibo has a heavy, instrument-rich platform so multiple receivers can be maintained simultaneously and switched at the press of a button. FAST has a much lighter platform, so changing receivers becomes a much bigger procedure.

No-one really know how well deforming the dish will work in practise. The largest deformations at the long wavelengths are of order 1 m, over an area 300 m across. For higher frequencies you'd need to do this with sub-mm precision. Dish design might help but you also need extreme accuracy on the tiedowns and an enormous amount of tension to beat the effects of thermal expansion (which over this large an area will vary from point to point). This may not be possible at all, regardless of dish design (it isn't yet clear how well it will even work at longer wavelengths).


Can I shoot down a satellite with a beam of powerful radio waves ?
Q : Rasers. Radio wavelength lasers. I am trying to conceive of a submarine that can use a RASER weapon while underwater, to attack targets in orbit. My question is: which radio wavelengths are good at traversing several meters of water and then the entire atmosphere, while being mostly unaffected by the ionosphere?

My current idea is that small openings in the metal hull will allow some of the radio beam to penetrate the ship. This turns the spaceship into a microwave oven where the beam bounces on the internal surfaces of spaceship millions of times. Even a 'radio transparent' materials will eventually absorb all of the radio beam's energy if it gets traversed millions of times... What about semiconductors in electronics? Will they suffer short circuits and ionization like the electric sparks you see when you put a fork in a microwave?

A : Interesting !

For transmission through the atmosphere you're good to go - radio telescopes like the GMRT wouldn't work otherwise. For underwater attenuation I'm far less sure. A quick Google says that submarine communications work at much longer wavelengths, but I guess they need more than a few metres of transmission. If you need a raser to penetrate the entire atmosphere but only a few metres of water, I can only assume you're building the world's first orbital fish-killer...

Except for superconductors, metals will always absorb some fraction of the radio waves - if they didn't, radio sets and radio telescopes alike wouldn't work. So I suspect the situation will be rather like trying to deflect a powerful laser with a reflective mirror - you might be able to limit the damage, but that's all. Possibly it will even make things worse because radio waves induce electrical currents in conductors, which at energy levels necessary for spaceship-zapping are going to be... bad. Getting the radio waves to bounce around is, I think, unnecessary : if you have enough energy in the wave to theoretically damage a spaceship, then it likely will suffer damage.

The same is true for insulators : some of the energy will be absorbed, although there won't be much in the way on induced currents, just heating.

What I'm uncertain about is how to quantitatively calculate how much energy is absorbed by the metal from the radio wave. I suspect this is highly non-trivial and dependent on the material, frequency of the wave, and the geometry of the situation, e.g. :
http://teacher.nsrl.rochester.edu/phy122/Lecture_Notes/Chapter36/chapter36.html
http://www.physics.umd.edu/courses/Phys122/Streets/25_EM_Induction_and_Waves.pdf

The best I've been able to find online in relation to satellites is this : http://www.madsci.org/posts/archives/2002-03/1015162213.Eg.r.html "Example: a silvered mirror reflects about 95 % of light power and about 95% of radio power and absorbs about 5 % of light and radio power. "

I think it's safe to presume that circuitry is going to be fried.


If you were a dictator how would you fund the space program ? And why don't ESA or other space agencies build more space telescopes ?
Q : [This one was relation to my stated intent to become an evil dicator* and build a fleet of Hubbles.
* Evil from the perspective of the military; somewhat less so from the perspective of scientists.]


NASA (taxpayer funded) paid for the first HST. Can your country pay to build the next one after JWST (NASA built that one, too). Where are the other countries in building their own space telescopes, instead of counting on the US?

A : As an evil dictator funding won't be a problem. I'll just cancel the Olympics or some other useless project. :)
http://astrorhysy.blogspot.cz/2012/08/ten-things-we-could-have-done-instead.html
Though building Hubble is a bit pricey, so I might have to sell a few aircraft carriers. Oh well.

ESA was and is involved with Hubble :
https://www.spacetelescope.org/about/history/esa/
... and the JWST :
http://sci.esa.int/jwst/45728-europe-s-role/
... and has built Gaia all by itself...
http://sci.esa.int/gaia/
... and led the Herschel project...
http://sci.esa.int/herschel/
... also Planck...
http://www.esa.int/Our_Activities/Space_Science/Planck/Telescope

... and many more, but that's probably enough. :D


As an evil dictator, what would you do to the space program ?
Q : [In relation to my statement that I'd cancel the Olympics to fund exploration] It's OK to cancel the winter Olympics . Not a fan of the ice and snow. BUT, leave my swimsuit Summer Olympics alone!!! Or we might have an "issue".

[Different questioner] As the Grand Vizier to Emperor Rhys, I should commission a fleet of robots capable of repairing, upgrading, refuelling and repositioning scientific vehicles on orbit. I'd re-park Hubble in a geosynchronous orbit right away and build another good UV Great Telescope. Infrared and UV seem to be the most scientifically productive regions of the spectrum.

A : The principal goal of my reign will be to complete the following checklist :
https://imgs.xkcd.com/comics/telescope_names_2x.png

JWST can stay, but will be renamed the Why The Hell Did We Think Calling It The James Webb Space Telescope Would Be A Good Idea Telescope instead. That'll teach 'em. And whoever it was that decided to pull the plug on GALEX is going to live to regret that.

The NSF, a.k.a. "where science goes to die", are not going to enjoy my administration. I shall do unto them what they did unto Arecibo : continuously shave the funding, hold lengthy bureaucratic meetings producing voluminous yet ambiguous outputs, make unfounded allegations as to their mediocrity, and produce a detailed deconstruction plan on the grounds that, "just in case". Oh, and I shall make them live somewhere nice and remote where only a fraction of the locals speak English and the infrastructure is lacking, just to keep it realistic.

And fear not, the future of the swimsuit contest is under no threat from increased space funding.
https://www.youtube.com/watch?v=lKq-tScUWdg

Finally, once we're done solving world hunger, which will take about twenty minutes for a sufficiently ruthless despot, all interferometry will be banned on the grounds that "it's too bloody complicated". Interferometers shall be replaced with equivalent-sized single dish telescopes constructed in space, thus getting terrifying resolution and sensitivity whilst having trivial data reduction procedures.


Isn't the Fermi Paradox easy to solve ?
Q : [This was an interesting little discussion, which I've edited into a slightly more readable format here]
The solution to the #FermiParadox is that there is no paradox. One does not simply colonize other planets. Note that research facilities or tourism do not count as colonization - nor does an actual colony that still depends on its mother planet. To credibly claim that there is a paradox one should first show how to colonize space. Not fantasize. Show. Let us not simply assume it can be done.

[I asked which part of the process of colonisation was not yet adquetely demonstrated to be possible]

Supply chains at the destination. Moving mass is not colonisation. It is transportation. Colonisation is about creating a viable ecosystem at the destination. That has not been demonstrated. Transportation sure has been. In my original post I mention research facilities and tourism, which I do believe we can do. Generational, self-supporting colonies on other planets, or even on Earth inside airtight domes, without any support from outside, have not been demonstrated and cannot be done. Generational means that people can be born, live, and die inside the colony over indefinite generations. If you don't have that you can't colonise space.

My basic point is that it requires life adapted to the destination. If the ultimate destination is very far, the intermediate destination would be empty space - and empty space is not another Earth-like planet. Mars is not Earth-like either - and even if it was, going to a neighboring planet is almost meaningless to the goal of colonising other star systems. Colonising a neighboring planet means little or nothing to the Fermi paradox. And I am also very skeptical of just that. I don't think anyone has shown how to colonise Mars. Visit Mars - sure, but not colonise.

A : "Generational, self-supporting colonies on other planets, or even on Earth inside airtight domes, without any support from outside, have not been demonstrated..."

Agreed.

"... and cannot be done."

Why ? In a completely closed system recycling abilities must be finite, but finite can be a very, very long time. On another planet having equal resources to, say, Earth, I don't see it making much difference if there are airtight domes or not. So long as resources can be used to support the colony, they don't need to come from Earth. I'm not seeing any fundamental barrier preventing in-situ resource utilisation.

I am not seeing any reason why another planet cannot be adapted to provide an Earth-like environment. It does not have to be over the whole planet, just enough to provide a biosphere for sustained generations. Say an underground complex or above-ground glass domes. Either way, other planets possess raw materials and I don't see what would prevent colonists from exploiting them. Mining and electrochemcial techniques for extracting elements are already known, so there's no reason they couldn't be used on other planets. Thus there's no reason that a colony couldn't be self-sustaining.


How come radar resolution is high when the instruments are so small ?
Q : Like in this suitcase-sized instrument, for example :
http://www.bbc.com/news/science-environment-42725349

A : The neat thing about radar is that its resolution is dependent on frequency and time resolution, not aperture size. I'm impressed it has the power to get such a good image.


Has Virgin Galactic ceased manned flights ?
Q : I've lost track of that.

A : As I understand it, they are still conducting manned flights with SpaceShip Two, but they haven't yet attempted a suborbital flight.
https://www.space.com/g00/36679-virgin-galactic-spaceshiptwo-unity-feather-system-test-flight.html?i10c.encReferrer=aHR0cHM6Ly93d3cuZ29vZ2xlLmNvbS8%3D&i10c.ua=1


Could a parachute bring a rocket engine safely down to Earth ?
Q : [United Launch Alliance have this scheme to re-use rocket engines by discarding them while the rocket is in flight, parachuting them to the lower atmosphere and then catching them with a helicopter. Compare this to SpaceX's approach of landing the entire rocket : https://spaceflightnow.com/2018/03/20/ula-touts-new-vulcan-rocket-in-competition-with-spacex/]
Why can't they just parachute all the way to the ground...?

A : It sounds like they need a drone ship (or similar) to collect the engine. I guess they want to fly over sea to minimise risk of errors over populated areas. And presumably, like Space X, they want to prevent the engines from getting contaminated with salt water. Isaac Kuo rightly adds that launching over water isn't done by choice because there's no existing launch complex where that would be an option.


Nuclear weapons are risky things to use against asteroids and would be as bad as if it impacted.
Q : I think nuclear weapons should be the last option for defending against asteroids. If the rocket after lunch will explode in our atmosphere, we would have the same problem as like the meteorid would impact. The rocket would also be a good target for 'terrorists'.

A : It obviously depends on the details, but a nuclear deflection doesn't necessarily need to have nearly as much energy as the impact energy the asteroid would have if it hit - provided it's launched early enough. Changing the asteroid's velocity by a few cm/s may be enough, and the energy requirements for that are modest. Additionally, airbursts are far less destructive than impacts (witness the Chelyabinsk event, where the explosion was reckoned at about 0.5 megatonnes but did no more than damage windows).

Of course if you want to blast the asteroid into little bits, that's another matter.


Is Orion crap because of the spherical explosions ?
Q : inefective since majority of the blast is travelling in a radial way

A : No, for two reasons.
1) Efficiency doesn't matter if you have the energy of a nuke at your disposal. It's a bit like saying that solar panels are inefficient because they don't capture the entire energy output of the Sun : it's true, but irrelevant. You have way, way, way, WAY more than enough energy captured to get the job done. Far more than any chemical rocket.
2) By configuring the mass distribution of the bomb, you do actually capture most of the mass with the pusher plate. The mass is not distributed in a spherical blast, but in a quite narrow cone.


Is NASA's sterilisation procedure just the same as natural daylight and therefore pointless ?
Q : Guess what they use to sterilise those spacebound artifacts? Yep. Ultraviolet light. UV-C to be precise. You can buy 'em on Amazon. And we have this ozone layer over our atmosphere which has kept every DNA-based organism safe from ultraviolet light since life emerged on this planet.

A : In addition to other answers which clarify actually the sterilization procedures are more complex than that, I'd just add that it's also known that bacteria can survive direct exposure to space :
https://www.popsci.com/technology/article/2010-08/bacteria-survive-553-day-exposure-exterior-iss


Could a giraffe take over the Arctic ?
Q : [On SpaceX's Tesla not being sterilised and people worrying it will infect the rest of the Solar System]
It's always a bit puzzling why people seem to assume that bacteria from earth would be able to run rampant on another planet anyway. That's no more true than a Giraffe being able to take over the Arctic if you transported some there. Maybe it might happen, but far more likely they'll get wiped out. Sounds more like newspaper filler or publicity seeking.

A : ... I, for one, would watch a movie about a giraffe that takes over the Arctic.

Though I think the problem is rather different. If you don't sterilise a spacecraft at all, you have no idea how many bacteria/microbes are on it or how many species (quick Googling suggests that human skin is home to billions of individual microbes and thousands of different species). Give evolution a large enough starting population of extremeophiles and time will do the rest.

... but while the little buggers may be hard to kill, they're not invincible. If they can survive an uncontrolled crash on Mars, then they've likely been introduced there by Earth-originating meteorites anyway. Spacecraft designed for controlled landings need to be sterilised, but I don't think there's much point in worrying about accidental collisions.


Could we build a solid ring around the Earth for aliens to detect ?
Q : [On the claim that we could detect alien satellites around distant planets, if they had sufficient density : https://plus.google.com/u/0/+RhysTaylorRhysy/posts/PHKnFZ41dT7?cfem=1. I pointed out that this requires really extreme densities, basically amounting to a solid ring, which the author's don't acknowledge. They also require vast numbers (hundreds of billions) of satellites, and that to me seems pointless.]
Then again, a solid geostationary belt is a thing in moderately advanced SF.

A : Yes, but the claim in the paper is that this structure doesn't require any fundamental advances. That's true if you just have a genuine cloud of satellites. It's not the same if you're building a solid band, i.e. a megastructure. This is worth emphasising because the authors insist in several press releases that this idea is interesting because it requires no advances fundamentally beyond today's capabilities, but their own numbers show that this is not so. As another alien megastructure, it's fine, but it's wrong to claim this is something we can envisage construction : building a solid ring is very different from just launching lots of satellites !


How about that Alcubierre drive, eh ?
Q : do you have some trace on the alcubierre drive? I've seen fan fiction and a popular explanation on the solution which looked as if he sucked some mathematics from his fingers, but this is as far as I got. Has someone tried to go for a small-scale prototype?

A : As far as I'm aware it's purely hypothetical, with no consensus on whether it's even theoretically possible - let alone how to go about testing it, even on a very small scale.


Your Orion-drive variant of the Discovery from 2001 : A Space Odyssey is all wrong, isn't it ?
Q : [See the video here and the explanatory blog post here]
Except that the long shape in the movie version of Discovery was to save shielding mass by using distance to shield from the radiation of a nuclear thermal engine. A Project Orion ship would not be concerned about shielding mass and indeed the more mass the better with a nuclear external pulse drive. A Project Orion Discovery ship would be stouter, with a wide base, something like a large bullet. Also it could easily be a 300 man ship. Given that the crew in the movie was quite small, Clarke might've been thinking of NASA's trimmed down version that was designed to be launched in a Saturn V hence a 60 foot diameter blast shield and longish shape. Project Orion was intended to average 1g acceleration on one nuke per second, that would be a bit interesting in the centrifuge which Clarke designed to mimic Lunar gravity. To those who think a nuke would blow it apart, do some research, the whole point of the mass and shocks was to even out the stress and the nukes were to be small shaped charges with plastic disks to form a directed plasma burst to the blast shields, the nuke sizes were from sub kiloton to one megaton, all of this was calculated and tested even to testing steel plates in actual nuclear explosions, it would not have shook itself apart. It would take thousands of nukes, 8,000 sub kiloton for a ground launch from Earth, about 2,000 for an Interplanetary transit (they were thinking of a quick run to Saturn) and 300,000 megaton nukes for a fly by mission to Alpha Centauri (I find the 1 megaton reference in Wiki to be somewhat suspect). No, those shouldn't be heat radiators, a nuclear thermal or nuclear electric would need heat radiators but a Nuclear External Pulse just needs to be big, massive and have plenty of shielding though some heat radiators for environmental would of course exist. Project Orion was killed because of the Nuclear Arms Treaties preventing nuclear weapons in space not because of environmental concerns though a ground launch would have an impact similar to the open bomb testing of the 50's. The construction of thousands of mini-nukes for a Project Orion would certainly attract the ire of other countries and the attention of terrorists who would no doubt desire a few to go missing.

A : See the link above. The design is based on the original, official concept art depicting Discovery with an Orion drive. I then took some liberties with this very utilitarian, ugly design, and tried to slim it down and make it more elegant. The intention was to produce something that looked movie-worthy rather than realistic. Hence I kept the overly-large radiators because they I felt they benefited the style more than they detracted from the realism.


Are telescope eyepieces overrated ?
Q : Referring to this new telescope for amateur observing :
https://www.space.com/40625-stellina-telescope-no-eyepiece-digital-interface.html
I think there are cases where having a true optical view would be preferred over electronic, but this is an interesting development.

A : Mixed feelings about this, though I don't do much amateur astronomy.

On one hand, professional astronomers haven't used eyepieces in decades, so in that sense this is just bringing the public up to date. It would also be much easier to use than a classical telescope and better for sharing and recording the experience. There would be lots of great uses for this at star parties, school trips etc. where you don't want some silly technical issue at the wrong moment.

On the other hand a keen amateur (also a professional) friend of mine has often told me that it's remarkable how much you can improve your observing technique through experience. I don't think you'd get that with this - you'll just be taking pictures, which will be basically the same to everyone using the same instrument.

I'd prefer a hybrid system where you get the benefits of an easy to use machine with a shareable electronic screen but an eyepiece (if necessary detachable) that gives you a raw view of the sky.


Should we nuke the ISS from the ground to take out the xenomorphs ?
Q : [This was in response to this spoof graphic :
https://www.theonion.com/international-space-station-tented-to-spray-for-xenomor-1826362305. Naturally the response has to be to misquote Aliens, "I say we land and nuke the entire site from the ground."]
It's still perfectly logical to nuke it from orbit. Once nuked, it is no longer orbiting. Also, nuking from orbit would likely be much cleaner than performing an atmospheric detonation. Previous work: https://en.wikipedia.org/wiki/Starfish_Prime

A : But all our nukes are on the ground. I suppose we could launch one into orbit first before it detonates, but then we've got a philosophical "true cheeseburger" definition problem of where the nuke was from. :P
https://www.smbc-comics.com/index.php?id=4015

Lest there be any confusion, the nuke is going to have to be at the same altitude to do any damage. A lower altitude atmospheric blast has no way of affecting the ISS except possibly through the EMP, but taking out the ISS's electricity supply isn't what I had in mind. I want the Xenomorphs to be dead and quickly.

The nuke could be launched on a suborbital (parabolic) or hyperbolic escape trajectory, or go into orbit first, I don't mind which. Either way, it's got to a) come very close to the ISS to do any damage, because blast waves aren't nearly as much fun in a vacuum; b) be launched from the ground. :P


Could we detect alien civilisations by searching for their orbiting satellites ?
Q : This article says yes !
https://www.centauri-dreams.org/2018/06/15/enter-the-clarke-exobelt/

A : Yeah but the paper's own numbers say "no". At least, it wouldn't be anything like as advertised. The main selling point is that we already know how to launch satellites, so instead of the much more speculative prospect of vast megastructures like Dyson spheres, we ought to be on much surer footing with this one. It sounds like a sensible, reasonable extrapolation of existing technology into the near future. Unfortunately, it isn't.

The problem is that the numbers in the paper show that a detectable exobelt would actually be a solid ring. Launch rates to build it in the time frame specified average out to 80 satellites per second. This isn't really anything like an extrapolation of current technologies, it's another sort of advanced alien megastructure. There's nothing wrong with in in principle, but it's not what's advertised - building a solid ring is very different from launching lots of satellites (and why would you want a ring around a planet anyway ?). It's not that the idea is flawed so much as it is that it's been sold in a very misleading way.

More details here : https://plus.google.com/u/0/+RhysTaylorRhysy/posts/PHKnFZ41dT7


Could a multi-generational space mission be made smaller using stocks of frozen genetic material instead of living humans ?
Q : I suppose frozen embryos might be a good idea anyway. They're lightweight, and may be useful for repopulation without introducing an excessive extra genetic bottleneck after unforeseen die-offs. The benefits/costs ratio is massive.

A : The author of
this particular study, Frederic Marin, is a former housemate of mine and we collaborated on this research. Since my input was limited to providing comments on the papers and not doing any of the hard coding work, I'm in the acknowledgements but not on the author list. We're actually writing a more detailed press release on this right now.

I did tell him about using frozen genetic material making the need for a minimum crew size potentially very small indeed, and this is discussed in the first paper (there doesn't seem to be much need to repeat it in the second). So the goal here is really to examine the minimum size of a self-sustaining population requiring no artificial genetic controls, rather than the optimum configuration of a space mission. Which is an interesting to number to have in the unlikely event of a complete cryogenic failure.

Actually this minimum number of about 100 is what you need to guarantee success, all being well. If you want to live a bit more dangerously you can push it below that. Maybe even quite a ways if you're prepared to accept a little inbreeding (even without things getting Lannisterian).

See also : https://plus.google.com/u/0/+RhysTaylorRhysy/posts/eTc1Lym9wsR


Wouldn't it be very difficult to dock with a rotating space station ?
Q : [Regarding my video "
Space Station V"]
In order to go in you have to time it because your ship would get destroyed

A : You do have to be careful but it's not quite that bad. There's no need to land on the ring, which is rotating quickly. Instead you land in the central axis. Since everything is rotating at the same angular speed (the same number of revolutions per minute), this means the centre is actually moving much more slowly in terms of equivalent linear speed (i.e. m/s). So all you have to do is get your rotation to match that of the station and fly slowly into the centre. Of course if you get things really very badly wrong, then you're in trouble. But this is true for any docking operation.


Did the Chinese not know their plants on the Moon would all die ?
Q : [In response to this article :
https://www.space.com/43025-china-moon-mission-plants-dead.html]
Let me get this straight. They went to moon to grow plants and could not have anticipated this would happen? Something wrong with this story.

A : No, they knew this would happen. As it says, this was an extra experiment, not a core project. Also :
https://www.abc.net.au/news/2019-01-17/change-probe-china-cotton-seeds-moon-dies/10723336

"Professor Xie Gengxin of Chongqing University, who led the design of the space experiment, told Chinese newspaper Inkstone his team had anticipated the plant's short lifespan. He explained: "Life in the canister would not survive the lunar night."


MISCELLANEOUS

Dude, aliens !
Q : What is the probability that another complex carbon based life form exists within our Universe?

A : Short answer : nobody knows. Longer answer :
http://www.space.com/25219-drake-equation.html

We only know for certain of one place where life started, so it's very hard to judge how probable it is that life gets going if conditions are suitable. However, the first life on Earth is generally have thought to have appeared pretty soon after the Earth formed, pretty much as soon as it was habitable :
http://www.astrobio.net/news-exclusive/when-did-life-on-earth-begin-ask-a-rock/
The last time I had a chance to talk to an expert on this (some years ago now), most of the evidence for the earliest life on Earth was considered a lot more controversial than generally appears in the popular press. If so, life might have taken a lot longer to get going, meaning it may be relatively difficult for life to begin.

The Universe is so large it seems very unlikely that there's no other life out there somewhere. However, I also like Arthur C. Clarkes's quote :
“Two possibilities exist: either we are alone in the Universe or we are not. Both are equally terrifying.”


Time travel
Q : I've read where if you eat only good, plant based foods and practice fasting and meditation, you'll see visions. Is this time travel? [Yes, this was a real question - someone actually asked this !]

A : Either that or hallucinations brought on by malnutrition.


"Sensors are picking up a massive gravitational field, Captain !"
Q : How can we detect gravity ?

A : The simple answer : we detect gravity because it makes things accelerate. Apples fall from trees. Cats fall over. The Moon goes round the Earth (circular motion is acceleration). Planets go round the Sun. So in that sense we're detecting the gravity of the Sun by the very fact that we're going around it.

The complicated answer : how strong gravity is depends on the mass of an object. How much acceleration an object of any given mass produces is related to a parameter called the gravitational constant, G. Measuring this is very much more difficult than measuring acceleration : http://phys.org/news/2014-06-method-gravitational-constant.html

Yeah, yeah, but what about if you're a windowless box, eh smarty pants ?
[Questioner did not actually say this]

Ahh, now I get you. You can't detect the acceleration inside the box since everything is accelerating at the same rate. Well... almost. In practise the box will have some finite size, so the acceleration experienced by the bottom of the box will be slightly higher than at the top. This is only going to matter near an object like a black hole.
http://science.howstuffworks.com/science-vs-myth/what-if/what-if-fell-into-black-hole2.htm

Or, you could fire a beam of light from one side of the box to the other. The light is trying to go straight, but the box is accelerating, so it doesn't hit the exact opposite point : it appears to curve.

http://www.dummies.com/how-to/content/einsteins-general-relativity-theory-gravity-as-acc.html


Should we ignore stupidity ?
Q : Should we care about harmless pseudoscience ? Do stupid people matter ?
[This was not exactly how the question(s) were asked, but I need a standard response for such matters]

Stupid people do not matter. Stupid people (or of course intelligent people with misunderstandings) with enough talent to make their nonsense go viral, on the other hand...

A : When pseudoscience goes viral, it undermines a lot of very hard work that has gone on in trying to understanding the Universe. Which is something we don't do for fame (unless you're Tyson or Cox, you won't get any) or money (even less chance of that), or because it's a cushy job (competition is just as fierce as in any other sector), or because we have any political axes to grind (galaxy evolution doesn't care if you're left-wing, a fascist, or a small turtle). Astronomy is, as much as is humanly possibly, knowledge for knowledge's sake - there are no campaigns to ban dangerous chemicals based on observations of the Orion nebula, no decisions about people's rights because of the Triangulum galaxy.

We do astronomy because we think it's worth doing. At a professional level, astronomy is funded almost entirely by taxpayers, and we want to give them the truth, whatever that may be, because that's what they pay us for. It typically takes around 7 years of higher education before you start making meaningful contributions in astronomy, let alone coming up with ground-breaking results. Moreover, it's based on exactly the same proven physics that's led to things like rockets, radios, telecommunications, microwaves, radar, satellites, electrical power... pretty much the entire basis of the modern world really. It is not a soft science. So, if a non-scientist comes along and makes a fancy video with some rudimentary, but easily correctable errors, claiming to have overturned an extremely basic fact of a subject that tens of thousands of people choose as a career for the sole reason that they think knowledge is worth knowing.... well, you can imagine how we feel about that. Instead of communicating our latest hard-won discoveries to the tax-paying public, we have to spend time convincing them about things that were established beyond all doubt centuries ago.

For a thorough analysis, see my article on quackery; for a really comprehensive review see this.


What is time ?
Q : Well what is it then ?

A : I don't know.


What's the difference between gravitational waves and tidal forces ?
Q : I have heard of "gravitational waves" in space, but not "tidal waves". Are they different ?

A : This was in reference to a press release describing the collision of galaxy clusters as a "tidal wave" (http://www.ras.org.uk/news-and-press/2619-giant-cosmic-tsunami-wakes-up-comatose-galaxies).

The simplest answer is that "tidal wave" generally means a wave in the normal sense - in a fluid - whereas "gravitational wave" refers to a change in gravity.

The tricky part is that tidal forces (not waves !) are caused by a difference in gravity between two places : e.g. one side of the Earth experiences less force from the Moon than the other. This is very similar to a gravitational wave, but not quite the same. A gravitational wave, as I understand it, is a change of gravity that moves through space, whereas tidal forces - at any given distance from their source - don't vary over time.


Can the impossible happen anyway ?
Q : So if everything is bound to happen in infinite time and space , happenings which are impossible are bound to happen.

A : No. Impossible things don't happen. It's like saying that if you have an infinite number of empty rooms, you must have some monkeys. Actually you don't have any monkeys, just an infinite number of empty rooms.

There are plenty of truly impossible things. You can't fit a square peg through a round hole of equal diameter, for instance. You cannot put two 1 metre rulers next to each other (in flat Euclidean space, smarty-pants) and have them stretch a total of three metres. There are also lots of shapes which are impossible in the real world, e.g. http://thelastpsychiatrist.com/images/impossible%20fork.JPG

Granted, if the Universe is infinite, then all unlikely things - even if extraordinarily unlikely - should happen somewhere. An impossible thing may be defined as something which cannot happen even in an infinite Universe.

I personally don't believe in a infinite Universe, for reasons I explain in detail here : http://astrorhysy.blogspot.cz/2015/04/the-most-astounding-fact.html


What will the first aliens we discover be like ?
Q : If there extra terrestrial life forms on other planets, what do you believe the first ones humanity would encounter would resemble? My opinion ? Life forms not dependent on heat. Cold blooded beings. I imagine they would be considerably smaller than humans.

A : There's no way to know. It will depend entirely on the conditions on the planet they've evolved on.


How can photons last for billions of years ?
Q : Does anyone know how long a photon of light can exist for ? What is its lifespan and how come it can travel over billions of light years and over billions of years ? I just don't understand how it can keep going...

[Several people had answered this by invoking relativity. The proper time - the time "experience" by a photon - is zero, no matter how far it travels. While true, and tremendously interesting, it seemed obvious to me that this was somewhat beyond the questioner's level of understanding, and in any case absolutely unnecessary. There's a much simpler and equally correct explanation.]
A : Even ordinary particles of matter will keep going indefinitely provided they don't hit anything - Newton's first law of motion :
https://www.grc.nasa.gov/www/k-12/airplane/newton1g.html
If you want to make something speed up or slow down, you have to give it energy or take energy away from it. You can't do that unless it collides with something. The question, "why does it keep going ?" would be better stated as, "why would it stop ?". In everyday life things slow down because there's always some medium around to stop them (like air). In space, that's not the case, so they keep going.

Photons don't need to have very much energy to keep going for billions of years, it's just that they're not losing any.


Don't photons get chilly in the cold of space, and lose energy ?
Q : Space is pretty cold in places. Why doesn't the photon's energy dissipate ? How can it travel through such extremely cold temperatures and yet maintain its speed and energy ?

[This question followed on from the previous one. I could have mentioned that photons lose energy as space expands, but I felt that this would confuse the issue and decrease understanding.]
A : Temperature is a property of matter. The higher the temperature of a substance, the higher the velocities of its atoms (speed of movement through space for liquids gases, vibrations around some fixed point for solids). When the atoms collide, they may lose energy as photons. The higher the temperature, the faster the motions, the greater the rate of collisions, and so the more photons are emitted.

A single atom doesn't really have a temperature. If left in complete isolation, the electrons inside it would keep orbiting the nucleus indefinitely, and if in motion, it will keep going. It needs to interact with something to lose energy. You can't really say that any individual atom in a substance has a temperature - it's the effect of many interactions throughout the substance that gives it a temperature. It's a bulk property.

"Space is cold" only in the sense that the atoms which are present have very small motions. But the space itself between them doesn't really have a temperature as such. In most of the Universe, the density of matter is quintillions of times lower than in Earth's atmosphere. So the chance of a photon colliding with anything in this very cold gas is small. To answer your question, it's best to think of space as being completely and utterly empty.

More importantly, photons don't really have a temperature either. Most photon "collisions" just result in the photons passing straight through each other. Photons are the way in which energy is lost by atoms, but they themselves don't lose energy except when they interact with other atoms. In a true vacuum, there's nothing for them to interact with so they lose no energy.


Would two black holes with the same electrical charge repel each other ?
Q : Let's say the Black hole in the Milky Way is negatively charge and the black hole at the center of Andromeda is positively charged, that would mean they would merge. Yet, if both were of the same charge, would they not repel each other... sorta like magnets ?

A : Yes, that's true - if they were charged. In practise, it's not thought to be likely that a black hole could gain a significant charge. If it was formed from charged matter, it would be extremely difficult to compress it enough to overcome gravity.
https://en.wikipedia.org/wiki/Charged_black_hole

In principle a neutral black hole could gain a charge by swallowing some charged matter. But if it did that (most matter is electrically neutral overall so it's not terribly likely) it would have a charge... and therefore attract matter with the opposite charge more strongly, and become neutral again.


Is there any known way of reversing time ?
Q : Is there any known event or force that could reverse time on a spatial plane. (Hoping I used that term correctly). My thinking is this: Since the "Big Bang" created the multiverse, instantaneously, could the reverse happen. What got me thinking was Newton's 3rd law: "For every action, there is an equal and opposite reaction". Is there an event like a time portal, that could appear, reverse time and then move it forward at a different chronological timeframe.

A : Travelling to the future is easy : just go really fast.
http://www.dummies.com/how-to/content/slowing-time-to-a-standstill-with-relativity.html

Travelling into the past is much more difficult and might be impossible. The closest thing I can think of to a "time portal", which, so far as is known, is not prohibited by the laws of physics, would be a wormhole.

A wormhole is a a short-cut through space that allows you to travel from one place to another almost instantaneously - much faster than the speed of light through ordinary space. To use them for time travel, the idea is that you keep one end fixed, then drag the other off at near-light speed and then bring it back again. Time passes more slowly at high velocities, so the moving end of the wormhole will have aged less. You can use this to travel back in time, but not to a point before you starting mucking about with the wormhole.
http://www.andersoninstitute.com/wormholes.html


If the speed of light isn't constant, why do we measure distance in light years ?
Q : I just want to know. Why is distance still measured in light years? I thought the speed of light was proven not to be constant and that it's affected by temperature and gravity.

A : Nope, it's a constant. Gravity causes it to shift frequency, but not speed.
http://astronomy.swin.edu.au/cosmos/G/Gravitational+Redshift
In the laboratory it is possible to slow down light but AFAIK not under any conditions we know of that can occur naturally.
http://news.harvard.edu/gazette/1999/02.18/light.html


Can you escape the gravity of an object ?
Q : If you move far enough away from an objects, will you eventually be totally unaffected by its gravity ?

A : No. Relativity says that the speed of gravity equals that of light, which is finite. Objects which are separated by enough distance do not feel each other's gravity. However, since you can't travel faster than the speed of light, you can never escape the gravitational field of any object which is already affecting you.

You can get far enough away from an object so that its gravitational field becomes negligible, but how far depends on the mass of the object (and what you define as "negligible").


Do gravitational waves ever stop ?
Q : What I am wanting to know is at the instance of a gravity wave, does the wave have an "ending" or does it go on, til there is some sort of other event? The reason I ask is, from my standpoint, space is "endless". So, I'm not sure if a gravity wave would continue on, but weaken, over time.

A : That's exactly what would happen. Like gravity itself, its strength decreases the further it goes, but it never reaches zero.


What's the latest in the search for gravitational waves ?
Q : Speaking of gravitational waves, I have yet to hear any news about gravitational waves. Where can I get better sources of information ?

A : I seem to recall that the LIGO detector is finally at the stage where they expect to be able to detect gravitational waves :
https://ligo.caltech.edu/news/ligo20150918
So in principle they could make an announcement at any moment. Given that the search has been going on for decades, I wouldn't count on it being any time soon though. If they've got any sense they'll wait until they're really sure.


If two gravitational waves collided, what would happen ?
Q : Would they just cancel each other out, or create a new wave ?

A : Actually, when two waves meet, they don't create a new wave. If two wave peaks meet, they add up to produce a higher wave but only temporarily - they keep going afterwards as though nothing had happened. If a wave peak peaks a trough, they can cancel out completely, but again it's only temporary and they keep going as though nothing had happened.
It's much easier with pictures :
http://physics.bu.edu/~duffy/py105/WaveInterference.html


Have there been any radio signals from aliens yet ?
Q : Have there been any signals from space using the existing radio telescopes around the world ? Also, are we transmitting anything into space for other worlds to pick up ?

A : Signals from aliens no, with the cautious caveat of the "Wow" signal :
http://www.universetoday.com/122409/what-was-the-wow-signal/

There have been a handful of attempts to deliberately transmit signals that aliens could detect :
https://en.wikipedia.org/wiki/Active_SETI#Realized_projects
Most (all ?) of these wouldn't be powerful enough for our technology to detect if we were on the receiving end, and rely on the aliens having much more sensitive equipment.


Are you sure it's not aliens ?
Q : Why do you say: "It's still premature to assume it's aliens" ? Since the universe is some 13-15 Billion yrs old, wouldn't the earlier segments of the universe have already developed some sort of technology to communicate ?

A : It's not about whether aliens exist or not - they might very well do. The problem is that we have no idea how advanced aliens are or what their capabilities might be. So you can use aliens to explain literally any unsolved mystery : fast radio bursts, where are my socks, anything. If you assume it's aliens, you risk missing discovering interesting natural phenomena that could tell you a lot about the Universe that wasn't known before.

Of course that doesn't mean that you can say it's definitely, definitely not aliens, or that SETI projects shouldn't take a look at potentially interesting targets. It's only that if you say it definitely is aliens that you run into trouble.


Why do we assume aliens use known technologies to communicate ?
Q : I don't understand why we assume any alien civilization would be using radio waves for communication. Just because we haven't discovered something else yet doesn't mean that it doesn't exist.

A : Well if they were using a method we haven't discovered yet, we wouldn't be able to search for aliens using that method. If we want to do the search at all, we have to assume they're using methods we understand.


Do gravitational waves cause winds in space ?
Q : Well, do they ?

A : No, winds in space are caused by something simpler : heat. The heat from a star is enough to blast off part of its atmosphere.
http://solarscience.msfc.nasa.gov/SolarWind.shtml


Can gravitational waves be dangerous ? They took out Praxis, the Klingon moon, in Star Trek VI...
Q : So, since I believe what you say, in that gravitational waves are so weak, the explosion of "Praxis" (the Klingon moon in Star Trek VI: The Undiscovered Country", couldn't happen ? Or wouldn't be as devastating ?

A : f I recall correctly, Praxis was destroyed by some mining accident thingy, not a gravitational wave. Still, the explosion might itself produce a gravitational wave. In the movie a huge shockwave hits the Excelsior which is some light years away. This relies on Trek physics since in the real world faster-than-light is not permitted in physics as we currently understand it.

So, let's have the Excelsior be at least somewhere in the Klingon star system. There's an interesting thing about expanding (or exploding) spheres : you won't notice any change in gravity until you're inside the debris cloud. The reason is that until that moment, there was still just as much mass "beneath" you (i.e. between you and the centre of Praxis), so you're feeling the same gravitational force from "Praxis". The fact that Praxis is now much, much larger turns out to make no difference. Except to everyone on Praxis, of course.

Once the debris stars moving past you, there is now less mass "beneath" you, so you feel less gravity. But, unless the Excelsior was very close to Praxis (like in orbit), the gravity from Praxis would have been negligible anyway, so there wouldn't be any appreciable change. Also, there's a subtle difference between "gravitational waves" and "a change in gravity", which I don't fully understand, but it turns out that spherical explosions don't produce gravitational waves at all (I am wondering if this is strictly correct, I'll get back to you on that) :
http://www.ligo.org/science/GW-Sources.php

The visible blast wave in the movie is presumably supposed to be debris from the explosion. While I'm loathe to criticize anything Trek, this follows the classic movie "circular shockwave" effect, which looks cool but is totally wrong. There's no reason at all to think the explosion would be confined to a thin plane.

However, if the explosion was powerful enough to destroy a moon, the debris cloud would certainly be dangerous since the debris would be moving at several kilometres per second at the very least. The difference between reality and the movie is that the dangerous blast radius would be very much smaller because the explosion should be spherical (well, very nearly).

So, no gravitational wave destroying Praxis, and no gravitational wave from the explosion either. The movie gets right that the explosion itself would be extremely dangerous, but gets just about everything else wrong.

However, in the extreme case that for some reason the explosion of Praxis was non-circular, which would produce gravitational waves, they might be dangerous if the Excelsior was close enough. Here's a very nice link : http://stuver.blogspot.cz/2012/06/q-what-would-gravitational-wave-feel.html

In summary, while gravitational waves can be dangerous under extreme circumstances, as far as we're concerned here on Earth they are probably one of the least dangerous things in the Universe.


Why do different planets move at different speeds ?
Q : I have been informed by folks here, that the Earth is traveling at around 66,000 MPH. What I was wondering is do different planets and moons in different regions of space travel at different speeds in space ?

A : Here's my attempt at a simple, non-mathsy answer.

The speed an object moves depends on the strength of gravity where it is. Gravity depends on the mass and distance of other objects. For example, if you fall over on the Earth you fall down more quickly than if you fell over on the less massive Moon.

So, Mercury (for example) moves more quickly than the Earth because it's closer to the Sun. If it moved more slowly it would fall in.

Here's quite a nice little simulator. You can alter the starting speed and direction of various different objects and see how they react.
https://phet.colorado.edu/sims/my-solar-system/my-solar-system_en.html


Was Giodano Bruno a scientist ?
Q : Was Giordano Bruno not doing science when he claimed that the stars were in fact suns, despite having no conceivable way of measuring such a claim for hundreds of years ?
[This is a question about the philosophy of science, relating to the very interesting article here : http://www.forbes.com/sites/startswithabang/2015/12/10/why-trust-a-theory-physicists-and-philosophers-debate-the-scientific-method/]

A : A very brief background for those who didn't follow the discussion : Bruno was a 16 century Italian thinker who claimed that the universe was full of inhabited worlds. He had no actual evidence for this, just sheer philosophical reasoning. He had no way of testing his claims either. He was burned at the stake by the Church, though whether this was because of his astronomical beliefs - and belief is absolutely the correct word in this case - is a matter of controversy.

In my opinion, Bruno was probably not a scientist or doing science.
You could fairly call Bruno a martyr to the cause of religious freedom, but his cosmic worldview was neither a deduction nor a guess. It was a philosophical corollary of his heterodox belief that God and souls filled all of the universe.
http://blogs.discovermagazine.com/outthere/2014/03/10/cosmos-pick-wrong-hero/#.VmvKUvkrKhc

However, that article states that the charges against Bruno are known, whereas a 2009 book states that the list didn't survive. It also says that Bruno wasn't even a brilliant mathematician, but a plagiarist :
http://astrorhysy.blogspot.cz/2014/07/review-gods-philosophers.html

So, there is still some controversy over Bruno.

One could also ask if Kekulé was doing science when he dreamt about a snake and came up with the molecular structure of benzene : https://en.wikipedia.org/wiki/August_Kekul%C3%A9#The_ouroboros_dream

A personal example also comes to mind. During my PhD, a friend of mine was working on a particularly tricky problem. Something involving galactic dust extinction, I think. Being prone to bouts of obsessiveness, he was working on this problem rather a lot. One night he had a dream in which we were both in a dingy Cardiff nightclub. Over the noise I was shouting to him the answers to the equations he'd been working on. When he woke up, he wrote them down and found that they worked. He cited this in his thesis as, "Rhys Taylor, private communication."

I guess my take would be that it's possible to do science by irrational means. Sheer blind luck is sometimes very important. But, Bruno was both using irrational methods and coming up with untestable conclusions. And that's definitely not science, because God knows how many other people at that time were doing the same thing. The fact that he got something right is somewhat incidental, because almost everyone else who does things in this way gets it wrong. Scientists can get lucky, but being lucky doesn't make you a scientist.

Which also reminds me of Wegener's Law : http://astrorhysy.blogspot.cz/2013/02/i-wanna-be-pseudoscientist.html


Are the constants really constants ?
Q : How certain are we today of the, well, constance of the physical constants ?

A : Pretty sure, at least as far the speed of light goes. If it had changed, we would see differences in the spectral lines of different elements. Thus far, no significant results have been reported.
http://cosmoquest.org/x/365daysofastronomy/2012/05/05/may-5th-encore-were-the-fundamental-constants-of-physics-different-in-the-early-universe/ I know some of the researchers involved in this. The last I heard there was no sign of the very weak detections being confirmed, so they were most likely not real.

Sometimes people invoke, "if the speed of light wasn't constant then it would mean the Universe was much smaller so it's not billions of light years across at all and everything is wrong !". This is true. However, you can't alter the speed of light without causing massive other changes : radiation transport would be totally different for one thing, so star formation and evolution would be drastically different. Since even galaxies billions of light years away look basically similar to ones nearby, there's no sign that the speed of light has drastically altered. Small changes, however, are not ruled out.


What's the opposite of a black hole ?
Q : I have heard that "white matter is at the opposite end of a black hole. What is white matter. When I type in "white matter" I got The Beattles White Album... I thought that was odd, to say the least.

A : I never heard of white matter before. Google tells me it's some brain thing, which I know nothing about.
https://en.wikipedia.org/wiki/White_matter

A white hole is the theoretical opposite end of a black hole. Just as you can't escape from a black hole, you could never enter a white hole. The idea is that a black hole curves space into a tunnel which opens somewhere else (maybe somewhere far away from the black hole, maybe in another universe entirely). So any stars or planets which get swallowed by a black hole get spewed out as hot plasma from a white hole somewhere.

A white hole would look like an intense source of energy apparently coming from nowhere. Claims that they've been spotted turn up from time to time, but thus far there's no definitive proof that they exist. It's believed that even if a black hole did produce a tunnel in space, it would quickly collapse.

So black holes probably aren't really "holes" in the normal sense. If you fall into one, you don't get shot out somewhere else - you just die, quickly and painfully, ending up as a very, very small lump in the centre.


What would happen if a black hole collided with a white hole ?
Q : I'm wondering what were to happen if a white hole from one region of space came in contact with a black hole from another region of space.

A : Very difficult question. The rest of the internet seems to want to dodge the question completely by saying that "white holes don't exist". That may be true, but it's still an interesting hypothetical question.

After thinking about it, I would tentatively suggest that a black hole and a white hole could never collide. One is trying to draw everything in, the other is pushing everything away. Both go to infinity at the center, which is another way of saying, "the maths is broken, cap'n", but before that happens everything will stay sane.

With a black hole, nothing that falls below the event horizon can ever escape - or in other words you're safe as long as you keep your distance. With a white hole nothing can ever fall below its event horizon. So the white hole pushes the black hole aside before everything gets wacky. I'm not too certain about this answer though.


What causes inertia ?
Q : I DEMAND ENLIGHTENMENT !!!

A : After looking into this, I have to admit defeat. The internet is full of pseudoscience, possibly valid science but totally obfuscated in dense mathematics, and people saying, "it just is, so there". I can't make head nor tail of it. Sorry !


If black holes and white holes have the same strength gravity, doesn't that mean they're identical ?
Q : But the white hole would have the same gravity field as the black and, would therefore, behave the same surely ? How could matter, and energy, escape a white hole with the same gravity field as the black hole ?

A : A black hole has extreme gravity. In general relativity, gravity is the curvature of space. A very nice explanation is here :
https://www.youtube.com/watch?v=MTY1Kje0yLg

Gravity is like a depression in a rubber sheet, things roll around it but ultimately fall in if they're not moving fast enough. Anti-gravity (which is what you'd get from a white hole) would be like pulling the rubber sheet up into a hump. So things would again roll around it, but ultimately get pushed away unless they're moving fast enough. Another way to think of it would be to say that a white hole has negative mass.

Black holes and white holes take this to extremes. In a black hole, you can't escape even if you travel at the speed of light. Since that's the fastest it's possible to go, if you get too close to the hole there's no way to escape. For a white hole, the only way to enter the hole would be to travel faster than the speed of light. Since you can't do that, nothing ever gets in.

IIRC, a white hole could theoretically form when a black hole forms as it curves spacetime into a tunnel which opens somewhere else. Where it might open gives everyone a philosophical headache. Somewhere else in our Universe ? Another universe entirely ? It is also thought that the tunnel would be extremely unstable (I guess because you'd need exactly equal amounts of positive and negative mass), and anything trying to enter it would immediately cause it to collapse.

If it did exist, a white hole would look like an incredibly bright object with no apparent energy source. Thus far, while a few candidates have turned up from time to time, there have always been more conventional explanations that could explain them.

EDIT : This answer is totally wrong ! I'm preserving it only to show what a stupid mistake I made. White holes aren't anything like what I've described, they're far more complicated than that. It turns out that white holes do indeed have gravity just like a black hole, so things can orbit them just as they can a black hole at a safe distance. Rather than being a sort of anti-gravity, white holes are actually black holes running in reverse :
https://en.wikipedia.org/wiki/Talk%3AWhite_hole#DO_white_holes_mathmatically_resemble_black_holes_in_reverse_time.3F
The upshot is that because general relativity is Bloody Complicated, white holes still have strong gravity like a black hole but are nonetheless completely different objects. Frankly this is too mind-warping for me and I can't find a good maths-free explanation. So I'll be a lot more cautious about answering questions about white holes in future.


Where does matter falling into a black hole go ?
Q : I mean it's got to go somewhere, right ?

A : Yes - it goes into the hole. "Hole" does not equal "tunnel", it's most probably just a pit. Throw stuff into the hole in a bin and it stay in the bin and nothing else happens.


How can we prove there's life on other planets ?
Q : How do we prove there is the possibility of life on other far away galaxies? Since they are an older society, they would have already figured out "fire" and how best to use solar power and could very well be far superior in technology. On Earth, we have smoke stacks spewing CO2. What if other planets had a different chemical that they used, how would we be able to detect that? Hoping I made sense with that question.

A : The way to do that would be to look for oxygen in the exoplanet's atmosphere. Oxygen isn't stable, it's only because of photosynthesis that 20% or so of our atmosphere is oxygen. There are other chemicals we could search for (such as methane) but a 20% oxygen atmosphere would be pretty solid. Detecting trace gases like CO2 would be much harder. To do this, we would need to use spectroscopy : splitting light into its component colours and looking for signatures of specific chemicals :
http://loke.as.arizona.edu/~ckulesa/camp/spectroscopy_intro.html

If I recall correctly, the great sensitivity needed to do this even for relatively nearby exoplanets requires next-generation giant ground-based telescopes or planned future space missions. It's not easy, because planets are so faint and also overwhelmed by the glare of their parent star.

Here's a very nice, very readable article on the challenges of looking for life based on the chemistry of planetary atmospheres :
http://www.scientificamerican.com/article/csi-aliens-astronomers-prep-to-detect-cryptic-exoplanet-biosignals/


What is the escape velocity of Jar Jar Abrahams and is there any way to reduce it to zero ?
Q : You heard me. Make with the Jar Jar !

A : The escape velocity of JJA is small because his mass is small. Alas, his movies are transmitted at the speed of light, making them impossible to escape according to current physics.


How do you know black holes exist ?
Q : Well ?

A : For now, through a combination of evidence and theory. The observations show that objects exist above a certain density. The theory says that objects above this density cannot avoid collapsing into black holes. Cygnus X1 is a nice example of a good stellar mass black hole candidate :
http://hyperphysics.phy-astr.gsu.edu/hbase/astro/blkbin.html#c2

Sagittarrius A* is probably the best candidate supermassive black hole since we can observe the orbits of individual stars around the compact object and thus determine its mass very precisely (we can also estimate it from X-ray emitting gas) :
https://www.eso.org/public/news/eso0846/
According to current theory there's probably nothing else it could be AFAIK. Still, one should never rule out observations detecting something not predicted by theory - it wouldn't be the first time.

But the really nice thing about Sag A* is that with the development of the Event Horizon Telescope, it's so large that we should actually be able to image the event horizon and finally see the black dot at the centre of the accretion disc. That would be really solid evidence, bordering on actual proof.
http://www.eventhorizontelescope.org


How long will it take for all the heavy elements to decay ?
Q : If the last star in the universe were to explode today, how long would it take for all the heavy elements (like uranium) to break down in to lesser elements ? And I am excluding Black Holes which might still exist and be responsible for creating new stars in nearby nebulae. Let us assume these are gone already too.

A : I won't be infinte since there are a finite number of atoms. The numbers keep reducing until there's only one left, which, eventually, would also decay. It's very hard to put a number on this but my guess would be hundreds of trillions of years.

There are also some theories which say that protons themselves are not stable. So if you wait long enough you don't even need to worry about radioactive decay because protons and neutrons won't even exist. Timescale estimates for that vary wildly, from as little as 10^40 years to 10^200 years.
https://en.wikipedia.org/wiki/Future_of_an_expanding_universe#Nucleons_start_to_decay


Is it possible stop microscopic black holes from shrinking ? I am totally not plotting to destroy the world.
Q : I asked a question if a man made black hole could be created in space, to make it grow large you could place it near a star, or maybe inside a moon for it to consume matter quickly to not disappear, and or evaporate. But Hawking radiation is a problem with this, and someone said on a answers forum "You'd have to precisely aim the mass of a mountain range at it in 10^-20 seconds (say), to keep it from evaporating, and anything that approached it would be exposed to super high temperatures blasting away from it".

Also another person also said on the answers forum "The smallest microscopic black hole that you can theoretically make is has a mass of 1 Planck Mass, and a diameter of 1 Planck Length. Unfortunately, this black hole will also evaporate in about 1 Planck Time, so you won't even be able to feed anything into it at the speed of light before it disappears". So how to get around this problem, if the microscopic black hole instantly has matter to feed on if you created the microscopic black hole in either a moon, or a planet, to get larger in mass, would this fix the problem. But would the radiation push the moon matter away from the microscopic black hole, so would the black hole not be able to grow big this way.

A : I would say the fact that microscopic black holes don't grow any larger is something to be thankful for, rather than a problem to be avoided. :)

I suspect there is a threshold for the black hole's mass below which Hawking radiation is so strong that it becomes literally impossible for the hole to grow any more. At the atomic scale, radiation pressure is going to push any infalling matter away, delaying it so that the hole completely evaporates. At that size (I can't give you a numerical estimate) there is no hope - the initial mass of the hole simply has to be larger.

Your best bet for creating a black hole which stands the greatest chance of growing would be to start inside a neutron star where the density is fantastically high. That will give you the maximum rate of mass accretion into the hole, so you can start with the smallest hole possible. You can't do this on Earth though, because without the tremendous mass of the neutron star to pressurise it, neutron-degenerate matter is extremely unstable and explodes.


Can something turn into a black hole simply by going really fast ?
Q : I remember reading that if a particle goes fast enough, it would theoretically collapse into a black hole because it would contain so much kinetic energy.

A : I'm not at all expert in relativity, but the internet is providing me with some contradictory answers. First, a straightforward Google search says categorically, no, you can't make a black hole just by going faster. Which makes intuitive sense to me because you don't really add more mass by going faster - relativistic mass is not the same as rest mass.
http://physicsfaq.co.uk/Relativity/BlackHoles/black_fast.html
http://www.universetoday.com/56051/can-a-really-really-fast-spacecraft-turn-into-a-black-hole/

But on the other hand, some normally impeccable sources say that energy does act in the same way as gravity :
https://plus.google.com/+CiroVilla/posts/an3iaFoLwoo
http://what-if.xkcd.com/140/

So I'm thoroughly confused. Can anyone help clear up this mess ? UPDATE : Jonah Miller to the rescue ! He adds :

Since there is no preferred inertial reference frame (and indeed we can always find one where a non-accelerating spaceship is not moving), moving really fast alone definitely does not translate into becoming a black hole. Mass and energy are the same, but the subtlety, I think, is that the relevant quantity is the energy-momentum tensor, which is a complicated combination of energy and momentum that explicitly stays invariant when you move between inertial (and even non-inertial) reference frames. I am extremely confident about that.

But acceleration?

I am less confident about the following.

But... that's perhaps not a totally satisfying answer, so let's ask a different question. If we were to want to go at high speed relative to our current reference frame, we'd have to accelerate. Could that process form a black hole? I think... sort of.

Suppose we wanted to accelerate up to some very high speed within a time of one second. Then we'd need to output gamma m c^2 of energy per one second to do it. That means we need gamma m c^2 of energy NOW in a single packet, which might come from mass. The packet of energy we need may well be massive enough to form a black hole.

On the other hand, if we accelerated adiabatically, we'd need to expend that energy slowly and we could take it (and expend it via acceleration) in chunks. Also there is a way to mimic a black hole with motion: constant acceleration can create an effect analogous to an event horizon.



What do you think of this ridiculous hour-long video ? Doesn't this nebula look just like a finger ?
Q : What do you think of this ridiculous hour-long video ?
https://www.youtube.com/watch?v=MtQNEKZ5oKg

A : [First, for the record, I'm generally fine with reading short articles no matter how ridiculous. But if you're going to ask me to spend a full hour of my time doing something else, please remember I'm under no obligation to do so. I'm doing this entirely voluntarily. And if that video turns out to be so awful that I hate every second of it, then forget it - I'm not going to waste an hour of my free time doing something I hate. I do, however, absolutely guarantee to give my honest opinion if I do watch it, but be warned that if I honestly think it's a big pile of poop, I'm going to tell you as much. You did ask for it, after all.]

This one comes from a long and silly discussion regarding the difference between pareidolia (seeing things which aren't there) and image recognition (seeing things which are there).
https://plus.google.com/u/0/112582243977859176055/posts/B13D1Uq4SGe

The questioner professes to be a geometry teacher but apparently doesn't understand the difference between one thing resembling another and actually being the thing it looks like. A nebula that looks like a finger is exactly an example of pareidolia because it isn't really a finger. It only looks like one at a very specific wavelength and (likely) only at a very specific angle. Even if it did look like a finger from all angles and at all wavelegnths, it still wouldn't have any significance. Which I explain at length here :
http://astrorhysy.blogspot.cz/2015/04/things-that-look-like-other-things.html

The video turned out to be so unbearably stupid that I gave up after 15 minutes. It claims that a Renaissance painting resembles the Orion nebula. It doesn't. It was equivalent to saying, "hey, that turnip looks a bit like a cat !". The questioner than linked to a website claiming that the Orion nebula looks like a brain. Again, it flat-out doesn't. The image of the nebula had clearly been cut to fit and even then it wasn't doing a good job.

The questioner further claimed that wavelength doesn't matter to geometry. This leads me to conclude the questioner is actually properly delusional, because this is objectively wrong. It's a bit like pointing to a moose aand saying, "nice elephant !". It just doesn't make any sense at all. I don't get it.


Does my stupid website prove that gravity doesn't exist and the world is flat ?
Q : Why didn't you watch this 25 minute video ?

A : Because I read the text on the website and that was enough. Frankly, ordinarily I could happily watch Kate Upton in zero gravity for hours on end, but even that wasn't enough to persuade me to watch this obvious effluence.
http://www.timetounite.com/gravity-does-not-exist/

I offer some thoughts on why some ideas aren't even worth considering here :
http://astrorhysy.blogspot.cz/2015/10/not-so-open.html

I have great respect for people who try and persudae people that their objectively wrong ideas are, well, wrong, but there comes a limit when I refuse to get involved. I'm just not convinced some people will ever see reason even if it bites them in the groin. The best I can say for them is that they are genuinely stupid - they literally lack the intellecutal capacity for basic reasoning.
http://astrorhysy.blogspot.cz/2015/12/a-grand-unified-theory-of-stupidity.html


Could there be black holes made of anti-matter ?
Q : I was just thinking, if there is such a thing as an anti matter black hole ? I understand that the amount of anti matter required is mind blowing but could it exist in some distant past of the universe ?

A : There's no particular reason one couldn't form as far as I know (except that there probably isn't enough anti-matter but that's besides the point). Anti-matter is believed to cause gravity in the same way a regular matter. It shouldn't also be affected by gravity in the same way. AFAIK, you wouldn't be able to tell if a black hole was made from regular matter or anti-matter.
http://www.universetoday.com/121487/what-if-a-black-hole-met-an-antimatter-black-hole/


Apart form looking for aliens, what will China's new 500m telescope do ?
Q : The only thing the media are saying is "aliens".

A : The obsession with aliens annoys the heck out of me. Yes, looking for aliens is cool, but we aren't going to get results on that anytime soon. Meanwhile, there's a tonne of productive research to be getting on with. Pushing the aliens card so strongly is selling the public short by insisting they won't believe anything else is worthwhile. As for the research :
http://fast.bao.ac.cn/en/Science.html

It's basically a more limited but more sensitive version of Arecibo. It's more sensitive obviously because it's bigger. It's more limited because it will operate over a much smaller range of frequencies and won't have a radar transmitter installed. Like Arecibo, it will still search for pulsars, measure the gas and chemistry of distant galaxies, but it won't be studying the atmosphere or asteroids.


What do you think of this UFO report ?
Q : Here it is :
http://midnightinthedesert.com/ham-radio-operator-hears-american-airlines-pilot-report-ufo/

A : Well, "we had the recording, but we don't any more" doesn't exactly fill me with confidence. Maybe they'll be something to be comment on after the FOI request but until then there's no actual information here.


What do you think of this 80 minute documentary about UFOs ?
Q : Here it is :
https://www.youtube.com/watch?v=ny9PGHSYT6k

A : To be honest I'm more or less done with UFOs at this point. I spent far too much time reading about this stuff already and I never found anything really convincing. The few cases where I thought there might be something in it were just nowhere near enough to convince me this is something worth pursuing. I'm basically at the point where I won't accept any evidence short of a flying saucer landing on the White House lawn, because >99% of the "evidence", in my view, turns out to be utterly useless. I'll still entertain short articles but 80 minute documentaries are out, sorry.

Which is not to say I think no-one should investigate this, just that it's not for me.

Well could you watch the first 15 minutes at least ?

I watched the first 18 minutes because nothing actually happened in the first three minutes. Nothing here I haven't seen a million times before, so I'm going to watch War And Peace instead.


Do black holes rip you apart because they're made of anti-matter ?
Q : A black hole is made of antimatter right? That's why you get ripped up when you go inside cause they cancel each other out. I know its just a hypothesis but it makes sense.

A : No, it's made from ordinary matter. You get ripped apart because of the intense gravity. Anti-matter does not rip things apart : colliding matter and anit-matter annihilate into energy.


If we can see back in time, can we also see forwards in time ?
Q : While I know the fun of looking back in time teaches us about the history of our universe, is there anyone looking forward, to see what kind of obstacles might be in front of us ? Like the collapse of the universe ? Is that even a possibility ? What kind of technology would we need ?

A : There's certainly no easy way to do this as there is with seeing into the past. Travelling into the future turns out to be a straightforward matter of travelling very fast :
http://www.huffingtonpost.com/2013/09/13/time-travel-possible-physicist-machine-future-only_n_3921944.html

(and by straightforward I mean, "with access to more energy than has ever been produced in the entire history of mankind")

But to send a signal or anything else backwards in time (i.e. for someone to look into the future) turns out to be incredibly difficult at best, widely considered impossible. There are a few exotic possibilities, though none (AFAIK) allow you to travel back to a point before the time machine was constructed (and by "time machine" I mean anything which permits backwards time travel, e.g. a wormhole, particular arrangements of cosmic string, etc.). So seeing into the future, at best, won't be possible until we build a time machine. A very nice, quite comprehensive summary is here :
http://science.howstuffworks.com/science-vs-myth/everyday-myths/time-travel.htm
See also :
https://en.wikipedia.org/wiki/Ronald_Mallett#Time_travel_research

Even if time machines do turn out to be possible, they're fraught with paradoxes. The essence of it being : if I see what's coming and change my actions, where did the information come from ? E.g. I peer into the future and see myself getting run over by a bus. So I stay home that day and don't get run over by a bus... but what was the future I originally saw ? Was it real ? Or, more dramatically, what if I go back in time and kill myself - if I'm dead, how was I able to go back in time ? #MassiveHeadache
#SoTotallyNotWorthIt



Would it take infinite time to cross the galaxy at the speed of light since time stops ?
Q : I saw this meme saying it would take 100,000 years to cross the galaxy. That's obviously wrong, right ?

A : You're wrong that it's wrong because the meme is right. It would indeed take about 100,000 years to cross the galaxy at the speed of light. Although it does depend on your point of view (whether you're the astronaut or someone back on Earth), no-one will see it take an infinite amount of time.

The time the astronaut experiences is always less than the time someone back on Earth experiences. An observer travelling close to the speed to light sees the outside universe going by extremely quickly, so from their point of view it takes almost no time at all. To an external observer it would take them 100,000 years, but they would see them ageing much more slowly. Their clock would appear to slow down, but their speed would not.
http://www.emc2-explained.info/Time-Dilation/#.VuG3bJwrKhc


When black holes merge, do they form a wormhole ?
Q : Earlier we knew that when two black holes combine they form a worm hole and now , we have found a binary system of black holes also ... Are there any conditions concerning these two different things like if these conditions will get satisfied then a worm hole or a binary system will be formed ?

A : Wormholes are theoretical - there's no good observational evidence that they exist. It's though that (at best) if a wormhole did form naturally from a black hole, it would be incredibly unstable and collapse if even slightly perturbed. So merging black holes is likely to destroy wormholes rather than creating them, if they even exist at all. It just creates a bigger black hole.

That said, the recent detection of gravitational waves shows the small black holes do merge, but how the black holes get close enough to do this is a controversial area. When they're really close, the gravitational waves themselves carry away huge amounts of energy, so the holes get closer. At larger distances, friction with the surrounding gas can help slow them down, causing them to move closer. But there's a range of distances at which neither of these effects are thought to be very important - yet there's pretty good evidence that mergers happen even so. It's an area of ongoing research.
http://physics.stackexchange.com/questions/221111/the-final-parsec-problem 


Do black holes convert energy into mass ?
Q : E = mc^2
Nuclear weapons
Blackholes
Nukes release mass into energy yes.
Blackholes squish energy into mass ?

A : No, black holes just squish mass into really dense mass.
http://science.nasa.gov/astrophysics/focus-areas/black-holes/


Can you do science without the scientific method ?
Q : If you use the scientific method to come to an infinitely-wrong conclusion, is it junk science? And alternately, if you construct a more-coherent story line that comes to the right conclusion, but without using the scientific method, is that even science? Would you rather be right or scientific? (not that those two outcomes are typically contradictory).

A : I would say the key point is how you test it. If you test things as objectively as can you can by the standard scientific approaches and still get it wrong, it isn't junk. Just about everything is eventually disproven, that's just the nature of the beast. If you come up with a conclusion that happens to be right without testing it scientifically, you have really no idea of knowing whether it's right or not - so you'd just be making wild claims, even if they were eventually proven correct.


Should the Arecibo radar be subject to ethics review boards to prevent us accidentally signalling our presence to aliens ?
Q : I can almost see +Rhys Taylor rolling his eyes already. [Original article : https://sites.psu.edu/astrowright/2016/05/19/a-question-for-meti-critics/]

A : Back when there was that ill-fated project to send twitter messages to the supposed source of the "Wow" signal, I sat in on one of the initial meetings. I asked one of the radar operators if the signal would be detectable to an alien equivalent of Arecibo at the distance of the target stars (~100 light years, IIRC). The reply was an unhesitating, almost contemptuous, "no". Given that Arecibo is a 1 MW transmitter, I'm highly skeptical of the claims for searches of ~10 W laser signals in the article. Anyway, any civilization advanced enough to actually visit us will already have detected us without our sporadic, non-repeating, pathetically weak radio signals.


Will the new FAST radio telescope be better at signalling/detecting aliens ?
Q : I hope that the numbers will go up (a bit) with the new Chinese radio telescope. They say here it will be ready in September... do you have any more info ?

A : AFAIK FAST will not even have a transmitter of any kind. If it ever does get one, I would not expect the numbers to increase that dramatically - maybe a factor of a few. Arecibo's 1 MW transmitter is quite capable of imaging Saturn, so I doubt there would be a need for a dramatic increase in transmitted power unless they were deliberately trying to do active SETI. What the technical challengers of having a more powerful transmitter would be, I'm not sure. FAST has a much smaller receiver cabin than Arecibo - last I heard they need to lower the entire platform to change receivers (Arecibo has them on a rotary floor, kindof like a revolver only with more science). I don't know if it would be feasible to install a transmitter in that sort of limited space/capacity. Arecibo's transmitter is not the sort of thing you'd want to be taking out every time you want to observe something else.

As far as regular SETI goes I would expect a fairly significant increase in sensitivity. The dish size increase is not quite what you might expect. Arecibo typically only uses 225 m of the 305 m dish (otherwise it could only point straight up). FAST will typically use 300 m of the 500 m dish, so that's an area increase of only 1.7. However, it's planned to have a 19 beam receiver compared to Arecibo's current 7 (though a 40 beam receiver is planned/under construction), so it should be able to survey much larger areas more quickly (numbers depends on the performance of the receiver instrument, which I don't know). It will also have about twice the area coverage that Arecibo has.

I believe the plan with FAST is for first light in September, which right now looks entirely feasible given the extremely impressive construction pace. My guess would be that initial science results will follow within a few months, with the first approximately design-spec stuff coming sometime in 2017.


How about these space facts, eh ?
Q : Isn't space COOL!
http://www.forbes.com/sites/quora/2016/06/03/5-facts-about-outer-space-that-hardly-anyone-knows/#5119a88e639c

A : Awww, man, I hate stuff like this. Totally random assorted nuggets of trivia that are usually just wrong. And this list is no exception.

"the Eridanus supervoid is very empty of ordinary matter. Dark matter likely is very abundant here, which would prove as an explanation."
That doesn't make any sense as an explanation. Dark matter is abundant in normal galaxies; it's thought to be a major reason why they form. In standard cosmological models voids are devoid of both dark matter and normal matter.

"Everything in the universe will return to its position it is in today in 10^10^10^10^2.08 years, according to the Poincare-Recurrence time."
That assumes the Universe is either static (it isn't) or oscillating (for which there is little or no evidence). Currently the data favours the Universe expanding forever, which means there's no chance matter will ever return to its original position.

"Black holes will live longer than the universe."
If there's no Universe, there can't be any black holes. Black holes may well live longer than anything else in the Universe, but they can't live longer than the Universe itself.

"Every galaxy in the observable universe is headed for the Great Attractor, minus the ones being pulled away from the attractor due to the expansion of the universe."
Since the number of galaxies which are being pulled away due to the expansion is much, much greater than the ones which are not, the first half of that sentence makes absolutely no sense. It's like saying, "Everyone in the world is currently in Wales, apart from the ones who aren't."


If nothing can escape from a black hole, why do they have jets ?
Q : If nothing, not even light escapes, where are the jets generated... and where are they escaping from ?

A : Nothing can escape from within the event horizon of a black hole. Outside that distance things can still escape provided they have enough speed.
http://www.physicsoftheuniverse.com/topics_blackholes_event.html

The formation of the jets is not well understood, but a huge amount of energy can be released by matter falling into the black hole - enough to drive some of it back out into the jets. The jets are likely escaping from the discs of accreted material surrounding the hole.
http://www.einstein-online.info/spotlights/accretion


Does centripetal gravity affect different masses differently ?
Q : Does spin generated gravity affect objects equally, regardless of mass, or does it accelerate objects of different masses at different rates ? I think it affects all masses equally, however I also recall that centrifuges are used to separate masses that are intermingled, like how a uranium centrifuge is used to separate the heavier U-238 from the lighter radioactive U-235, which would make me suspect that spin-generated gravity does not affect all masses equally, but there are many other factors that could be at work in the case of centrifugal separation. am not aware of any other sources that could answer this question.

A : I would think that things in a centrifuge are separated just because of their different densities, same as under normal gravity just much stronger. Centripetal acceleration doesn't depend on mass, though force does - same as gravity. The force experienced is directly proportional to the mass, but acceleration is always inversely proportional to mass (F = m*a), so they exactly cancel.
http://hyperphysics.phy-astr.gsu.edu/hbase/cf.html


Have we reached the end of physics ?
Q : I've read this article, and... hey, wait, I never asked for your opinion ! STOP IT !!!
http://www.express.co.uk/news/science/691046/END-OF-PHYSICS-near-as-physicists-reach-their-limit-on-understanding-the-Universe

A : Saying, "the end of physics" is a bit like saying "Voyager has left the Solar System". This is flagrant clickbaiting.

"for the first time in the history of science, we could be facing questions that we cannot answer.” There's never, ever been a period where that wasn't the case. It's a sort of defeatist humblebrag to state "we've got this huge mystery we'll probably never solve". For one thing, the multiverse is but one possible explanation. For another, whether you can learn everything or not is not the point. The point is to try.

The infinite multiverse, like any theory of an infinite universe, lets you get away with scientific murder. You don't have to explain anything because everything happens an infinite number of times, so obscenely unlikely events become certainties. Instead of searching for a deeper answer, it's all, "nope, it's a statistical fluke". Maybe it's even true, but it's not helpful.


If aliens visited Earth, what would be the hardest universal human behaviour to explain ?
Q : Well, what ?

A : It's impossible to answer without know what the aliens are like. They could be every bit as batshit crazy as the rest of us.


Are there any known lifeforms which can survive in space ?
Q : Is there any other type of life which exist in space. I got this question because some persons say that earth got its components like water, oxygen and many other things from space. What if those components were from a planet which have those components and life forms in it.

A : There are organisms known which can survive in space, for example, tardigrades :
http://www.smithsonianmag.com/science-nature/how-does-the-tiny-waterbear-survive-in-outer-space-30891298/?no-ist

We also know that not just water but also more complex chemical compounds like amino acids (which are important for life) can be found in space :
https://www.newscientist.com/article/dn17628-found-first-amino-acid-on-a-comet/

But what we don't know is whether other planets or comets would have been more suitable places for life to begin than the early Earth, or if any lifeforms could survive crashing into the planet. Some people think that bacteria might have seeded the Earth if they were deep instead meteorites, which is one type of controversial "panspermia" theories.
http://www.panspermia-theory.com/ It could also have worked the other way around, with bacteria from Earth (presumably unsuccessfully) seeing other planets in the Solar System.


Are black holes gateways to other dimensions ?
Q : I really wonder about black holes! Are they an actual cosmic doorway maybe to another universe or another dimension!?? You never know! And sadly I don't think we will ever find out!?

A : We can't know for certain without visiting one. And even then, anyone who fell through wouldn't be able to tell us about it unless there happened to be another black hole to send them back to our Universe. Which is unlikely as there's no evidence for any white holes.

All we have to go on are our theories and (somewhat limited) observations. Currently it doesn't seem likely. It's possible that wormholes do form inside a black hole, but they would be extremely unstable and not likely to last long enough for anyone to pass through them. For smaller, stellar-mass black holes you'll get ripped apart by tidal forces anyway, though for supermassive black holes you might have a chance.
http://www.smithsonianmag.com/science-nature/would-astronauts-survive-interstellar-trip-through-wormhole-180953269/
http://www.universetoday.com/1605/maximizing-survival-time-inside-the-event-horizon-of-a-black-hole/#

So we really don't know, but current theory says probably not. Black holes could simply be pits : throw something in one and it ends up in the bottom of the pit, feeling very unpleasant and not doing anything very interesting.


Could Sagittarius A*, the supposed supermassive black hole at the centre of the Galaxy, actually be a white hole ?
Q : I read this article on the internet. Is is true ?
http://physics.stackexchange.com/questions/207317/could-the-black-hole-in-the-center-of-the-galaxy-be-a-white-hole

A : Every so often, evidence for white holes comes along, then it fades away again. I would be very skeptical about any such claim. They're theoretical concepts but in a very different way to black holes - it shouldn't be possible that they form at all. Not everyone agrees with this though, because we're dealing with things like time dilation and the reversal of space-time.
http://www.askamathematician.com/2013/04/q-are-white-holes-real/
http://www.universetoday.com/122715/what-are-white-holes/
https://en.wikipedia.org/wiki/Talk%3AWhite_hole#DO_white_holes_mathmatically_resemble_black_holes_in_reverse_time.3F

(If you only visit one of those links, I recommend the second one)

I'd always thought that white holes had anti-gravity and that's why they push stuff away. As the third link makes clear, it is much more complicated than that. So my initial answer of, "Sag. A* can't possibly be a white hole, things are orbiting it" won't do. Darn. That would have been too easy.

Still, that link does have some misconceptions I'm more confident about :
1) As the link in this point makes clear, this isn't so unusual. Black holes are known to be messy eaters. Given the complex environment of the hole, I'd want to be really sure we knew just how much mass should be accreting.
http://www.forbes.com/sites/startswithabang/2016/06/09/how-do-black-holes-eat-like-cookie-monster/#3b2b829820c0
3) The black hole doesn't "energise" anything directly. It's just gravity with a tiny, negligible amount of Hawking radiation. But material orbiting in an accretion disc can be extremely hot and a powerful source of X-rays, which can ionise the surrounding region.
4) Again, black holes are messy eaters. The density of material being particularly high near the centre, it's not surprising we see stars being born. It's certainly not surprising we've never seen a star being eaten - although the density is much higher than in our locals stellar neighbourhood, it's still very, very low - too low to expect to see stars colliding either with each other or the hole. The radius of the hole is ~0.08 AU whereas the nearest star is typically ~1000 AU away.
5) A former colleague of mine was saying ever since the G2 cloud was discovered that nothing was going to happen.
7) Urrgh, this is complicated :
https://medium.com/starts-with-a-bang/is-energy-conserved-in-a-universe-with-dark-energy-a26572cc6853#.f0dgodl4a
8) We can't observe singularities. However within the next few years we should be able to observe the event horizon of Sag A* :
http://www.eventhorizontelescope.org/
And white holes have singularities too.


Why do you want physics to be broken ?
Q : Why do you want physics to be broken ? Most people who want physics to be broken are people with an existential interest in fantasies that simply can't come true in a universe as defined by the current physics.

A : [Edited version of my original answers. It is indeed true that most cranks have minds which are so open their brains fall out, but this hardly means that scientists should consider their theories inviolable ! This "scientists are dogmatic" attitude is something we should actively be fighting against, not encouraging. Anyone who thinks current science has solved all the problems or is impervious to attack simply does not understand how the scientific method works.]

And most scientists also enjoy breaking physics. Scientists love revolutionary discoveries because a) it makes us feel important and b) we get to learn something new instead of just validating the existing model. E.g., http://www.bbc.com/news/science-environment-31162725

The vast majority of scientists would agree that we don't have all the fundamental pieces of the puzzle, for instance, we don't know what dark matter/energy are, we can't reconcile quantum mechanics with general relativity, etc. So it's entirely fair to say that physics is already broken. In fact it's usually broken, thus far we just keep improving theories. Nothing wrong with that, it's the nature of the beast. And it doesn't mean that the theories we currently have don't do a fantastic job in most regards.

I'm an observational astronomer by training, so I take whatever the Universe throws at me. Sometimes that means it's dull and boring, occasionally it throws up anomalies we can't explain. I'm also involved in numerical simulations. Specifically, I study extragalactic neutral hydrogen. A few years ago I found some dark hydrogen clouds in the Virgo cluster that aren't easily explained. Currently I'm running simulations to test three proposed scenarios to test which (if any) is the most likely. Thus far, one of the more controversial solutions is emerging as the clear winner. But I really, honestly do not care which solution works best as long as we find something that does work.

One can go from being a genuine skeptic to a denier by adopting too extreme a position in either direction. It's as foolish to cling too strongly to the idea that we've solved a problem as it is to the idea that we haven't solved it. While I'd agree that there are a lot of quacks out there screaming LOOK AT MY PIE CHART IT DISPROVES PHYSICS (believe me I get those kinds of emails), it's equally important to remember the history of science is not one of merely establishing facts with ever-greater precision, but also of radical, drastic revisions to its interpretations of the world. The trick is to walk the middle ground, not to go nuts over every tiny anomaly, but not to fall victim to hubris at every vindication either.

A tremendous amount of work goes into getting results one way or the other. The Universe is the way it is, it's not obliged to agree or disagree with our theories. Verifying them or disproving them are both "victories" of a sort. But for me personally, the really exciting stuff is when we find something we really can't explain. Knowing that existing theories are still working is great, but doesn't help us solve the known problems.


How do we know black holes are spinning ?
Q : How can we determine that the black hole is spinning ? Does black hole spin on its own axis ?

A : Most black holes are probably spinning because most objects in the Universe are spinning. Obviously we know the Sun is rotating because we can watch features on its surface move. We can do this to some extent with other stars, but mainly this can be estimated from the spectra of the stars. Lines in the spectra look wider than they otherwise would due to the rotation.
https://en.wikipedia.org/wiki/Stellar_rotation#Measurement

Since black holes are collapsed stars, they must also be spinning, and because they are small they have to be spinning fast. Which is easily demonstrated by spinning on a chair and pulling your legs in.
http://www.einstein-online.info/spotlights/angular_momentum

To actually measure how fast a black hole is spinning we need to measure the material around it. There's no way to measure the spin directly since the hole has no solid surface. But we can estimate the spin from material close to the event horizon.
http://stronggravity.eu/how-to-measure-black-hole-spin/

Another, weirder effect of the extremely strong gravity and spin of black holes is frame dragging - the distortion of space by the black hole's spin. This also has been observed, confirming the rotation of the hole :
https://www.sciencenews.org/article/general-relativity-caught-action-around-black-hole


Do all astronomical imaging systems have the same contrast range ?
Q : Contrast between two spots in an image is the same no matter what number of pixels devoted to it. Am I wrong about that ? Aren't all imaging systems limited to the same contrast range ?

A : I think that would only be true if you had identical receivers performing the same observing setup (but with different integration times), the sources were both above the noise level but below the saturation limit, and the images were created using the same algorithm. In that case you'd measure different absolute fluxes but the relative contrast would be the same. However, what should be more resilient is the inferred intrinsic luminosity of the source after calibration - as long as it's a point source to both receivers, that should be identical (if it's detected and not saturated) regardless of the receiver size, integration time, observing and imaging setup.

But, in general, this isn't the case. Telescopes are different sizes with different angular resolutions, so a point source to one telescope may be resolved to another, so the flux is spread out over more pixels. Or velocity channels for that matter. Instruments can have different dynamic range sensitivities. Images can be reconstructed in ways which favour point sources or extended emission, from the same raw data (number of pixels in the image doesn't usually relate to number of receiver pixels). Choice of observing setup can strongly affect the end result, especially (as in the OP) where there's very bright emission filling the entire beam and the data must be calibrated accordingly or things just don't work at all. In a word... no. :)


How could we detect dark matter with an unlimited budget and/or resources ?
Q : What could a Kardashev type I, type II or type III civilisation build to detect dark matter/test its existence ? What could modern astronomers build, given an unlimited (say, trillion-dollar) budget, to detect dark matter ? A giant Ice-cube type neutrino detector using the entire surface of Ganymede ? A particle accelerator running around the moon ?

A : Let's take this on a scale of ascending technological advancement, starting with our own puny civilization.

I'm no expert in the direct detection experiments but offhand I'd say yes : bigger, more sensitive detectors would be the way to go. Not necessarily on the ice moons though : there's plenty of ice on Antarctica for expanding IceCube (https://icecube.wisc.edu/) and we could expand other detectors relatively easily (e.g. LUX uses just 300 kg of xenon - let's use more ! https://news.brown.edu/articles/2016/07/lux). Whether a larger particle accelator would help I wouldn't like to comment. Still, giant space projects are probably more in the realm of a type-I civilisation - unlimited budgets won't remove the need for a massive space infrastructure that would take decades to develop.

But there are also improvements in the indirect detection methods that could us much stronger evidence. For instance, one possible experiment with the Square Kilometer Array will be to measure how the velocities of galaxies change as the Universe expands (https://arxiv.org/abs/1501.03822). So with some serious hardware upgrades possible in the next 10-20 years (i.e. the sort of problem that just needs money thrown at it) we might be able to measure how the dynamics of individual galaxies change in real time. For extreme objects, that might just give us the capabilities to distinguish between dark matter and alternative theories of gravity. We'd also need more advanced numerical modelling to give us precise predictions : so a bunch more supercomputers. Couple that with more radio telescopes - say, ten Arecibos or FASTs, which would give us enormous sensitivty and sky coverage (and resolution if linked as an interferometer). That would give us the prospect of detecting objects that just couldn't exist without dark matter : the sky remains only poorly explored at radio wavelengths. Telescopes could also play a role in more direct detection methods, such as searching for signals of self-annihilating dark matter expected in galaxy cores in some models. But really there's not much we could do today with an unlimited budged that we're not doing already, except bigger and faster.

But what could we do if we did have the much greater energy and resources of type I-III civilizations ? One option would be to do tests of general relativity in the very low acceleration regime where alternative theories say gravity works differently. The famous "Pioneer Anamoly", where the Pioneer spacecraft drifted slightly off-course, now seems to be exlained by the uneven heating of the probe by the Sun. But the principle is sound. It would be trivial for a type I/II civilization to send thousands of probes throughout the Solar Sytem and monitor their positions as they get further from the Sun, where the gravity from the Sun is far less than near Earth. Probes would be sent in different directions at different speeds, with different sizes, masses and designs to account for thermal effects. Although technically possible to do this for our own civilization with an unlimited budget, it would be far easier and more reliable for a more advanced civlization : they'd deep to know about the distribution of matter in the Solar System as precisely as possible. For example, powerful deep space radar and imaging might give them a much better idea of the distribution of comets in the Oort Cloud, helping to distinguish the effects of different theories of gravity from the tiny effects of close encounters with bits of rock and the like.

Another option would be a relativistic probe that exploits the effects of time dilation. Accelerate a ship to very nearly the speed of light and the crew could watch galaxies interact and collide in a few minutes - the ultimate way to test their predictions. Of course they wouldn't be able to relay this information back, since for the rest of the Universe billions of years would have passed, but this question postulates a species who are clearly going to put this "£@*ing dark matter issue to rest ONCE AND FOR ALL, DAMMNIT.

Perhaps the grandest method, only available to a no-fooling-around type III species, would be redistributing matter. Such a species could start moving stars to different parts of the galaxy. At least some alternative theories of gravity, such as MOND, predict that the distribution of matter is important to its motion, not just the mass. If there's really dark matter holding our galaxy together, moving the stars to different orbits should result in different motions to if this is due to the nature of gravity. It would taken tens of millions of years, but that's faster than waiting for the relativisitc probe to return.


Are smartphones as good as telescopes ?
Q : Each tiny phone can take images that have the same value as any big mirrors. The matter is how you look at a picture, as each ever taken has a complete information of all frequencies in it. I have a way of doing it, so the answer is yes, all the frequencies including invisible are present in any imprint, so a radio signal can be converted to visual what is nothing new as TV works that way. besides it can all be seen in a still image that is still only in our comprehension while containing huge amount of motion. Problem is that people make machines specified for different frequencies without knowing that they are always all there. Receivers as you know may be specified by manufacturer, but they all have much wider potential

A : No. Just no. Seriously, no.

Receivers are only sensitive to specific frequencies. Neither your phone camera nor photographic film can even detect radio wavelengths, and if they could, they would be nowhere near as sensitive as a large telescope like FAST. No amount of clever processing of the data will change that. You'll never get the same information from a smaller receiver - it's fundamentally impossible. A smartphone camera isn't even sensitive to radio wavelengths at all because that's not how it works. No amount of image processing will ever recover this. Anyone claiming they've found a way to do so should patent it and make millions ASAP... :)


Is there sound in space ?
Q : A friend of mine is convinced that there is sound in space. There isn't, right ?
This question wasn't sent to me. I found it in Randall Munroe's excellent "What If ?" book, which I would highly recommend to anyone. However in this case I have to find a fault, as Munroe relegates this question to the "weird and worrying" section - questions which don't get answered but are just stated for the record. And while their weirdness is normally self-evident and amusing, that's not really the case here. In space no-one can hear you scream... but there's more to sound than screaming.

A : Right ! Well, sort-of. Actually not really. No.

The smart-alec answer would be that planets are in space, and there's certainly sound on other planets as long as they have an atmosphere. That's clearly not what the question was getting at though. Obviously it's about sound far away from the atmospheres of any planets.

Sound waves are just propagating variations in the density of any medium, be that solid, liquid or gas. Like with ordinary water waves, the air (say) itself doesn't move much, it's just that the molecules bunch up for a little while and then move apart again as the wave passes, before settling back into their ordinary density level. It's this density wave that moves, not the molecules themselves. This can happen in solids and liquids as well as gases, as anyone who's ever dunked their head underwater in the bath can confirm.

So all you need for sound is... well, stuff for it to travel through. And "space" is misnamed, because it's NOT totally empty. Oh, it looks empty, but that's only because most of the material there emits at wavelengths we can't see with our eyes. Actually you can see this indirectly, at least on a dark, clear night : the dark band across the Milky Way isn't dark because there are no stars there, but because there's intervening dust that blocks the view. Look at it in other wavelengths and it's actually a bright, interesting place.

So there IS stuff in space for sound to travel through. In numerical simulations, sound speed is an important parameter in calculations of fluid dynamics - it tells you how disturbances travel through the gas. So in that very real sense, there most certainly is sound in space, and it plays a very important role in determining the evolution of gas structures and ultimately star formation. We can even, in a few rare extreme cases, witness waves moving through interstellar material in real time, such as in this movie of the Crab pulsar.

But don't go nuts. "Space" is not a totally stupid name, because although there is stuff there it's incredibly thin : typically quintillions of times thinner than ordinary air, and thinner than the best vacuum ever created in a laboratory. So calling it empty is pretty reasonable by our standards. Of course if exposed to this near-vacuum for more than a few minutes you'll die, but what if only your ears were exposed ? Or, better, what if you used a microphone that was equal in sensitivity to your ears ? Would you hear anything ?

Although human senses don't fare very well compared to other animals, even so they're still incredibly sensitive. Your eyes can, astonishingly, detect a single photon of light - only your brain's processing normally puts a slightly higher limit on this to give you a cleaner image. But individual photons can stimulate the movement of individual electrons relatively easily. And while your ears can detect variations as low as one billionth of normal atmospheric pressure, those density variations have to be able to move your eardrum - and individual atoms and molecules just can't do that. You certainly wouldn't be able to hear anything in space, which is easily demonstrated with a small vacuum and a loudpseaker. Even at levels way higher than in space, we wouldn't be able to hear anything.

Of course, there are also grey areas. Even though even intergalactic space isn't totally empty, most of those spectacular pictures you see of nebulae are still thinner than our best laboratory vacuums. But sometimes that density has to increase dramatically, otherwise stars and planets would never form.

So the answer is yes, no and maybe all at the same time. Yes there is sound, but no it's not audible except maybe for some rare exceptions. Randall Munroe, you've made a rare error here - this question shouldn't be consigned to the "weird and worrying" section at all !


Time doesn't exist.
Q : If time is based on the distance the earth travels around the sun then it would be different on mars and every other place in space. Time doesn't exist.

A : Time isn't based on the speed of the Earth (or any other planet) around the Sun, it's based on the speed of light. According to relativity this is an absolute and doesn't depend on the speed of who's measuring it. This isn't something that can really be explained in short post - try a book. I recommend this one :
http://www.goodreads.com/book/show/1000555.Einstein_for_Beginners


What do you think of UFOs ?
Q : I know saying something negative will likely shut you down, but I think it's arrogant to suggest that ETs and their crafts are not real when high ranking, respectable people say, beyond a doubt, that they exist and have been visiting us for a long time. Here's just a handful of names for you...

A : I've looked at this stuff before - extensively. I don't doubt that at least some people sincerely believe what they're seeing. Recently I read an interesting (albeit strange) book about people who believe that the claims of all the world's religions are literally true (https://plus.google.com/u/0/+RhysTaylorRhysy/posts/NfTwhxM3kuZ). Some of them claim to see angels talking to them as vividly as they see other people. I don't doubt their sincerity either, but I don't think they're correct.

The thing is that the more I looked into this stuff, the less and less convincing it became. The evidence doesn't ever seem to increase in quality. UFOs remain either as fuzzy blobs on low-quality images, which seems incredibly unlikely given the smartphone and dashcam era, or as more elaborate, overblown hoaxes. Claims of a mass conspiracy seem to utterly fly in the face of governmental stupidity and incompetence. I see no remotely plausible evidence that they're somehow clever enough to play this incredible double-bluff, let alone have it be exposed by a bunch of guys on YouTube.

I reached saturation levels of this stuff many years ago; I got sick of it. Although it's just my opinion that they UFOs aren't alien visitors, I won't consider any claims unless they're presenting really extraordinarily high levels of evidence - something akin to a flying saucer landing on the White House lawn. Fully visible in broad daylight with independent photographic documentation by thousands or tens of thousands of witnesses. Not one or two supposed eyewitnesses with dodgy photographs or some weird burn marks on a couple of trees. Not "I can't reveal my sources but I trust 'im" testimony. Perhaps that's arrogant and closed-minded of me, but after so, so many times when I've seen incredibly dubious claims proclaimed as incontrovertible truth, I simply see no other way to proceed. Not if I wish to remain sane, at any rate...


Why doesn't the FAST radio telescope have a radar transmitter to study asteroids ?
Q : Wondering why there is no active radar unit at FAST. It´s probably not a question of budget, is it ? How they can give up of such a chance to monitor NEOs and other interesting rocks while it is so big and complex observatory ?... :o Sorry, probably stupid question, but can´t get it out of my head.

A : I believe it's a question of weight and design. FAST has a very different, much lighter instrument platform than Arecibo. This means it can be moved laterally and vertically, so as the dish is deformed to change the focal point the whole instrument platform moves too - which gives it a much larger field of view than Arecibo. The disadvantage is that the platform is too light to hold many instruments at once - especially radar transmitters which are heavy, bulky things (because of the ~1 MW of power being transmitted they need large cooling systems).
In fact rather than taking new instruments up to the platform, FAST's approach is to be able to lower the platform to the ground. Arecibo's heavier design has an advantage here : it has the receivers installed on a turntable which can be rotated to select whichever instrument is required for the observations at the push of a button. I forget the exact numbers, but Arecibo has around 10 available receivers at any one time whereas FAST will have something closer to 1.


Could you survive falling in to a supermassive black hole ?
Q : Yeah but it is said that really big black holes do not crush you ! For example the one in the galactic centre of out galaxy does not crush you when you get close.
[The questioner was under the impression that you a singularity in a supermassive black hole wouldn't crush you, and wasn't just saying that you could survive falling through the event horizon.]

A : Singularities will indeed always crush you... if you hit them. But long before that the tidal forces of the black hole will tear you apart. It's not just that gravity is very strong, it's that it's even stronger on your feet than it is on your head.

The way this works is that the size of the event horizon of a black hole - the region which if you fall into you're never coming back out again, ever - depends on its mass. So you can think of black holes as having a sort of "surface", in a very loose sense. For more massive black holes this is further from the singularity, so you actually experience less acceleration and tidal stretching from a supermassive black hole than a smaller one. It could even be so little that you'd survive, for a really massive hole. But of course as you (inevitably) fall further and further in, you still die in the end.
http://hubblesite.org/explore_astronomy/black_holes/encyc_mod3_q16.html

Calculating the surface gravity turns out to be extremely complicated, so I'm not going to attempt to estimate any numbers.


Can you hear the sounds of space using a radio telescope ?
Q : This article claims that you can.

A : Well, real sound waves in space certainly do exist, but most of them would be of a totally inaudible volume and outside the human frequency range anyway.
http://astrorhysy.blogspot.cz/p/q-a.html#inspacenoonecanhearyouaaaaaarrgh

But "converting electromagnetic vibrations into sound", while interesting, even meaningful, doesn't mean that you're really "hearing" the planets. It would depend on how the radio waves are generated. If it's generated by density waves moving through the plasma and if the frequency of the sound wave is somehow converted into exactly the same frequency at radio wavelengths, then yes, you're "hearing" space. But this is exceedingly unlikely. Radio sets only transmit real sounds because they've been carefully engineered to do so.
http://electronics.howstuffworks.com/radio.htm

Much more likely is that you're hearing radio emission generated by various different processes in the plasma - which will be more complicated than sound waves. Just as music player software can generate pretty visualisations that depend on the sound being played, what you're hearing may be - at best - equivalent to the original sound but it's not the same thing. So this is not completely fictitious, but it's a very large step to call this "what space sounds like".

At a press conference in August of 2012, Don Gurnett, the principal investigator for Voyager 1’s plasma wave science instrument, demonstrated a series of sounds the instrument had picked up. “Strictly speaking, the plasma wave instrument does not detect sound. Instead, it senses waves of electrons in the ionized gas or ‘plasma’ that Voyager travels through. These waves, however, do take place at frequencies that humans can detect. We can play the data through a loudspeaker and listen. The pitch and frequency tell us about the density of gas surrounding the spacecraft.”
http://www.inquisitr.com/1545488/nasa-recorded-sounds-in-space-the-results-will-amaze-you-video/


Can gravitational waves push black holes around the Universe ?
Q : There seems to be a problem with this article, Gravitational waves are a result of very large objects moving around one another and maybe merging under the effects of gravity. The waves do not go flinging super massive black holes around the universe.
http://www.independent.co.uk/news/science/nasa-supermassive-black-hole-discovery-a7650656.html

A : There's actually a nice video in the article explaining how this can happen. The effect is called gravitational recoil and can happen as two black holes merge. A strange effect emerges in general relativity when two massive, compact objects start orbiting each other very closely - the gravitational waves can be "beamed" preferentially in one direction more strongly than another. Huge amounts of energy can be carried in gravitational waves, even though they're bloody hard to detect (unless you're really close) - so directing this energy in one direction is very much like how a rocket works, by chucking mass out in one direction.
https://en.wikipedia.org/wiki/Binary_black_hole#Black-hole_merger_recoil


Could we push a black hole away using an electric charge ?
Q : In fact, we could even move a black hole fairly easily if we ever needed to do so... black holes have an electrical charge, so we could use the same charge to "push" a black hole, or an opposite charge to "pull" one... theoretically that is... we can't generate such large charges at this time.

A : Black holes can have charges in principle, but they aren't expected to do so in practise. To accumulate a charge they'd have to absorb more particles of one charge than another, but since the Universe appears to be electrically neutral overall, there's no reason to think it would ever encounter any strongly charged matter. Even ionised gas (which might have local overdensities of one charge or another) is neutral overall.

But in theory, we could artificially give the black hole a strong electric charge by shooting beams of protons or electrons into it. Eventually the charge would become so strong it would cause charge induction in the surrounding matter and preferentially attract the opposite charges, neutralising it. But this would take time, so let's ignore it. Then the question becomes how much charge we'd need to push it away - that is, to equal the acceleration caused by the hole's gravity.

Let's assume we have a device of mass m that charges the hole while becoming equally charged itself. Then we can say that the attractive gravitational force (http://hyperphysics.phy-astr.gsu.edu/hbase/grav.html) is equal but opposite to the repulsive electrical force (http://hyperphysics.phy-astr.gsu.edu/hbase/electric/elefor.html). This leads to the equation Q = sqrt(G.M_hole.m / k) where Q is the charge, G is the gravitational constant, and k is Coulomb's constant.

Interestingly, the charge required doesn't depend on distance because both electric and gravitational force depend on distance in the same way, so they cancel. However it does depend on the mass of the device. For a 1 kg device and a 5 solar mass black hole (about the smallest it's believed they can get) this gives a charge of 270,000 Coloumbs, which is a terrifyingly scary number. But the number of charged particles required is surprisingly modest, equivalent to a mass of a few grams. So, in principle, it does make more sense to charge the black hole than try to use another large mass to drag it along.

The problem would be in generating this incredibly high charge on the device. That massive charge is going to be trying to blast the thing apart. At a typical density, our 1 kg device would be experiencing accelerations billions of times the surface gravity of Earth. No conventional material could survive that - it will rip atomic bonds apart. If we make the device more massive, the charge required increases even further. If we make the device very low density, it will be exposed to more of the interstellar medium and tend to neutralise itself more rapidly. About the only material that could survive such a collosal acceleration is that of a neutron star, which naturally experiences such high inward accelerations due to its gravity. Unfortunately such material is very, very unstable if you take it outside of the high gravity of the neutron star itself (https://www.youtube.com/watch?v=o_EBqZPCZdw - well worth watching !) so we'd end up needing something almost as massive as the black hole itself.

All in all, it would probably be easier to move the hole by sending large masses past it to tug on it gravitationally.


Could the reported speed of this black hole be in error ?
Q : [From a larger discussion on this article, which you can read in full here. Although this exchange doesn't go anywhere, I thought I'd preserve it for its sheer oddness.]
Their estimated speed, is considerably in err. Perhaps an independent laboratory would be a tad more objective in their findings. At first, we thought that number was a misprint, being so far off the mark. As independents, we believe they merely made what they thought to be an educated guess. We are at a loss to find any other logical conclusion to their carelessness and lack of concern for their readers.

A : So, would you explain how you came to the conclusion that their measured speed is in error ?

Q : The theories they offer as proof of their findings will differ from ours. I will only go so far as to suggest that they may be wrong. If I could show you a proof of absolute certainty, I have found long ago when ones indoctrination is tied to their income, neither hell or high water will change their mind. Differing from accepted and well documented theory may arouse fears of becoming a pariah. (lol) Like me!!!

A : Well then, if you won't offer any any reason why you think they're wrong, then I'm afraid I don't see the point of saying, "they're wrong" without also saying, "here's why".

Q : The little I have learned in life has taught me to respect others, regardless of what they think. Another important fact that I've learned along the way, the great majority of theories, hypothesis or call it whatever you like, have been wrong. When they come back and say their analysis of the new finding was wrong by so much, (and logic dictates in all probability, they will) what do those who are so indoctrinated and willing to believe and accept too readily, tell themselves??? Have a good life, stay safe and read. It forces us to think.

A : Though science admits its mistakes loudly and publically. As opposed to, "I think it's wrong but I won't say why".

Q : Will you please expound a bit on that last comment? We may just have to agree to disagree here. I may be diametrically opposed to your reasoning. One question may bring about a prime example of my income/indoctrination logic. Do you work in the medical field? Pharmacology, prostheses or anything similar?

A : Sure. I'm not a medical doctor, I'm an astronomer. Did my PhD in Cardiff, my first postdoc in Arecibo, the current second one in Prague.

My comment was about two aspects of things you've raised : 1) "I have found long ago when ones indoctrination is tied to their income, neither hell or high water will change their mind."
Honestly, I could go on at great length about why this is not applicable. As a scientist, I publish papers in which I describe new discoveries. If I thought I should only publish things which support existing ideas and not challenge them, I'd immediately quit and do something else. Nothing would be more tedious or boring than only publishing things we already know about. No, it's discovery that's the name of the game in science; there is even a direct financial motivation to challenge the existing ideas. But - and this is the really crucial thing - you have to be able to demonstrate really well when something flies in the face of accepted wisdom. You can always speculate about what would happen if things were different, but to publish something claiming that things actually are different requires real rigour. It's not enough to simply say, "I disagree". But if you can say why you disagree, and those reasons are not nonsensical (which they all too often are), then I assure you that you will not find indoctrination much of hurdle.

The example I point most often to is the discovery of the acceleration of the expansion of the Universe. No-one predicted this; everyone thought the expansion should be slowing. But when the research was published and the quality of the data was clear, it was quickly accepted by the community. Not entirely, mind you, because that's not how science works (or people for that matter). Alternatives were considered (they still are, from time to time) but found wanting.

2) You keep refusing to say why you think there's a problem with the measurements. Now, suppose you're right, and eventually everyone else agrees. Then what ? You can say, "I told you so", but since you don't have any written proof of your argument, it will be meaningless. You will not have a leg to stand on, no-one will believe you really did discover it for the reasons you claim because it will be impossible to validate. You might have believed things for totally different reasons to those you then claim - wrong theories being far more numerous than correct ones, but often giving the correct results in extremely specific circumstances. Similarly you can keep saying, "you're wrong but I won't tell you why" if you like, but what will this achieve ? Nothing much, except to identify yourself as a possible madman. Alternatively you can actually state your reasoning and put it open for discussion as scientists do and take the same risks the rest of us take whenever we propose a new idea. It may well be that you are a madman; at least then we'll have something to judge you on. If you are, you won't care about anyone else's opinion anyway, so you may as well state it. If you're not, you may have just been misinformed or misunderstood something, and if so we can correct you and you'll go away having learned something; then again you might be right and can teach the rest of us something. Put simply, if you don't actually tell us your reasoning, you will certainly earn yourself a bad reputation, but if you do describe it, then there's a chance of a positive outcome for everyone.

I thought my response was entirely reasonable, but, predictably, nothing useful came of it. First there was a reply which was deleted (so I won't repeat it here) that left me convinced this person is a moron. Then there was one about asking what I thought of the 9/11 attacks. I lost patience at that point. To be afraid of being denounced as a lunatic is one thing. But to be afraid of being called a lunatic because of your explanations, yet unafraid to state your claims, and simultaneously refuse to answer simple questions while expecting that everyone else answers your own questions - this is mad. It's impossible to argue rationally with such a person, so I gave up. Repeated statements that "we should all be nice and respect one another" only look like so much bollocks if you're not actually prepared to do the most basic courtesy of answering a simple question.


When will we get these pictures of the black hole in Sag A* ?
Q : so when are we going to get pictures of this black hole in Sag A everyone's been working on? Is it true, as I suspect, that those images will be later in coming than next season's Better Call Saul, what can be done to motivate them? A case of Macallan when they're done? Everyone knows they're all soaks....

A : Tough to say. If I have to guess, I'd say, "later this year" and refuse to be any more specific than that. There are many difficulties. The Event Horizon Telescope is really a collection of telescopes observatories that also have other competing projects to juggle so it doesn't get many opportunities for suitable observations. Although there are many antennas in the array (http://www.eventhorizontelescope.org/array/index.html) IIRC at any one time only three will be able to observe the black hole - because, by necessity, they're spread out over the surface of the Earth. That's popularly explained as giving you a resolution equivalent to a telescope the size of the Earth. And it does... but there's a huge loss in sensitivity as a result, much more than due to the fact you're not using a collecting area the size of the Earth. Example : the VLA has a total collecting area about one-fifth as much as Arecibo, but a sensitivity to diffuse gas about a thousand times lower. This can be compensated for (somewhat) by so-called "Earth rotation synthesis", letting the Earth's rotation effectively move the telescopes so you're sampling from different points. So you need as much observing time as possible, which is difficult when you can only point three antennas at it any one time, the weather is bad and there are other projects to juggle.

Oh, and more pedantically, it's possible the first image of the event horizon won't be of Sag A* at all, but the much more distant M87 (which is thought to be much larger).

So don't hold your breath. On the other hand, astronomers are a lot less fickle than TV producers and don't cave in to the braying of the mob nearly so easily (witness gravitational waves !). If they say they can do it, then they probably will.


Surely radio bursts in space aren't the least bit surprising ?
Q : [In relation to this article : http://newworldwow.com/science/radio-bursts-detected-coming-outer-space/]
I would imagine that since "radio" is electromagnetic radiation, and pretty much everything in the Universe emits it, we're going to see "radio bursts" in space. Facepalm.

A : Constant signals are perfectly normal, but "bursts", i.e. something which is detectable for a while but then stops, are not. At extragalactic distances that implies an incredibly powerful and compact (as the article says, they only last millseconds) source. Signals from the entire galaxy should not fluctuate in a coherent way.

Also note that although "extragalactic" isn't in the headline, it's clear in the article. In fairness, as it stands, yes we will definitely see "radio bursts in space" - that much is facepalm-worthy. :)


How can you tell the difference between stars, planets and satellites just using the naked eye ?
Q : Is there an easy way to tell the difference between stars, planets, and satellites when looking at the night sky unaided (especially in urban areas) ?

A : Satellites are easy because they move rapidly (you can easily see them moving). Jupiter, Saturn and Venus are easy because they're relatively bright (Jupiter and Venus are brighter than any star). Mars is harder because it's often faint. Mercury is probably the hardest because you can only see it just before and after sunset.

It's often said that planets don't twinkle, but I have absolutely no idea why people say this, because I've never noticed any difference between the twinkling of planets and stars.

By far the easiest and most reliable way is to consult a chart of the positions of the planets beforehand, or with an app on a phone or tablet while observing. Not very clever, but it works !


What's the difference between dark matter and anti-matter and are there any practical applications ?
Q : Well, what's the answer then ?

A : Anti-matter is basically normal matter but with the opposite electrical charge (this is a simplification, but it'll do as a start). When particles of normal and anti-matter collide they annihilate each other, releasing an enormous amount of energy. Anti-matter can be produced in laboratories but so far only in tiny amounts.

Dark matter is thought to be an unknown type of particle that does not interact with normal matter except through gravity. It has not been observed directly - it's inferred from observations of the motions of galaxies (among other things). It's not the only explanation but it is by far the best one.

Some models suggest that dark matter could be its own anti-particle. If so, it would be slowly annihilating itself into energy, which we'd detect as gamma rays. There have been a few claims for detecting the sort of signal we'd expect if that was the case, but they seem to be getting weaker. We don't yet know if dark matter is its own anti-particle - if it isn't, it could be very hard indeed to directly detect it.

Practical applications of anti-matter - well, if it could be produced in large amounts, we'd have unlimited clean energy. Dark matter... well, there's not so much you can do with matter that only interacts through gravity. :) It seems to have a very low density, making it difficult to think of any obvious practical uses.


Why are black holes such messy eaters ?
Q : So I've been looking at various aspects of the [galactic centre] problem. Chandra did a great long observation in 2013 at x-ray frequencies. VLT in Chile looked at it in the infrared and came up with some theories about the G2 gas cloud, that it was once a star - or several stars, ripped apart by interaction with the black hole. But what really surprised me was how little the black hole consumes. Conditions have be perfect for the material to cross the event horizon. Angular momentum, especially.

A : Yes, black holes are indeed messy eaters :
https://www.forbes.com/sites/startswithabang/2016/06/09/how-do-black-holes-eat-like-cookie-monster/#1aac593b7774
Thing is that if you fall on a trajectory which even just slightly misses the surface, you'll have some angular momentum relative to it. And then you get spat back out on an elliptical orbit :
https://blogger.googleusercontent.com/img/b/R29vZ2xl/AVvXsEiyPschI_1sZ4nXTMs5M8B4nDIMFTVprINifPnTQnx-PxAxLAIlqoLeD9YMroaUEGvASCE1Xndq-X4JjXTTn37_KyUUXZzyNbYeWh70DVPe4rUzq7sUPdPKwhBXta3HYpBx1qw8DvOR6Gk/s1600/FallingOrbitsSmall.gif
(old, "falling off a space elevator" gif, watch what happens to the highest dot)
So unless you happen to fall on a direct collision course (or in practise through the accretion disc or other intervening matter that could slow you down), you're safe from the event horizon. And since black holes are tiny, they're very hard to hit. They can scatter things to the left and right of them like nobody's business, but they eat very little.


Should we all turn off all our lights to enjoy the beautiful stars and galaxies ?
Q : It should be called "Space Day".

A : There's already Earth Hour, but it's very controversial. For one thing, there are extremely good reasons we have lights - preventing crime, traffic accidents, etc. It may sound nice, but in practise it's a terrible idea.
https://en.wikipedia.org/wiki/Earth_Hour


Could we all be aliens ?
Q : According to the evolution theory we all are evolved from a unicellular organism. Where did that unicellular come from ? I know that there are many theories to prove that ..but I am just thinking about an really deferent idea about where it comes from. I am just thinking that why it's not possible that any another alien species found our planet millions of years ago and they just send a unicellular organism here to make a new colony here ....

A : Well presumably the aliens would have had to have come from some unicellar organism, so it wouldn't really explain anything about how life started. That's certainly a difficult question. A few people think that the chances of life arising on the early Earth were so low that it must have originated from somewhere else - not necessarily by deliberate alien intervention but just by bacteria surviving in asteroids/comets. This means that life could have arisen originally on any one of the estimated billions of planets in the galaxy, greatly increasing its chances to develop. This isn't a very popular idea, though there's more support for possibility that cells could survive interplanetary journeys.
http://www.panspermia-theory.com/


If spatial topology reveals itself to be constitutively hyper-toroidal (Einstein-Rosen bridges, EPR thought experiment/non-locality), are doughnuts a good source of nutrition ? #IndirectSugaryOntologies
Q : Well ? I wanna eat doughnuts, dammit.

A : Doughnuts are always a good source of nutrition and are one of the under-utilised major food groups.


Why don't we turn into buckets of goo ?
Q : I thought gravity was all around me. Isn't that why we don't turn into buckets of goo??? Isn't it the internal and external forces that make up "gravity"?

A : In fact there is gravity in every direction from other distant objects, but on Earth (or indeed close to any massive object) this is so small you can ignore it. You don't get squished into a bucket of goo simply because atomic bonds are more than strong enough to resist 1g of acceleration from the Earth.


What do you think of this video about NASA being speechless about the discovery of a super-powerful object billions of light years away ?
Q : Yeah, I didn't actually last you for comments but you're going to anyway, aren't you ?
https://www.youtube.com/watch?v=cD5MnHZUnOw

A : Pro tip : SecureTeam is not a reliable source of information. They completely neglect the fact that ASASSN-15lh was an explosive event. Sure it's mysterious... but it's not that mysterious. If it had a continuous output as strong as its peak, then yeah, that would be bloody weird. But it doesn't.
https://astronomynow.com/2016/01/14/possibly-the-most-powerful-supernova-ever-seen/

The existence of large quasar groups is also disputed :
https://briankoberlein.com/2014/09/16/calling-question/

Makes my blood boil every time someone says "scientists baffled" or "have no idea". If you believe the press we must be some kind of schizophrenic crazy people who are perpetually switching from solving mysterious to a state of total bewilderment. Aaarrgghh.


Does the Arecibo telescope need to be kept clean ?
Q : I'm curious; how important is it to keep the dish clean? If a bird poops on it does someone have to go out there and scrub it? And with the work that was just done, has the sensitivity of the receivers/transmitter increased?

A : Not very. Actually the dish is filthy ! There is (or was) a photo of the dish in the library back from when the aluminium panels had just been installed (before then it was just a wire mesh) and the difference is huge. There was talk of designing a robot to clean the dish, but that never seemed to come to anything. Too many other, more serious issues with funding.

What matters more are plants growing underneath the dish that can push up and deform it. Dirt and plants are pretty well transparent to radio waves, and it doesn't even matter about a few holes in the dish, but if you change the shape of the dish over a wide area, that changes the focal position. That loses you significant amounts of sensitivity (this is also why it's hard to clean), so plants have to be regularly cut. There was talk of getting in a herd of goats, but alas that never happened either.

As far as I know recent work has been limited to repairs and painting the platform. That helps prevent rust but there haven't been any major changes to any instruments. An exception is the ionospheric heater (which I know very little about but I don't think it's operational yet).


What does Physicist Valhalla look like ?
Q : This notion was raised in a discussion about a physicist who was, for reasons unknown, described as legendary and mythical.

A : Physicist Valhalla is a lot like Physicist Daily Real Life, except with unlimited funding, no grant proposals, unlimited telescope time, no teaching, no appraisals, no group meetings, no travel request forms, no deadlines, and a decent canteen. Conferences are exactly like they are now, except that everyone keeps to time and no-one has graphs with labels too small to read. Ale is quaffed in evenings while tea is - very carefully - quaffed in copious quantities throughout the day along with the gluttonous consumption of truly obscene amounts of cake that would make a baker blush. Papers are full of wit and verve with a narrative flair that makes even the most tedious report a delight to the soul. Letters and social media posts from crackpots claiming that the Universe is actually entirely made out of potatoes still exist, but at a rate so low that they remain an entertaining diversion.


Can you tell us more about this graph?
Q : I mean this one : https://twitter.com/astronomolly/status/918854680050061313

A : I can tell you that it's difficult to read and the paper isn't publically accessible... :P


Is this story about a wormhole absolute rubbish ?
Q : I mean this one : https://www.quantamagazine.org/newfound-wormhole-allows-information-to-escape-black-holes-20171023/
[This discussion involved some needless criticism of the scientific method. See, science is sometimes about hard facts and sometimes about wild, wholly unjustified ideas. Both have their place. Legitimate criticism requires the reader to understand when each is appropriate.]

A : Of course, it's theoretical physics, not observational astronomy. Theorists are supposed to and encouraged to think outside the box. Seeing if the model is applicable to the real world only occurs at a later stage.


What do you think of this article on panpsychism ? Q : I mean this one : http://bigthink.com/aeon-ideas/panpsychism-is-crazy-but-its-also-most-probably-true

A : [Unfortunately there are quite a few big websites able to pretty much guarantee that ANYTHING will go viral and have little or not regard for the contents of what they post]

The author has completely misunderstood both probability - offering precisely zero evidence supporting panpsychism and then declaring it to be "highly probable" - and Occam's Razor, taking it to mean the sci-fi movie version that "the simplest explanation is the right one". This is nonsense and bears no resemblance to what Occam said, which was more like, "complexity should not be increased beyond necessity". If you start thinking that an explanation is more likely to be correct because it's simple, then you rapidly end up with, "a wizard did it" as your solution to every problem.
http://astrorhysy.blogspot.cz/2016/03/youll-have-your-eye-out-with-that.html

TLDR : scientifically, simpler explanations are preferably because they are easier to test and harder to fudge. The idea that they're also correct is utterly at variance with observation. The Universe is not, and I say this as an observational astronomer, even remotely close to anything I call simple.


Is nuclear winter just a theory ? Q : Nuclear winter is a theory not fact.

A : Well of course it's a bloody theory, if it was a fact we'd all be dead !

Aaaaaarrrggghhh.


What events in astronomy are you looking forward to in 2018 ?
Q : We appear poised to enjoy some significant space launches and exploration which is never a bad thing. I suspect that Osiris-Rex will be this year's rockstar mission. What about astronomy ? An image of the event horizon of Sag A* from EHT, Gaia DR (2). Anything else you're particularly looking forward to ?

A : I really hope Gaia is going to make one of those "hmmm, that's funny..." discoveries, but we may have to wait a while for that, sooo much data !

I don't know what there schedule is like, but I'm looking forward to future development with the SKA pathfinders ASKAP, MeeKAT and APERTIF. It's the things which have an unknown discovery potential that most excite me. And looking very slightly further ahead, 1st January 2019 should see New Horizons at its next target...

But personally I think the highlight is going to be the first Falcon Heavy flight launching a Roadster into space. It's gonna be just like Star Trek Voyager !
https://vignette.wikia.nocookie.net/memoryalpha/images/8/81/Ford_truck_the_37s.jpg/revision/latest?cb=20060825032855&path-prefix=en
[And I wad dead right about that, because it was supremely cool and I doubt anything else is going to have as much sheer fun and joy about it as seeing a car floating through space. Those who disagree probably have no soul.]


Will we see the event horizon of a black hole in 2018 ?
Q : Well, will we ?

A : I'm optimistic on a first EHT image this year. But I wouldn't expect it to be any good for a fair few years yet. This one requires as much data as possible. The Event Horizon Telescope is an interferometer with its component antennas spread very far apart. This gives outstanding resolution, but there's a penalty : information of certain scales is lost. A crude analogy might be looking through a fence with a few holes poked through and trying to work out what's on the other side : things get a lot easier if you're allowed to poke even just a few more holes. The effect is the same for the EHT : the longer we observe it for, the more different telescope configurations we'll use and the more telescopes that are added to the network, and the better the end result will be.
(Note that this analogy is VERY crude - interferometry is super complicated and I hate it - but it gets the point across !)


What do you think of the notion that scientific ideas must be falsifiable ?
Q : What are your thoughts on falsification as a necessity for scientific theories ?
[This all started with a very nice post by cosmologist Sean Carrol in which he states that we must move beyond falsification as an absolute requirement for a theory to be deeemed scientific.
https://arxiv.org/abs/1801.05016
Predictably, as he notes in his blog (https://www.preposterousuniverse.com/blog/2018/01/17/beyond-falsifiability/) this triggered a wealth of responses, of which the ones I found the most interesting were :
http://www.math.columbia.edu/~woit/wordpress/?p=9938
https://platofootnote.wordpress.com/2018/01/22/peter-woit-vs-sean-carroll-string-theory-the-multiverse-and-popperazism/ https://www.npr.org/sections/13.7/2018/01/22/579666359/scientific-theory-and-the-multiverse-madness
And see also this earlier article :
https://www.nature.com/news/scientific-method-defend-the-integrity-of-physics-1.16535]


A : This is something I've covered before in general terms. First, on falsifiability : http://astrorhysy.blogspot.com/2017/05/i-told-you-he-was-tricksy.html
And on the closer related issue of simplicity : http://astrorhysy.blogspot.cz/2016/03/youll-have-your-eye-out-with-that.html
So I don't have much to add here, except for the specific issues of the multiverse.

I agree with Sean Carrol about there being shades of grey as to what constitutes a scientific or unscientific theory. I also agree with him regarding some people taking falsification to extremes, even if the references he cites don't (as the rebuttals note) actually demonstrate this. However, I agree with the rebuttal articles that the multiverse isn't business as usual, because you can't test - not falsify, but merely even evaluate it against other theories - its major prediction. I think the Nature article linked by Carrol says it best :

"As we see it, theoretical physics risks becoming a no-man's-land between mathematics, physics and philosophy that does not truly meet the requirements of any."

Cosomology, someone once said, is always on the edge of mysticism. And regarding the Many Worlds interpretation - surely the philosophical next of kin to the inflationary multiverse - someome else once said, "surely it doesn't take the creation of an entire universe to kill one cat." And again from the Nature article :

"In our view, cosmologists should heed mathematician David Hilbert's warning: although infinity is needed to complete mathematics, it occurs nowhere in the physical Universe."

However, I take issue with a couple of points in the Nature article :

"In our view, the issue boils down to clarifying one question: what potential observational or experimental evidence is there that would persuade you that the theory is wrong and lead you to abandoning it? If there is none, it is not a scientific theory... Dawid argues that the veracity of string theory can be established through philosophical and probabilistic arguments about the research process... Instead of belief in a scientific theory increasing when observational evidence arises to support it, he suggests that theoretical discoveries bolster belief."

Well, this clearly does show that Carrol was not attacking a straw man when he said that some people take falsification and even testability too far. Yet while it's wrong to suggest that belief in a theory should increase because of theoretical arguments, that doesn't mean that theoretical arguments can't lend a preference for it. It's a very good thing, in my opinion, to prefer simple ideas that avoid infinities. You just shouldn't cling militantly to that mere preference in the face of the evidence.

I'll further muddy the waters by noting that "scientific" and "useful" are certainly not the same thing, nor is being useful even necessary for a scientific theory. The idea that the Universe may extend beyond the visible horizon is logical, rational, eminently scientific and perhaps even escapable given current knowledge. The idea that it extends to infinity and perhaps contains other spacetimes with different physical laws, well, I don't know if that's scientific or not - it's certainly not pseudoscience or mystical woo, so perhaps it's a new class of weird - but it's definitely useless (though see Hossenfelder's blog post).

There are certainly interesting theories which are useful, useless theories which are interesting, and useless theories which are not interesting. The infinite multiverse (and its variants) currently fall in the middle ground, in my opinion. It might be true. But "everything happens somewhere" currently lacks much credence to be a scientific theory even if it is true. It's the measure problem : you can't have meaningful probabilities if reality is infinite. Worse, multiversers seem (often) to use this as an asset, trying to explain specific problems because "everything happens somewhere" whilst trying to stick to hard, proper physics elsewhere. It's trying to have your cake and eat it.

I would venture to go even further, and say that any statistical explanation for why the Universe is the way it is (i.e. just chance alone) is not really science. You have to have a mechanism by which things act. Electrons don't flow "because that's just the way they happened to be moving", objects don't fall to the ground "because they just do", the value of G isn't 6.67E-11 because "why not ?". It takes more than dice to make a theory.

Which does not preclude the proto-science multiverse or string theory from becoming genuine science, one day. If the measure problem could be solved, so that you could somehow assign a probability even given infinities, and still show that physical processes could remain meaningful (i.e. not everything is occuring due to chance, but that there really are actual fundamental mechanisms driving physical processes), then it would be much more interesting and maybe even useful.


Does the IRAM 30 m telescope complain when it gets too cold ?
Q : Does the build-up of snow on the dish affect the telescope's performance at all ? What band are you observing in? I always assumed radio telescopes to be lower altitudes, I guess it's either something that gets absorbed by water vapour or it helps the sensors stay cool and reduce the noise level?

A : I can probably answer both questions at once. It's a mm-wave telescope, which to some more pedantic astronomers (not me though) is not the same as radio because the frequencies are much higher - enough that ice and water vapour make a difference. How much depends on exactly what frequency is observed. Ice and snow do affect performance (not least, as I understand it, because they can cause instruments to seize up - their are moving parts in the antenna), so they have to be cleaned off. There's even a 1 MW (! - I did a double-take when they told me that, though they don't use it at full power) heat source to help melt the ice off the dish, though for the receivers people have to go up there and brush/hack it off.


Does that telescope have its own ski lift ?
Q : Surely no-one ever gets any work done ! Bloody freeloading astronomers...

A : Well, not quite... :P The telescope is basically on a ski resort; it's not there especially for telescope users. Still, I would have thought a ski lift would be terrible for RFI, but they seem to manage somehow...


I'm an Aries. Any advice for my love life?
Q : Juuuuuuuuuust kidding.

A : Tough, I'm answering it anyway. Ah, Aries, the ram. Head-butt your opponents into submission and claim dominance over your harem. That's science, that is.


Have you tried giving the telescope a nice hot chocolate when it's snowing ?
Q : I find it very soothing.

A : The telescope's not getting any tea until my stupid cold clears up. :P


Are you a rah-rah scientist ?
Q : I do like that you're not a rah-rah kind of scientist. Objective science often requires the "I doubt it". Even when it is obvious. Just look at the road-side debris of outcast theories.

A : What's a rah-rah scientist ?
Q : ummm ... evangelist, cheerleader etc
A : Ahh well, just get me some pom-poms and/or a call-in show where I ask people for money... ;)

I'm a cheerleader for the overall ideal process of self-examination and improvement. I've got theories I prefer over others and people I like less than other people. I don't see the point of being a fanboy of any of 'em though; there only ever (with rare exceptions) theories which are better given the current evidence. Not much point waving a pom-pom for something you expect is eventually gonna end up in the gutter. :P


Is that my timeline of the Universe Stephen Hawking is presenting ?
Q : Wait, is that an image of Stephen Hawking presenting a timeline of the Universe ? Because if so, that's one of mine, and my ego might actually explode.
http://www.rhysy.net/other-1.html
https://plus.google.com/u/0/+RhysTaylorRhysy/posts/RhXtJAU2Uxz

A : I think it probably is...


What do we call the points in orbits closest to and furthest away from the central mass, specifically with regard to black holes ?
Q : I just found out that there are terms for the highest and lowest points in an orbit around a black hole... apobothron and peribothron. Honestly, I tend to think specialized terms for orbits about specific bodies just makes it harder to follow what's going on. Apogee? Aphelion? Apojove? Apoaerion? That last one gets confused with "apeiron" way too easily. Maybe we should just call them all apoapsides, and rely on context to establish which body is being orbited.

A : Given that it's about black holes, I propose aposcary and periscary.


Is mainstream science a colossal douchebag because it pretends the Big Bang is real ?
Q : It's unbelievable what mainstream science will do to keep this bogus theory known as the Big Bang alive. They are stooping so low as to mislead the laymen. Not only do they assert that the universe is static in their assertions to disprove an infinite universe in a state of constant flux, but also disregard the limitations of our technology. Unbelievable!

A : Sigh. No-one, literally no-one at all is out there to deceive the laymen or anything even remotely like that; pretty much everything you've written is wrong on every level. Honestly, when the arguments are written plainly and clearly - the maths is sometimes very complex but the concepts aren't that hard - I just throw up my hands in despair. But for whatever it's worth, here's my little summary (and see also references therein).
http://astrorhysy.blogspot.cz/2015/04/the-most-astounding-fact.html


What shape are black holes? And what is the reasoning for that shape ?
Q : Well, what's the answer ?

A : A black hole has three properties : mass, electrical charge, and spin. Electrical charge is unlikely to be significant since if it became strongly charged it would rapidly accrete material of the opposite charge, neutralising it. So effectively just mass and spin.

Although it has mass, according to general relativity all of this mass is concentrated in the singularity at the centre of the hole. If the hole isn't rotating, the standard model predicts that the singularity will be a point of infinite density : it has no shape because it has no size. If, however, the hole is spinning, then the singularity will be an infinitely thin ring rather than a point. Singularities aren't continuously collapsing, they just have infinite density (most people believe that these infinities, where the equations break down and no longer make sense, point to a flaw in the theory - they may not actually be infinitely dense, but would have some physical structure).

Since there's no other mass to the black hole, the rest of it can't be said to have a shape in the way matter has shapes. However, the gravitational field does vary, and that shape can be described. The usual convention is to refer to the event horizon, which is where the gravitational field means that to escape from the hole you'd have to travel at the speed of light. For a non-rotating hole, this is spherical. But it isn't a solid surface - there's no physical substance to it.

For a non-rotating hole things get unpleasantly complicated. At its simplest, as well as the event horizon there will also be an elliptical-shaped "ergosphere", from which objects can escape but must remain in motion : http://www.astronomysource.com/tag/what-is-an-ergosphere/
Which, interestingly, can be exploited to make a bomb :
https://www.youtube.com/watch?v=ulCdoCfw-bY
The longer, vastly more complicated answer is that rotating black holes can have multiple event horizons, in which time and space become somewhat more interchangeable than usual :
https://www.quora.com/I-recently-read-a-paper-that-stated-two-event-horizons-exist-around-a-black-hole-Inside-the-first-horizon-causality-would-be-flipped-and-passing-through-the-second-horizon-would-re-flip-causality-to-normal-How-is-this-possible-and-why-are-there-two-event-horizons


Did you just predict the battle of Callisto from The Expanse ?
Q : [Regarding my video "Deep Space Force"]
I dunno, did I ? I didn't get along so well with that show, so I haven't seen the latest season.

A : Winchell Chung writes : I don't think so.
Yes, in The Expanse there was a battle between the Earth and Mars fleets over Callisto. But they were not Orion-drive ships, and certainly not US and Soviet ships.


How much stuff has been found using microlensing ?
Q : About microlensing, how much stuff has been found out there, ultimately? Can it differentiate between close asteroids and distant neutron stars? What can we expect to find in interstellar space?

A : As to how much has actually been detected, I don't know. However there are upper limits from observations as to how much there could potentially be, which is a few percent of the mass of the halo. Loads of interesting things from a stellar or geophysical perspective, but negligible for galactic dynamics [which was the context of the question]. https://kellyoakes.wordpress.com/2010/09/24/machos-wimps-and-the-mystery-of-the-missing-mass/

IIRC, microlensing is able to estimate the mass of the object, so it ought to be able to distinguish asteroids and neutron stars. The problem is that it can detect stuff which is so damn faint you can't find it with other instruments, and the configuration for a lensing event is so precise you might not see it again even for an object orbiting a star. It gives very limited but interesting information.
https://www.universetoday.com/138141/gravitational-microlensing-method/


Is the Fermi Paradox really much of a paradox ?
Q : Well, is it ?
[This was in response to a post on Quora where I described the limitations of Arecibo as a transmitter. Despite being the most powerful transmitter on the planet, any signals it sends out would require even larger telescopes to detect, at least at distances greater than a few light years. Strangely, I've never done a proper blog post where I explain my views on Fermi in detail, although I suppose this one comes closest.]

A : I should probably add to that that while an important criterion for SETI is that the signal repeats, both times Arecibo transmitted a message it did so only once and briefly. Since we ourselves have not transmitted any kind of repeating signal to anyone who might be listening, I think the Fermi paradox is easily solved when it comes to the lack of detected direct communication signals. They would already have to know that we're here to even bother with the reasonably sophisticated and energy-consuming signal we'd be able to detect. So in terms of human-level technological civilisations, the galaxy could be full of them and we'd have no clue who was out there.

What I find much less easy to dismiss is the lack of galactic empires. We can already envisage the kind of technologies needed to cross interstellar space. The only thing we're lacking are the economic resources to do so. And our technological advancement took a long time to get going, but when it did, it spiked. Thus the galaxy ought to be teeming with super-advanced civilisations but that doesn't seem to be the case at all. Personally, I don't find any of the proposed explanations convincing : with ~400 billion stars and likely trillions of planets, plus 10 billion years of time, someone ought to have colonised the galaxy by now. I see no plausible mechanism that could stop them.


Do we need a better measure of brightness to describe the naked-eye visibility of astronomical objects ?
Q : The apparent magnitude is the widely used measure to indicate the brightness of night sky objects as viewed from Earth. This fits with our observations while observing stars and planets. Venus appears brighter than Sirius which is brighter than Saturn. Their apparent magnitudes agree too. But we can see 4th magnitude stars from light polluted skies but we cannot see the Andromeda galaxy at 3.44 magnitude. We know that the galaxy is a diffuse object with light scattered over a large angular area in sky. So for diffuse objects, apparent magnitude alone won't confirm if it is a naked eye object. Should we not have a different measure of magnitude for diffuse and fuzzy objects like galaxies, comets and nebulae? For galaxies, we can possibly measure the brightness of its core alone without its spiral arms. For comets, we can possibly measure the brightness of its nucleus without its coma and tail. Would this measure help common people understand if a comet or galaxy stays within naked eye visibility? In other words, do we need a different measure of brughtness that indicates naked eye visibility?

A : The term we use is called surface brightness. It's a measure of the total amount of light emitted per unit area :
https://dept.astro.lsa.umich.edu/resources/ugactivities/Labs/brightness/index.html
For stars, this doesn't matter : except for the Sun they're all point sources. For galaxies and nebulae it's important, but off the top of my head I don't know how many examples there are where it would be worth knowing for naked-eye observations.

This provoked a follow-up question :
Q : Surface brightness gives a better understanding but it still does not tell us how bright the object is to the naked eye. The surface brightness of Andromeda galaxy is 22 and that makes it look like an impossible target. Even from our light polluted skies, we can see it even with a good but modest 10 x 50 binocular. That is because the area around the nucleus of the galaxy is considerably brighter than magnitude 22. So should we not have a brightness measure for such diffuse objects done differently?

A : Surface brightness doesn't have a single value for each object. The centre of most galaxies is much brighter than their outer regions, so an average value for the whole galaxy won't give you an accurate induction of whether it's visible at all. A better value would be the peak surface brightness, but this gets a bit complicated since total brightness will also depend on size.

Another option would be isophotal radius : how large an object's appears on the sky at the radius at which its surface brightness is just detectable. That will tell you both if the object is detectable at all (so long as its angular size is better than the resolution of the eye) and of it is, how large it will appear. It won't work well for everything but it should be good for most nebulae and galaxies.

14 comments:

  1. How firm is the data indicating that the expansion of the universe is accelerating? Is this something that with new data or better analysis might be found not to be the case 20 or 30 years from now?

    ReplyDelete
  2. Great question ! Added my answer to the Cosmology section. For your convenience I'll also provide it here :

    I'd say it's pretty solid. Maybe not 100% certain, but good enough for government work. :)

    The evidence that the expansion comes from supernovae explosions - specifically, type Ia supernovae. These happen when a white dwarf gains enough mass from a companion star that it re-start fusion. The point at which this happens is thought to only depend on mass so the resulting explosion should always be of the same energy. Knowing that energy, we can work out the distance to the supernova pretty accurately.

    The benefit of using supernovae is that we can measure both their redshift and distance even in distant galaxies. Because light travels at a finite speed, the further away a galaxy is, the younger (and therefore smaller) the Universe was when light left that galaxy. After accounting for this, the supernovae data indicate that the acceleration of the Universe is increasing - which is something nobody was expecting.

    http://stardate.org/astro-guide/btss/cosmology/dark_energy

    There was an alternative interpretation of the supernovae data : we live in a void, a region of the Universe which happens to have much less matter than the rest. Since there's less matter inside the void, there's less gravity to slow down its expansion, which would look like acceleration.

    This led me to a rather interesting paper-chase. Here's an original press release from 2009 categorically stating that we don't live in such a void :
    http://phys.org/news152374990.html
    And here, on the exact same day, is another wesbite interpreting this to mean that we DO live in a void and dark energy is wrong !
    http://archive.news.softpedia.com/news/Void-Theory-to-Replace-Dark-Energy-Hypothesis-103226.shtml

    More recent, independent evidence futher supports the notion that we don't live in a void. As far as we can tell, the expansion really is accelerating.
    http://www.pbs.org/wgbh/nova/blogs/physics/2012/04/whos-afraid-of-the-dark-alternatives-to-dark-energy/

    ReplyDelete
  3. Dear Rhys, by what factor is Galactic astronomy cooler than extragalactic astronomy? Is it 7, 17, or 70?

    ReplyDelete
    Replies
    1. Since Galactic astronomy is a mere sub-branch of extragalactic astronomy, it is obviously the less cool of the two. Exactly how much less awesome it is will depend on the number of galaxies. Clearly, you're talking about powers of ten. There are definitely more than 10^7 galaxies, as you well know, so I expect that one was a red herring. 10^17 is a plausible number given that the observable Universe is probably not the whole Universe. However, if inflationary theory is correct then the whole Universe is likely to be very much larger indeed, in which case Galactic astronomy could easily be a factor 10^-70 cooler than extragalactic astronomy.

      Delete
    2. A mere sub-branch?! I seriously laughed at that one. Based on this backward reasoning, we should invert your numbers and deduce that Galactic astronomy is 10^70 times more awesome than extragalactic astronomy. Sounds good - we should've settled this back at Arecibo. :)

      Delete
    3. Bah, you're just jealous because your sample size is 10^70 times smaller.

      Delete
  4. This comment has been removed by a blog administrator.

    ReplyDelete
  5. This comment has been removed by a blog administrator.

    ReplyDelete
  6. This comment has been removed by the author.

    ReplyDelete
  7. This comment has been removed by the author.

    ReplyDelete
  8. Regarding your orion discovery model ( the #sillyspaceshipissilly section of this page), do remember that even though an external pulse drive has no need for radiators, you will still need radiators to handle the heat generated by internal systems (computers, the crew...) and you'll especially need radiators if you have an internal nuclear reactor to supply electrical power to the ship. Even better one should remember that with the Power=const*A*T^4 law low temperature radiators, for crew and computer temperatures (as well as for if the cold end of your nuclear reactor's generator is particularly cold), tend to be pretty big while high temperature radiators can often be much smaller even though they transfer far more watts of heat. While this might not have been your original thoughts when designing your orion discovery ship you can certainly provide physically plausible justifications for the existence of those radiators by saying they are big ones to dump low temperature heat.

    ReplyDelete
  9. Best Corporate Video Production Company in Bangalore and top Explainer Video Company in Bangalore , 3d, 2d Animation Video Makers in Chennai

    Awesome article. good read blog. Thanks for sharing

    ReplyDelete
  10. Best Corporate Video Production Company in Bangalore and top Explainer Video Company in Bangalore , 3d, 2d Animation Video Makers in Chennai

    Awesome article. good read blog. Thanks for sharing

    ReplyDelete

Due to a small but consistent influx of spam, comments will now be checked before publishing. Only egregious spam/illegal/racist crap will be disapproved, everything else will be published.